Sie sind auf Seite 1von 343

1

LIFE ASSURANCE MATHEMATICS

W.F.Scott

c
°1999 W.F.Scott
Department of Mathematical Sciences
King’s College
University of Aberdeen
Aberdeen
AB24 3UE
U.K.
2

Preface
This book consists largely of material written for Parts A2 and D1 of the U.K. actuarial exami-
nations (old system). It is hoped that the material given here will prove useful for much of Subjects
104 and 105 of the new examinations and certain similar examinations at universities, and that it
may also be useful as a general reference work on life assurance mathematics.

William F. Scott. November 10, 1999


Contents

1 NON-SELECT LIFE TABLES 9


1.1 Survivorship functions . . . . . . . . . . . . . . . . . . . . . . . . . . . . . . . . . . . 9
1.2 Probabilities of death and survival . . . . . . . . . . . . . . . . . . . . . . . . . . . . 9
1.3 The force of mortality, µx . . . . . . . . . . . . . . . . . . . . . . . . . . . . . . . . . 12
1.4 The expectation of life . . . . . . . . . . . . . . . . . . . . . . . . . . . . . . . . . . . 15
1.5 The assumption of a uniform distribution of deaths . . . . . . . . . . . . . . . . . . . 17
1.6 Central death rates . . . . . . . . . . . . . . . . . . . . . . . . . . . . . . . . . . . . . 18
1.7 Laws of mortality . . . . . . . . . . . . . . . . . . . . . . . . . . . . . . . . . . . . . . 19
1.8 Exercises . . . . . . . . . . . . . . . . . . . . . . . . . . . . . . . . . . . . . . . . . . 22
1.9 Solutions . . . . . . . . . . . . . . . . . . . . . . . . . . . . . . . . . . . . . . . . . . 24

2 SELECT LIFE TABLES 29


2.1 What is selection? . . . . . . . . . . . . . . . . . . . . . . . . . . . . . . . . . . . . . 29
2.2 Construction of select tables . . . . . . . . . . . . . . . . . . . . . . . . . . . . . . . . 30
2.3 The construction of A1967-70. . . . . . . . . . . . . . . . . . . . . . . . . . . . . . . . 31
2.4 Some formulae for the force of mortality. . . . . . . . . . . . . . . . . . . . . . . . . . 32
2.5 Select tables used in examinations. . . . . . . . . . . . . . . . . . . . . . . . . . . . . 32
2.6 Exercises . . . . . . . . . . . . . . . . . . . . . . . . . . . . . . . . . . . . . . . . . . 33
2.7 Solutions . . . . . . . . . . . . . . . . . . . . . . . . . . . . . . . . . . . . . . . . . . 34

3 ASSURANCES 37
3.1 A general introduction . . . . . . . . . . . . . . . . . . . . . . . . . . . . . . . . . . . 37
3.2 Whole life assurances . . . . . . . . . . . . . . . . . . . . . . . . . . . . . . . . . . . . 37
3.3 Commutation functions . . . . . . . . . . . . . . . . . . . . . . . . . . . . . . . . . . 39
3.4 The variance of the present value of benefits . . . . . . . . . . . . . . . . . . . . . . . 39
3.5 Assurances payable at the end of the year of death. . . . . . . . . . . . . . . . . . . . 41
3.6 Assurances payable at the end of the 1/m of a year of death. . . . . . . . . . . . . . 44
3.7 Temporary and deferred assurances . . . . . . . . . . . . . . . . . . . . . . . . . . . . 44
3.8 Pure endowments and endowment assurances . . . . . . . . . . . . . . . . . . . . . . 46
3.9 Varying assurances . . . . . . . . . . . . . . . . . . . . . . . . . . . . . . . . . . . . . 48
3.10 Valuing the benefits under with profits policies . . . . . . . . . . . . . . . . . . . . . 50
3.11 Guaranteed bonus policies . . . . . . . . . . . . . . . . . . . . . . . . . . . . . . . . . 54
3.12 Exercises . . . . . . . . . . . . . . . . . . . . . . . . . . . . . . . . . . . . . . . . . . 55
3.13 Solutions . . . . . . . . . . . . . . . . . . . . . . . . . . . . . . . . . . . . . . . . . . 57

4 ANNUITIES 61
4.1 Annuities payable continuously . . . . . . . . . . . . . . . . . . . . . . . . . . . . . . 61
4.2 Annuities payable annually . . . . . . . . . . . . . . . . . . . . . . . . . . . . . . . . 63
4.3 Temporary annuities . . . . . . . . . . . . . . . . . . . . . . . . . . . . . . . . . . . . 65
4.4 Deferred annuities . . . . . . . . . . . . . . . . . . . . . . . . . . . . . . . . . . . . . 67

3
4 CONTENTS

4.5 Annuities payable m times per annum . . . . . . . . . . . . . . . . . . . . . . . . . . 68


4.6 Complete annuities (or “annuities with final proportion”) . . . . . . . . . . . . . . . 70
4.7 Varying annuities . . . . . . . . . . . . . . . . . . . . . . . . . . . . . . . . . . . . . . 71
4.8 Exercises . . . . . . . . . . . . . . . . . . . . . . . . . . . . . . . . . . . . . . . . . . 77
4.9 Solutions . . . . . . . . . . . . . . . . . . . . . . . . . . . . . . . . . . . . . . . . . . 78

5 PREMIUMS 81
5.1 Principles of premium calculations . . . . . . . . . . . . . . . . . . . . . . . . . . . . 81
5.2 Notation for premiums . . . . . . . . . . . . . . . . . . . . . . . . . . . . . . . . . . . 82
5.3 The variance of the present value of the profit on a policy. . . . . . . . . . . . . . . . 83
5.4 Premiums allowing for expenses . . . . . . . . . . . . . . . . . . . . . . . . . . . . . . 83
5.5 Premiums for with profits policies . . . . . . . . . . . . . . . . . . . . . . . . . . . . 85
5.6 Return of premium problems . . . . . . . . . . . . . . . . . . . . . . . . . . . . . . . 86
5.7 Annuities with guarantees . . . . . . . . . . . . . . . . . . . . . . . . . . . . . . . . . 87
5.8 Family income benefits . . . . . . . . . . . . . . . . . . . . . . . . . . . . . . . . . . . 89
5.9 Exercises . . . . . . . . . . . . . . . . . . . . . . . . . . . . . . . . . . . . . . . . . . 91
5.10 Solutions . . . . . . . . . . . . . . . . . . . . . . . . . . . . . . . . . . . . . . . . . . 93

6 RESERVES 95
6.1 What are reserves? . . . . . . . . . . . . . . . . . . . . . . . . . . . . . . . . . . . . . 95
6.2 Prospective reserves . . . . . . . . . . . . . . . . . . . . . . . . . . . . . . . . . . . . 95
6.3 Net premium reserves . . . . . . . . . . . . . . . . . . . . . . . . . . . . . . . . . . . 96
6.4 Retrospective reserves . . . . . . . . . . . . . . . . . . . . . . . . . . . . . . . . . . . 99
6.5 Gross premium valuations and asset shares. . . . . . . . . . . . . . . . . . . . . . . . 102
6.6 The variance of L . . . . . . . . . . . . . . . . . . . . . . . . . . . . . . . . . . . . . . 104
6.7 Zillmerised reserves . . . . . . . . . . . . . . . . . . . . . . . . . . . . . . . . . . . . . 106
6.8 Full preliminary term reserves. . . . . . . . . . . . . . . . . . . . . . . . . . . . . . . 109
6.9 Reserves for with-profits policies . . . . . . . . . . . . . . . . . . . . . . . . . . . . . 110
6.10 Exercises . . . . . . . . . . . . . . . . . . . . . . . . . . . . . . . . . . . . . . . . . . 113
6.11 Solutions . . . . . . . . . . . . . . . . . . . . . . . . . . . . . . . . . . . . . . . . . . 115

7 APPLICATIONS OF RESERVES 119


7.1 Surrender values . . . . . . . . . . . . . . . . . . . . . . . . . . . . . . . . . . . . . . 119
7.2 Paid-up policy values . . . . . . . . . . . . . . . . . . . . . . . . . . . . . . . . . . . . 120
7.3 Alterations and conversions . . . . . . . . . . . . . . . . . . . . . . . . . . . . . . . . 121
7.4 The actual and expected death strains . . . . . . . . . . . . . . . . . . . . . . . . . . 124
7.5 Mortality profit and loss . . . . . . . . . . . . . . . . . . . . . . . . . . . . . . . . . . 125
7.6 Other sources of profit and loss . . . . . . . . . . . . . . . . . . . . . . . . . . . . . . 129
7.7 Exercises . . . . . . . . . . . . . . . . . . . . . . . . . . . . . . . . . . . . . . . . . . 133
7.8 Solutions . . . . . . . . . . . . . . . . . . . . . . . . . . . . . . . . . . . . . . . . . . 135

8 EXTRA RISKS 137


8.1 Introduction . . . . . . . . . . . . . . . . . . . . . . . . . . . . . . . . . . . . . . . . . 137
8.2 A constant addition to the force of mortality . . . . . . . . . . . . . . . . . . . . . . 137
8.3 A variable addition to the force of mortality . . . . . . . . . . . . . . . . . . . . . . . 139
8.4 Rating up . . . . . . . . . . . . . . . . . . . . . . . . . . . . . . . . . . . . . . . . . . 140
8.5 Debts . . . . . . . . . . . . . . . . . . . . . . . . . . . . . . . . . . . . . . . . . . . . 142
8.6 Exercises . . . . . . . . . . . . . . . . . . . . . . . . . . . . . . . . . . . . . . . . . . 145
8.7 Solutions . . . . . . . . . . . . . . . . . . . . . . . . . . . . . . . . . . . . . . . . . . 147
CONTENTS 5

9 PROFIT-TESTING 151
9.1 Principles of profit-testing . . . . . . . . . . . . . . . . . . . . . . . . . . . . . . . . . 151
9.2 Cash flow calculations . . . . . . . . . . . . . . . . . . . . . . . . . . . . . . . . . . . 152
9.3 The profit vector and the profit signature . . . . . . . . . . . . . . . . . . . . . . . . 155
9.4 The assessment of profits . . . . . . . . . . . . . . . . . . . . . . . . . . . . . . . . . 157
9.5 Some theoretical results about {σt } . . . . . . . . . . . . . . . . . . . . . . . . . . . . 159
9.6 Withdrawals . . . . . . . . . . . . . . . . . . . . . . . . . . . . . . . . . . . . . . . . 160
9.7 The actual emergence of profits . . . . . . . . . . . . . . . . . . . . . . . . . . . . . . 162
9.8 Exercises . . . . . . . . . . . . . . . . . . . . . . . . . . . . . . . . . . . . . . . . . . 165
9.9 Solutions . . . . . . . . . . . . . . . . . . . . . . . . . . . . . . . . . . . . . . . . . . 167

10 STATIONARY POPULATIONS 171


10.1 Some Definitions . . . . . . . . . . . . . . . . . . . . . . . . . . . . . . . . . . . . . . 171
10.2 The Central Death Rate . . . . . . . . . . . . . . . . . . . . . . . . . . . . . . . . . . 175
10.3 Relationships Between mx and qx . . . . . . . . . . . . . . . . . . . . . . . . . . . . . 176
10.4 Stationary Funds . . . . . . . . . . . . . . . . . . . . . . . . . . . . . . . . . . . . . . 176
10.5 Exercises . . . . . . . . . . . . . . . . . . . . . . . . . . . . . . . . . . . . . . . . . . 178
10.6 Solutions . . . . . . . . . . . . . . . . . . . . . . . . . . . . . . . . . . . . . . . . . . 180

11 JOINT-LIFE FUNCTIONS 185


11.1 Joint-Life Mortality Tables . . . . . . . . . . . . . . . . . . . . . . . . . . . . . . . . 185
11.2 Select Tables . . . . . . . . . . . . . . . . . . . . . . . . . . . . . . . . . . . . . . . . 188
11.3 Extensions to More than 2 Lives . . . . . . . . . . . . . . . . . . . . . . . . . . . . . 188
11.4 The Joint Expectation of Life . . . . . . . . . . . . . . . . . . . . . . . . . . . . . . . 190
11.5 Monetary Functions . . . . . . . . . . . . . . . . . . . . . . . . . . . . . . . . . . . . 191
11.6 Last Survivor Probabilities (two lives only) . . . . . . . . . . . . . . . . . . . . . . . 195
11.7 Last Survivor Monetary Functions . . . . . . . . . . . . . . . . . . . . . . . . . . . . 196
11.8 Reserves for Last Survivor Assurances . . . . . . . . . . . . . . . . . . . . . . . . . . 198
11.9 Exercises . . . . . . . . . . . . . . . . . . . . . . . . . . . . . . . . . . . . . . . . . . 200
11.10Solutions . . . . . . . . . . . . . . . . . . . . . . . . . . . . . . . . . . . . . . . . . . 201

12 CONTINGENT ASSURANCES 205


12.1 Contingent Probabilities . . . . . . . . . . . . . . . . . . . . . . . . . . . . . . . . . . 205
12.2 Contingent Assurances . . . . . . . . . . . . . . . . . . . . . . . . . . . . . . . . . . . 206
12.3 Premiums and Reserves for Contingent Assurances . . . . . . . . . . . . . . . . . . . 210
12.4 A Practical Application – The Purchase of Reversions . . . . . . . . . . . . . . . . . 211
12.5 Extension to Three Lives . . . . . . . . . . . . . . . . . . . . . . . . . . . . . . . . . 212
12.6 Exercises . . . . . . . . . . . . . . . . . . . . . . . . . . . . . . . . . . . . . . . . . . 213
12.7 Solutions . . . . . . . . . . . . . . . . . . . . . . . . . . . . . . . . . . . . . . . . . . 215

13 REVERSIONARY ANNUITIES 219


13.1 Reversionary Annuities Payable Continuously . . . . . . . . . . . . . . . . . . . . . . 219
13.2 Reversionary Annuities Payable Annually or mthly . . . . . . . . . . . . . . . . . . . 220
13.3 Widow’s (or Spouse’s) Pension on Death after Retirement . . . . . . . . . . . . . . . 221
13.4 Actuarial Reduction Factors . . . . . . . . . . . . . . . . . . . . . . . . . . . . . . . . 224
13.5 Exercises . . . . . . . . . . . . . . . . . . . . . . . . . . . . . . . . . . . . . . . . . . 225
13.6 Solutions . . . . . . . . . . . . . . . . . . . . . . . . . . . . . . . . . . . . . . . . . . 227
6 CONTENTS

14 PROFIT TESTING FOR UNIT-LINKED POLICIES 231


14.1 Unit-Linked Policies . . . . . . . . . . . . . . . . . . . . . . . . . . . . . . . . . . . . 231
14.2 Mechanics of the Unit Fund . . . . . . . . . . . . . . . . . . . . . . . . . . . . . . . . 231
14.3 The Sterling Fund (or Sterling Reserves) . . . . . . . . . . . . . . . . . . . . . . . . . 233
14.4 The Assessment of Profits . . . . . . . . . . . . . . . . . . . . . . . . . . . . . . . . . 235
14.5 Zeroisation of the Profit Signature . . . . . . . . . . . . . . . . . . . . . . . . . . . . 236
14.6 Withdrawals . . . . . . . . . . . . . . . . . . . . . . . . . . . . . . . . . . . . . . . . 237
14.7 Exercises . . . . . . . . . . . . . . . . . . . . . . . . . . . . . . . . . . . . . . . . . . 240
14.8 Solutions . . . . . . . . . . . . . . . . . . . . . . . . . . . . . . . . . . . . . . . . . . 243

15 MULTIPLE-DECREMENT TABLES 247


15.1 Introduction . . . . . . . . . . . . . . . . . . . . . . . . . . . . . . . . . . . . . . . . . 247
15.2 The Associated Single-Decrement Tables . . . . . . . . . . . . . . . . . . . . . . . . . 249
15.3 The Relationships between the Multiple-Decrement Table and its Associated Single-
Decrement Tables . . . . . . . . . . . . . . . . . . . . . . . . . . . . . . . . . . . . . . 249
15.4 Dependent Rates of Exit . . . . . . . . . . . . . . . . . . . . . . . . . . . . . . . . . . 250
15.5 Practical Construction of Multiple-Decrement Tables . . . . . . . . . . . . . . . . . . 253
15.6 Further Formulae . . . . . . . . . . . . . . . . . . . . . . . . . . . . . . . . . . . . . . 256
15.7 Generalization to 3 Modes of Decrement . . . . . . . . . . . . . . . . . . . . . . . . . 257
15.8 “Abnormal” Incidence of Decrement . . . . . . . . . . . . . . . . . . . . . . . . . . . 258
15.9 Exercises . . . . . . . . . . . . . . . . . . . . . . . . . . . . . . . . . . . . . . . . . . 263
15.10Solutions . . . . . . . . . . . . . . . . . . . . . . . . . . . . . . . . . . . . . . . . . . 264

16 FINANCIAL CALCULATIONS USING MULTIPLE-DECREMENT TABLES 267


16.1 Principles . . . . . . . . . . . . . . . . . . . . . . . . . . . . . . . . . . . . . . . . . . 267
16.2 The Use of “Defective” Variables . . . . . . . . . . . . . . . . . . . . . . . . . . . . . 267
16.3 Evaluation of Mean Present Values . . . . . . . . . . . . . . . . . . . . . . . . . . . . 268
16.4 Benefits on Death by a Particular Cause . . . . . . . . . . . . . . . . . . . . . . . . . 272
16.5 Extra Risks Treated as an Additional Mode of Decrement . . . . . . . . . . . . . . . 272
16.6 Calculations Involving a Change of State . . . . . . . . . . . . . . . . . . . . . . . . . 274
16.7 Exercises . . . . . . . . . . . . . . . . . . . . . . . . . . . . . . . . . . . . . . . . . . 275
16.8 Solutions . . . . . . . . . . . . . . . . . . . . . . . . . . . . . . . . . . . . . . . . . . 277

17 MULTIPLE-STATE MODELS 281


17.1 Two Points of View . . . . . . . . . . . . . . . . . . . . . . . . . . . . . . . . . . . . 281
17.2 Kolmogorov’s Forward Equations . . . . . . . . . . . . . . . . . . . . . . . . . . . . . 281
17.3 Life Tables as Stochastic Processes . . . . . . . . . . . . . . . . . . . . . . . . . . . . 283
17.4 Sickness Models . . . . . . . . . . . . . . . . . . . . . . . . . . . . . . . . . . . . . . . 284

18 SICKNESS FUNCTIONS 287


18.1 Rates of Sickness . . . . . . . . . . . . . . . . . . . . . . . . . . . . . . . . . . . . . . 287
18.2 Valuing Sickness Benefits . . . . . . . . . . . . . . . . . . . . . . . . . . . . . . . . . 289
18.3 Various Other Points . . . . . . . . . . . . . . . . . . . . . . . . . . . . . . . . . . . . 292
18.4 Exercises . . . . . . . . . . . . . . . . . . . . . . . . . . . . . . . . . . . . . . . . . . 295
18.5 Solutions . . . . . . . . . . . . . . . . . . . . . . . . . . . . . . . . . . . . . . . . . . 298
Appendix The Manchester Unity Experience 1893-97 . . . . . . . . . . . . . . . . . . . . . 301
CONTENTS 7

19 PENSION FUNDS 303


19.1 General Introduction . . . . . . . . . . . . . . . . . . . . . . . . . . . . . . . . . . . . 303
19.2 Valuation Principles . . . . . . . . . . . . . . . . . . . . . . . . . . . . . . . . . . . . 303
19.3 Service Tables . . . . . . . . . . . . . . . . . . . . . . . . . . . . . . . . . . . . . . . . 304
19.4 Salary Scales . . . . . . . . . . . . . . . . . . . . . . . . . . . . . . . . . . . . . . . . 304
19.5 The Value of Future Contributions . . . . . . . . . . . . . . . . . . . . . . . . . . . . 306
19.6 The Value of Pension Benefits . . . . . . . . . . . . . . . . . . . . . . . . . . . . . . . 308
19.7 Fixed Pension Schemes . . . . . . . . . . . . . . . . . . . . . . . . . . . . . . . . . . . 309
19.8 Average Salary Schemes . . . . . . . . . . . . . . . . . . . . . . . . . . . . . . . . . . 311
19.9 Final Salary Schemes . . . . . . . . . . . . . . . . . . . . . . . . . . . . . . . . . . . . 313
19.10Lump Sums on Retirement . . . . . . . . . . . . . . . . . . . . . . . . . . . . . . . . 316
19.11Death and Withdrawal Benefits . . . . . . . . . . . . . . . . . . . . . . . . . . . . . . 317
19.12Return of Contributions on Death or Withdrawal . . . . . . . . . . . . . . . . . . . . 318
19.13Spouse’s Benefits . . . . . . . . . . . . . . . . . . . . . . . . . . . . . . . . . . . . . . 320
19.14Preserved Pensions on Leaving Service . . . . . . . . . . . . . . . . . . . . . . . . . . 322
19.15Exercises . . . . . . . . . . . . . . . . . . . . . . . . . . . . . . . . . . . . . . . . . . 324
19.16Solutions . . . . . . . . . . . . . . . . . . . . . . . . . . . . . . . . . . . . . . . . . . 327
Appendix Formulae for valuing a return of contributions . . . . . . . . . . . . . . . . . . . 331

APPENDICES 333
A Some notes on examination technique . . . . . . . . . . . . . . . . . . . . . . . . . . 333
B Some technical points about the tables used in examinations . . . . . . . . . . . . . 334
C Some common mistakes . . . . . . . . . . . . . . . . . . . . . . . . . . . . . . . . . . 335
D Some formulae for numerical integration . . . . . . . . . . . . . . . . . . . . . . . . . 336

SUPPLEMENT 337
8 CONTENTS
Chapter 1

NON-SELECT LIFE TABLES

1.1 Survivorship functions


We consider a certain population from age α to a “limiting age” w , i.e. the youngest age to which
no-one survives. (In theoretical work we may have w = ∞.) Let (x) be a shorthand notation for “a
life aged (exactly) x”; we define the survivorship function

s(x) = P r{ (α) survives to age x } (1.1.1)

In practice one usually uses the function

lx = lα s(x) (1.1.2)

where lα is called the “radix” of the table, and is usually a large number such as 1, 000, 000. If lives
are considered from birth, we naturally have α = 0, and we have lx > 0 for x < w, lw = 0, unless
w is infinite, in which case lx → 0 as x → ∞. The function lx is assumed to be continuous. By
formulae (1.1.1) and (1.1.2) ,

lx
P r{ (α) survives to age x } = (1.1.3)

Example 1.1.1. English Life Table No.12 - Males refers to the male population of England and
Wales in 1970-72. In this table α = 0 , w may be taken to be about 106, and the radix of the table is
100, 000. We observe (for example) that, since l50 = 90, 085 , the probability that a new-born child
will survive to age 50 is s(50) = l50 /l0 = 0.90085, or 90.1%.

We now consider survival from age α in terms of a random variable. Let T (or T (x) if the age x

is not clear; Tx is sometimes used, but this may be confused with the symbol Tx = lx ex used in
stationary population problems) be the variable representing the future lifetime (in years, including
fractions) of (x). According to our definitions,

P r{ T (α) ≤ t } = the probability that (α) will die within t years


= 1 − the probability that (α) will survive for at least t years
= 1 − s(α + t) (1.1.4)

9
10 CHAPTER 1. NON-SELECT LIFE TABLES

1.2 Probabilities of death and survival


We now consider how to deal with lives aged between α and w. To simplify matters, we shall suppose
here that w = ∞. The variables {T (x) : x ≥ α} are assumed to be related as follows:
for each x1 , x2 , x3 such that α ≤ x1 ≤ x2 ≤ x3 ,
P r{T (x1 ) > x3 − x1 } = P r{T (x1 ) > x2 − x1 }P r{T (x2 ) > x3 − x2 } (1.2.1)
In particular, when x1 = α, x2 = x and x3 = x + t, we have
P r{T (α) > x − α + t} s(x + t) lx+t
P r{T (x) > t} = = = (1.2.2)
P r{T (α) > x − α} s(x) lx

We introduce the important notation:

t px = P r{T (x) > t} = P r{ (x) survives to age x + t} (1.2.3)

t qx = P r{T (x) ≤ t} = P r{ (x) dies between ages x and x + t} (1.2.4)


When t = 1 , it may be omitted, so that

px = P r{ (x) survives for at least a year } (1.2.5)

qx = P r{ (x) dies within a year } (1.2.6)


qx is called the (q-type) rate of mortality at age x. (The m-type, or central, rate of mortality,
mx , is considered in section 1.6.) It is clear that, for all t ≥ 0 ,

t px + t qx = 1 (1.2.7)

and, by the formula (1.2.2),

s(x + t) lx+t
t px = = (1.2.8)
s(x) lx
and
lx+t
t qx =1− (1.2.9)
lx
That is, the (cumulative) distribution function of the variable T is
(
1 − lx+t
lx , if t≥0
F (t) = P r{T ≤ t} = t qx = (1.2.10)
0 , if t<0
We may say that “the expected number of survivors at age x + t among lx lives aged x is lx+t ”
because the chance that a given life aged x will survive to age x + t is lx+t
lx . In practice, the word
“expected” is sometimes omitted, but it is always to be understood; assuming that the lx lives are
independent, the number of survivors at age x + t will follow a binomial distribution with mean
lx · t px = lx+t

and variance
µ ¶
lx+t
lx · t px (1 − t px ) = lx+t 1 −
lx
1.2. PROBABILITIES OF DEATH AND SURVIVAL 11

(Even if the lives are not independent, the formula for the expected number of deaths is correct.)
Note that
lx+1
px = (1.2.11)
lx
lx+1 dx
qx = 1 − = (1.2.12)
lx lx

where

dx = lx − lx+1 (1.2.13)
= the (expected) number of deaths at age x last
birthday among lx lives aged x

Example 1.2.1. The following extract from English Life Table No. 12 - Males illustrates the
functions lx , dx , px and qx .

age
x lx dx px qx

0 100,000 2,499 0.97551 0.02449

1 97,551 153 0.99843 0.00157

2 97,398 96 0.99901 0.00099

3 97,302 67 0.99931 0.00069

4 97,235 60 0.99938 0.00062

(The high death rate at age 0, relative to the rates at ages 1 to 4, is quite noticeable.)

We also observe that, for all ages x3 ≥ x2 ≥ x1 ≥ α , we have the “rule of multiplication of
probabilities of survival”:
x3 −x1 px1 = x2 −x1 px1 · x3 −x2 px2 (1.2.14)
(which is merely a re-statement of equation (1.2.1). )

Note. The definition of t px is sometimes taken to be


P r{(α) survives to age x + t | (α) survives to age x }, which equals

P r{T (α) > x − α + t and T (α) > x − α} P r{T (α) > x − α + t}


=
P r{T (α) > x − α} P r{T (α) > x − α}
= P r{T (x) > t}

in agreement with our earlier definition. Further, t px is sometimes written as Sx (t). When α = 0
we sometimes write S0 (t) = S(t), so S(t) and s(t) are equal. The notation S(t) is much used in
survival analysis, in which t denotes the time lived by a patient since the start of a given treatment.
In reliability engineering the notation R(t) may be used instead of S(t).
12 CHAPTER 1. NON-SELECT LIFE TABLES

1.3 The force of mortality, µx


This may be defined as the “instantaneous rate of mortality”, i.e.
h qx
µx = lim (1.3.1)
h→0+ h

Theorem 1.3.1. Suppose that µx is continuous on [α, w). We have

lx0
µx = − (1.3.2)
lx
Proof. we have
−[s(x + h) − s(x)] −s0+ (x)
µx = lim =
h→0+ hs(x) s(x)
where s0+ (x) denotes the R.H. derivative of s(x). But s(x) and s0+ (x) = −s(x)µx are continuous
on [α, w), so s0 (x) exists and equals −s(x)µx (see McCutcheon and Scott, An Introduction to the
Mathematics of Finance, Appendix 1. )
Hence
s0 (x) l0
µx = − =− x
s(x) lx

We remark that
d
µx = − log lx (1.3.3)
dx
Hence
Z x Z x
d
− µy dy = (log ly ) dy
α α dy
x
= [ log ly ]α
= log lx − log lα
µ ¶
lx
= log

from which we obtain the important formulae:


µ Z x ¶
lx = lα exp − µy dy (1.3.4)
α

and
µ Z x ¶
s(x) = exp − µy dy (1.3.5)
α

It follows that
µ Z x+t ¶ µ Z t ¶
lx+t
t px = = exp − µy dy = exp − µx+r dr (1.3.6)
lx x 0

If µx is piecewise continuous, we may apply the above formulae over each age-range and combine
the results; the above formulae remain true, except that lx0 does not exist at the points at which
1.3. THE FORCE OF MORTALITY, µx 13

µx is not continuous (the R.H. and L.H. derivatives of lx differ at these points.) Alternatively, we
may regard the actual position as closely approximated by a model in which the force of mortality
is continuous.
We may now express the probability density function of the variable T (the future lifetime of (x))
in terms of the force of mortality. The p.d.f. f (t) of T is the derivative of the distribution function
F (t), i.e.  ³ ´
d
0 1 − lx+t
lx , if t > 0
f (t) = F (t) = dt
0 , if t < 0
By the chain rule
d 0
lx+t = lx+t
dt
= −lx+t µx+t (by formula (1.3.2))

so we obtain (
lx+t µx+t
lx = t px µx+t , if t > 0
f (t) = (1.3.7)
0 , if t < 0
In view of the following general formula (connecting the d.f. and p.d.f.):
Z t
F (t) = f (r) dr
−∞

we have Z t
t qx = r px µx+r dr (1.3.8)
0

An alternative notation
Some statisticians refer to the force of mortality as the “hazard rate” or “transition intensity.”

Let f (t) and F (t) denote the p.d.f. and d.f. respectively of T = T (x) . The hazard rate at age x + t
(often considered to be “at time t years after entry to assurance (say) at age x”) is defined in terms
of f (t) and F (t) by the formula
f (t)
h(t) = (1.3.9)
1 − F (t)
which equals µx+t (by formulae (1.2.7) and (1.3.7).) If α = 0 and x = 0, we naturally have T = the
future lifetime of a new-born child, and

f (t)
µt = = hazard rate at age t years (1.3.10)
1 − F (t)

Example 1.3.1. Suppose that, for x ≥ α,

µx = cδxδ−1 (δ > 1, c > 0) (1.3.11)

(This is “Weibull’s law of mortality”.) Find a simple expression for Sx (t) = t px .


14 CHAPTER 1. NON-SELECT LIFE TABLES

Solution.
µ Z x ¶
s(x) = exp − µy dy
µ Zαx ¶
δ−1
= exp − cδy dy
³ £α ¤x ´
= exp −c y δ α
¡ ¢ ¡ ¢
= exp −cxδ . exp cαδ

hence
¡ £ ¤¢
t px = Sx (t) = exp −c (x + t)δ − xδ

Numerical estimation of µx from lx


If lx is known only at integer values of x, we may use numerical differentiation to estimate µx . The
formula
f (x + 1) − f (x − 1)
f 0 (x) + (1.3.12)
2
which is exact if f is a cubic between x − 1 and x + 1 , may be used to show that
lx0 lx−1 − lx+1
µx + − = (1.3.13)
lx 2lx

Deferred probabilities
The symbol m | indicates deferment for m years; for example,
m |n qx = P {(x) will die between ages x + m and x + m + n } (1.3.14)
By elementary probability, this equals
P r{(x) will die before age x + m + n} − P r{(x) will die before age x + m}
= m+n qx − m qx
= m px − m+n px
lx+m − lx+m+n
=
lx
That is,
lx+m − lx+m+n
m |n qx = (1.3.15)
lx
This may be remembered by the following rule: of lx lives aged x, lx+m − lx+m+n is the (expected)
number of deaths aged between ages x + m and x + m + n. If n = 1 it may be omitted, so we have

m |qx = P r{(x) will die between the ages x + m and x + m + 1 }


dx+m
= (1.3.16)
lx
We remark that there is no such thing as m |n px .

We may also use the result that

m |n qx = P r{(x) survives to age x + m}.P r{(x + m) dies within n years}


= m px .n qx+m (1.3.17)
1.4. THE EXPECTATION OF LIFE 15

Example 1.3.2. Consider the force of mortality to be as in Example 1.3.1. Find an expression for
the chance that (x) will die between ages x + m and x + m + n.

Solution. We require:
lx+m − lx+m+n
m |n qx =
lx
s(x + m) − s(x + m + n)
=
s(x)
exp[−c(x + m)δ ] − exp[−c(x + m + n)δ ]
=
exp[−cxδ ]

1.4 The expectation of life



Let T be the future lifetime (in years, including fractions) of (x). The mean of T is written ex , and
is called the complete expectation of life at age x. That is,

ex = E(T ) (1.4.1)
Z ∞
= t · t px µx+t dt (1.4.2)
0

by formula 1.3.7. If w < ∞ , the upper limit of the integral should be replaced by w − x.

Theorem 1.4.1. Z ∞

ex = t px dt (1.4.3)
0
(with ∞ replaced by w − x if w is finite).
Proof. By integration by parts, with u = t , v = −t px ,
Z ∞ Z ∞
◦ ∞
ex = t · t px µx+t dt = [−t · t px ]0 − −t px dt
0 0
Z ∞
= t px dt
0

since it may be shown that t · t px → 0 as t → ∞ (using the fact that E(T ) < ∞. ).
We may also use the formula
Var(T ) = E(T 2 ) − [E(T )]2 (1.4.4)
where
Z ∞
E(T 2 ) = t2 · t px µx+t dt
0
Z ∞
£ ¤∞
= −t2 · t px 0 + 2t · t px dt
0

(on setting u = t2 , v = −t px )
Z ∞
=2 t · t px dt
0
16 CHAPTER 1. NON-SELECT LIFE TABLES

(since t2 · t px → ∞ , because E(T 2 ) < ∞ ).


Hence Z ∞

Var(T ) = 2 t · t px dt − (ex )2 (1.4.5)
0

The median future lifetime of (x) is the solution, t, of the equation


F (t) = 0.5
i.e., t qx = 0.5
i.e., lx+t = 0.5 lx (1.4.6)
x + t may be estimated by linear interpolation, as in the next example.

Example 1.4.1. Find the median future lifetime of (10), according to English Life Table No.12 -
Males.

Solution. We must find the value of t such that


l10+t = 0.5 l10 = 48, 469.5
On inspecting the tables, 10 + t lies between 72 and 73. By linear interpolation in y = 10 + t , we
have
ly − l72
= y − 72
l73 − l72
48, 469.5 − 48, 625
i.e. y = 72 +
45, 430 − 48, 625
= 72.05, so t = 62.05 years.
The modal future lifetime of (x) is the time, t0 years, at which the p.d.f. t px µx+t (or, equivalently,
the function lx+t µx+t ) reaches its maximum. The modal age at death of (x) is y = x + t0 , which
is the value of y for which ly µy attains its maximum in the range y ≥ x. (The function ly µy is
sometimes called the “curve of deaths”.) If ly µy has a unique maximum at age 80 (say), this is the
modal age at death for all lives aged under 80, and hence the modal future lifetime of a life aged
x < 80 is 80 − x. (See Exercise 1.7 for an example in which there is a unique modal age at death.)
We now consider the discrete random variable K (or K(x) if the age x is not clear) defined by

K = the integer part of T


= the number of complete years to be lived
in the future by (x) (1.4.7)

Now it follows by formula 1.3.16 that

P r{K = k} = k |qx ( k = 0, 1, 2, ...) (1.4.8)

This variable is used in many actuarial calculations. In particular, the curtate expectation of
life at age x, written ex , is the mean of K ; that is

X ∞
X
ex = k · k |qx = k.k |qx (1.4.9)
k=0 k=1
1.5. THE ASSUMPTION OF A UNIFORM DISTRIBUTION OF DEATHS 17

Theorem 1.4.2.

X lx+1 + lx+2 + . . .
ex = k px = (1.4.10)
lx
k=1

Proof.
dx+1 dx+2 dx+3
ex = +2 +3 + ...
lx lx lx
1
= [(dx+1 + dx+2 + dx+3 + . . .)
lx
+ (dx+2 + dx+3 + . . .)
+ (dx+3 + . . .)
+ . . .]
lx+1 + lx+2 + lx+3 + . . .
=
lx

[since ly = (ly − ly+1 ) + (ly+1 − ly+2 ) + . . .


= dy + dy+1 + . . . ]

We may also evaluate Var(K) by the formula

Var(K) = E(K 2 ) − [E(K)]2



X dx+k
= k2 − (ex )2 (1.4.11)
lx
k=1

As is clear by general reasoning,


1 ◦
ex + ex + (1.4.12)
2
A more precise approximation may be obtained from the Euler-Maclaurin formula, which we shall
discuss later.

1.5 The assumption of a uniform distribution of deaths


Let x be fixed. We may say there is a uniform distribution of deaths (U.D.D.) between ages x
and x + 1 if, for 0 ≤ t ≤ 1,
lx+t = (1 − t)lx + t lx+1 (1.5.1)
(i.e. ly is linear for x ≤ y ≤ x + 1)
This equation may be written in the form

lx+t = lx − tdx (0 ≤ t ≤ 1) (1.5.2)

Theorem 1.5.1. The following conditions are each equivalent to the assumption of U.D.D. between
ages x and x + 1:

t qx = t · qx (0 ≤ t ≤ 1) (1.5.3)
t px µx+t = qx (0 ≤ t < 1) (1.5.4)
18 CHAPTER 1. NON-SELECT LIFE TABLES

Proof. Assume 1.5.2 holds. Then


t px = 1 − t · qx (1.5.5)
But t px = 1 − t qx , so t qx = t · qx ; therefore 1.5.3 holds.
This argument may be reversed, so 1.5.2 and 1.5.3 are equivalent. Now suppose that (1.5.3) holds.
By differentiation with respect to t, we obtain (1.5.4), and we may obtain (1.5.3) from (1.5.4) by
integration.

Example 1.5.1. In a certain non-select mortality table, there is a uniform distribution of deaths
between any two consecutive integer ages. Find formulae in terms of l30 , l31 and l32 for

(i) 1.5 p30.5


(ii) µ30.5

Solution.

l32 l32
(i) = 1 since l30+t is linear for 0 < t < 1
l30.5 2 (l30 + l31 )

q30 l30 − l31 l30 − l31


(ii) t p30 µ30+t = q30 for 0 ≤ t ≤ 1, so µ30.5 = = =
1
2
p30 l30 12 1
2 (l30 + l31 )

1.6 Central death rates


For simplicity of notation, we again suppose that the limiting age of our mortality table is infinity.
Define
Z ∞
Tx = lx+t dt (1.6.1)
Z0 ∞
= ly dy (on setting y = x + t)
x

from which we obtain


R∞
◦ x
ly dy
ex =
lx
Tx
=
lx

and hence

Tx = lx ex (1.6.2)

We also define
Z 1
Lx = lx+t dt (1.6.3)
0
1.7. LAWS OF MORTALITY 19

Note that
Z x+1
Lx = ly dy
Zx∞ Z ∞
= ly dy − ly dy
x x+1
= Tx − Tx+1 (1.6.4)

Assume that there is a Uniform Distribution of Deaths (U.D.D) between the ages x and x + 1, i.e.
lx+t is linear for 0 ≤ t ≤ 1.
We have
Z 1
Lx = lx+t dt = 12 (lx + lx+1 ) (1.6.5)
0

(as the trapezoidal rule is exactly correct, not just an approximation).


Since lx+1 = lx − dx , we have

Lx = lx − 21 dx (1.6.6)

Formulae (1.6.5) and (1.6.6) are sometimes used as approximations when U.D.D. does not hold.
The central death rate at age x is defined as
dx
mx = (1.6.7)
Lx

Note the following important approximation:


R1
0
lx+t µx+t dt
mx = R1 + µx+ 12 (1.6.8)
0
lx+t dt
Relationships between mx and qx
If there is U.D. of D. between ages x and x + 1, formula (1.6.6) shows that
dx
mx =
Lx
dx
=
lx − 12 dx
qx
= (1.6.9)
1 − 21 qx

We may rearrange this equation to get


mx
qx = (1.6.10)
1 + 21 mx
If U.D.D. does not hold, these results may be used as approximations.

1.7 Laws of mortality


The term ‘law of mortality’ is used to describe a mathematical expression for µx (or possibly qx
or mx ) which may be explained from biological or other arguments (rather than being just a best-
fitting curve.) The most famous law of mortality is that of Gompertz (1825), who postulated that
20 CHAPTER 1. NON-SELECT LIFE TABLES

µx satisfies the following simple differential equation:


dµx
= kµx (x ≥ α)
dx
This may be solved to give
µx = Bcx (x ≥ α) (1.7.1)
Gompertz’ Law is often found to be quite accurate (at least as a first approximation) for ages over
about 25 or 30, the value of c usually being between 1.07 and 1.12.

Example 1.7.1. Show that, if Gompertz’ Law holds for all ages greater than or equal to α, there
is a positive constant g such that
x
(ct −1)
t px = gc for x ≥ α, t ≥ 0 (1.7.2)

Solution.
· Z t ¸
t px = exp − µx+r dr
0
· Z t ¸
= exp −Bcx cr dr
0
· ¸
Bcx (ct − 1)
= exp −
log c
· ¸
x
(ct −1) B
= gc with g = exp −
log c

In 1860 Makeham suggested the addition of a constant term to Gompertz’ formula for µx , giv-
ing Makeham’s law:
µx = A + Bcx (x ≥ α) (1.7.3)

Example 1.7.2. Show that, if Makeham’s law holds for all ages greater than or equal to α, there
are positive constants s and g such that
x
(ct −1)
t px = st g c for x ≥ α, t ≥ 0 (1.7.4)

Solution.
· Z t ¸
t px = exp − µx+r dr
0
· Z t ¸
= exp − (A + Bcx cr )dr
0
x
(ct −1)
= exp[−At]g c (using Example 1.7.1)
· ¸
x
(ct −1) B
= st g c with g = exp − , s = e−A
log c
1.7. LAWS OF MORTALITY 21

Weibull’s law of mortality has already been mentioned in Example 1.3.1. In practice it is usu-
ally less successful than those of Gompertz and Makeham in representing human mortality. The
fitting of laws of mortality is complicated by the fact that mortality rates may be varying with time.

If a law of mortality holds, certain mortality and monetary functions may be evaluated analyt-
ically (rather than numerically), but this point is of little practical importance in the computer
age. There are also certain simplifications in the evaluation of joint-life functions (that is, functions
depending on the survival of more than one life.)
22 CHAPTER 1. NON-SELECT LIFE TABLES

Exercises

1.1 Calculate the following probabilities on the basis of English Life Table No. 12-Males.

(i) 40 p25 ,
(ii) 60 |10 q25 ,
(iii) the probability that (30) survives for at least 10 years,
(iv) the probability that (40) survives to age 65,
(v) the probability that (50) dies within 10 years,
(vi) the probability that (50) fails to reach age 70,
(vii) the probability that (60) dies between ages 80 and 85,
(viii) the probability that (60) dies within the first five years after retiring at age 65.
In addition, express (i) and (ii) in words.

1.2 On the basis of E.L.T. No. 12 - Males, find the probability that a life aged 30 will

(i) survive to age 40,


(ii) die before reaching age 50,
(iii) die in his 50th year of age, i.e. between ages 49 and 50,
(iv) die between his 40th birthday and his 50th birthday,
(v) die either between exact ages 35 and 45 or between exact ages 70 and 80.

1.3 A man aged 50 has just retired because of ill health. Up to exact age 58 he will be subject to
a constant force of mortality of 0.019803 p.a., after which his mortality will be that of E.L.T.
No. 12 - Males. Find the probability that he will

(i) die before age 55,


(ii) live to age 65,
(iii) die between ages 55 and 60.

1.4 For the first 5 years after arrival in a certain country, lives are subject to a constant force of
mortality of 0.005. Thereafter lives are subject to mortality according to English Life Table
No. 12- Males with an addition of 0.039221 to the force of mortality.

(i) A life aged exactly 30 has just arrived in the country.

(a) Show that the probability that the life will survive to age 35 is 0.97531.
(b) Find the probability that the life will survive to age 60.

(ii) What is the probability that a life aged exactly 33 who has been in the country for 3
years will die between ages 50 and 51?
(Assume that these lives will remain in the given country.)

1.5 For a certain animal population,


l0
lx = (x ≥ 0)
(1 + x)2
Calculate

(i) the complete expectation of life at birth,


(ii) the force of mortality at age 1 year,
(iii) the chance that a newly-born animal will die between ages 1 and 2 years.
1.8. EXERCISES 23

1.6 Suppose that t px µx+t is decreasing for 0 ≤ t ≤ 1. Show that qx < µx .

1.7 Suppose that Gompertz’ law, µx = Bcx , holds for all x ≥ α, c being greater than 1. Assume
that µα < log c.

(i) Give a formula for s(x).


(ii) Show that lx µx attains a maximum when µx = log c, and has no other stationary points.

1.8 (Difficult) Suppose that there is a “uniform distribution of deaths” from age x to age x + 1,
i.e. lx+t is linear in t for 0 ≤ t ≤ 1.
Show that, for all 0 ≤ s ≤ t ≤ 1,
t−s
t−s qx+s = qx
1 − sqx

1.9 (Difficult) Suppose that the “Balducci hypothesis” holds from age x to age x + 1, i.e.
1−t qx+t = (1 − t)qx for 0 ≤ t ≤ 1.

Show that, for all 0 ≤ s ≤ t ≤ 1,


t−s
t−s qx+s = qx
1 − (1 − t)qx

1.10 Suppose that, for some a > 0,


s(x) = (1 − x/w)a (0 ≤ x ≤ w)

Give simple formulae for

(i) µx ,

(ii) ex (Note finite limiting age),
(iii) 10 p70 ,
(iv) 40 |5 q35

1.11 A certain group of lives now aged 60 experience mortality according to a(55) males ultimate
with addition to the force of mortality. The addition is 0.0005 at age 60, increasing linearly to
0.0025 at age 80, at which level the addition remains constant.

Find the probability that a life aged exactly 60 dies within 20 years.
24 CHAPTER 1. NON-SELECT LIFE TABLES

Solutions

l65
1.1 (i) 40 p25 = = 0.71528
l25
l85 − l95
(ii) 60 |10 q25 = = 0.10021
l25

l40
(iii) 10 p30 = = 0.98452
l30

l65
(iv) 25 p40 = = 0.73025
l40

l50 − l60
(v) 10 q50 = = 0.12389
l50

l50 − l70
(vi) 20 q50 = = 0.39162
l50

l80 − l85
(vii) 20 |5 q60 = = 0.16173
l60

l65 − l70
(viii) 5 |5 q60 = = 0.17338
l60

In words,

(i) is “the probability that a life aged 25 will survive to age 65.”
(ii) is “the probability that a life aged 25 will survive to age 85 and die before age 95.”

l40
1.2 (i) 10 p30 = = 0.98452
l30
l30 − l50
(ii) 20 q30 = = 0.05437
l30

d49
(iii) 19 |q30 = = 0.00613
l30

l40 − l50
(iv) 10 |10 q30 = = 0.03889
l30

(v) 5 |10 q30 + 40 |10 q30 =

(l35 − l45 ) + (l70 − l80 )


= 0.35789
l30

1.3 Let us use an asterisk to denote the mortality table of the life concerned, while lx etc. refers
to E.L.T. 12-Males. Let k = 0.019803.

(i) 5 q50 = 1 − 5 p∗50 = 1 − exp(−5k) = 0.09427
1.9. SOLUTIONS 25

∗ l65
(ii) 15 p50 = 8 p∗50 7 p∗58 = exp(−8k) = 0.71188
l58

(iii) 5 |5 q50 = 5 p∗50 − 10 p∗50 = 5 p∗50 − 8 p∗50 2 p∗58

l60
= exp(−5k) − exp(−8k) = 0.08540
l58

µ Z 5 ¶
1.4 (i) (a) Prob. of survival for 5 years = exp − 0.005dt = 0.97531
0
ELT
l
(b) 0.97531 exp (−0.039221 × 25) 60
ELT = 0.3051
l35

(ii) Prob. = prob. of survival for 17 years - prob. of survival for 18 years
ELT
l50
= exp(−2 × 0.005) exp (−0.039221 × 15) ELT
l35
ELT
l51
− exp(−2 × 0.005) exp (−0.039221 × 16) ELT
l35

= 0.02377

Z ∞ Z ∞ · ¸∞
◦ lx −1
1.5 (i) e0 = dx = (1 + x)−2 dx = =1
0 l0 0 1+x 0
l0 2
(ii) µx = − x = , so µ1 = 1
lx 1+x

l1 − l2 1 1
(iii) 1|
q0 = = − = 0.13889
l0 4 9

1.6 Let f (t) = t qx − µx .t (0 ≤ t ≤ 1)


Z
d t
f 0 (t) = r px µx+r dr − µx = t px µx+t − µx < 0 for 0 < t < 1
dt 0

Now f (0) = 0, and f (t) is decreasing for 0 ≤ t ≤ 1, so f (1) < 0; therefore qx < µx .

1.7 (i) For x ≥ α, · Z t ¸


x+s
t px = exp − Bc ds
0
£ ¤
= exp −Bcx (ct − 1)/ log c

Therefore s(x) = x−α pα = exp [−B(cx − cα )/ log c].


26 CHAPTER 1. NON-SELECT LIFE TABLES

(ii) It follows that lx = k. exp [−Bcx / log c] for some k. Therefore


d
(lx µx ) =lx Bcx log c
dx
[−Bcx log c]
+ Bcx k exp [−Bcx / log c]
log c
=lx µx (log c − Bcx ) (∗)
=0 when µx = Bcx = log c

Let x0 be the unique point at which this occurs. Notice that (by equation (∗) above)
d
(lx µx ) > 0 for x < x0 , and is negative for x > x0 .
dx
Therefore x0 is a maximum point of lx µx .

1.8 Under U.D.D.,


lx+t = tlx+1 + (1 − t)lx (0 ≤ t ≤ 1)
and similarly for lx+s .

lx+s − lx+t (t − s)lx − (t − s)lx+1


Therefore = = (t − s)qx
lx lx

(t − s)qx lx 1
Therefore t−s qx+s = , since = .
1 − sqx lx+s 1 − sqx
1.9 µ ¶
lx+t lx+1 lx+t lx+1
t−s qx+s = 1− =1− − 1−
lx+s lx+s lx+s lx+t
= q
1−s x+s − (1 − q ) q
t−s x+s 1−t x+t

1−s qx+s − 1−t qx+t


Therefore t−s qx+s = (on rearranging).
1 − 1−t qx+t

Now apply “Balducci” to the R.H.S. of the equation. This gives the desired result.
d a
1.10 (i) µx = − [log s(x)] =
dx w−x
(ii)
Z w−x Z w

ex = t px dt = ly dy/lx
R0w x

x
(1 − y/w)a dy
=
(1 − x/w)a

Substitute z = 1 − y/w to obtain (calculus exercise!) the result that


◦ w−x
ex =
a+1
µ ¶a
s(80) w − 80
(iii) 10 p70 = =
s(70) w − 70
¡ ¢a ¡ ¢
80 a
s(75) − s(80) 1 − 75
w − 1− w
(iv) 40 |5 q35 = = ¡ ¢a
s(35) 1 − 35
w
1.9. SOLUTIONS 27

1.11 Addition to force of mortality = 0.0005 + 0.0001t (0 ≤ t ≤ 20)


½ Z 20 ¾
Prob. of survival for 20 years = exp − (µ60+t + 0.0005 + 0.0001t)dt
0

= 20 p60 exp(−0.003) on integration

= 0.4108
∴ Ans. = 0.5892
28 CHAPTER 1. NON-SELECT LIFE TABLES
Chapter 2

SELECT LIFE TABLES

2.1 What is selection?

In the previous chapter we considered mortality rates to depend only on age: life tables of this form
are sometimes called aggregate life tables. We now consider the situation when mortality rates (or
the force of mortality ) depend on two factors: (i) age, and (ii) the time (duration) since a certain
event, known as “selection”.

One important example of “selection” is the acceptance of a proposal for life assurance at normal
rates of premium: the mortality rates of lives who have been recently accepted for life assurance
at normal rates may be expected to be lower than those of the general population. After a certain
period the difference in mortality between those who have been accepted for life assurance and the
general population of the same age decreases, but it is not correct to say that the effect of selection
“wears off” entirely (since the general population contains some lives who would never have been
accepted for life assurance at normal premium rates). This point may be confirmed by a comparison
between the mortality rates of English Life Table No.12 - Males and A1967-70 ultimate, which we
will discuss below. (The process by which life offices decide whether to accept lives for assurance,
and on what terms, is called “underwriting”.)

Another form of selection is the “self-selection” exercised by those who buy annuities: it may nor-
mally be assumed that such lives are in good health (for their age), for the purchase of an annuity
is otherwise likely to be a poor investment.

In these examples, the mortality of those selected is lower than that of the general population,
particularly in the period soon after selection. In “reverse selection”, mortality rates are higher than
those of the general population (or some other reference group). An example of reverse selection
is early retirement due to ill-health. After a certain period from the date of ill-health retirement,
the mortality of these lives may be expected to become closer to that of lives who retired in normal
health, or who are not yet retired.

29
30 CHAPTER 2. SELECT LIFE TABLES

2.2 Construction of select tables


Let us, for definiteness, consider “selection” to be the acceptance of a proposer for life assurance at
normal rates by a life office.
Let
x = age at entry to assurance, i.e. at the date of selection
t = duration (in years) since the date of selection

The current (or attained) age of such a life is

y =x+t
We write “[x] + t” as a shorthand notation for a “life aged x at selection and duration t years since
selection”. Thus for example,

h q[x]+t = P r{“[x] + t” will die within h years}


= P r{a life aged x + t, who has selected t years ago,
will die within h years} (2.2.1)
µ[x]+t = the force of mortality of a life aged x + t who has selected t years ago
h q[x]+t
= lim (2.2.2)
h→0+ h

The select period. We assume that there is a period, s years, such that the mortality of those
selected at least s years ago depends only on the attained age, x + t. That is,
q[x]+t = qx+t (t ≥ s) (2.2.3)
µ[x]+t = µx+t (t ≥ s) (2.2.4)
where qx+t and µx+t refer to those who were selected at least s years ago: such people are called
ultimate lives.

For each fixed entry age x, we may define the family of random variables.
T ([x] + t) = the future lifetime of “[x] + t” (t ≥ 0)
Regarding x as fixed, we may construct a “life table” for those selected at age x by methods similar
to those of chapter 1 (with t in place of x, and α = 0). We have, for example,

h q[x]+t = P r{T ([x] + t) ≤ h}


l[x]+t+h
=1−
l[x]+t
where l[x]+t is the expected number number of survivors at age x + t of l[x] lives selected at age x.
But instead of fixing each radix l[x] arbitrarily, we choose them to be such that l[x]+t depends only
on x + t when t ≥ s. More precisely, we first construct a life table for the ultimate lives by the
methods given in chapter 1; the function l[x]+t is then constructed to be such that
l[x]+t = lx+t (t ≥ s) (2.2.5)
This ensures that relationships such as
l[x]+t+h
h p[x]+t = (2.2.6)
l[x]+t
2.3. THE CONSTRUCTION OF A1967-70. 31

(which are found by replacing “x” by “[x] + t” in the formulae of chapter 1) are true. We may omit
the square brackets enclosing x in expressions such as h p[x]+t and l[x]+t+h if t ≥ s or t + h ≥ s
respectively.

We now give formulae for l[x]+t for 0 ≤ t ≤ s. If q[x]+t is given for t = 0, 1, 2, . . . , s − 1 , we proceed
recursively, using the formulae
l[x]+t+1
= 1 − q[x]+t (t = s − 1, , s − 2, . . . , 0) (2.2.7)
l[x]+t
That is
l[x]+s−1 = l[x]+s /(1 − q[x]+s−1 )
l[x]+s−2 = l[x]+s−1 /(1 − q[x]+s−2 )

······························

l[x] = l[x]+1 /(1 − q[x] )


(2.2.8)
If µ[x]+t is given for 0 ≤ t ≤ s , we use the formula
l
l[x]+t = ¡ R x+s
s ¢ (t < s) (2.2.9)
exp − t µ[x]+r dr

2.3 The construction of A1967-70.


We illustrate the construction of select tables by reference to A1967-70, which has a select period of
2 years. This table refers to the mortality of male assured lives in the U.K. during the period 1967 to
1970, and is based on data collected by the Continuous Mortality Investigation Bureau (C.M.I.B.).
Mortality rates were extended to young ages (including “ultimate” lives aged under 2) for which no
data existed.

The “building blocks” of the table were the mortality rates:


qy (y = 0, 1, 2, . . . )
q[x]+t (x = 0, 1, 2, . . . ; t = 0, 1)
The stages of construction of ly and l[x]+t were as follows:
1. l0 was fixed arbitrarily at 34,489.
2. l1 , l2 , l3 , . . . , were computed as shown (working downwards in the third column of Table
2.3.1).
3. By working to the left from l2 , l[0]+1 and l[0] were found; similarly for l[1]+1 and l[1] , etc.

select select
duration 0 duration 1 ultimate
34, 489 = l0
34, 489(1 − q0 ) = 34, 463.8 = l1
l
[0]+1
l[0] = 1−q [0]
← l[0]+1 = 1−ql[0]+1
2
← 34, 463.8(1 − q1 ) = 34, 440.4 = l2
··············· ··············· ← 34, 440.4(1 − q2 ) = 34, 418.7 = l3
..
.
32 CHAPTER 2. SELECT LIFE TABLES

Table 2.3.1

2.4 Some formulae for the force of mortality.


We assume that l[x]+t and µ[x]+t are continuous in t (t ≥ 0).
h q[x]+t
µ[x]+t = lim
h→0+ h
l[x]+t − l[x]+t+h
= lim
h→0+ hl[x]+t
d
− l[x]+t
= dt (t ≥ 0) (2.4.1)
l[x]+t
(the derivative is, strictly speaking, a L.H. derivative, but arguments similar to those given in chapter
1 show that the l[x]+t is differentiable in t ).
We may thus write
µ[x]+t = f 0 (t) (t ≥ 0)
where f (t) = − log(l[x]+t ).
We now consider how best to estimate µ[x]+t for t = 0, 1, 2, . . . , s − 1 from a table of l[x]+t (t =
0, , 1, 2, . . . , s). To estimate µ[x] = µ[x]+0 , we may use the differentiated form of Newton’s forward
difference formula, which gives
1
f 0 (0) + ∆f (0) − ∆2 f (0) (2.4.2)
2
where
∆f (0) = f (1) − f (0)
∆2 f (0) = f (2) − 2f (1) + f (0)

To estimate µ[x]+t , where t = 1, 2, . . . , s − 1 , we may use the formula

f (t + 1) − f (t − 1)
f 0 (t) +
2
to obtain
1
µ[x]+t + − log(l[x]+t+1 /l[x]+t−1 ) (2.4.3)
2

2.5 Select tables used in examinations.


Formulae and Tables for Actuarial Examinations give
1. A1967-70 (select period 2 years) and
2. a(55) males and females.
Note that a(55) consists of 2 tables, a(55) males and a(55) females. In each case the select period is
1 year, and functions are given only for ages 60 and over. These tables were constructed on the basis
of C.M.I.B. data and were intended to be suitable for those purchasing annuities in about 1955, an
allowance being made for improvements in mortality .
One should be careful not to look up “select” values (such as q[x] )when ultimate values (such as qx )
are required, or vice versa. In the case of a(55), one should be careful not to look up the “males”
table when the “females” table is required, and vice versa.
2.6. EXERCISES 33

Exercises

2.1 A mortality table has a select period of three years.


(i) Find expressions in terms of the life table functions l[x]+t and ly for
(a) q[50]
(b) 2 p[50]
(c) 2 | q[50]
(d) 2 |3 q[50]+1

(ii) Calculate 3 p53 given that:

q[50] = 0.01601, 2 q[50] = 0.96411,


2 |q[50] = 0.02410, 2|3 q[50]+1 = 0.09272

2.2 In its premium rate basis, an office assumes a 3-year select period. Functions on this table are
indicated by an asterisk. The table is such that:
∗ ∗ ∗ ∗
q[x+7] = q[x+3]+1 = q[x]+2 = qx+1

and ultimate mortality follows A1967-70 ultimate. Assuming further that ly∗ = ly on A1967-70

ultimate, calculate l[45]+t for t = 0, 1 and 2.

2.3 A certain life table has a select period of 1 year. At each integer age x , the select rate mortality
is 50% of the ultimate rate. Calculate e[60] , given that e60 = 17.5424 and q60 = 0.0142.

2.4 Explain briefly the concept of selection in relation to mortality tables. Define the term “select
period” and, using the A1967-70 table as an example, explain how a select table differs in
construction from an aggregate table such as English Life Table No. 12 - Males.
34 CHAPTER 2. SELECT LIFE TABLES

Solutions
l[50] − l[50]+1
2.1 (i) (a) q[50] = = .01601 (A)
l[50]
l[50]+2
(b) 2 p[50] = = .96411 (B)
l[50]

l[50]+2 − l53
(c) 2| q[50] = = .02410 (C)
l[50]

l53 − l56
(d) 2|3 q[50]+1 = = .09272 (D)
l[50]+1

(ii) Put l50 = 100, 000 (say); then


l[50] = 98, 399 from (A)
l[50]+2 = 96, 411 from (B)
l53 = 94, 001 from (C)
l56 = 84, 877 from (D)
l56
Then 3 p53 = l53 = .90294.
2.2
∗ l48 l48
l[45]+2 = ∗ = = 33, 033
1 − q[45]+2 1 − q46
∗ l[45]+2 33, 033
l[45]+1 = ∗ = = 33, 101
1 − q[45]+1 1 − q43


l[45]+1 33, 101
l[45] = ∗ = = 33, 144
1 − q[45] 1 − q39

2.3
l61 + l62 + . . .
e[60] =
l[60]
l60 l61 + l62 + . . .
= ·
l[60] l60
l60
= e60
l[60]

But q[60] = 0.5q60 , which implies that


l61
l[60] =
1 − 12 q60
l60 l60 (1− 12 q60 ) (1− 1 q
2 60 )
Therefore l[60] = l61 = 1−q 60
= 1.007202 so
e[60] = 1.007202 × 17.542 = 17.668

2.4 Mortality rates depend on the age x at “selection” [e.g. entry to assurance, purchase of an
annuity (self selection)] and duration t years since selection. Thus q[x]+t = Pr{ a life aged x + t
2.7. SOLUTIONS 35

who was selected t years ago will die within a year }, etc. The select period, s years, is such
that
q[x]+t = qx+t for t ≥ s
i.e. for lives who were selected at least s years ago (these being called “ultimate” lives) mortality
depends only on the attained aged. The l[x]+t function is constructed to be such that
l[x]+t = lx+t (t ≥ s)

In A1967-70, s = 2. In a non-select or aggregate life table, such as E.L.T. No. 12 - Males,


mortality depends on one variable only (age) and there is no “selection”.
Note. It is not correct to say that the effects of selection “wear off” entirely (and that the
mortality of ultimate lives is the same as that of the general population). If this were true,
A1967-70 ultimate would resemble ELT 12 - Males, and it does not. The general population
includes some lives who would never have been acceptable for assurance cover (at normal
premium rates).
36 CHAPTER 2. SELECT LIFE TABLES
Chapter 3

ASSURANCES

3.1 A general introduction


A life assurance (or life insurance) policy is a contract which promises to pay a specified sum,
S say, on the death of a given life (the life assured) at any future time, or between certain specified
dates. The death benefit S is called the sum assured; it may be level (constant) or it may increase
or decrease in a manner specified in the contract. Policies under which payments depend on the
death or survival of more than one life may also be issued.

In the case of with profits policies, the sum assured may be increased by additions, called bonuses,
which depend on the experience of the office and its bonus distribution policy. With profits policies
may be found in traditional or unitised form (see later discussion). One also encounters unit-
linked policies, in which the benefits are directly linked to the performance of certain assets, e.g. a
certain portfolio of equities. If the benefits (and premiums) are completely specified in money terms
in the contract, the policy is said to be without profits (or non-profit). Policies may also provide
endowment benefits, i.e. sums payable on the survival of the life (or lives) assured until a certain
date (or dates).

In this chapter we shall consider contracts issued on the life of one person. The benefits and expenses
are paid for by premiums, which we shall discuss later.

3.2 Whole life assurances


Suppose that, when calculating the value of benefits, the life office assumes that its funds will earn
interest at a constant rate, i per annum. The corresponding force of interest per annum is

δ = log(1 + i)

and the present value of 1 due at time t years is

v t = (1 + i)−t = e−δt (3.2.1)

Now consider a life aged x, who is subject to a certain non-select mortality table, and let T be the
future lifetime of (x). A whole life assurance is a policy providing a certain sum assured, S, on
the death of (x) at any future date. The present value of this benefit (assumed for the moment to
be payable immediately on death) is
Z = g(T ) = Sv T (3.2.2)

37
38 CHAPTER 3. ASSURANCES

The mean of this variable, E(Z), is called the mean (or expected) present value (M.P.V.) of
the whole life assurance benefit, and is
Z ∞
E(Z) = g(t)f (t) dt (3.2.3)
0

where f (t) is the p.d.f. of T (for t > 0); hence


Z ∞
E(Z) = S v t t px µx+t dt (3.2.4)
0

When S = 1 , we write
Z ∞
E(Z) = v t t px µx+t dt = Āx (3.2.5)
0

where A stands for “assurance” and the bar indicates that the sum assured is payable immediately
on the death of (x). The M.P.V. of the sum of £S payable immediately on the death of (x) is
therefore
S Āx

Example 3.2.1. Find the M.P.V. of a whole life assurance of £10, 000 , payable immediately on
the death of (40), according to E.L.T. No. 12-Males with interest at 4% p.a.

Solution.

M.P.V. = 10, 000 Ā40 on E.L.T. No. 12-Males, 4% interest


= 10, 000 × 0.31981 from Tables
= £3, 198.10

Select tables
For a select life (i.e. a person who has just been selected) aged x, we replace x by [x], and for an
“[x] + t” life (i.e. a life now aged x + t who was selected t years ago) we replace x by [x] + t. Thus,
letting T = T ([x]) denote the future lifetime of “[x]”, we write

Ā[x] = M.P.V. of v T
Z ∞
= v t t p[x] µ[x]+t dt (3.2.6)
0

If T = T ([x] + t) = future lifetime of “[x] + t” , we have

Ā[x]+t = M.P.V. of v T
Z ∞
= v r r p[x]+t µ[x]+t+r dr (3.2.7)
0
3.3. COMMUTATION FUNCTIONS 39

3.3 Commutation functions


Functions such as Āx may easily be evaluated by numerical integration, and are often tabulated
directly at various rates of interest. (In fact, a modern computer can easily compute Āx directly at
any rate of interest required, e.g. 4.032% ).

Commutation functions are numerical devices (developed by Griffith Davies and others) which allow
the calculation of certain common assurance (and annuity) values at a specified rate of interest from
a small number of columns, labelled Dx , M̄x etc. In view of their importance in the Tables for
Actuarial Examinations we shall look at them in detail.

At this point we consider only whole life assurances payable immediately on death. Define
Dx = v x l x (3.3.1)
Z 1
C̄x = v x+t lx+t µx+t dt (3.3.2)
0

X
M̄x = C̄x+t (3.3.3)
t=0

Now
R∞
v x+t lx+t µx+t dt
0
Āx =
v x lx
C̄x + C̄x+1 + C̄x+2 + . . .
=
Dx
Z r+1 Z 1
x+t
(on writing v lx+t µx+t dt = v x+r+u lx+r+u µx+r+u du
r 0
by the change of variable u = t − r)
so that P∞
C̄x+t M̄x t=0
Āx =
= (3.3.4)
Dx Dx
We shall consider commutation functions again in relation to temporary and deferred assurances,
etc.

3.4 The variance of the present value of benefits


We recall that Z = g(T ) = Sv T is the present value of S due immediately on the death of (x), and
that the mean of Z is S Āx . What is the variance of Z?
Answer. We have
2
var(Z) = E(Z 2 ) − [E(Z)]
Z ∞
¡ ¢2
= S 2 v 2t t px µx+t dt − S Āx
0
·Z ∞ ¸
2 ∗ t
¡ ¢2
=S (v ) t px µx+t dt − Āx
0
(where v ∗ = v 2 )
h ∗ ¡ ¢ i
2
= S 2 Āx − Āx (3.4.1)
40 CHAPTER 3. ASSURANCES

where the rate of interest i∗ is such that


1 1
v∗ = ∗
= v2 =
1+i (1 + i)2
i.e. i∗ = 2i + i2 (3.4.2)

Note that the force of interest, δ ∗ , corresponding to i∗ is such that



v ∗ = e−δ = v 2 = e−2δ
i.e. δ ∗ = 2δ (3.4.3)

The standard deviation of Z is, of course, found by taking the square root of var(Z). If we consider
a block of n identical whole life policies on independent lives aged x, the total present value of the
assurance benefits is

Z = Z1 + Z2 + . . . + Zn

where, for i = 1, 2, . . . , n,

Zi = present value of benefit under ith policy


(i)
= Sv T

with T (i) = future lifetime of ith life. Since the variables {T (i) } are assumed to be independent, the
variance of Z is
n
X
Var(Z) = Var(Zi )
i=1
Xn h ∗ ¡ ¢ i
2
= S 2 Āx − Āx
i=1
h ∗ ¡ ¢ i
2
= nS 2 Āx − Āx (3.4.4)

and so the standard deviation of the total present value is


rh
√ ¡ ¢2 i
S n Ā∗x − Āx (3.4.5)

We remark that the mean present value of a group of policies is the sum of their separate M.P.V’s
even if the lives are not independent. Formulae 3.4.4 and 3.4.5 may be generalised to cover the case
when the sums assured and/or ages of the lives are different (see exercise 3.3).

Jensen’s inequality
Let g(t) be convex, which is the case of g 00 (t) ≥ 0 for all t > 0. Jensen’s inequality shows that

E[g(T )] ≥ g[E(T )] (3.4.6)

An application. Let g(t) = e−δt , where δ > 0. We have

g 00 (t) = δ 2 e−δt ≥ 0 for all t > 0


3.5. ASSURANCES PAYABLE AT THE END OF THE YEAR OF DEATH. 41

By Jensen’s inequality,

E(v T ) ≥ v E(T )

but

E(T ) = ex

and

E(v T ) = Āx

so we have shown that



Āx ≥ v ex (3.4.7)

That is, at any positive rate of interest, v ex (which is the present value of £1 on the assumption
that (x) dies when he reaches his expectation of life) is less than or equal to the mean present value,
Āx .

Example 3.4.1. Verify formula 3.4.7 for x = 50 on the basis of E.L.T. No.12-Males, 4% p.a. interest.


Solution. e50 = 22.68

so v e50 = 0.41085

which is less than Ā50 = 0.44429

3.5 Assurances payable at the end of the year of death.


Suppose that the death benefit S is payable at the end of the year of death (years being measured
from the date of issue of the policy). It is now convenient to use the random variable

K = [T ] = the integer part of T

which has discrete probabilities

P r{K = k} = k |qx (k = 0, 1, 2, . . . )

The present value of the assurance is now

Z = g(K) = Sv K+1
which has mean

X
E[g(K)] = Sv k+1 k |qx (3.5.1)
k=0
= SAx
42 CHAPTER 3. ASSURANCES

where

X
Ax = v k+1 k |qx (3.5.2)
k=0

We define the commutation functions

Cy = v y+1 dy (3.5.3)
X∞
My = Cy+k (3.5.4)
k=0

and find that


P∞
k=0 v x+k+1 dx+k
Ax =
Dx
Mx
= (3.5.5)
Dx

Example 3.5.1. Show that the variance of Z (= Sv K+1 ) is


h ∗ i
2
S 2 Ax − (Ax ) (3.5.6)


where indicates at the rate of interest 2i + i2 .

Solution.

Var(Z) =E(Z 2 ) − [E(Z)]2


"∞ #
X
=S 2 (v ∗ )k+1 k |qx − (Ax )2
k=0
£ ¤
=S 2 A∗x − (Ax )2

The relationship between Ax and Āx .


Assuming that deaths occur on average mid-way through each policy year (i.e. the year between two
consecutive policy anniversaries), benefits payable at the end of the year of death will be received,
on average, 6 months later than those immediately payable on death. We thus have the approximate
relationship
1
Āx + (1 + i) 2 Ax (3.5.7)

This may be confirmed by establishing the following approximate relationship


1
C̄x + (1 + i) 2 Cx (3.5.8)

which follows from the mean value theorem for integrals. Formulae (3.3.3) and (3.5.4) now show
that
1
M̄x + (1 + i) 2 Mx (3.5.9)
3.5. ASSURANCES PAYABLE AT THE END OF THE YEAR OF DEATH. 43

and hence
1
M̄x (1 + i) 2 Mx 1
Āx = + = (1 + i) 2 Ax (3.5.10)
Dx Dx

Theorem 3.5.1. If there is a uniform distribution of deaths between the ages x + k and x + k + 1
(for k = 0, 1, 2, . . . ), we have
i
Āx = Ax (3.5.11)
δ
£P∞ ¤± P∞
Proof. It is sufficient (since Āx = k=0 C̄x+k Dx and Ax = [ k=0 Cx+k ]/ Dx ) to show that

i
C̄y = Cy for y = x + k (k = 0, 1, 2, . . . )
δ
Now, since we have U.D. of D. between ages y and y + 1,

ly+t µy+t = dy (0 ≤ t < 1)

Therefore
Z 1
C̄y = v y+t ly+t µy+t dt
0
Z 1
y
= dy v v t dt
µ0 ¶
y 1−v
= dy v
δ
µ ¶
i
= dy v y+1
δ
i
= Cy
δ

Note. By the mathematics of finance, if i is small we have


i 1
+1+ i
δ 2

and

1 1
(1 + i) 2 + 1 + i
2
so we sometimes find the following approximations (which are suitable only when i is small):

1
Āx = (1 + i)Ax 

2 


1
C̄x = (1 + i)Cx (3.5.12)
2 



1 
M̄x = (1 + i)Mx 
2
44 CHAPTER 3. ASSURANCES

3.6 Assurances payable at the end of the 1/m of a year of


death.
Suppose that £1 is payable at the end of the 1/m year (measured from the issue date) following the
death of (x); for example, if m = 12 , the sum assured is payable at the end of the month of death.
The position is illustrated in Figure 3.6.1 below, in which ∗ indicates the time when (x) dies.

death occurs here payment is made here


JJ
^ À­
| | | | |­ - time
(years)
1 2 . . . . . . . . . r (r+1)
0 m m m m

Figure 3.6.1:

The mean present value of this benefit is



X r+1
A(m)
x = v m r 1
m|m
qx (3.6.1)
r=0

As m → ∞ , we find that (as one would expect from general reasoning)


A(m)
x −→ Āx (3.6.2)
Corresponding to formula 3.5.7 , we have the useful approximation
Ax + v 1/(2m) Āx (3.6.3)
where the factor of v 1/(2m) allows for interest for 1/(2m) year, which is the average “delay” in
(m)
receiving death benefit. The function Ax is not often used in practice.

3.7 Temporary and deferred assurances


A term (or temporary) assurance contract pays a sum assured of £S, say, on the death of (x)
within the term of the policy, which we usually write as n years. If the benefit is payable immediately
on death, the present value (as a random variable) is
½
Sv T if T ≤ n
Z = g(T ) =
0 if T > n

The mean of this variable is written as S Āx1:n , the “1” indicating that the status (x) must fail
before the status n (a fixed period of n years) for a payment to be due (and that the payment is
due when this event occurs). Thus
Z n
Āx1:n = v t t px µx+t dt (3.7.1)
0
This may be evaluated by numerical integration or by commutation functions, i.e.
n−1
X
C̄x+t
t=0 M̄x − M̄x+n
Āx1:n = = (3.7.2)
Dx Dx
3.7. TEMPORARY AND DEFERRED ASSURANCES 45

(provided that n is an integer)


If the sum assured is payable at the end of year of death (if this occurs within the n year term), we
have the present value ½
Sv K+1 if K < n
Z = g(K) =
0 if K ≥ n
The M.P.V. of this benefit is written as S Ax1:n , and we have

n−1
X
Ax1:n = v t+1 t |qx (3.7.3)
t=0

(It is assumed that n is an integer.) In terms of commutation functions,


n−1
X
Cx+t
t=0 Mx − Mx+n
Ax1:n = = (3.7.4)
Dx Dx

In view of the relationship between Cx and C̄x , we have the approximations

1 i 1
Āx1:n ; (1 + i) 2 Ax1:n ; A (3.7.5)
δ x :n
A deferred assurance provides a sum of £S (say) on the death of (x) if this occurs after a certain
period, called the deferred period (or, more correctly, the period of deferment), which we usually
write as n years. If the benefit is payable immediately on death of (x) after the deferred period has
elapsed, the present value is
½
Sv T if T > n
Z = g(T ) =
0 if T < n
if the sum assured is payable immediately on death, or
½
Sv K+1 if K ≥ n
Z = g(K) =
0 if K < n
if the sum assured is payable at the end of year of death.
The corresponding M.P.V.’s are written as

n | Āx = v n n px Āx+n (3.7.6)


and
n |Ax = v n n px Ax+n (3.7.7)
We have the formulae
Z ∞
n | Āx = v t t px µx+t dt (3.7.8)
n

X
C̄x+t
t=n
=
Dx
M̄x+n
= (3.7.9)
Dx
46 CHAPTER 3. ASSURANCES

and

X
n |Ax = v t+1 t| qx (3.7.10)
t=n
X∞
Cx+t
t=n
=
Dx
Mx+n
= (3.7.11)
Dx

In view of the relationship between C̄x and Cx , we have the approximations:


i1
n | Āx ; (1 + i) 2 n |Ax ;
n |Ax (3.7.12)
δ
The evaluation of term and deferred assurance functions is often simplified by the observation that a
whole life policy may be thought of as a combination of a term assurance and a deferred assurance,
so that

Ax = Ax1:n + n |Ax (3.7.13)


and Āx = Āx1:n + n | Āx (3.7.14)

3.8 Pure endowments and endowment assurances


We now consider a pure endowment of £1 of term n years, i.e. a policy providing the sum of £1
at time n years if (x) is then alive.
The present value of this benefit is
½ n
v if T ≥ n
Z=
0 if T < n

and hence the M.P.V. is

v n n px
(since P r{T ≥ n} = n px ). This M.P.V. is written as

n Ex or Ax:n1 or Āx:n1
i.e.
n Ex = Ax:n1 = Āx:n1 = v n n px (3.8.1)
In terms of commutation functions we have the useful result that
Dx+n
n Ex = (3.8.2)
Dx
Returning to the formulae (3.7.6) and (3.7.7) , we see that the M.P.V. of a deferred assurance may be
written as the product of n Ex and Āx+n or Ax+n . That is, the value is obtained by multiplying the
M.P.V. of the assurance at the “vesting date” (when the life attains age x + n) by a pure endowment
factor.

We finally consider an endowment assurance of term n years with sum assured £S. In this
policy, the sum assured is payable if (x) dies within n years or on maturity of the contract at time
3.8. PURE ENDOWMENTS AND ENDOWMENT ASSURANCES 47

n years, whichever occurs first. It follows that an endowment assurance is a combination of a term
assurance and a pure endowment (of the same term). If we assume that the death benefit is payable
immediately on death, if death occurs within n years, the present value of the benefits is
½
Sv T if T < n
Z = g(T ) =
Sv n if T ≥ n
The mean of Z is written S Āx:n .Thus

Āx:n = Āx1:n + Āx:n1 (3.8.3)


= Āx1:n + n Ex
M̄x − M̄x+n + Dx+n
= (3.8.4)
Dx
Similarly, if the death benefit is payable at the end of the year of death, the present value is
½
Sv K+1 if K < n
Z = g(K) =
Sv n if K ≥ n
and the M.P.V. is written as S Ax:n . Thus

Ax:n = Ax1:n + Ax:n1 (3.8.5)


= Ax1:n + n Ex
Mx − Mx+n + Dx+n
= (3.8.6)
Dx

In view of the fact that the term Dx+n occurs in formulae (3.8.4) and (3.8.6), it is not correct to
write

1 A COMMON
Āx:n ; (1 + i) 2 Ax:n
MISTAKE

Correct relationships are (for example):


1
Āx:n ; (1 + i) 2 Ax1:n + Ax:n1 (3.8.7)

and
1
(1 + i) 2 (Mx − Mx+n ) + Dx+n
Āx:n ; (3.8.8)
Dx

Select tables The adjustments to the formulae when a select table is used are straightforward: one
need only replace ‘x + t’ by ‘[x] + t’ (or ‘[x]’ if the life assured has just been selected) and dispense
with [ ] if the duration since selection is at least equal to the select period, s years, and in compound
interest functions.
For example,
C[x]+t = v x+t+1 d[x]+t

Example 3.8.1. On the basis of A1967-70 select mortality and 4% p.a. interest, calculate the mean
present value of each of the following assurance benefits for a life aged 30:
48 CHAPTER 3. ASSURANCES

(i) A whole life assurance for £10, 000, payable immediately on death;

(ii) A 20-year term assurance for £50, 000, payable at the end of the year of death;

(iii) A 20-year endowment assurance for £50, 000, with the death benefit payable immediately on
death;

(iv) A deferred temporary assurance for £100, 000, payable at the end of the year of death, if death
occurs between ages 40 and 50 exactly.
1
[Use the factor (1 + i) 2 for accelerating payments from end of year of death to the moment of death.]

Solution.

1
(i) 10, 000Ā[30] = 10, 000(1.04) 2 A[30]

= £1, 934.87

(M[30] − M50 )
(ii) 50, 000 = £1, 012.90
D[30]

1 D50
(iii) (1.04) 2 × 1, 012.90 + 50, 000 = £23, 070.56
D[30]

(M40 − M50 )
(iv) 100, 000 = £1, 360.91
D[30]

3.9 Varying assurances


Suppose that a contract provides the sum of β(t) immediately on the death of (x) at time t years.
The present value of this benefit is

Z = g(T ) = β(T ) v T (T > 0)

and hence the M.P.V. is


Z ∞
v t β(t)t px µx+t dt (3.9.1)
0

If β(t) = t for all t > 0 , the M.P.V. is written as (I¯Ā)x , i.e.


Z ∞
(I¯Ā)x = tv t t px µx+t dt (3.9.2)
0
3.9. VARYING ASSURANCES 49

If β(t) = [t] + 1 , where [t] indicates the integer part of t, the M.P.V. is written as (I Ā)x , and it may
easily be shown that

X
(t + 1)C̄x+t
t=0
(I Ā)x =
Dx
½ ¾Á
£ ¤ £ ¤
= C̄x + C̄x+1 + · · · + C̄x+1 + C̄x+2 + · · · + · · · Dx

M̄x + M̄x+1 + · · ·
=
Dx
R̄x
= (3.9.3)
Dx

where

X
R̄x = M̄x+k (3.9.4)
k=0

In view of the fact that the average benefit is 12 less when β(t) = t than in the present case, we have
the approximation
1
(I¯Ā)x ; (I Ā)x − Āx (3.9.5)
2
We now consider the case when a death benefit of β(k) in year k is payable at the end of the year
of death. The present value is

Z = g(K) = β(K + 1) v K+1 (K = 0, 1, 2, . . .)

and hence the M.P.V. is



X
β(k + 1)v k+1 k |qx (3.9.6)
k=0

When β(k) = k for k = 1, 2, . . . , the M.P.V. is written (IA)x , so we have



X
(IA)x = (k + 1)v k+1 k |qx (3.9.7)
k=0

In terms of commutation functions we have



X
(k + 1)Cx+k
k=0
(IA)x =
Dx
Mx + Mx+1 + · · ·
=
Dx
Rx
= (3.9.8)
Dx

where

X
Rx = Mx+k (3.9.9)
k=0
50 CHAPTER 3. ASSURANCES

1
In view of the approximations C̄x ; (1 + i) 2 Cx , etc., we find that
i 1
R̄x ; Rx ; (1 + i) 2 Rx (3.9.10)
δ

and

i 1
(I Ā)x ; (IA)x ; (1 + i) 2 (IA)x (3.9.11)
δ
We may also define similar symbols for increasing temporary and endowment policies, e.g.
n−1
X
(IA)x1:n = (k + 1)v k+1 k |qx
k=0
n−1
X
(k + 1)Cx+k
k=0
=
Dx
½
= [Cx + 2Cx+1 + · · · ] − [Cx+n + 2Cx+n+1 + · · · ]
¾Á
− n [Cx+n + Cx+n+1 + · · · ] Dx

Rx − Rx+n − nMx+n
= (3.9.12)
Dx

and

(IA)x:n = (IA)x1:n + n · n Ex (3.9.13)

3.10 Valuing the benefits under with profits policies


So far we have studied without profits contracts, for which the premiums and benefits are com-
pletely fixed (in money terms). In with profits contracts, the premiums are higher but the policy-
holder has the right to a share of the profits of the office. In the U.K., the usual system for sharing
profits is by means of additions to the basic benefit (sum assured, pension, etc.) in the form of
bonuses. These may be either reversionary bonuses, which are usually added annually and are not
subsequently reduced, or terminal bonuses, which are paid only on death or maturity (and possibly
on surrender) and are not guaranteed to continue.

Nowadays one may also encounter unitised with profit policies. These are not the same as
unit-linked policies: the maturity proceeds of the latter are linked to the performance of a unit
trust or internal fund of the life office, while for U.W.P. policies the office credits each policy with
internal units which are subject to certain guarantees.

In this book we shall study reversionary bonuses only. There are 3 systems of adding bonuses
in common use:
(1) the simple bonus system;
(2) the compound bonus system, and
(3) the two-tier (or supercompound) bonus system.
3.10. VALUING THE BENEFITS UNDER WITH PROFITS POLICIES 51

(1) Simple bonuses. In this system, bonuses are calculated only on the basic sum assured (B.S.A.),
which we shall denote by S. Consider a whole life with profits policy issued t years ago to a life
then aged x. Suppose that bonuses vest annually in advance, let the bonuses already vested be B,
and suppose that a bonus is about to be added. the formula used to calculate new bonus addition
is
new bonus = S × rate of bonus p.a. (3.10.1)
To value the benefits we must estimate the rate of simple bonus p.a. (applying now and for the
forseeable future) as b (say). The total sum assured is thus
S + B + bS in year 1 from the present time,
S + B + 2bS in year 2 from the present time,
and so on, giving a total benefit in year k of
S + B + kbS
(3.10.2)
If the benefits are payable at the end of the year of death, the M.P.V. of the benefits is

X
[S + B + (k + 1)bS] v k+1 k |qx+t
k=0
=(S + B)Ax+t + bS(IA)x+t (3.10.3)

Notes. (1) At the inception of the policy, we have t = 0 and B = 0.


(2) If the benefits are payable immediately on death, the M.P.V. is
(S + B)Āx+t + bS(I Ā)x+t (3.10.4)
(3) In the case of an n-year endowment assurance with profits, the benefits on maturity
are normally equal to those on death in the final year. This leads to the following
formulae for the value of benefits:

at end of year
: (S + B)Ax+t:n−t + bS(IA)x+t:n−t
of death (3.10.5)
immediately
: (S + B)Āx+t:n−t + bS(I Ā)x+t:n−t
on death (3.10.6)

(2) Compound bonuses In this system, the formula for bonus additions is
 
basic sum
 assured 
 
new bonus =  plus bonuses  × rate of bonus p.a. (3.10.7)
 
already
vesting
Assuming a bonus rate of b p.a. applies now and for the forseeable future, the benefits will be
(S + B)(1 + b) in year 1
2
(S + B)(1 + b) in year 2
and so on, giving
(S + B)(1 + b)k in year k
(3.10.8)
52 CHAPTER 3. ASSURANCES

If the benefits are payable at the end of the year of death, their M.P.V. is

X
(S + B)(1 + b)k+1 v k+1 k |qx+t
k=0
∞ µ
X ¶k+1
1+b
=(S + B) k |qx+t
1+i
k=0
=(S + B)A∗x+t (3.10.9)

where A∗ is at rate of interest


i−b
(3.10.10)
1+b
If the benefits are payable immediately on death, their M.P.V. is approximately
1
(S + B)(1 + i) 2 A∗x+t (3.10.11)
¡ ¢
NOT (S + B)A∗x+t
For endowment assurances, the corresponding formulae are

(S + B)A∗x+t:n−t (3.10.12)

and h i
1
(S + B) (1 + i) 2 A∗ x+t
1
:n−t + A ∗ 1
x+t:n−t (3.10.13)

Notes. (1) In formula (3.10.13), we observe that the pure endowment part is as in formula
(3.10.12): only the term assurance part is “accelerated”
(2) At inception, we have t = 0 and B = 0.

(3) Supercompound (two-tier) bonuses The formula is


à !
bonus rate p.a.
new bonus =(B.S.A.) in respect of
B.S.A.
 
µ ¶ bonus rate (3.10.14)
bonuses already p.a. in respect
+  
vesting of bonuses
already vesting

Let us assume that the bonus rate p.a. in respect of the B.S.A. and previous bonus additions are
a and b respectively (now and for the forseeable future).

Let B(k) = total bonus in year k (from the present time), and let us define B(0) = B. We have

B(1) = B(1 + b) + aS
B(2) = B(1 + b)2 + aS[1 + (1 + b)]

and so on, giving the conjecture

B(k + 1) = B(1 + b)k+1 + aS × sk+1 b (3.10.15)


3.10. VALUING THE BENEFITS UNDER WITH PROFITS POLICIES 53

Proof by induction. Since

B(1) = (aS + bB) + B = B(1 + b) + aS

the result is true for k = 0. Now assume formula (3.10.15) is true for B(k + 1), k being some value.
We have

B(k + 2) = [aS + bB(k + 1)] + B(k + 1)


£ ¤ £ ¤
= (1 + b) (1 + b)k+1 B + aS (1 + b)sk+1 b + 1
¡ ¢
= (1 + b)k+2 B + aS sk+2 b

so the result is also true for B(k + 2), as required.

It follows that, in this bonus system, the M.P.V. of a whole life assurance is

X
[S + B(k + 1)]v k+1 k |qx+t
k=0

(assuming death benefits are payable at end of the year of death)

∞ ½
X · ¸ ¾
(1 + b)k+1 − 1
= S + aS + B(1 + b)k+1 k |qx+t
b
k=0
∞ n
³
X a´ ³a ´ o
= S 1− + S + B (1 + b)k+1 k |qx+t
b b
k=0
³ a ´ ³ a ´
=S 1 − Ax+t + S + B A∗x+t (3.10.16)
b b
i−b
where A∗x+t is at rate
1+b

Notes. (1) If the sum assured is payable immediately on death, the M.P.V. is approximately
³ a´ ³a ´ 1
S 1− Āx+t + S + B (1 + i) 2 A∗x+t
b b (3.10.17)

(2) At inception of the policy, we have t = 0 and B = 0.


(3) The “ordinary” compound bonus system is a special case of the two-tier system with
a = b.
(4) For an endowment assurance the formulae are:
³ a´ ³a ´
at end of year
: S 1− Ax+t:n−t + S + B A∗x+t:n−t
of death b b (3.10.18)
³ a ´
immediately
: S 1− Āx+t:n−t
on death b (3.10.19)
³a ´h 1
i
+ S + B (1 + i) 2 A∗ x+t
1
:
∗ 1
n−t + A x+t:n−t
b
(Note that in formula (3.10.19) the term assurance benefits are “accelerated”
but the pure endowment parts are the same as in formula (3.10.18).)
54 CHAPTER 3. ASSURANCES

3.11 Guaranteed bonus policies


As their name suggests, these are not with profits contracts, as they provide additions to the basic
benefit at rates which are fixed at the outset and do not depend on future the experience of the life
office. But many of the formulae used to value these benefits are the same as those we have derived
in the previous section.

Example 3.11.1. A company issues 20-year endowment assurances each with a basic sum assured
of £1, 000 to male lives aged 45.

Guaranteed simple reversionary bonuses at the rate of 2.25% of the current sum assured vest on pay-
ment of each annual premium. Alternatively, at the outset of a policy, a life assured may elect that
compound reversionary bonuses should be added to the policy instead of simple reversionary bonuses.

The sum assured and added bonuses will be payable at maturity, or at the end of the year of
death, if earlier.

Derive an expression from which can be calculated the guaranteed rate of compound bonus the
company should offer so that the value of the benefits at the outset is the same.

Solution. Let b be the annual rate of guaranteed compound bonus. The equation of value for b is:

1000 [A45:20 + 0.0225(IA)45:20 ]


= 1000A∗45:20 at rate i∗ = i−b
1+b

This may be solved for i∗ , and hence b may be found.


3.12. EXERCISES 55

Exercises

3.1 Evaluate (a) Ā60 and (b) A60 on the bases:

(i) A1967-70 ultimate, 4% p.a. interest;


(ii) E.L.T. No.12 - Males, 4% p.a. interest.

3.2 (i) Show that Ax = vqx + vpx Ax+1


(ii) Given that p60 = 0.985, p61 = 0.98, i = 0.05 and A62 = 0.6 , evaluate A61 and A60

3.3 Consider n lives now aged x1 , x2 , . . . , xn respectively. Let Z be the total present value of a
contract providing the sum of £Si immediately on death of (xi ), i = 1, 2, . . . , n. The n lives
are subject to the same non-select mortality table, Table B, and the interest is taken to be
fixed at rate i p.a.

(i) Show that E(Z) = S1 Āx1 + · · · + Sn Āxn on Table B with interest at rate i p.a.
(ii) Assuming further that the future lifetimes T (xi ) of the lives are independent variables,
show that
Xn
Var(Z) = Sj2 [Ā∗xj − (Āxj )2 ]
j=1

where ∗ refers to an interest rate of 2i + i2 p.a.

3.4 Using commutation functions or otherwise calculate the values of the following:

(i) A[40]:10 on A1967-70, 4% p.a. interest;


1
(ii) A30 :20 on A1967-70 Ultimate, 4%;
1
(iii) Ā30 :20 on A1967-70 Ultimate, 4%;
(iv) Ā30:20 on A1967-70 Ultimate, 4%;
(v) Ā30:20 on English Life Table No.12 Males, 4%.

3.5 A life aged 50 who is subject to the mortality of the A1967-70 Select table, effects a pure
endowment policy with a term of 20 years for a sum assured of £10,000.

(i) Write down the present value of the benefits under this contract, regarded as a random
variable.
(ii) Assuming an effective rate of interest of 5% per annum, calculate the mean and the
variance of the present value of the benefits available under this contract.

3.6 What are the random variables (in terms of K = curtate future lifetime of (x)) whose means
are represented by the following symbols?

(i) n Ex
(ii) Ax1:n
(iii) Ax:n1

3.7 A life aged exactly 60 wishes to arrange for a payment to be made to a charity in 10 years’
time. If he is still alive at that date the payment will be £1000. If he dies before the payment
date, the amount given will be £500. Assuming an effective interest rate of 6% per annum and
56 CHAPTER 3. ASSURANCES

mortality according to ELT No.12-Males, calculate the standard deviation of the present value
of the liability.
3.8 (Difficult)
You are given that

(i) 1000 (IA)50 =4,996.75 ,


1
(ii) 1000 A50 :1 =5.58,
(iii) 1000 A51 = 249.05 and
(iv) i =0.06.

Calculate 1000(IA)51 .
3.13. SOLUTIONS 57

Solutions
1
3.1 (i) (a) Ā60 = (1 + 0.04) 2 A60
1
= (1.04) 2 0.51726
= 0.52750
(b) A60 = 0.51726 from tables

(ii) (a) Ā60 = 0.58317 from tables


1
(b) A60 = (1.04)− 2 Ā60
= 0.57185

X
3.2
(i) Ax = k |qx v k+1
k=0


X
= qx v + px k−1 |qx+1 v k+1
k=1


X
= qx v + px v k |qx+1 v k+1
k=0

= qx v + px vAx+1
1
(ii) A61 = (0.02 + 0.98 · 0.6) = 0.57905
1.05
1
A62 = (0.015 + 0.985 · 0.57905) = 0.55749
1.05
3.3 (i)

We have
n
X
Z= Sj v T j
j=1

so
n
X n
X
E(Z) = Sj E(v Tj ) = Sj Āxj
j=1 j=1

(ii) By independence of Txj , we have


n
X
Var(Z) = Sj2 var(v Tj )
j=1
n
X
= Sj2 [Ā∗xj − (Āxj )2 ]
j=1

M[40] − M50 + D50 1904.8595 − 1767.5555 + 4597.0607


3.4 (i) = = 0.67812
D[40] 6981.5977
M30 − M50 1981.9552 − 1767.5555
(ii) = = 0.020550
D30 10433.310
58 CHAPTER 3. ASSURANCES

1
1 1
(iii) Ā30 :20 ; (1 + i) 2 A30:20 = 0.020956

1 D50
(iv) Ā30:20 = Ā30 :20 + = 0.46157
D30
1
Ā30 :20 1 M̄30 − M̄50
(v) 1
A30 :20 ; 1 = (1 + i)− 2 = 0.03286
(1 + i) 2 D30

3.5 (i) ½
10, 000v 20 if T ≥ 20
P.V. =
0 if T < 20

where T = future lifetime of (50).


(ii)
E(P.V.) = (20 p[50] × v 20 + (1 − 20 p[50] ) × 0)10, 000
= 10, 000 × 0.725539 × 1.05−20 = 2734.48
E[(P.V.)]2 = 20 p[50] (10, 000v 20 )2 = 0.725539 × 10, 0002 × 1.05−40 = 10, 305, 968
⇒ V ar(P.V.) = E[(P.V.)2 ] − [E(P.V.)]2
= 2, 828, 587 = 1, 681.842

3.6 (i) ½
vn if K ≥ n
Z=
0 if K < n

(ii) ½
0 if K ≥ n
Z=
v K+1 if K < n

(iii) Same as 3.6(i)

3.7 Let X payment to be made at time 10 years.


½
1000 with prob. 10 p60
X=
500 with prob. (1 − 10 p60 )

∴ E(X) = 100010 p60 + 500(1 − 10 p60 )


= 500(1 + 10 p60 ) = £847.207
Var(X) = 10002 10 p60 + 5002 (1 − 10 p60 ) − (mean)2
1
= 1, 000, 000[10 p60 + (1 − 10 p60 )] − (mean)2
4
= 1, 000, 000[0.25 + 0.7510 p60 ] − (mean)2
= 770, 811.1 − (847.207)2
= 53, 051.39
∴ s.d. of X = 230.33

Present value is v 10 (X) and so has s.d. 230.33v 10 at 6% = £128.615


3.13. SOLUTIONS 59

3.8

C50 + 2C51 + · · · 1 (C51 + C52 + · · · ) + (C51 + 2C52 + · · · )


(IA)50 = = A50 :1 +
D50 D50
1 D 51
= A50 :1 + D (A51 + (IA)51 )
50
1
Now vq50 = A50 :1 = 0.00558
D51
∴ = v − vq50 = 0.93782
D50
∴ 4.99675 = 0.00558 + 0.93782[0.24905 + (IA)51 ]
∴ (IA)51 = 5.07305
∴ 1, 000(IA)51 = 5, 073
60 CHAPTER 3. ASSURANCES
Chapter 4

ANNUITIES

4.1 Annuities payable continuously


Let (x) be entitled to £1 p.a. for life, payable “continuously”, i.e. the rate of payment at time t
years is £1 p.a. if (x) is then alive. The present value of this benefit, expressed as a random variable,
is
g(T ) = āT for T > 0 (4.1.1)

This variable has mean āx , i.e.


Z ∞
āx = E(āT ) = āt t px µx+t dt (4.1.2)
0

An alternative definition is:

Z ∞
āx = vt t px dt (4.1.3)
0 | {z } | {z }
interest (or probability
discounting) factor of survival
of (x) for t years

To show that these definitions are equivalent, we may use integration by parts, as follows:

Z ∞
āt t px µx+t dt
0 | {z }
| {z }

Z t d d
āt = v r dr (−t px ) = (t qx )
0
dt dt

d = t px µx+t
So (ā ) = v t
dt t

61
62 CHAPTER 4. ANNUITIES

Z ∞ Z ∞
£ ¤∞
āt t px µx+t dt = āt (−t px ) 0 + v t t px dt
0 0
Z ∞
= v t t px dt
0

Note the following conversion relationship (i.e. a formula connecting assurance and annuity
values):
Āx = 1 − δāx (4.1.4)
which follows from:
Z ∞
āx =
āt t px µx+t dt
0
∞µ Z ¶
1 − vt
= t px µx+t dt
0 δ
1
= (1 − Āx ).
δ
We may also argue as follows. By the Mathematics of Finance,

1 = δāT + vT (for all T > 0)


| {z } | {z } | {z }
Capital p.v. p.v. of
invested of interest return of capital
Take expected values of each side, which gives
1 = δāx + Āx
The variance of the present value of an annuity. Since
1 − vT
āT =
δ
we have
1
Var(āT ) = Var(v T )
δ2 ¡ ¢
Ā∗x − Ā2x
= (4.1.5)
δ2
where ∗ indicates “at the rate of interest 2i + i2 p.a.”; the corresponding force of interest is δ ∗ = 2δ,
so some writers use 2 Āx rather than Ā∗x .
Example 4.1.1. Express Var(āT ) in terms of āx and ā∗x .

Solution.
Ā∗x = 1 − (2δ)ā∗x since force of interest is 2δ
so, by formula (4.1.5),
1 − 2δ(ā∗x ) − (1 − δāx )2
Var(āT ) =
δ2
1 − 2δ(ā∗x ) − 1 − δ 2 (āx )2 + 2δāx
=
δ2
1£ ¤
= 2(āx − ā∗x ) − δ(āx )2
δ
4.2. ANNUITIES PAYABLE ANNUALLY 63

Commutation Functions. Define

Dx = v x lx (as stated in Chapter 3) (4.1.6)


Z 1
D̄x = v x+t lx+t dt (4.1.7)
0

X
N̄x = D̄x+t (4.1.8)
t=0

Example 4.1.2. Show that


N̄x
āx = (4.1.9)
Dx
Solution.

X
N̄x = D̄x+t
t=0

Z t+1
but (on change of variable) D̄x+t = v x+r lx+r dr for t = 0, 1, 2, . . ., so
t

Z ∞
N̄x = v x+r lx+r dr
0

from which we obtain Z ∞


N̄x
= v t t px dt = āx .
Dx 0

4.2 Annuities payable annually


Now consider the cases when an annuity of £1 p.a. is payable (i) annually in advance, (ii) annually
in arrear. (The former is called an annuity-due.)
In case (i), the p.v. of the benefit is
g(K) = äK+1 (4.2.1)
In case (ii), it is
g(K) = aK = äK+1 − 1 (4.2.2)
where we take a0 = 0. The means are denoted by äx and ax respectively. Note that

äx = 1 + ax VITAL!
(4.2.3)
(since the annuitant gets extra immediate payment of 1 in case (i)).
Now
¡ ¢
äx =E äK+1

X
= äk+1 k |qx (4.2.4)
k=0

But

1 − v K+1
äK+1 = (K = 0, 1, 2 . . . )
d
64 CHAPTER 4. ANNUITIES

so
· ¸
1 − v K+1 1 − E(v K+1 )
äx =E =
d d
1 − Ax
= (4.2.5)
d
We thus have the conversion relationship

Ax = 1 − däx (4.2.6)

Important formulae for ax and äx . We note the following results:



X
ax = v t t px (4.2.7)
t=1
X∞
äx = v t t px (4.2.8)
t=0

It is sufficient to prove (4.2.8) since ax = äx − 1 then gives (4.2.7).

Two Proofs
(1) Regard the annuity as sum of pure endowments due at times 0,1,2,. . .. Thus

äx = 0 Ex + 1 Ex + 2 Ex . . .
X∞
= v t t px
t=0

(2) By the definition,



X
äx = äk+1 k |qx
k=0
µ ¶ µ ¶ µ ¶
dx dx+1 dx+2
= ä1 + ä2 + ä3 + ...
lx lx lx
1 © ª
= dx + (1 + v)dx+1 + (1 + v + v 2 )dx+2 + . . .
lx
© ª
(dx + dx+1 + dx+2 + . . . ) + v(dx+1 + dx+2 + . . . ) + v 2 (dx+2 + . . . ) + . . .
=
lx
lx + vlx+1 + v 2 lx+2 + . . .
=
lx
X∞
= v t t px ,
t=0

as required.

Evaluation of äx by Commutation Functions Define



X
Nx = Dx+t
t=0
4.3. TEMPORARY ANNUITIES 65

We obtain
v x lx + v x+1 lx+1 + . . .
äx =
v x lx
Dx + Dx+1 + . . .
=
Dx
Nx
=
Dx
Remark If we require ax , use ax = äx − 1 or
Nx+1
ax = (since Nx+1 = Dx+1 + Dx+2 + . . . )
Dx
The variance of äK+1 .
We use the formula
µ ¶
¡ ¢ 1 − v K+1
Var äK+1 = Var
d
1 ¡ K+1 ¢
= 2 Var v
d
A∗ − (Ax )2
= x 2 (4.2.9)
d
where ∗ indicates the rate of interest i∗ = 2i + i2 p.a.
Note For an annuity payable in arrear, we use the result that
¡ ¢
Var (aK ) = Var äK+1 − 1
¡ ¢
= Var äK+1 (4.2.10)

4.3 Temporary annuities


Let us first consider the “continuous payments” case, and suppose that (x) is entitled to £1 p.a.
payable continuously for at most n years. (that is, payments cease when x dies or after n years,
whichever is earlier.) The p.v. is
(
āT if T < n
g(T ) = = āmin{T,n} (4.3.1)
ān if T ≥ n

The mean of g(T ) is


Z n Z ∞
āx:n = āt t px µx+t dt + ān t px µx+t dt
0 n
Z n
= v t t px dt (4.3.2)
0

by integration by parts. (Check this!)

Note. THERE ARE NO SUCH THINGS AS ā 1 and ā 1 !!


x:n x:n

Commutation Functions.
If n is an integer, Rn
0
v x+t lx+t dt N̄x − N̄x+n
āx:n = x
= (4.3.3)
v lx Dx
66 CHAPTER 4. ANNUITIES

If payments are made annually in advance, and are limited to at most n payments, we obtain
(
äK+1 if K < n h i
äx:n = mean of = E ämin{K+1,n} (4.3.4)
än if K ≥ n

n−1
X
= äk+1 k |qx + än n px
k=0

which may be shown to be equal to

1 + vpx + v 2 2 px + · · · + v n−1 n−1 px

(i.e., the sum of the values of n pure endowments each for £1)

Dx + Dx+1 + · · · + Dx+n−1
=
Dx

Hence

Nx − Nx+n
äx:n = (4.3.5)
Dx
If the payments are made annually in arrear, we obtain
(
aK if K < n h i
ax:n = mean of = E amin{K,n} (4.3.6)
an if K ≥ n
leading to
Dx+1 + Dx+2 + · · · + Dx+n Nx+1 − Nx+n+1
ax:n = = (4.3.7)
Dx Dx
Note that
(a) äx:n = 1 + ax:n−1 (4.3.8)
(b) äx:n and ä[x]:n are tabulated for certain values of x+n in the A1967-70 section of “Formulae
and Tables.”

Example 4.3.1. By expressing Ax:n and äx:n as expectations of appropriate random variables, or
otherwise, prove the conversion relationships

(i) Ax:n = 1 − däx:n , and

(ii) Āx:n = 1 − δāx:n .


4.4. DEFERRED ANNUITIES 67

Proof.

1 − v min{K+1,n}
(i) By Maths. in Finance, ämin{K+1,n} = .
d
Take expected values to obtain
1 − Ax:n
äx:n =
d
which gives the required result.
(ii) Take expected values in the equation

1 − v min{T,n}
āmin{T,n} = .
δ

4.4 Deferred annuities


These are annuities which commence in m (say) years’ time, provided that the annuitant is then
active. Thus
m |āx = M.P.V. of annuity of 1 p.a. to (x), payable con- (4.4.1)
tinuously, beginning in m years’ time
It is often best to evaluate m |āx by the formula
µ ¶
Dx+m
|ā
m x = × āx+m (4.4.2)
Dx
| {z } | {z }
pure annuity
endowment factor
factor at age x + m
Similarly, µ ¶
Dx+m
m |äx = äx+m (4.4.3)
Dx
and so on. Note that
äx = äx:n + n |äx (4.4.4)
and µ ¶
Dx+m Nx+m Nx+m
m |äx = = (4.4.5)
Dx Dx+m Dx
N.B. Pensions are (essentially) deferred annuities.

Select Tables The modifications needed for select tables are straightforward, and are illustrated in
Table 4.4.1 below.
68 CHAPTER 4. ANNUITIES

In terms of Commutation
Type of Annuity Symbol In terms of t p[x] Functions


X N[x]+1
Immediate a[x] v t t p[x] D[x]
t=1

X N[x]
Annuity-due ä[x] v t t p[x]
t=0
D[x]

n
X N[x]+1 − Nx+n+1
Temporary (n years) a[x]:n v t t p[x] (n ≥ 1)
D[x]
t=1

n−1
X
Temporary annuity-due N[x] − Nx+n
ä[x]:n v t t p[x] (n ≥ 2)
(n years’ payments) D[x]
t=0


X Nx+m+1
Deferred (m years) v t t p[x] (m ≥ 1)
m |a[x] D[x]
t=m+1

X
Deferred annuity-due Nx+m
m |ä[x] v t t p[x] (m ≥ 2)
(m years) D[x]
t=m

Table 4.4.1: Expression for Values of Single-Life Curtate Annuities. Life aged x.
Select Mortality Table (select period 2 years)

4.5 Annuities payable m times per annum


We require the following formula from numerical analysis:
The Euler-Maclaurin formula:
Z ∞ ∞
X 1 1
f (t) dt + f (t) − f (0) + f 0 (0) (4.5.1)
0 t=0
2 12

Woolhouse’s formula may be then deduced:


∞ µ ¶ ∞ µ ¶ µ ¶
1 X t X m−1 m2 − 1
f + f (t) − f (0) + f 0 (0) (4.5.2)
m t=0 m t=0
2m 12m2

(We have assumed that f (t) → 0 and f 0 (t) → 0 as t → ∞.)

Example 4.5.1. Use the Euler-Maclaurin formula to deduce an approximate formula for āx in terms
of äx
4.5. ANNUITIES PAYABLE M TIMES PER ANNUM 69
³ R ´
t
Solution. Let f (t) = v t t px = exp − 0 (δ + µx+r ) dr . Hence
³ R ´
t
f 0 (t) = −(δ + µx+t ) exp − 0 (δ + µx+r ) dr . ∴ f (0) = 1 and f 0 (0) = −(δ + µx ). By E.-M.

1 1 1 1
āx + äx − − (µx + δ) or ax + − (µx + δ) (4.5.3)
2 12 2 12
In practice the final term is usually ignored, giving
1 1
āx + äx − = ax + (4.5.4)
2 2


Example 4.5.2. Find an approximate formula for ex in terms of ex .

Solution. Set i = 0 in example 4.5.1. This gives


◦ 1 1
ex + ex + − µx (4.5.5)
2 12
The final term is usually omitted, giving
◦ 1
ex + ex + (4.5.6)
2
(m) (m)
The symbols äx and ax refer to the expected present values of an annuity of 1 per annum payable
monthly in advance and monthly in arrear respectively. Thus
X∞
1 mt
ä(m)
x = v t (4.5.7)
t=0
m m px
and
X∞
1 mt
a(m)
x = v t (4.5.8)
t=0
m m px

(m)
Example 4.5.3. Find an approximate formula for äx in terms of äx .

Solution. Apply Woolhouse’s formula to f (t) = v t t px . This gives


µ ¶ µ 2 ¶
(m) m−1 m −1
äx + äx − − (µx + δ) (4.5.9)
2m 12m2
In practice one usually uses
µ ¶
m−1
ä(m)
x + äx − (4.5.10)
2m
Note.
1
a(m)
x = ä(m)
x −
µ m ¶
m−1 1
= äx − − (ignoring the final term)
2m m
µ ¶
m−1
= ax + (4.5.11)
2m
70 CHAPTER 4. ANNUITIES

Temporary mthly annuities

Define
(m)
äx:n = M.P.V. of £1 p.a. payable mthly in advance to (x)
for at most n years
= ä(m)
x − n |ä(m)
x (4.5.12)
D x+n (m)
= ä(m)
x − ä
Dx x+n
· µ ¶¸ µ ¶· µ ¶¸
m−1 Dx+n m−1
+ äx − − äx+n −
2m Dx 2m
µ ¶ µ ¶
m−1 Dx+n
= äx:n − × 1− (4.5.13)
2m Dx
| {z }
CARE!
DO NOT OMIT
THIS TERM

We also have µ ¶µ ¶
m−1 Dx+n
a(m)
x:n
= ax:n + 1− (4.5.14)
2m Dx

4.6 Complete annuities (or “annuities with final proportion”)


Suppose that (x) is to receive 1 p.a. monthly in arrear with a final payment immediately on death at
t 1
time r since the last full payment, i.e. on death at time + r, where t = 0, 1, 2, . . . and 0 ≤ r < m
m
1 1
This final payment varies from 0 to m and hence is on average about 2m . This annuity is called
“complete” or “with final proportion”, and we define

◦ (m)
ax = M.P.V. of complete annuity
1
+ a(m)
x + Āx (4.6.1)
2m

◦ (m) ◦
When m = 1 we may write ax =ax , giving

◦ 1
ax + ax + Āx (4.6.2)
2

More accurate formulae for ax . If we have U.D. of D in each of the age-ranges x to x + 1, x + 1
to x + 2, etc., we have the exact formulae
µ ¶ µ ¶
◦ i−δ i−δ
ax = ax + Ax = ax + Āx (4.6.3)
δ2 iδ

Proof. The final payment is shown below: Hence (using the formula for a varying assurance)
4.7. VARYING ANNUITIES 71
final
payment
(on death of (x) ) 6

1
· · · ·
· · · ·
· · · ·
· · · · .
· · · · ..
.
· · · · ..
· · · ·
· · · · ··
· · · · · - time (years)
0 1 2 3 4


X Z 1
◦ t
ax = ax + v t px rv r r px+t µx+t+r dr
t=0 | {z } 0 | {z }
pure endowment value at time t
factor to age x + t of death benefit
in year t + 1

By U.D. of D.,

X Z 1

ax = ax + v t t px qx+t rv r dr
t=0 0
| {z }
¯ ā −v
= (Iā) 1 =
1
δ

X∞ · ¸
t+1 ā1 − v
= ax + v t |qx (1 + i)
t=1
δ
µ ¶ µ ¶
s̄1 − 1 (1 + i) − 1
= ax + Ax = ax + Ax − 1 /δ
δ δ
µ ¶
i−δ
= ax + Ax
δ2

The second part of (4.6.3) follows from Āx = δi Ax (which holds by U.D.D.)

4.7 Varying annuities


Suppose, firstly, that payments are made continuously so long as (x) survives at the rate b(t) p.a. at
time t. The present value of these payments (as a random variable depending on the future lifetime
T of (x)) is (by the Mathematics of Finance)
Z T
g(T ) = v t b(t) dt (4.7.1)
0

The mean present value is thus


Z ∞
E[g(T )] = g(t)t px µx+t dt (4.7.2)
0
72 CHAPTER 4. ANNUITIES

Using the following important formula (obtained from integration by parts):

Z ∞ µZ t ¶ Z ∞ µZ ∞ ¶
u(t) v(r) dr dt = v(t) u(r) dr dt (∗)
0 0 0 t

we obtain the alternative expression


Z ∞
E[g(T )] = vt b(t) t px dt (4.7.3)
0 | {z } | {z } | {z }
interest rate of probability
factor payment of survival
at time t

To prove this, let


v(t) = v t b(t)
and
u(t) = t px µx+t
so that Z ∞
u(r) dr = t px then use formula (∗)
t

Example 4.7.1. Define (Iā)¯ x = M.P.V. of an increasing annuity to (x) in which


the rate of payment at time t is t.
¯ x in terms of an integral.
Find an expression for (Iā)

¯
Solution. Let b(t) = t, t ≥ 0. This gives g(T ) = (Iā) T , and we have
¯ x = M.P.V. of annuity to (x) with payment at rate t p.a. at time t
(Iā)
Z ∞
= tv t t px dt (4.7.4)
0

An approximation. Let f (t) = tv t t px , (t ≥ 0). By Euler-Maclaurin,



X
¯ x+ 1 1
(Iā) tv t t px − f (0) + f 0 (0)
t=0
2 12
d
But f (0) = 0 and f 0 (0) = [v t t px + t dt (v t t px )]t=0 = 1, so

X
¯ x+ 1
(Iā) tv t t px +
t=1
12
1
= (Ia)x + (see definition of (Ia)x below) (4.7.5)
12
Now suppose that 

1 in year 1
b(t) = [t] + 1 = 2 in year 2


...
4.7. VARYING ANNUITIES 73

The M.P.V. of the corresponding annuity is written (Iā)x . Thus


Z ∞
(Iā)x = ([t] + 1)v t t px dt
0
µZ ∞ Z ∞ Z ∞ ¶
= + + v t t px dt
0 1 2
= āx + 1 |āx + 2 |āx + . . .
N̄x + N̄x+1 + . . .
=
Dx
S̄x
= (4.7.6)
Dx

where


X
S̄x = N̄x+t (4.7.7)
t=0

Now suppose that benefits are paid annually so long as (x) survives, the benefit at time t years being
b(t) (t = 0, 1, 2, . . . ). The present value of the benefits (regarded as a random variable) is

K
X
g(K) = v t b(t) (4.7.8)
t=0

since the last payment is made at time K. (If b(0) > 0, this is a variable annuity-due). The M.P.V.
of the varying annuity is thus

X
E[g(K)] = g(k)k |qx
k=0
1
= {g(0)dx + g(1)dx+1 + . . . }
lx
1
= {g(0)[dx + dx+1 + . . . ]
lx
+ [g(1) − g(0)][dx+1 + dx+2 + . . . ]
+[g(2) − g(1)][dx+2 + dx+3 + . . . ] + . . . }
1 © ª
= b(0)lx + vb(1)lx+1 + v 2 b(2)lx+2 + . . .
lx

X
E[g(K)] = b(t) vt t px (4.7.9)
t=0 | {z } | {z } | {z }
benefit interest survival
at time t factor factor

This formula forms the basis of “spreadsheet”


calculations for pension schemes, etc.
[This may also be found by summing the pure endowments]
74 CHAPTER 4. ANNUITIES

Applications
Let b(t) = t + 1 (t = 0, 1, 2, . . . ). The M.P.V. is defined as (Iä)x , so we have


X
(Iä)x = (t + 1)v t t px
t=0
" ∞
#
X
= (t + 1)Dx+t /Dx
t=0
Dx + 2Dx+1 + . . .
=
Dx
(Dx + Dx+1 + . . . ) + (Dx+1 + Dx+2 + . . . ) + . . .
=
Dx
Nx + Nx+1 + . . .
=
Dx
Sx
= (4.7.10)
Dx

where

X
Sx = Nx+t (4.7.11)
t=0

Similarly, when b(t) = t (t = 1, 2, . . . ) we get the M.P.V.


X X∞
t Dx+t
(Ia)x = tv t px = t
t=1 t=1
Dx
Sx+1
= (4.7.12)
Dx

Example 4.7.2. Show that


(Ia)x = (Iä)x − äx

Solution.

X
(Ia)x = tv t t px
t=0

X
= (t + 1 − 1)v t t px
t=0

X ∞
X
= (t + 1)v t t px − v t t px
t=0 t=0
=(Iä)x − äx

Temporary increasing annuities


4.7. VARYING ANNUITIES 75

Define
(Iä)x:n = M.P.V. of payments of t + 1 at time t
(t = 0, 1, 2, . . . , n − 1), provided that (x) is
then alive "n−1 #
n−1
X X
t
= (t + 1)v t px = (t + 1)Dx+t /Dx
t=0 t=0
Dx + 2Dx+1 + · · · + (n − 1)Dx+n−2 + nDx+n−1
=
Dx
(Dx + 2Dx+1 + . . . ) (Dx+n + 2Dx+n+1 + . . . )
= −
Dx Dx
(nDx+n + nDx+n+1 + . . . )

Dx
Sx − Sx+n − nNx+n
= (4.7.13)
Dx
Similarly,
Dx+1 + 2Dx+2 + · · · + nDx+n
(Ia)x:n =
Dx
(Dx+1 + 2Dx+2 + . . . ) (Dx+n+1 + 2Dx+n+2 + . . . )
= −
Dx Dx
(nDx+n+1 + nDx+n+2 + . . . )

Dx
Sx+1 − Sx+n+1 − nNx+n+1
=
Dx
Also,
Z n
¡ ¢
¯
Iā = tv t t px dt
x:n
0
S̄x − S̄x+n − nN̄x+n
(Iā)x:n = (4.7.14)
Dx
Note that (for example) µ ¶
Dx+m
m | (Iä)x:n = (Iä)x+m:n
Dx

which enables us to express m| (Iä)x:n in terms of commutation functions, viz

Sx+m − Sx+m+n − nNx+m+n


m| (Iä)x:n =
Dx
Conversion relationships for increasing annuities and assurances
¡ ¢ ¡ ¢
¯
āx = δ Iā + I¯Ā x (1)
x
äx = d (Iä)x + (IA)x (2)
¡ ¢
äx = δ (Iā)x + I Ā x (3)
Proof
To show (1), take expected values on each side of the maths of finance formula
¡ ¢
¯
āT = δ Iā + T vT
T
76 CHAPTER 4. ANNUITIES

To show (2), take expected values on each side of

äK+1 = d (Iä)K+1 + (K + 1)v K+1

To show (3), we need to use the formula

ä[t]+1 − ([t] + 1)v t


(Iā)t = for all t ≥ 0 (∗∗)
δ
where

(Iā)t = p.v. of an annuity-certain payable continuously at rate 1 p.a. in year


1, 2 p.a. in year 2, . . . , ceasing at time t exactly.
Z t
= v r ([r] + 1) dr
0

(∗∗) holds for t = 0, 1, 2, 3, . . . by McCutcheon and Scott, formula 3.6.6. It may be proved for general
t ≥ 0 by letting t = n + r ( n integer, 0 ≤ r < 1) and observing that
Z n+r
(Iā)t = (Iā)n + v s (n + 1) ds
n
µ ¶
än − nv n 1 − vr
= + (n + 1)v n
δ δ
t
ä − (n + 1)v
= n+1
δ
Note. There are also conversion relationships for temporary increasing annuities, e.g.
¯ x:n + (I¯Ā)x:n
āx:n = δ(Iā)
4.8. EXERCISES 77

Exercises

4.1 The following is an extract from a life table with a select period of 1 year.
Age x l[x] lx+1 age x + 1
55 90,636 90,032 56
56 89,739 89,132 57
88,151 58
87,094 59
85,874 60
84,586 61

Evaluate ä56:5 and ä[56]:5 at 5 % per annum interest.

4.2 Given that ax = 20, ax:n = 18 and ax+n = 8, find the values of n Ex and äx:n .

4.3 (i) Write down an expression for ax in terms of v, px , and ax+1 .


(ii) On a certain select mortality table the select period is one year. Express a[x] in terms of
v, p[x] and ax+1 . Given that q[x] = .6qx and that at 4 14 % p.a. interest a45 = 15.719 and
a46 = 15.509, find the value of a[45] (at the same rate of interest).
4.4 Using the A1967-70 table with 4% p.a. interest find the values of
ä[40]:30 , ä[39]+1:30 , ä40:30 , (Iä)40 , 5 |ä[40]:25 , 5 |(Ia)40:25

4.5 (i) Find the present value of a deferred annuity of £1000 p.a. to a man aged 40. Payments
commence on his 60th birthday, if he is then alive, and continue thereafter annually for
life.
Basis: A1967-70 ultimate mortality, 4 % p.a. interest.
(ii) As above, but select mortality (at entry.)
(iii) As above, but A1967-70 ultimate mortality to exact age 60 and a(55) males ultimate
mortality above exact age 60.

4.6 According to a certain mortality table, which has a select period of 1 year and is such that
q[x] = 0.6qx for each x,
a70 at 10% interest = 5.641, and,
a71 at 10% interest = 5.449.

Find a[70] at 10% interest.


78 CHAPTER 4. ANNUITIES

Solutions

4.1
l56 + vl57 + · · · + v 4 l60
ä56:5 = = 4.451
l56
l[56] + vl57 + · · · + v 4 l60
ä[56]:5 = = 4.463
l[56]

4.2
1
ax = ax:n + n Ex ax+n =⇒ n Ex =
4
äx:n = ax:n − n Ex + 1 = 18.75

4.3 (i) ax = vpx (1 + ax+1 )


(ii) Note that with a one-year select period
a[45] = vp[45] ä46
= (1.0425)−1 16.509p[45] . (a)

We must determine p[45] from the given data. Now


a45 = vp45 ä46

i.e.
(1 + i)a45 15.719
p45 = = 1.0425 × = .9926136
ä46 16.509
Hence
q45 = 0.0073864
Then
q[45] = 0.6q45 = 0.0044318
Hence
p[45] = 1 − q[45] = 0.9955682
From (a) we then get
a[45] = 15.7658

4.4
ä[40]:30 = 16.960 directly tabulated

ä[39]+1:30 = (N[39]+1 − N70 )/D[39]+1 = 16.953

ä40:30 = 16.949 directly tabulated

(Iä)40 = S40 /D40 = 276.468

5 |ä[40]:25 = (N45 − N70 )/D[40] = 12.342

5 |(Ia)40:25 = (S46 − S71 − 25N71 )/D40 = 122.337


4.9. SOLUTIONS 79

4.5 (i) 1000.N60 /D40 = 5130.08, say £5130


(ii) 1000.N
³ 60 /D
´[40] = 5133.68, say £5134
(iii) 1000. D ∗ D60
D40 .ä60 , where the factor D40 is evaluated on A1967-70 mortality and the factor
60

ä∗60 is evaluated on a(55) males mortality. The value is thus 1000 × .40873 × 12.625 =
5160.22, say £5160.
4.6 a70 = v(1 − q70 )ä71 , so 1 − q70 = 0.96218. Hence q[70] = 0.6q70 = 0.022918, and q[70] =
v(1 − q[70] )ä71 = 5.730
80 CHAPTER 4. ANNUITIES
Chapter 5

PREMIUMS

5.1 Principles of premium calculations


Premiums calculated without an allowance for expenses are called net premiums, whilst premiums
which are actually charged are called office or gross premiums. Office premiums are usually cal-
culated with an explicit allowance for expenses, and in some cases for a profit to the office. A life
assurance policy may be issued with (a) a single premium payable at the date of issue, or (b) reg-
ular premiums payable in advance and usually (but not always) of level amount. The frequency of
regular payments may be, for example, yearly or monthly, and the maximum number of premiums
payable may be limited to (for example) 20. Premiums should, of course, cease on the death of the
assured life, when the policy matures, or when there is no longer any possibility of future benefits:
for example, on the expiry of the term of a temporary (term) assurance policy.

Premiums are usually calculated by the equivalence principle, which may be stated as follows:
E(Z) = 0 (5.1.1)
where
Z = present value of profit to the life office on the contract
In some cases an explicit loading for profit is included in the calculation; that is, we replace equation
(5.1.1) by
E(Z) = B (5.1.2)
where B = M.P.V. of profit on the contract. In the absence of information to the contrary, however,
we shall assume that the equivalence principle (5.1.1) is to be used, although in some cases the use
of “conservative” assumptions regarding mortality, interest and expenses means that the office has
an implicit margin of expected profit. The rate of interest is usually taken to be fixed (not random).

Example 5.1.1. Consider a whole life assurance of 1 payable immediately on the death of (x),
and suppose that premiums are payable continuously at rate P p.a. until the death of (x). Give a
formula for P in terms of annuity functions, assuming that the equivalence principle applies.

Solution. Let
Z = g(T ) = P.V. of profit to office on this contract
= P āT − v T

81
82 CHAPTER 5. PREMIUMS

Since we require that E{g(T )} = 0, we have

E{P āT − v T } = 0
¡ ¢
∴ P.E (āT ) = E v T
∴ P āx = Āx
Āx 1
∴ P = = −δ using the conversion relationship Āx = 1 − δāx
āx āx
The equations (5.1.1) and (5.1.2) may be expressed as equations of payments, or equations of
value, i.e.

M.P.V. of premiums =M.P.V. of benefits


+ M.P.V. of expenses (if any) (5.1.3)
+ M.P.V. of profit to the office (if any)
In example 5.1.1, the equation of value is

P āx = Āx

5.2 Notation for premiums


The International Actuarial Notation (see Formulae and Tables for Actuarial Examinations) should
be used, at least for straightforward policies. If the policy is complicated it is best to just use the
symbol P (or P 0 , P 00 ) for the premium (single or regular). The general symbols P, P 0 , P 00 may be
used for any sum assured, but the standard symbols Px , P̄ (Āx ) , etc. , refer to a sum assured of
£1.

Some of the more important rules of the International Notation are the following:
1. The symbols P ( ), or P̄ ( ), or P (m) ( ) indicate the level net annual premium for the benefit
indicated in the brackets. Premiums are assumed to continue for as long as the contract can
provide benefits, i.e. n years for a n-year contract, provided of course that the life assured is
still alive. P, P̄ , P (m) indicate that payments are made annually in advance, continuously
and mthly in advance respectively.
2. If premiums are limited to, at most t years’ payments (where t < the term of contract, i.e. the
term of the benefits), we write t P ( ), t P̄ ( ), t P (m) ( ).
e.g.

10 P (Ax:20 ) = net annual premium for 20-year EA with S.A. =1


(payable at end of year of death or on survival
for 20 years.), limited to at most 10 payments
(i.e. payments cease on death or after 10
payments are made, whichever occurs first)
A
= x:20
äx:10

3. When benefits are payable at end of year of death we may (optionally) shorten the notation
as follows:
(e.g.) P (Ax ) = Px
5.3. THE VARIANCE OF THE PRESENT VALUE OF THE PROFIT ON A POLICY. 83

Table 5.2.1 relates to a mortality table with select period 2 years, for example A1967-70, and refers
to policies in which premiums are payable annually in advance.

5.3 The variance of the present value of the profit on a policy.


Consider again the policy of Example 5.1.1. We have

g(T ) = p.v. of profit to office on contract


= P āT − v T

(We need not assume the equivalence principle holds: if it does, E[g(T )] = 0.) Note that
µ ¶
1 − vT
g(T ) = P − vT
δ
µ ¶
P P +δ
= − vT
δ δ
µ ¶2
P +δ
∴ Var[g(T )] = Var(v T )
δ
µ ¶2
P +δ £ ∗ ¡ ¢2 ¤
= Āx − Āx (5.3.1)
δ
| {z }
at rate
2i + i2

Note the idea of “collecting” all the terms involving v T before taking the variance: a similar technique
may be used for endowment assurance policies (but not generally).

5.4 Premiums allowing for expenses


Most expenses may be classified as either:
(a) initial (incurred at the outset only) , or
(b) renewal (incurred on the payment of later premiums).
There may also be expenses of payment of benefits (especially for pensions and annuities) and the
expenses of maintaining records of policies with continuing benefits after premiums have ceased.
Expenses may also be divided into:
(i) commission payments, and
(ii) other costs.
Some offices employ a policy fee system, whereby a fixed addition of (say) £15 is added to the
annual premium; for example the office annual premium for a policy with a sum assured for £20,000
may be quoted as “£10 per £1,000 plus a policy fee of £15”, giving £20 × 10 + £15 = £215. This
system reflects the fact that certain administrative costs do not depend on the size of the benefit.

Example 5.4.1. Consider n-year endowment assurance without profits with sum assured £1 issued
to a select life aged x. Expenses are e per premium payment (including the first) plus I at issue
date (so the total initial expense is I + e ). Find formulae for the level office annual premium, P 00 .
CHAPTER 5. PREMIUMS

Net Annual Premium Net Annual Premium


Type of payable throughout duration of contract Limited to t payments
Assurance
In terms of In terms of commutation In terms of In terms of commutation
Symbol Symbol
A and ä Functions (n ≥ 2) A and ä Functions (n ≥ 2 and t ≥ 2)
1 1
1 A[x] :n M[x] − Mx+n 1 A[x] :n M[x] − Mx+n
Temporary P [x] :n t P [x]:n
ä[x]:n N[x] − Nx+n ä[x]:t N[x] − Nx+t
1 1
Deferred ¡ 1
¢ m |A[x]:n Mx+m − Mx+m+n ¡ 1
¢ m |A[x]:n Mx+m − Mx+m+n
P m |A[x]:n tP m |A[x]:n
Temporary ä[x]:m+n N[x] − Nx+m+n ä[x]:t N[x] − Nx+t
A[x] M[x] A[x] M[x]
Whole-life P[x] t P[x]
ä[x] N[x] ä[x]:t N[x] − Nx+t
Deferred m |A[x] Mx+m m |A[x] Mx+m
P (m |A[x] ) t P (m |A[x] )
Whole-life ä[x] N[x] ä[x]:t N[x] − Nx+t
Endowment A[x]:n M − Mx+n + Dx+n
[x] A[x]:n M[x] − Mx+n + Dx+n
P[x]:n t P[x]:n
Assurance ä[x]:n N[x] − Nx+n ä[x]:t N[x] − Nx+t
Table 5.2.1: Expressions for Annual Premiums. Select Mortality Table (select period 2 years)
Note. When n < 2 or t < 2, these formulae should be suitably modified.
84
5.5. PREMIUMS FOR WITH PROFITS POLICIES 85

Solution. The equation of value is

P 00 ä[x]:n = A[x]:n + eä[x]:n + I (5.4.1)

where P 00 = level annual premium. Suppose further that e = kP 00 (i.e. renewal expenses are a
proportion k of the office premium): we have

(1 − k)P 00 ä[x]:n = A[x]:n + I


A[x]:n + I
∴ P 00 =
(1 − k)ä[x]:n
· ¸
1 I
= P[x]:n + (5.4.2)
(1 − k) ä[x]:n

Sometimes I is a proportion of the first premium, cP 00 , say. This leads to

(1 − k)P 00 ä[x]:n = A[x]:n + cP 00


A[x]:n
∴ P 00 =
(1 − k)ä[x]:n − c

(For example, k = 2 21 % and c = 47 12 % if expenses are 50% of the first premium and 2 12 % of all
subsequent premiums .)

5.5 Premiums for with profits policies


In early days of life assurance, premiums for policies with the right to participate in the profits of the
life office were calculated on conservative assumptions but without an explicit allowance for bonus
declarations. In modern conditions there is often an explicit allowance for possible future bonus
rates for with profits (or “participating”) policies, as illustrated in the next example.

Example 5.5.1. A with-profits whole-life assurance is about to be issued to a man aged 45. The
basic sum assured is £12, 000. The office assumes that at the start of each policy year there will be
bonus additions at the rate of 1% per annum compound. The basic sum assured and bonuses are
payable immediately on the man’s death. The policy has half-yearly premiums, payable for at most
20 years. Calculate the half-yearly premium on the following basis: (Note: at 3.75% per annum
interest, the value of A[45] is 0.34587).

Solution. Let annual premium be P .


1
Value of benefits + 12, 000(1 + i) 2 [1.01vq[45] + (1.01)2 v 2 1 |q[45] + · · · ]
1
= 12, 000(1 + i) 2 A∗[45] (see formula (3.10.11))

i − 0.01
where ∗ is at rate + 3.96%
1 + 0.01

By linear interp., A∗[45] ; 0.32804.

∴ Value of benefits = £4033.69


86 CHAPTER 5. PREMIUMS

Equation of value is
(2) (2)
0.96P ä[45] :20 = 0.16P ä[45] :1 + 200 + 4033.69
| {z } | {z }
12.362 0.98753
∴ P = £361.58
Hence the half-yearly premium is £180.79

5.6 Return of premium problems


Consider, as an example, a policy issued to (x) with level annual premiums, P , providing:
(i) £1, 000 on survival for n years, and
(ii) a return of all premiums paid, at the end of the year of death, if (x) dies within n years.
Assume firstly that the premiums are returned without interest (sometimes described as “Return
No Interest”, abbreviated to R.N.I.). If there are no expenses, the equation of value for P is:
Dx+n
P äx:n = 1000 + P (IA)x1:n (5.6.1)
Dx
| {z } | {z }
survival return of
benefit premiums
on death
This may be solved for P .
Note. If the premiums are returned immediately on death, replace (IA)x1:n by (I Ā)x1:n .

Now suppose that the premiums are returned with compound interest at rate j p.a. In practice,
such policies are often described as “ Return With Interest” (R.W.I.). If the death benefit is paid
at the end of the year of death, the term P (IA)x1:n in equation (5.6.1) must be replaced by
n−1
X n o
v k+1 k |qx P s̈k+1 j
k=0
| {z }
death benefit
in year k + 1
X · (1 + j)k+1 − 1 ¸
n−1
=P v k+1 k |qx
dj
k=0
P
= . {A∗ x1:n − Ax1:n } (5.6.2)
[j/(1 + j)]
i−j
where ∗ indicates the rate of interest 1+j

Notes. 1. If the death benefit is payable immediately on death, formula (5.6.2) should be multi-
1 1 1
plied by (1 + i) 2 /(1 + j) 2 = (1 + i∗ ) 2 .
2. If i (the rate of interest used to discount premiums and benefits in the equation of
value) equals j, some of these problems may be simplified: if there are no expenses,
mortality (within n years) may be ignored giving the compound interest equation
P än = 1000v n
5.7. ANNUITIES WITH GUARANTEES 87

This result may best be proved by reference to reserves, which we shall discuss later.

Example 5.6.1. A life office sells policies each providing a cash sum at age 65. Premiums of £1, 000
are payable annually in advance during the deferred period. On the death of the policyholder during
the period of deferment, the premiums paid are returned immediately without interest. In respect
of a life now aged 45, find the cash sum at age 65, given that the office uses the following basis:
A1967-70 select
4% p.a. interest
expenses are ignored

Solution. Let C be available at age 65. The equation of value is


D65 1
1000ä[45]:20 = C + 1000(I Ā)[45] :20
D[45]
£ ¤
1000 (N[45] − N65 ) − (1.02)(R[45] − R65 − 20M65 )
∴ C=
D65
1000[76, 722.7 − 1.02 × 7, 407.24]
= = £32, 258
2144.1713

5.7 Annuities with guarantees


Annuities are sometimes sold with the provision that payments will certainly continue until their
total equals the purchase price, B say (or possibly some proportion of this purchase price.)

Let us consider an annuity of £1 p.a., payable continuously with this guarantee to a life aged x
at the issue date. Ignoring expenses, the equation of value for B is

B = āB + B |āx (5.7.1)

Theorem 5.7.1. Equation (5.7.1) has a unique solution


Proof. Let

f (B) = āB + B |āx − B (B ≥ 0)


Z B Z ∞
= v t dt + v t t px dt − B
0 B

Hence

f 0 (B) = v B − v B t px − 1
= v B B qx − 1
<0 for all B > 0

1
Now f (0) = āx > 0 and, as B → ∞, f (B) → −∞ (as āx → δ and B |āx → 0). It follows that
f (B) = 0 has a unique solution
88 CHAPTER 5. PREMIUMS

Note. The purchase price of an annuity with this guarantee may be considerably larger than for
an ordinary annuity.

If the annuity instalments are paid annually in arrear we must solve the equation

B = an + n |ax (5.7.2)

subject to the condition n − 1 < B ≤ n. (Here n is the guarantee period, which must be an integer.)
This is solved by trial and error.

Another type of guaranteed annuity is that in which the balance (if any) of the purchase price
(or a proportion of it) over the total annuity payments received is paid immediately on the death
of the annuitant. For example, if the guarantee consists of a payment of the balance of 85% of the
purchase price over the total annuity instalments received, equation (5.7.1) would be replaced by
Z n
B = āB + [0.85B − t]v t t px µx+t dt (5.7.3)
0

where n = 0.85B (the time when the annuity payments equal 85% of the purchase price).

Example 5.7.1. A man aged 65 buys a guaranteed annuity payable continuously for a purchase
price of £32, 258. The annuity payments are guaranteed to continue until the total payments reach
£20, 000. The office issuing the contract uses the following basis:

A1967-70 ultimate;
4% p.a. interest;
expenses are ignored.

Let n be the guarantee period. Give an equation for n, and prove that this equation has a unique
solution, which lies between 6 and 7 years.

Solution. Let Z be the annual rate of payment of the annuity. We have the equation
32, 258 = Z(ān + n |ā65 ) (1)
subject to nZ = 20, 000 (2)
Replace Z by 20, 000/n in (1) to obtain
32, 258
ān + n |ā65 = n = 1.6129n
20, 000

i.e. solve f (n) = ān + n |ā65 − 1.6129n = 0


Now f (0) = ā65 > 0, and, as n → ∞, f (n) → −∞.
µZ n Z ∞ ¶
0 d t t
f (n) = v dt + v t p65 dt − 1.6129n
dn 0 n
= v n − v n n p65 − 1.6129
= v n n p65 − 1.6129 < 0, since, for n > 0, v n n p65 < 1

∴ f (n) decreases, so f (n) = 0 has a unique solution.


5.8. FAMILY INCOME BENEFITS 89

Try n = 7
1, 289.7567
f (n) = 6.12136 + × 7.772 − 11.2903
2, 144.171
<0
Try n = 6
1, 401.6093
f (n) = 5.34626 + × 8.112 − 9.6774
2, 144.171
>0
Hence 6 < n < 7

5.8 Family income benefits


A family income benefit of term n years is a series of instalments payable from the date of death of
the assured life, if he dies within n years, for the balance of the term. It may be considered as a
decreasing term assurance in which the benefit on death is an annuity-certain for the balance of the
term.

Case 1.The benefits are payable continuously


Suppose that F.I.B. payments are at rate £B per annum, and the life assured is aged x at the issue
date. The present value of the benefits is
B(ān − āT ) if T < n
(5.8.1)
0 if T ≥ n
The P.V. of the benefits may thus be written in the form
h i
B ān − āmin{T,n} (5.8.2)

and hence their M.P.V. is


B (ān − āx:n ) (5.8.3)
thly
Case 2.The benefits made m in arrear, beginning at the end of the 1/m year of death (mea-
sured from the issue date.)
Consider the combination of a F.I.B. and an mthly temporary annuity of term n years, payable in
(m)
arrear. It is clear that their total present value is Ban no matter when (x) dies. Hence the M.P.V.
of the family income benefit is ³ ´
(m)
B a(m)
n − a x:n (5.8.4)
Case 3. As in case 2, but with payments beginning immediately on death.
1
As payments are received on average 2m year earlier than in case 2, the M.P.V. is approximately
1
³ ´
(m)
B(1 + i) 2m a(m) n − a x:n (5.8.5)
Premiums. These are found by an equation of value in the usual way. There is, however, a
danger that, if premiums are payable for the full n-year term, the policy may have a negative reserve
at the later durations (see later discussion of reserves.) This means that the policyholder may be
able to lapse the contract leaving the office with a loss, and possibly obtain the same benefits more
cheaply by effecting a new policy. In practice, the effect of expenses and other factors may make this
possibility hardly profitable, especially if the F.I.B. is part of a more general assurance contract, as
in Example 5.8.1 below. The possibility of lapse option may be avoided by making the premium -
paying term shorter than the benefit term.
90 CHAPTER 5. PREMIUMS

Example 5.8.1. Ten years ago a woman aged exactly 35 effected an assurance policy by level annual
premiums payable for a maximum of 25 years. The policy provided the following benefits:
(i) a whole life assurance benefit of £2, 000 payable at the end of the year of death,

(ii) a family income benefit of term 25 years, with payments of £300 per month, beginning im-
mediately on death, if this occurs within 25 years. The final payment is made in the month
ending 25 years after the issue date.
Calculate the annual premium on the basis given below:

A1967-70
6% interest
expenses are 3% of all premiums.

Solution. Let P be the annual premium.


1
³ ´
(12)
0.97P ä35:25 = 2000A25 + 3600(1 + i) 24 a(12)
25
− a 35: 25
1
2000 × 0.12187 + 3600(1.06) 24 (13.1312 − 12.8527)
P =
0.97 × 13.282
= £96.93

Mortgage protection policies are similar to those for family income benefits, although the
death benefits are payable in one sum rather than in instalments over balance of the term. The
similarity arises from the fact that the loan outstanding after the tth payment has been made is
equal to the value of the future loan instalments (see Mc Cutcheon and Scott, “An Introduction to
the Mathematics of Finance”, Table 3.8.1).
5.9. EXERCISES 91

Exercises

5.1 A life aged 40 effects a 25-year without profits endowment assurance policy with a sum assured
of £50,000 (payable at the end of the year of death or on survival to the end of the term). Level
premiums are payable annually in advance throughout the term of the policy or until earlier
death of the life assured. Calculate the level premium, P , using the following premium basis
Mortality: A1967-70 Ultimate;
Interest: 6% p.a.
Expenses: none

5.2 An office issues a large number of 25-year without-profit endowment assurances on lives aged
exactly 40. Level annual premiums are payable throughout the term, and the sum assured of
each policy is £10,000, payable at the end of the year of death or on survival to end of the
term. The office’s premium basis is:
A1967-1970 ultimate;
4% p.a. interest;
expenses are 5% of each annual premium including the first, with additional initial ex-
penses of 1% of the sum assured.

Calculate the annual premium for each policy.

5.3 A 5-year temporary assurance, issued to a woman aged 55, has a sum assured of £50,000
in the first year, reducing by £10,000 each year. The sum assured is payable at the end of the
year of death. Level premiums, limited to at most 3 years’ payments, are payable annually in
advance. Calculate the annual premium.
Basis:
A1967-1970 select mortality
4% p.a. interest
expenses are 10% of all office premiums

5.4 A life office sells immediate annuities, using English Life Table No. 12 - Males, 4% p.a. interest
with no expenses as the premium basis. Assuming that the mortality of annuitants does follow
this table, that investments will earn 4% per annum, and that expenses are negligible, find the
probability that the office will make a profit on the sale of an annuity payable continuously to
a life aged 55.
5.5 (i) Let g(T ) be the present value of the profit to the life office, at the issue date, in respect
of an n-year without profits endowment assurance to (x) with sum assured (payable
immediately on death if this occurs within n years) and premium P per annum, payable
continuously for the term of the policy. Expenses are ignored in all calculations.
(a) Write down an expression for g(T ).
(b) Derive expressions for
(1) the mean, and
(2) the variance of g(T ).
(c) For what value of P is the mean of g(T ) equal to zero?
(ii) An office issues a block of 400 without profits endowment assurances, each for a term of
25 years, to lives aged exactly 35. The sum assured under each policy is £10, 000 and the
premium is £260 per annum, payable continuously during the term. The sum assured is
payable immediately on death, if death occurs within the term of the policy.
92 CHAPTER 5. PREMIUMS

Assuming that the office will earn 4% interest per annum, that the future lifetime of the
lives may be described statistically in terms of the A1967-70 ultimate table, and that
expenses may be ignored, find
(a) the mean present value of the profit to the office on the block of policies, and
(b) the standard deviation of the present value of this profit.
£ ¤
Ā35:25 = 0.15646 on A1967-70 ultimate 8.16%

5.6 A life office issued a certain policy to a life aged 40. The benefits under this contract are as
follows:
On death before age 60: an immediate lump sum of £1, 000

On survival to age 60: an annuity of £500 p.a., payable continuously for the
remaining lifetime of the policyholder.

Level annual premiums are payable continuously until age 60 or earlier death.
Premiums are calculated according to the following basis:
Mortality: English Life Table No.12-Males
Interest: 4% p.a.
Expenses: Nil
Calculate the annual premium.
5.10. SOLUTIONS 93

Solutions

5.1 Equation of value is:


M.P.V. of premiums = M.P.V. of benefits
P ä40:25 = 50000A40:25 at A67-70 ultimate, 6% p.a.interest
=⇒ P × 13.081 = 50000 × 0.25955 (see page 67 of tables)
=⇒ P = £992.09

5.2 Let the annual premium be P 0 . We have


100 + 10000A40:25 = 0.95P 0 ä40:25
10000A40:25 + 100
∴P =
0.95ä40:25
= £276.70

5.3 Let the annual premium be P . M.P.V. of assurance benefits is (from first principles)
10000 © ª 1 1
5C[55] + 4C[55]+1 + 3C57 + 2C58 + C59 [= 60000A[55] :5 − 10000(IA)[55]:5 ]
D[55]

M.P.V. of premiums less expenses is


[N[55] − N58 ]
0.9P ä[55]:3 = 0.9P
D[55]

10000{5C[55] + 4C[55]+1 + 3C57 + 2C58 + C59 }


∴P =
0.9[N[55] − N58 ]
10000 × 371.511
= = £395.06
0.9 × 10449

5.4 Consider £1 p.a. of annuity. The purchase price is ā55 and the office will make a profit if death
occurs before time t, where
ā55 = āt at 4%
That is,
1 − vt log [1 − δā55 ]
= ā55 , so t=
δ log v
i.e.
log Ā55 log Ā55
t= = = 16.99
log v −δ
The probability of making a profit is thus
l55+t l71.99
t q55 = 1 − =1− = 0.434 (or 43.4%)
l55 l55

(by interpolation)

(
P āT − Sv T if T < n
5.5 (i) (a) g(T ) =
P ān − Sv n if T ≥ n
94 CHAPTER 5. PREMIUMS

(b) (1) E[g(T )] = P āx:n − S Āx:n


· ¸
1 − h(T )
(2) Write g(T ) = P − Sh(T )
δ
(
vT if T < n
where h(T ) =
vn if T ≥ n
µ ¶
P P
= − + S h(T )
δ δ
µ ¶2
P
∴ Var[g(T )] = +S Var[h(T )]
δ
µ ¶2 h
P ¡ ¢2 i
= +S Ā∗x:n − Āx:n
δ
where ∗ indicates the rate of interest 2i + i2 p.a.
S Āx:n
(c) By (b)(1), E[g(T )] = 0 when P =
āx:n
(ii) (a) First consider 1 policy.
E[g(T )] = 260ā35:25 − 10, 000Ā35:25
= 260 ×15.542 − 10, 000[ 1 − δā35:25 ]
| {z } | {z }
using 0.39043
1
ā35:25 + ä35 − 2

D60
− D35 (ä60 − 12 )

= 136.60
∴ For 400 policies, M.P.V. of profit = £54, 640
(b) For 1 policy, variance of p.v. of profit is
µ ¶2
260 £ 8.16% ¤
+ 10, 000 Ā35:25 − ( Ā35:25 )2
δ
| {z } | {z }

0.15646 0.39043
= (1.662915 × 104 )2 × 0.004024415 = 1, 112, 866

∴ s.d. for 400 policies = 20 × 1.662915 × 104 × 0.004024415
= £21, 098

5.6 Let P = annual premium.


1 N̄60
P ā40:20 = 1, 000Ā40 :20 + 500 D40
95.84 + 2, 040.81
∴ P = = £161.12
13.261
Chapter 6

RESERVES

6.1 What are reserves?


Reserves, or policy values, are sums of money held by financial institutions such as life offices
and pension funds to cover the difference between the present value of future liabilities (including
expenses) and the present value of future premiums or contributions. Alternatively, the reserve of
a contract may be considered as an accumulation of past premiums less expenses and the cost of
death claims (and other benefits). Reserves are required for various purposes, e.g.
(1) to pay surrender values (or transfer values in a pension fund);
(2) to work out the revised premium or sum assured if a policy is altered or converted to another
type;
(3) for inclusion in statutory returns to the Department of Trade and Industry (or other supervisory
bodies) for the purpose of demonstrating the office’s solvency;
(4) for internal office calculations to decide the bonus rates of with-profits policies, the distribution
of profits to shareholders, etc.

The bases used to calculate “reserves” for each of these purposes may be different, and some
practical considerations are beyond our present scope.

6.2 Prospective reserves


Consider a life assurance policy issued t years ago to a life then aged x. Define the NET LIABILITY
or PROSPECTIVE LOSS of the office in respect of this policy to be the random variable

L = present value of future benefits


+ present value of future expenses
− present value of future premiums (6.2.1)

The reserve or policy value of the contract (calculated prospectively, i.e. by reference to future
cashflows) is defined as

tV = E(L) (6.2.2)
= M.P.V. of future benefits and expenses
− M.P.V. of future premiums (6.2.3)

95
96 CHAPTER 6. RESERVES

If expenses may be ignored, we have

tV = M.P.V of future benefits


− M.P.V of future premiums (6.2.4)

The mortality, interest and expense assumptions used to evaluate t V are known as the reserving
basis. This may or may not agree with the premium basis. If these bases agree (or are assumed
to agree) and there are no expenses, we obtain net premium reserves, which we shall consider
in the next section. By convention, the reserve t V is calculated just before receipt of any premium
then due: the reserve just after payment of this premium is

tV + P 00 − e (6.2.5)

where P 00 is the premium paid at duration t years and e is the expense then incurred.

Are negative reserves allowable?

Certain formulae may give negative values of t V, at least for some policies and at early durations t.
A negative reserve should not normally be held because the life office is thereby treating the contract
as an asset: if the policy is discontinued there is no way in which the policyholder can be made to
pay money to the office! Similarly, the reserves stated in Statutory Returns should not be negative,
although negative reserves may be permissible in certain internal office calculations.
The general rule is therefore that, if a formula gives a negative value of t V, this should be replaced by
zero. Policies should in general be designed so that negative reserves do not arise (cf. the discussion
of family income benefit policies in Section 5.8).

6.3 Net premium reserves


These are reserves calculated without allowance for expenses, and on the assumption that the pre-
mium and the reserve bases agree. (In some cases, the actual premium basis is different, and the
premiums valued are calculated on the reserve basis). A net premium reserve basis is unambiguously
specified by
(i) a mortality table, and

(ii) a rate of interest.

By formula (6.2.4),
tV = net premium reserve

= M.P.V. of future benefits - M.P.V. of future premiums (6.3.1)


where the valuation assurance and annuity factors are calculated on the specified mortality and
interest basis. The premiums valued are also calculated on this basis.

Example 6.3.1. Consider a whole life policy with sum assured £1 without profits, payable imme-
diately on the death of (x). The policy was issued t years ago by level annual premiums payable
continuously throughout life. Find a formula for the net premium reserve t V (on a given mortality
and interest basis).
6.3. NET PREMIUM RESERVES 97

Solution. Let L be the net liability (a random variable). We have

L = v U − P āU
where U = future lifetime of (x + t), and P = P̄ (Āx ).
∴ t V = E(L) = Āx+t − P̄ (Āx )āx+t .

Notation. The International Notation for net premium reserves is similar to that for premiums,
with the addition of “t” to indicate the duration. If premiums are limited to (say) h years of payment,
the symbol h is placed above the duration t, as shown in Table 6.3.1. The general symbol t V may
be used for any sum assured and any reserve, whether net premium or not, but t Vx , t V̄ (Āx ), etc.,
refer to net premium reserves for a policy with a sum assured of £1. If select mortality tables are
used, we write t V[x] , etc.
CONVENTIONS

(1) The symbols t Vx , etc., refer to the reserves just before payment of the premium due at time
t (if a premium is then payable.) The net premium reserve just after receipt of this premium
is of course t Vx + Px , etc. Similar considerations apply if premiums are payable half-yearly,
monthly, etc.
(2) By formula (6.3.1) with t = 0, we have 0 V = 0 for all net premium reserves. In the case of
n-year endowment assurances or pure endowments, it is not always clear whether to take n V
as zero or the sum assured (S, say), i.e. whether to assume that the sum assured has or has
not already been paid. It appears that the convention n V = 0 is used in profit testing but not
elsewhere.

Type of
Notation Prospective Formula for Reserve
Policy
Whole Life
t V̄(Āx ) Āx+t − P̄ (Āx )āx+t
Assurance
n-year Term 1 1 1
t V̄(Āx:n ) Āx+t :n−t − P̄ (Āx:n )āx+t:n−t
Assurance
n-year Endowment
t V̄(Āx:n ) Āx+t:n−t − P̄ (Āx:n )āx+t:n−t
Assurance
h-Payment Years Āx+t − h P̄(Āx )āx+t:h−t t<h
h
Whole Life t V̄ (Āx )
Āx+t t≥h
Assurance
h-Payment Years
n-year h Āx+t:n−t − h P̄(Āx:n )āx+t:h−t t<h
t V̄ (Āx:n )
Endowment Āx+t:n−t t≥h
Assurance
n-year Pure 1
1
t V̄(Ax:n ) Ax+t:n−t − P̄ (Ax:n1 )āx+t:n−t
Endowment

Table 6.3.1: Net Premium Reserves (premiums payable continuously)


98 CHAPTER 6. RESERVES

Type of
Symbol Prospective Policy Value Retrospective Policy Value
Policy

Whole Life Dx £ ¤
t Vx Ax+t − Px äx+t Px äx:t − Ax1:t
Assurance Dx+t

Endowment Dx £ ¤
t Vx:n Ax+t:n−t − Px:n äx+t:n−t Px:n äx:t − Ax1:t
Assurance Dx+t

Temporary 1 Dx £ 1 ¤
1
t V x:n Ax+t 1
:n−t − P x:n äx+t:n−t P x:n äx:t − Ax1:t
Assurance Dx+t

Pure 1 1 Dx
t V x:n Ax+t:n−t − P x:n1 äx+t:n−t [P 1 ä ]
Endowment Dx+t x:n x:t

Table 6.3.2: Net Premium Reserves (premiums payable annually)

Example 6.3.2.

(i) What is the International Notation for the reserve of Example 6.3.1 ?

(ii) Express this reserve in terms of annuity functions.

Solution.

(i) t V̄ (Āx )

(ii)

t V̄ (Āx ) = Āx+t − P̄ (Āx )āx+t


1
= 1 − δāx+t − ( − δ)āx+t using conversion relationships
āx
āx+t
=1− (6.3.2)
āx

Reserves at non-integer durations


Let us consider (for example) a whole life non-profit policy with sum assured £1 payable at the end
of the year of death, effected by (x) by level annual premiums, Px . The net premium reserve at
integer duration t is
t Vx just before premium is paid
V
t x + Px just after premium is paid

Now consider duration r + k, integer r, 0 < k < 1. The reserve r+k Vx is estimated by linear
interpolation between r Vx + Px and r+1 Vx , i.e.

r+k Vx ; (1 − k)(r Vx + Px ) + k r+1 Vx (0 < k < 1) (6.3.3)

[The actual value is

v 1−k [1−k qx+r+k + (1 − 1−k qx+r+k )Ax+r+1 ] − Px v 1−k (1 − 1−k qx+r+k )äx+r+1
6.4. RETROSPECTIVE RESERVES 99

which is complicated to evaluate.] Similar formulae apply for other classes of business with premi-
ums payable annually in advance.

Reserves when premiums are payable mthly


Suppose that premiums are payable mthly in advance (e.g. m=12, which corresponds to monthly
premiums, which are very common in practice.) To simplify matters let us suppose that the sum
assured is payable immediately on death: this avoids the calculation of awkward assurance factors
at non-integer ages. Consider a policy providing £1,000 immediately on the death of (x), effected
by mthly premiums during the lifetime of (x). The annual premium, payable mthly in advance, is
1
P (m) (Āx ) = P , say, and the reserve at duration t (where t is an integer multiple of m ) is

(m)
tV = 1, 000Āx+t − P äx+t just before payment of the premium then due,

or

P
tV + just after payment of this premium.
m
k 1
The reserve at duration t = r + m (where r is an integer multiple of m and 0 < k < 1) is usually
P
estimated by linear interpolation between r V + m and 1 V
r+ m .
NOTE One may use standard symbols, e.g.

(12) (12)
tV (Āx ) = Āx+t − P (12) (Āx )äx+t
(4) 1 (4) (4)
tV (Āx1:10 ) = Āx+t 1
:10−t − P (Āx:10 )äx+t:10−t
1 1
if t is an integer multiple of 12 and 4 respectively, but these are seldom employed.

6.4 Retrospective reserves


So far we have calculated the reserve of a policy by referring to future cashflows, assuming that
the contract will not be surrendered. In practice, the policyholder may wish to surrender the
contract, and in that event will, at least in the early years of the policy, probably expect a surrender
value related to the accumulation of his premiums less expenses and the cost of life assurance
cover. Such a surrender value is related to the retrospective reserve of the contract, which is
obtained by accumulating the premiums less expenses and the cost of benefits, of a hypothetical large
group of identical policies whose mortality follows the office’s tables exactly (i.e. without random
fluctuations), and then dividing the hypothetical funds among the survivors.
Let us again assume that there are no expenses and that the premium and reserve bases agree.
Consider, for example, a whole-life policy, with sum assured £1 payable at the end of the year
of death, issued t years ago by level annual premiums to a life then aged x. The (prospective)
net premium reserve (on a specified mortality and interest basis, e.g. A1967-70 ultimate, 6% p.a.
interest) just before payment of the premium now due is

t Vx = Ax+t − Px äx+t

The corresponding retrospective reserve is found by accumulating the funds of (say) lx identical
policies until time t, and sharing the money out among the survivors. Since mortality is assumed to
follow the table exactly, there are dx deaths in the first policy year, dx+1 in the second policy year,
and so on, with lx+t survivors at time t. Interest is assumed to be earned at the rate i p.a. used in
100 CHAPTER 6. RESERVES

the premium and the reserving bases, so the accumulated funds at time t are

[lx Px (1 + i)t + lx+1 Px (1 + i)t−1 + · · · + lx+t−1 Px (1 + i)]


− [dx (1 + i)t−1 + dx+1 (1 + i)t−2 + · · · + dx+t−1 ]
=lx (1 + i)t [Px äx:t − Ax1:t ]

On division by lx+t (the assumed number of survivors at policy duration t), we find that
retrospective
Dx
reserve at = [Px äx:t − Ax1:t ]
Dx+t
duration t years
More generally, we have
 
M.P.V. (at issue date) of
retrospective
Dx premiums, minus M.P.V.
reserve at =   (6.4.1)
duration t years Dx+t of benefits and expenses, in
the first t years
If there are no expenses, we have
 
retrospective
Dx M.P.V. (at issue date) of
reserve at = premiums, less benefits in (6.4.2)
Dx+t
duration t years first t years
Some examples of formulae for retrospective reserves are given in Table 6.3.2 above.

Theorem 6.4.1. If the premium and the reserving bases agree, the prospective and the retrospective
reserves of a given policy are equal.
Proof. We illustrate the argument by means of the whole life policy discussed earlier in this section.
The retrospective and prospective reserves at policy duration t are
Dx
VR = [Px äx:t − Ax1:t ]
Dx+t

and

VP = Ax+t − Px äx+t

respectively. Hence

Dx+t
(VR − VP ) = Px äx:t − Ax1:t
Dx
Dx+t Dx+t
− Ax+t + Px äx+t
Dx Dx
= Px äx − Ax

(using the facts that äx = äx:t + t |äx and Ax = Ax1:t + t |Ax .)

But Px äx − Ax = 0, by the equation of value for the level annual premium, so we have proved the
desired result.
This argument may easily be extended to cover other policies, including situations in which there
are expenses.
6.4. RETROSPECTIVE RESERVES 101

It follows that, if the premium and reserving bases agree (or are assumed to agree in reserve calcu-
lations), one need not specify whether a prospective or retrospective reserve is required. In practice,
however, the prospective method is more often used.

Example 6.4.1. Give formulae for the following net premium reserves in terms of other monetary
functions:

(i) t Vx:n ,

(ii) t V̄ (Āx:n ),

by (a) the prospective method, and

(b) the retrospective method.

Solution.

(i) (a) Ax+t:n−t − Px:n äx+t:n−t


Dx
(b) {Px:n äx:t − Ax1:t }
Dx+t

(ii) (a) Āx+t:n−t − P̄ (Āx:n )āx+t:n−t


Dx
(b) {P̄ (Āx:n )āx:t − Āx1:t }
Dx+t

Example 6.4.2. Show that

äx+t:n−t
(i) t Vx:n =1− (6.4.3)
äx:n

āx+t:n−t
(ii) t V̄ (Āx:n ) = 1 − (6.4.4)
āx:n

Solution.

(i)

t Vx:n = Ax+t:n−t − Px:n äx+t:n−t


1
= 1 − däx+t:n−t − ( − d)äx+t:n−t
äx:n

= 1 − x+t:n−t
äx:n
102 CHAPTER 6. RESERVES

(ii)

t V̄ (Āx:n ) = Āx+t:n−t − P̄ (Āx:n )āx+t:n−t


1
= 1 − δāx+t:n−t − ( − δ)āx+t:n−t
āx:n

= 1 − x+t:n−t
āx:n

6.5 Gross premium valuations and asset shares.


We now allow for the possibility of expenses in the premium and reserving bases, which are no
longer assumed to be the same. The reserves obtained by the formula (6.2.3) are called prospective
gross premium reserves, whereas those obtained by a retrospective accumulation of premiums less
expenses and the cost of life assurance benefits are called retrospective gross premium reserves. (In
practice, however, the phrase “gross premium reserve” usually refers to prospective reserves only.) If
the premium and reserve bases do not agree, the two methods will normally give different results. An
important feature of gross premium reserves are the facts that the prospective reserve at inception
is not normally equal to zero, and the retrospective gross premium reserve of an n-year policy just
after it matures is not generally equal to zero.

We begin by considering prospective gross premium reserves. The premium basis (or the premi-
ums themselves) must be specified, together with the reserving basis, which gives

(i) a mortality table,

(ii) an interest rate, and

(iii) the allowance for future expenses.

If the allowance for future expenses is a proportion k of future premiums (including any due
now), the formula for the prospective reserve is

tV = M.P.V. of future benefits − M.P.V. of future premiums (6.5.1)


multiplied by the
factor (1 − k)

Example 6.5.1. Consider the whole life policy discussed in section 6.4 above, and suppose that

(i) the premium basis is A1967-70 ultimate, 6% p.a. interest, with no allowance for expenses; and

(ii) reserves are to be calculated prospectively by the gross premium method on the following
basis:

mortality: A1967-70 ultimate;


interest: 4% p.a.;
expenses: 5% of all future premiums.

Find a formula for the reserve at duration t years (just before receipt of the premium then due.)
6.5. GROSS PREMIUM VALUATIONS AND ASSET SHARES. 103

Solution
The annual premium is

P 0 = Px on A1967-70 ultimate, 6% p.a.

so the formula for the reserve is


tV = Ax+t − 0.95P 0 äx+t
where the factors Ax+t and äx+t are on A1967-70 ultimate, 4% interest.
If a retrospective gross premium reserve is required, one must specify the allowance for past expenses.
If, in the policy of example 6.5.1, one allowed for expenses of 10% of all past premiums, the formula
for the retrospective gross premium reserve would be

Dx £ ¤
tV = 0.9P 0 äx:t − Ax1:t
Dx+t

where the annuity and assurance factors, and Dx /Dx+t , are on A1967-70 ultimate at 4% p.a. inter-
est (or whatever mortality and interest basis is to be used.)

Asset shares
The asset share of a life assurance policy is a retrospective gross premium reserve calculated on the
basis of the mortality, interest and expenses actually experienced by the office. In order to illustrate
the calculations, let us consider an n-year policy issued t years ago to (x), and define

St = sum assured in year t (payable at end of year of death);


Pt = premium payable at the start of policy year t;
et = actual expenses incurred at the start of policy year t;
it = rate of interest earned by the office in policy year t;
q[x]+t = observed mortality rate among lives aged x + t who
entered assurance at age x.

Let t V denote the asset share at duration t of the policy, before payment of the premium then due.
We clearly have 0 V = 0, and the following relation holds:

t+1 V = (t V + Pt+1 − et+1 )(1 + it+1 ) − death cost, Dt+1 0≤t≤n−1 (6.5.2)

To calculate the “death cost”, we must consider the given policy’s share of the cost of making the
asset share (at the end of the policy year) up to the sum assured, St+1 , for those policies which
became claims in the year. Since the proportion of policies in force at the start of the year which
became claims was q[x]+t , the death cost is

Dt+1 =q[x]+t (St+1 − t+1 V) (6.5.3)

(which is negative if St+1 < t+1 V).

Notes

1. As will be stated in the next chapter, the quantity St+1 − t+1 V is called the death strain at risk
of the policy in year t + 1.

2. By substituting formula (6.5.3) in formula (6.5.2), we may obtain an expression for t+1 V in terms
of t V and other known quantities. In practice, however, one may perhaps replace Dt+1 by a
suitable approximate value and use formula (6.5.2) directly.
104 CHAPTER 6. RESERVES

3. The asset share at policy duration n years may of course differ from the maturity value, leading
to a profit or loss to the office at that time.

4. In the unlikely event that mortality , interest and expenses follow a certain valuation basis
exactly, the asset share of a policy will equal the retrospective gross premium reserve on that
basis.

5. Asset shares are particularly important for with profit policies, as their surrender and maturity
values are not fixed in advance and the asset share gives one method of deciding “fair” surrender
and maturity values. A number of other considerations are, however, also involved, and we shall
not pursue the discussion of asset shares.

Net premium versus gross premium reserves


A full description of the advantages and disadvantages of the various approaches is outside the scope
of this course, but we make some remarks.
It would at first seem obvious that a prospective gross premium valuation should be used, as
this represents the mean of the present value of the net liability. But this assumes that the policy
will not be surrendered: the policyholder may do this at any time, and (at least in the early years
of the contract) will expect a surrender value related to the accumulation of his past premiums
less expenses, i.e. a retrospective gross premium reserve. This suggests that the office should hold
reserves equal to the greater of

(i) a prospective premium reserve, and

(ii) a retrospective gross premium reserve (or asset share),

the bases being perhaps different (reflecting expected future conditions and actual past conditions
respectively.) The position is further complicated by the possible existence of guaranteed surrender
values and the difficulty of estimating future conditions. In practice, a net premium valuation at
a low rate of interest may be used, either by choice or because this is required by the supervisory
authorities: such a basis (though apparently artificial, since the premium value is unrelated to
the actual premium being charged) may have the property that the corresponding reserves are
comparable with, or generally greater than, the more “scientific” reserves given above. Similar
arguments lead to the “net premium method” for with-profit policies (provided that terminal bonuses
are allowed for separately): cf. Section 6.9 below.
The reason for the use of a low rate of interest in net premium reserves is that, by Lidstone’s
theorem, net premium reserves normally increase as the rate of interest falls.
The supervisory authorities may be prepared to allow Zillmerisation (see Section 6.7 below) of
the reserves (subject to limits on I), but many offices prefer to publish unmodified net premium
reserves, in case Zillmerisation is seen as a sign of financial weakness.
The above discussion refers mainly to the calculation of reserves for solvency testing, but reserves
are also used for internal purposes: deciding upon bonus levels, alterations to policies, etc. For these
purposes the office may use other bases.
The office must also allow for random fluctuations from the mean numbers of deaths (especially
when there is a small number of policies with very large sums assured or death strains at risk) and
consider the exchange or reinsurance of large risks.

6.6 The variance of L


We return to the definition of the random variable L given by formula (6.2.1). Its mean is defined
as the reserve t V, but in certain problems we also require its variance, Var(L). How may this best
be calculated?
6.6. THE VARIANCE OF L 105

Let us consider a policy providing £S immediately on the death of a life aged x at issue, with
premiums of P p.a. payable continuously throughout life. The random variable L is defined as
L = prospective loss on contract (at duration t)
= Sv U − P āU (6.6.1)
where U is the future lifetime of the policyholder, who is now aged x + t, and expenses are ignored.
Now we cannot use the formula
Var(L) = Var(Sv U ) + Var(P āU ) wrong!
(because Sv U and P āU are not independent), so we express L in terms of v U . That is,
µ ¶
U 1 − vU
L = Sv − P
δ
µ ¶
P P
= S+ vU −
δ δ
from which we obtain
µ ¶2
P
Var(L) = S+ Var(v U )
δ
µ ¶2 h
P ¡ ¢2 i
= S+ Ā∗x+t − Āx+t (6.6.2)
δ
where ∗ indicates a rate of interest of 2i + i2 p.a. This technique may also be used for certain other
types of contracts.
We now consider a number of life assurance policies, labelled from j = 1 to j = N . The total
reserves for the group of policies is
N
X
E[Lj ] = V1 + V2 + · · · + VN (6.6.3)
j=1

where Lj and Vj are, respectively, the net liability and the reserve of the jth policy. If we further
assume that the lives assured are independent,
 
XN XN
Var Lj  = Var(Lj ) (6.6.4)
j=1 j=1

where Var(Lj ) may be calculated by (say) formula (6.6.2). If we now suppose that all the policies
are identical (i.e. they are of the same sum assured, type and duration, and were issued at the same
date to N different lives all of the same age), the average of the net liabilities is
 
N
X
L̄ =  Lj  /N (6.6.5)
j=1

which has mean E(L) and variance Var(L)/N , where L is the net liability of a given policy. These
results are sometimes used in solvency calculations, etc. It is sometimes also assumed that N is so
N
X
large that the distribution of Lj or L̄ is approximately normal (by the central limit theorem
j=1

and related results), although this may only be accurate for very large values of N .
106 CHAPTER 6. RESERVES

6.7 Zillmerised reserves


Let us now suppose that the premium basis is the same as the reserving basis (or is assumed to be
the same for the purpose of calculating reserves.) As stated earlier, the retrospective and prospective
methods give the same results, but reserves may not be “net premium reserves” because of the effect
of expenses. In certain important classes of policy, however, the reserve is obtainable by means of
a simple adjustment to the net premium reserve: the corresponding reserve is called a “Zillmerised
reserve”, “Zillmerised net premium reserve” or sometimes “modified net premium reserve”.
Let us consider an n-year endowment assurance for a sum assured of £1 without profits, payable
at the end of the year of death (if this occurs within the term of the policy.) There are level premiums
of P 00 , payable annually in advance for n years or until earlier death of the policyholder, who was
aged x at the issue date. The reserve is required at duration t, just before payment of the premium
then due.
The premium and reserving basis includes the following allowances for expenses: expenses of e
on the payment of each premium, with additional initial expenses of I (so the total initial expense
is I + e).

Theorem 6.7.1. The reserve for the above policy is


Z
tV = (1 + I)t V − I (6.7.1)

where t V denotes the corresponding net premium reserve, i.e.

tV = t Vx:n =Ax+t:n−t − Px:n äx+t:n−t



=1 − x+t:n−t
äx:n
Proof. We first note that the annual premium P 00 is such that

P 00 äx:n = Ax:n + eäx:n + I


I + Ax:n
∴ P 00 − e =
äx:n
Now consider the reserve at duration t by the prospective method.
Z
tV = M.P.V. of future benefits and expenses less premiums
= Ax+t:n−t − (P 00 − e)äx+t:n−t
· ¸
I + Ax:n
= Ax+t:n−t − ȧx+t:n−t
äx:n
µ ¶
£ ¤ I
= Ax+t:n−t − Px:n äx+t:n−t − ȧx+t:n−t
äx:n

(where Px:n is the net annual premium)


· ¸
äx+t:n−t
= t Vx:n − I
äx:n

Now we use the fact that (by formula 6.4.3)


äx+t:n−t
t Vx:n =1−
äx:n
6.7. ZILLMERISED RESERVES 107

to give the formula

Z
tV = t Vx:n − I [1 − t Vx:n ]
= (1 + I)t Vx:n − I

as required.
Notes
1. The case of a whole life policy is similarly dealt with (put n = ∞ in the above formulae.)
2. A similar argument may be used to establish formula (6.7.1) if the sum assured is payable
immediately on death and premiums are payable continuously. In this case, t V = t V̄(Āx:n ).
3. Zillmer’s formula does not in general hold for other classes of policy, nor when the premium
due at time t has been paid. (It is, however, sometimes used in practice for all policies and all
durations.)
4. When the duration t is short (e.g. t = 1) Zillmer’s formula may give a negative reserve, which
should, as a rule, be replaced by zero, i.e. a policy should not be treated as an asset to the
office. Note that formula (6.7.1) gives
Z
0V = −I
which is correct if the additional initial expense I is thought of as being disbursed before the
first premium is received: if it is not, one should write
Z
0V =0

(as is always assumed in profit-testing: see later.)


5. If the sum assured is not £1, we naturally multiply formula (6.7.1) by the sum assured: for
example, the reserve per £1000 sum assured of a whole life without profits policy might be
quoted as
1020t Vx − 20
(corresponding to I= 2% ).

Example 6.7.1. Ten years ago life office issued a 20-year endowment assurance without profits to
(35). The sum assured is £10,000, payable at the end of the year of death (or on survival for 20
years), and premiums are payable annually in advance. The basis for premiums and reserves is:
A1967-70 ultimate;
6% p.a. interest;
expenses are 3% of all office premiums (including the first) with additional initial expenses of
1.5% of the sum assured.
Calculate
(i) the annual premium, and
(ii) the reserve,
(a) just before receipt of the premium now due, and
108 CHAPTER 6. RESERVES

(b) just after receipt of the premium now due.

Solution.

(i) The office premium is


· ¸
0010000 A35:20 + 0.015
P =
0.97 ä35:20
= £288.67

Note that tables at 6% interest are limited, so we use these formulae:

l55
ä35:20 = ä35 − v 20 ä55
l35
= 11.9969

and

A35:20 = 1 − dä35:20

(ii) One may use Zillmer’s formula in this case (or other methods). The reserve is

(a) 10000[1.01510 V35:20 − 0.015] = £3497


(b) £3497 + 0.97 × £288.76 = £3777 (see formula (6.2.5))

Example 6.7.2. Suppose that the policyholder of example 6.7.1 were to surrender his policy (before
paying the premium now due) and the office grants a surrender value equal to the reserve, as defined
in that question. What yield per annum would the policyholder have obtained?

Solution. The equation of value is

288.67s̈10 = 3497 at rate i=yield p.a.


∴ s̈10 = 12.11

We use the compound interest tables, i.e.

i s̈10
0.03 11.81
0.04 12.49

Interpolate:

i − 0.03 12.11 − 11.81


; = 0.44
0.04 − 0.03 12.49 − 11.81
∴ i ; 3.44%
6.8. FULL PRELIMINARY TERM RESERVES. 109

6.8 Full preliminary term reserves.


We again assume that the premium and reserving bases agree, and suppose that the reserve for a
whole life or endowment assurance policy is such that
1V =0 (6.8.1)
That is, the reserve just before payment of the second annual premium is zero. Let us again suppose
that the sum assured is £1, payable at the end of the year of death, and that expenses are as in the
previous section. We must have
1V = Ax+1:n−1 − (P 00 − e)äx+1:n−1 = 0.

Hence
Ax+1:n−1
P 00 − e = = Px+1:n−1
äx+1:n−1

Hence

tV = Ax+t:n−t − Px+1:n−1 äx+t:n−t


= A(x+1)+(t−1):(n−1)−(t−1) − Px+1:n−1 ä(x+1)+(t−1):(n−1)−(t−1)
= t−1 Vx+1:n−1
= the net premium reserve for the corresponding
policy on a life aged x + 1 at entry with term
n − 1 years at duration t − 1

This reserve is called the Full Preliminary Term reserve, which we shall write as t V F P T . We have
shown that
FPT
tV = t−1 Vx+1:n−1 (6.8.2)
In the case of a whole life policy, we may set n = ∞ to obtain
FPT
tV = t−1 Vx+1 (6.8.3)
If the sum assured is not £1, we naturally multiply these expressions by the sum assured. Similar
formulae hold if the sum assured is payable immediately on death, when premiums are payable
continuously or monthly, etc., and for certain other types of policy.

What level of initial expenses leads to a F.P.T. reserve? We observe that, by the retrospective
method,
FPT Dx £ 00 ¤
1V = (P − e)äx:1 − I − Ax1:1
Dx+1
=0
from which we obtain
P 00 = e + I + Ax1:1 (6.8.4)
This states that the first premium is exactly sufficient to pay the initial expenses, I + e, and the
cost of the first year’s life assurance cover, Ax1:1 .

The use of full preliminary term reserves was suggested by T. B. Sprague in 1870. Other actu-
aries (particularly in North America) have devised formulae for “modified full preliminary term
reserves”, in which the allowance for total initial expenses may be smaller than that of formula
(6.8.4), but we do not pursue this topic.
110 CHAPTER 6. RESERVES

6.9 Reserves for with-profits policies


We shall deal with “traditional” with profits policies only, and ignore terminal bonuses (although
these may in practice be very important.) Let us consider an n-year endowment assurance issued t
years ago to a life then aged x with basic sum assured S (payable, with attaching bonuses, at the
end of the year of death or on maturity.) We suppose that the policy was issued with level annual
premiums, and that the premium now due has not yet been paid. Reversionary bonuses are added
on the payment of each premium, and we denote the total bonus added to date by B (hence the
total death benefit in the year just ending is S + B.)
The principal methods used for calculating reserves for with profits policies are:
(1) the net premium method;
(2) the bonus reserve (or gross premium) method; and
(3) the asset share method.

(1) The net premium method. The reserve is taken to be the net premium reserve for corre-
sponding non profit policy, plus the mean present value of the bonuses already declared. In the
case of the endowment assurance policy discussed above, the reserve is given by the formulae:
WP
tV = St Vx:n + BAx+t:n−t (6.9.1)
= (S + B)Ax+t:n−t − SPx:n äx+t:n−t (6.9.2)
where all the actuarial functions are calculated on the given mortality and interest basis.

The rationale of this method is that the additional premiums for a with profits policy (relative
to the corresponding without profits policy) are considered to have “earned” the bonus B so far
declared, so an additional reserve is required to cover the value of these bonuses.

If the sum assured is payable immediately on death, we have


WP
tV = (S + B)Āx+t:n−t − P äx+t:n−t (6.9.3)
where P is the annual premium for the corresponding non-profit contract, i.e.
S Āx:n
P = = SP (Āx:n ) (6.9.4)
äx:n
Similar formulae may be given for whole life policies and for policies with monthly premiums,
etc.

Example 6.9.1. On 1st January 1993 a life office issued a with-profit assurance endowment
policy with a term of 10 years to each of 100 male lives then aged exactly 50. The basic sum as-
sured under each contract was £20, 000, and the basic sum assured, plus any bonuses, is payable
on maturity or immediately on earlier death. Level annual premiums are payable in advance
throughout the term. Among the group of policies there were 2 deaths during 1993 and 4 deaths
during 1994. There were no lapses or surrenders during 1993 or 1994. For this group of policies,
the office has declared a compound reversionary bonus of 3 43 % per annum vesting in advance on
each 1st January from outset. The office values the policies by a net premium method, using
A1967-70 ultimate mortality and interest at 3%p.a.

Calculate the total reserves which the office held for the group of policies on 31st December 1994.
6.9. RESERVES FOR WITH-PROFITS POLICIES 111

Solution. We require 94×reserve of one policy. Using the net premium method for with profits
policies,
reserve = S[2 V(Ā50:10 )] + B Ā52:8
per policy

where S = 20, 000, B = 20, 000[(1.0375)2 − 1] (i.e. bonuses already vesting)


i.e.

reserve = 20, 000(1.0375)2 Ā52:8 − P ä52:8


per policy
where P = net premium for corresponding non-profit policy
Ā50:10
= 20, 000 = 1, 760.94
ä50:10

[where we have used


1
1 1
Ā50:10 + (A50 :10 )(1.03) 2 + A50:10
1
= (1.03) 2 (A50:10 − v 10 10 p50 ) + v 10 10 p50
= 0.068 + 0.68419 = 0.75219 ]

∴ Reserves needed = 94 × [21, 528.125 Ā52:8 − 1, 760.94ä52:8 ]


| {z }

1
= (1.03) 2 (A52:8 − v 8 8 p52 ) + v 8 8 p52
1
= (1.03) 2 (0.06142) + 0.73329

= 0.79562

= £443, 417

(2) The bonus reserve method. As the name suggests, reserves are calculated on the assumption
that reversionary bonuses will be declared at certain annual rates. The value of the benefits
is therefore calculated by the formulae given in Section 3.10 above, according to the system of
bonus declarations used by the office, and the reserve is then found by subtracting the value of
the office premiums less projected future expenses. If, in the case of the endowment assurance
policy discussed above, it is assumed that future bonuses will be at the rate b per annum on the
compound system and a proportion k of future premiums will be absorbed in expenses, a formula
for the reserve is
WP
tV = (S + B)A∗x+t:n−t − (1 − k)P 00 äx+t:n−t (6.9.5)
where ∗ indicates the rate of interest (i − b)/(1 + b). Similar formulae may be derived for other
policies.

Since the premium valued is the actual (or office) premium for the policy, this method is some-
times referred to as the gross premium method for with profits policies.

As might be expected, the reserves calculated by the bonus reserve method depend greatly on the
assured future levels of bonus, and in the early years of a contract the method may give reserves
which are quite unrelated to the accumulation of premiums.
112 CHAPTER 6. RESERVES

(3) The asset share method. This has been described (for non-profit policies) in Section 6.5
above; the formulae are almost unchanged for with profit policies, the only difference being the
fact that the sum assured in each policy year includes the vested bonuses. Since these only affect
the death cost, the reserves given by the asset share method do not depend greatly on past bonus
declarations.

A complete description of the advantages and disadvantages of these methods of calculating


reserves is beyond the scope of this book.
6.10. EXERCISES 113

Exercises

6.1 (i) Express t Vx in terms of Ax and Ax+t . Hence, or otherwise, find the values of n Vx and
n Vx+n , given that
1 − Ax+2n = Ax+2n − Ax+n = Ax+n − Ax

(ii) Calculate 1 V40 given that P40 = .01536, p40 = .99647 and i = .05.

6.2 Consider an n-year pure endowment policy, issued to (x), with sum assured 1 and with annual
premiums payable throughout the duration of the policy. In the event of death within the n
years, all premiums paid will be returned without interest at the end of the year of death.
Obtain expressions for the reserve at duration t

(i) prospectively and


(ii) retrospectively.

Using commutation functions, show that your expressions are equal. (Assume that the premium
due at time t has not yet been paid.)

6.3 (Difficult)

Given that Px = 0.02, n Vx = 0.06, and P x:n1 = 0.25, find P x1:n .

6.4 Ten years ago a life office issued a 20-year endowment assurance without profits to (35). The
sum assured is £10000, payable at the end of the year of death (or on survival for 20 years),
and premiums are payable annually in advance. Using A1967-70 ultimate 6%, and ignoring
expenses, calculate

(i) the annual premium;


(ii) the reserve, assuming that the premium now due has been paid.

6.5 (Difficult) You are given:

(i) Px = 0.01212
(ii) 20 Px = 0.01508
(iii) P x:101 = 0.06942
(iv) 10 Vx = 0.11430

20
Calculate 10 Vx .

6.6 A whole life assurance with sum assured £100,000 payable at the end of the year of death was
purchased by a life aged 30. The policy has annual premiums payable throughout life.
The basis for calculating reserves for this policy is as follows:
net premium method: A1967-70 ultimate, 5% p.a. interest.
Estimate the policy value at duration 28 14 by interpolation.

6.7 Describe the following terms briefly:

(i) net premium reserves;


(ii) Zillmerised reserves;
114 CHAPTER 6. RESERVES

(iii) Full Preliminary Term reserves; and


(iv) gross premium reserves,
in each case giving suitable formulae in respect of a whole life assurance issued t years ago to
a life then aged x, with level premiums payable annually in advance throughout the term and
with a sum assured of £1 payable at the end of the year of death. Assume that the premium
now due has not yet been paid.
6.11. SOLUTIONS 115

Solutions

6.1 (i)
äx+t däx − däx+t Ax+t − Ax
t Vx =1− = =
äx däx 1 − Ax
The given information implies that
Ax+n = 2Ax+2n − 1

and
Ax = 2Ax+n − Ax+2n = 3Ax+2n − 2
Hence
Ax+n − Ax 1 − Ax+2n 1
n Vx = = =
1 − Ax 3 − 3Ax+2n 3
and
Ax+2n − Ax+n 1 − Ax+2n 1
n Vx+n = = =
1 − Ax+n 2 − 2Ax+2n 2
(ii) Retrospective method gives
l40 P40 (1 + i) − q40
1 V40 = [P40 − vq40 ](1 + i) = = 0.01264
l41 p40

6.2 The equation of value for the annual premium P is:

P äx:n = n Ex + P (Rx − Rx+n − nMx+n )/Dx

or
P (Nx − Nx+n ) = Dx+n + P (Rx − Rx+n − nMx+n ) (∗)
The expressions for the reserves are as follows:
Retrospective:
tV = [P (Nx − Nx+t ) − P (Rx − Rx+t − tMx+t )] /Dx+t (∗∗)
Prospective:
1 1
tV = n−t Ex+t + tP Ax+t :n−t + P (IA)x+t:n−t − P äx+t:n−t
= [Dx+n + tP (Mx+t − Mx+n )
+ P (Rx+t − Rx+n − [n − t]Mx+n )
− P (Nx+t − Nx+n )] /Dx+t (∗ ∗ ∗)

The expressions (∗∗) and (∗ ∗ ∗) are equal if and only if

P Nx − P Rx = Dx+n − tP Mx+n − P Rx+n − P [n − t]Mx+n + P Nx+n

i.e. if and only if


Dx+n + P (Rx − Rx+n − nMx+n ) = P (Nx − Nx+n )
which is true by (∗).

6.3 We know that


äx+n äx+n
n Vx =1− = 0.06, so = 0.94
äx äx
116 CHAPTER 6. RESERVES

1 1 äx − äx:n
− =
äx:n äx äx äx:n
n Ex äx+n
=
äx äx:n
= 0.94P x:n1
= 0.235
µ ¶ µ ¶
1 1 1 1
Px:n − Px = −d − −d = − = 0.235
äx:n äx äx:n äx
so

Px:n = 0.255
P x1:n = Px:n − P x:n1 = 0.005

6.4 (i) The annual premium is µ ¶


1
10000P35:20 = 10000 −d = £267.51
ä35:20
since
l55
ä35:20 = ä35 − v 20 ä55 = 11.997
l35
(ii) The reserve is µ ¶

1000010 V35:20 = 10000 1 − 45:10 = £3593
ä35:20
since
l55
ä45:10 = ä45 − v 10
ä55 = 7.687
l45
∴ Reserve after premium is paid = £3593 + £267.51
= £3861 (to nearest £1)

6.5 The prospective method does not work. By the retrospective method,
20
(reminder: 10 Vx refers to limited payments policy)

20 Dx
10 Vx − 10 Vx = [20 Px äx:10 − Px äx:10 ] (since benefits are the same
Dx+10 under both policies)
µ ¶
Dx äx:10
= (20 Px − Px )
Dx+10
P
20 x − P x
= = 0.04264
P x:101
∴ 20
10 Vx = 0.15694

6.6
28 V = 100, 000 28 V30 = 33, 101
29 V = 100, 000 29 V30 = 34, 754
Interpolate between 28 V + P and 29 V, where P is the annual premium, i.e. P = 100, 000
P30 = 724.00. This gives
3 1
28 14 V = (33, 101 + 724) + (34, 754)
4 4
= £34, 057
6.11. SOLUTIONS 117

6.7 (i) Net premium reserve = reserve calculated ignoring expenses and
assuming premium basis agrees with re-
serving basis
= t Vx = Ax+t − Px äx+t
in example given
(ii) Zillmerised reserve = (1 + I)t V − I per unit sum assured

(where t V = net prem. reserve, I = additional initial expenses per £1 sum assured)

= (1 + I)t Vx − I in example given.

(iii) F.P.T. reserve = net premium reserve for corresponding policy of


term n − 1, entry age x + 1, duration t − 1
= t−1 Vx+1:n−1 per unit sum assured for E.A.

= t−1 Vx+1 in example given

à !
³ ´ value of future
value of future
(iv) Gross prem. reserve = − gross premiums
benefits
less expenses.

= Ax+t − (1 − k)P 0 äx+t


in example, where
P 0 = office A.P. and
k = propn. of premiums allowed
for future expenses.
118 CHAPTER 6. RESERVES
Chapter 7

APPLICATIONS OF RESERVES

7.1 Surrender values


Most life assurance policies (except term assurances) provide for a sum of money known as a sur-
render value to be paid to the holder if the payment of premiums is discontinued and the contract
terminated. If the scale of surrender values is fixed in advance, the policy is said to have guaranteed
surrender values. For example, a whole life contract (with level annual premiums payable through-
out life) may be issued with guaranteed surrender values equal to the full preliminary term reserve,
i.e.
(SV )t = S.t−1 Vx+1 (7.1.1)
where x is the age at inception of the policy, t is the duration, (SV)t is the surrender value, S is the
sum assured, and t Vx is calculated on a specified mortality and interest basis. (It is assumed that
the premium due at time t is unpaid.) More usually, however, surrender values are not guaranteed,
although there may be guaranteed minimum surrender values, as is the case for Industrial Assurance
policies in the U.K. (i.e. those in which the premiums are collected regularly from the home of the
policyholder.)
Surrender values are normally related in a simple way to the reserve of the contract, but the
method of calculating the reserve for this purpose may be different from those used for other purposes,
e.g. statutory returns to the Department of Trade and Industry. Surrender value scales must take
several factors into account:

(i) the experience of the office since the policy was issued,

(ii) the expected future experience,

(iii) consistency between different classes of business,

(iv) competition between life offices (and possibly with a market for “second hand” life policies),
and

(v) guaranteed minimum surrender values (if applicable.)

A full discussion of all points involved is beyond our present scope. In some cases surrender values
are calculated from paid up policy values, which we discuss in the next section.
Transfer values in pension schemes correspond to surrender values of life assurance policies: a
transfer value is, however, payable only to another pension scheme and not to the scheme member
directly.

119
120 CHAPTER 7. APPLICATIONS OF RESERVES

7.2 Paid-up policy values


Instead of surrendering his policy, a policyholder who wishes to stop paying the premiums may ask
for the contract to continue in force with reduced benefits (payable when the original benefits were
to be paid.) The reduced benefit is called the paid-up sum assured (P.U.S.A.).
If the original sum assured on death differs from that on maturity, the paid-up benefits will normally
be in the same ratio.
The general symbol t W denotes the paid-up sum assured at duration t years, assuming that the
premium then due is unpaid. If expenses are ignored and all calculations are conducted on the same
mortality and interest basis, net premium paid-up policy values may be defined, using the notation
t Wx , t Wx:n , etc. For example,

t Wx = the paid-up sum assured at duration t in respect (7.2.1)


of a whole-life assurance issued to a life then aged
x by level annual premiums, with sum assured £1
payable at the end of the year of death
Suppose that, at the date of discontinuance of premiums of this whole life policy, the life office wishes
to make the mean net liability for the altered contract the same as that of the original contract.
This leads to the equation

t Vx = t Wx Ax+t (7.2.2)

from which we obtain


t Vx
t Wx = (7.2.3)
Ax+t
Similar calculations hold for other policies, e.g.
t Vx:n
t Wx:n = (7.2.4)
Ax+t:n−t
More generally, we obtain the following general formula for the P.U.S.A.:
tV
tW = (7.2.5)
A

where t V is the reserve or surrender value and A is an assurance factor, such as Ax+t or Āx+t:n−t .
(We note that no expenses have been allowed for at or after the date of alteration: if such an allowance
is desired, the formulae should be modified.)
In the important case of endowment assurances, the proportionate rule is sometimes used in
practice, i.e.
t
tW = S (7.2.6)
n
where S is the original sum assured, t is the total number of premiums actually paid and n is the
number originally payable.
Surrender values of endowment assurances (or other policies) are sometimes found from the P.U.S.A.
by the formula
(SV )t = (P.U.S.A.)A (7.2.7)
where A is the appropriate assurance factor at age x + t. In particular, if the proportionate rule for
endowment assurances is used, the surrender value at duration t years is sometimes taken as
t
SAx+t:n−t (7.2.8)
n
7.3. ALTERATIONS AND CONVERSIONS 121

if the sum assured is payable at the end of the year of death, if this occurs within the balance of the
term, n − t years, or
t
S Āx+t:n−t (7.2.9)
n
if the sum assured is payable immediately on death within this period.

Example 7.2.1.
(i) Four years ago a life then aged 35 effected a 25-year without profits endowment assurance by
annual premiums for a sum assured of £50,000 payable at the end of the year of death or on
survival for 25 years. He surrendered the policy just before paying the premium now due.
The life office calculated the premiums for this policy on the basis of A1967-70 select at 4%
interest with the following expense loadings:
2.5% of each premium, including the first, plus a further
initial expense of 1% of the sum assured.
The office also calculates reserves on this basis, and surrender values are equal to 95% of
reserves.
(a) Calculate the office annual premium.
(b) Calculate the surrender value of the policy.
(ii) Suppose that the policyholder of (i) had made his policy paid-up instead of surrendering it,
and that the office calculates paid-up values by the proportionate rule. Calculate the yield per
annum (to the nearest 0.1%) that he would have obtained on his entire transaction, assuming
that he survives to maturity date.

Solution.
(i) (a) Let A.P. be P 0 . We have
0.975P 0 ä[35]:25 = 50000(A[35]:25 + 0.01)
∴ P 0 = £1289.10

(b)
Reserve = 50000A39:21 − 0.975P 0 ä39:21
= £4790.00
∴ S.V. = 0.95 × reserve = £4550.50

(ii) PUSA=50000 × 4/25 = 8000. Solve


P 0 s̈4 = 8000v 21

i.e.
(1 + i)21 s̈4 = 6.20588
By trials and interpolation, i ; 1.9%.

7.3 Alterations and conversions


Life offices are frequently asked to alter a policy; for example, a policyholder with an endowment
assurance maturing at age 65 may request that the policy should be changed to an endowment
122 CHAPTER 7. APPLICATIONS OF RESERVES

assurance maturing at age 60. The policyholder might also request his policy to be converted to
another class of business: for example, from a whole life assurance to an endowment assurance.
The usual rule when carrying out these calculations is to equate the reserves before and after
conversion. If V1 and V2 denote the reserves before and after the conversion respectively, the rule
may be stated as follows:
V1 = V2 (7.3.1)

If there are expenses of C for the conversion itself, the formula becomes

V1 − C = V2 (7.3.2)

The bases used to calculate V1 and V2 may in practice be different, and V2 must be calculated
prospectively by the gross premium method.
To show that formulae (7.3.1) and (7.3.2) hold, we argue as follows. At the date of conversion,
the policyholder’s original policy is worth V1 on immediate surrender. (In practice, however, the
reserve V1 used in conversion calculations may exceed the surrender value, as the office is not losing
business on a conversion or alteration.) Imagine that V1 is applied as a special single premium
towards the new contract. The equation of value of the new contract is thus

V1 + M.P.V. of future premiums (including any due now)


= M.P.V. of future benefits + future expenses + C

Rearranging this equation gives:

V1 − C = M.P.V. of future benefits + expenses − premiums


= V2

The office then solves this equation for the unknown quantity (usually the new sum assured or
premium.) An example of the use of formula (7.3.1) has already been encountered in equation
(7.2.2) above.

Example 7.3.1. Ten years ago, a man now aged 40 effected a whole of life assurance for a sum
assured of £10000, payable at the end of the year of death, by level annual premiums. The premiums
were calculated using A1967-70 ultimate 4% and an allowance for expenses of 50% of the first year’s
premium and 5% of each subsequent premium.
Immediately before payment of the eleventh annual premium the man requests that the policy be
converted into an endowment assurance maturing on his sixtieth birthday, with the annual premium
remaining unaltered. Calculate his revised sum assured using a full preliminary term reserve (on
A1967-70 ultimate, 4% interest) for finding the reserve of the original policy, and
A1967-70 ultimate, 4% interest, expenses of 5% of premiums
for calculating the reserve of the new policy.

Solution. Annual premium is P 00 , where 0.95P 00 ä30 = 10000A30 + 0.45P . ∴ P 00 = £97.13.

V1 = Reserve of original policy = 10000 9 V31 = £949.85


| {z }
ä40
1−
ä31
7.3. ALTERATIONS AND CONVERSIONS 123

Equation of value at date of conversion is

reserve before conversion = (prosp.) reserve after conversion


∴ 949.85 = SA40:20 − 0.95 × 97.13 ä40:20
| {z }
allowing 5%
of premiums
for expenses
∴ S = £4717

Example 7.3.2. An office issued a ten-year endowment assurance to a man aged exactly 45. The
monthly premium was £25 during the first five years increasing to £50 thereafter. The sum assured,
payable immediately on death, was calculated using A1967-70 select 4%, allowing for expenses of
£1 per month with additional initial expenses of 2 12 % the sum assured.
After five years the man requested that the premium remain at £25 per month for the next five
years, with the death benefit staying unaltered. Calculate the reduced sum payable on survival using
the premium basis and allowing for a £30 alteration charge.

Solution. Let S =original sum assured.


D50 (12) h i
300ä(12)
[45] :10 + 300 ä 50 :5 = S 0.025 + Ā[45]:10 + 12ä(12)
[45] :10
D[45]
∴ S = £4871

D50
(using ä(12) (12)
[45] :10 = 8.154, ä 50 :5 = 4.489, = 0.8093, Ā[45]:10 = 0.6808 )
D[45]
Let S.A. on survival be reduced to S 0 . Equate reserves before and after conversion [allowing for
cost of conversion]:

4871Ā50:5 − (600 − 12)ä(12)


50 :5
− 30
D55
1
= 4871Ā50 :5 + S
0
− (300 − 12)ä(12)
50 :5
D50
D55
∴ (4871 − S 0 ) = 300ä(12)
50 :5
+ 30
D50
∴ S 0 = £3144

Example 7.3.3. In a certain country, the Universities operate the following superannuation scheme
for academic staff, of whom there are two grades, Lecturer and Professor. Lecturers retire at age 65
with a lump sum of £30, 000, while Professors retire at age 70 with a pension of £12, 000 per annum
payable monthly in advance. Staff pay for their benefits by means of level annual premiums, payable
annually in advance until retirement or earlier death. Premiums are returned without interest on
death before retirement, and withdrawals may be ignored. The employers do not contribute to the
scheme. Death benefits are paid at end of year of death. The benefits are provided by policies issued
by a life office which uses the following basis for all calculations:
A1967-70 ultimate
124 CHAPTER 7. APPLICATIONS OF RESERVES

4% per annum interest


expenses are ignored.
A Professor cannot be demoted to Lecturer.
(i) Calculate the annual premium payable by a Lecturer on entry to the scheme at 25.
(ii) Calculate the reserve, just before payment of the premium now due, for a Lecturer aged 45
with 20 years’ service.
(iii) The Lecturer of (ii) is now promoted to Professor. Calculate his or her new annual premium,
the first due at once.

Solution.
(i) Let the annual premium be P . The equation of value is
P ä25:40 = 30000A25:401 + P (IA)25
1
:40
=⇒ P = £274.44

(ii)
D65 1 1
V1 = 30000 + 20P A45 :20 + P (IA)45:20 − P ä45:20
D45
= £8536

(iii) Let the new premium be P 0 . We have,


V1 = V2
That is,
8536 = value of future benefits − future premiums for new contract
D70 (12) 1 0 1 0
= 12000 ä + 20P A45 :25 + P (IA)45:25 − P ä45:25
D45 70
=⇒ P 0 = £1525.87

7.4 The actual and expected death strains


Consider N identical policies on lives aged x at entry and now aged x + t, and assume that the
premiums now due are unpaid. The death strain at risk for a given contract in the coming policy
year is
S − t+1 V (7.4.1)
where S denotes the sum payable on death (assumed payable at the end of the policy year) and
t+1 V denotes the reserve at the end of the policy year. Since the life office may be considered to be
holding the reserve t+1 V for each policy in force at time t + 1 in an imaginary “bank account” for
the policyholder, the death strain at risk measures the extra cash required if he dies in policy year
t + 1. Let θx+t denote the number of deaths in the coming year. Note that (in the notation used for
mortality investigations)
N = Ex+t = the q-type exposed to risk at age x + t
(since each life is exposed to the risk of
death for the whole policy year)
7.5. MORTALITY PROFIT AND LOSS 125

Assuming independence of the lives we have

θx+t ∼ binomial(N, qx+t )

which has mean N qx+t .


Now consider the random variable

A.D.S. = actual = θx+t (S − t+1 V) (7.4.2)


death
strain

The mean of this variable (even without the assumption of independence of the lives) is

EDS = expected = N qx+t (S − t+1 V) (7.4.3)


death
strain

Note. Once the number of deaths θx+t , is known (at the end of the policy year), ADS ceases to be
a random variable: it is then a realisation of the variable.
The position may be generalised to cover cases in which policies considered are not identical,
provided that their premiums are due on the same date. The actual death strain is
X
ADS = (S − t+1 V) (7.4.4)
death claims
in policy
year t + 1

and the expected death strain is


X
EDS = qx+t (S − t+1 V) (7.4.5)
all policies in
force at the
start of policy
year t + 1

Note. The sum assured S may depend on the policy year, t + 1.

7.5 Mortality profit and loss


Consider a block of business in force at the start of policy year t + 1, the premiums now due being
unpaid, and suppose that the life office holds funds equal to the total reserves for this block of busi-
ness. Let us (i) ignore expenses, (ii) suppose that the premium and reserving bases agree, and (iii)
assume that interest will be earned in policy year t + 1 at the valuation rate, i p.a. The mortality
profit in policy year t + 1 is defined as the funds at the end of the year less the money needed to pay
death claims and to set up reserves for the survivors. Hence
X X X
mortality profit = (t V + P )(1 + i) − S− t+1 V (7.5.1)
all in force death survivors
at the claims
start of
year t + 1

Hence
126 CHAPTER 7. APPLICATIONS OF RESERVES

X X X
mortality profit = (t V + P )(1 + i) − (S − t+1 V) − t+1 V (7.5.2)
all in force deaths all in force
at the at the
start of start of
year year

Theorem 7.5.1. Under the present assumptions, the mortality profit to the life office in respect of
this group of policies, i.e. the difference between the actual funds at the end of the policy year and
the funds required to pay claims and set up reserves, is equal to E.D.S. minus A.D.S.; that is,
mortality profit in policy year t + 1 = E.D.S. − A.D.S. (7.5.3)

Proof. Consider a particular policy in the group. We have equation

(t V + P )(1 + i) = Sqx+t + (1 − qx+t )t+1 V (7.5.4)

which may be thought of as saying that the accumulated funds at the end of the year, (t V + P )(1 + i) ,
must provide:
(i) the expected cost of death claims, Sqx+t , plus

(ii) the expected cost of setting up reserves for the survivors, (1 − qx+t )t+1 V.

Equation (7.5.4) may be rearranged to give

(t V + P )(1 + i) = t+1 V + qx+t (S − t+1 V) (7.5.5)

On summing over all policies in force at the start of the year, we find that
X X
(t V + P )(1 + i) = t+1 V + EDS (7.5.6)
all in force all in force
at the at the
start of start of
year year

Hence, by equation (7.5.2),


mortality profit = EDS − ADS
as required.
Notes. 1. The sum assured S and the premium P may depend on the policy year, t + 1.
2. Equation (7.5.4) may be verified directly for particular policy types: e.g., for a whole
life policy with sum assured £1,

(t Vx + Px )(1 + i) = qx+t + (1 − qx+t )t+1 Vx (7.5.7)

Example 7.5.1. On 1st January 1988, a certain life office sold 100 10-year without profits endow-
ment assurance policies to lives then aged 50. Premiums are payable annually in advance, and the
sum assured under each contract is £30000, payable on maturity or at the end of the year of previous
death. The office calculates premiums and reserves on the following basis:
7.5. MORTALITY PROFIT AND LOSS 127

mortality: A1967-70 ultimate

interest: 5% per annum

expenses: nil

All 100 policies were still in force on 1st January 1994, and 2 policyholders died during 1994.
Calculate the mortality profit or loss to the office during 1994 in respect of this business, at 31
December 1994.

Solution. Death strain at risk for each policy at 31.12.94

= 30000 − 300007 V50:10


µ ¶
ä57:3
= 30000 = 10753
ä50:10

Hence

mortality profit = E.D.S. − A.D.S.


= (100q56 − 2) × 10753
= −11378 (Loss of £11378)

Example 7.5.2. On the 1st January 1979 a life office issued a number of 20-year non-profit endow-
ment assurance policies, with annual premiums payable in advance throughout the term and sums
assured payable on maturity or at the end of the year of death, to lives then aged exactly 45. At
31st December 1993, total sums assured of £4,000,000 remained in force. During 1993 sums assured
of £50,000 became death claims, and there were no other exits in 1993.
The office calculates premiums, and maintains net premium reserves, on the basis given below.
Calculate the profit or loss from mortality for this group of policies for the year ending 31st
December 1993.

Basis: mortality: A1967-70 ultimate

interest: 4% per annum

expenses: nil.

Solution.
Sums assured in force on 1.1.1993 = £4050000

∴ Expected death strain = q59 (4050000)[1 − 15 V45:20 ]


µ ¶
ä60:5
= q59 (4050000)
ä45:20

Actual death strain = 50000(1 − 15 V45:20 )


µ ¶
ä60:5
= 50000
ä45:20
128 CHAPTER 7. APPLICATIONS OF RESERVES

∴ mortality profit = E.D.S. − A.D.S.


µ ¶
4.489
= (4050000 × 0.01299373 − 50000)
13.488
= £874

Some generalisations

1. If expenses are included in the calculations, we need only alter t V + P to t V + P − e, where e


is the expense incurred at the beginning of policy year t + 1. The equation corresponding to
(7.5.4) is
(t V + P − e)(1 + i) = Sqx+t + (1 − qx+t )t+1 V (7.5.8)
and formula (7.5.3) remains correct.
(It is assumed that the actual expenses are as allowed for in the basis used for premiums and
reserves. If not, one obtains expense profits or losses, which we do not discuss here.)

2. If the premium basis differs from the reserving basis, formula (7.5.3) remains true provided that
reserves are calculated by a gross premium method (either retrospective or prospective.) This
follows from the fact that equation (7.5.8) remains true (with mortality, interest and expenses
according to the reserving basis and P on the premium basis.)

Example 7.5.3. For several years an insurance company has issued a large number of special en-
dowment assurances. Each policy matures at exact age 65 and is effected by annual premiums
payable on each January 1st throughout the term. The sum assured, payable at the end of the year
of death during the term, is one half of the sum assured that will be paid if the policyholder survives
to age 65. Details of the policies in force on 31 December 1988 are as follows:

Exact age Total sums assured Total annual premiums


payable on death (£) (£)
55 3500000 250000

The claims in 1989 were on policies with the following total sums assured and premiums:

Total sums assured Total annual


payable on death (£) premiums (£)
50000 4000

Assuming that at 31.12.88 the office’s funds were equal to its reserves, calculate the mortality profit
or loss in 1989, given that the following reserving basis is used:
prospective method, gross premium reserve using A1967-70 ult., 4% p.a. interest, no expenses.

Solution. Let EDS and ADS denote the expected death strain and the actual death strain in 1989.
Then X
EDS = q55 [Sj − {Sj (A56:9 + 9 E56 ) − Pj ä56:9 }]
all j
7.6. OTHER SOURCES OF PROFIT AND LOSS 129

where the summation is over all policies in force at the start of the year, i.e.
hX X X i
EDS = q55 ( Sj ) − ( Sj )(A56:9 + 9 E56 ) + ( Pj )ä56:9
X X
= q55 (694048) (since Sj = 3500000 and Pj = 250000)
all j all j

= 5859

The actual death strain is obtained by a summation of the death strains at risk over those policies
which become claims. Thus
X
ADS = [Sj − {Sj (A56:9 + 9 E56 ) − Pj ä56:9 }]
Claims
X X X
=( Sj ) − ( Sj )(A56:9 + 9 E56 ) + ( Pj )ä56:9
Claims Claims Claims
X X
= 13082 (since Sj = 50000 and Pj = 40000)
Claims Claims

The mortality profit is thus = 5859−13082=−7223, i.e. a loss of £7,223.

7.6 Other sources of profit and loss


Although some of these topics may also be considered under the heading of “profit-testing”, we shall
give certain straightforward results on interest and surrender profits here.

Interest profits. Suppose that the office earns interest at a rate i0 p.a. in policy year t + 1, the
interest rate in the reserving basis being i p.a. Assuming as before that the funds at the end of
policy year t equal the reserves for the business under consideration, the interest profit is
X
[ (t V + P − e)](i0 − i) (7.6.1)
all in force
at the
start of policy
year t + 1

Example 7.6.1. On 1st January 1978 a special twenty-year endowment assurance policy for an
initial sum assured of £20,000 was issued to a life then aged 40 exactly. The sum assured, which is
payable at the end of the year of death or on survival to maturity date, increased to £25,000 after 10
years and to £30,000 on maturity. The annual premium was P for the first ten years, and thereafter
increased to 1.25P . The office’s basis for premiums and reserves is

A1967-70 ultimate

4% per annum interest

expenses are ignored

(i) Calculate the initial annual premium P , and the reserve for the contract at (a) 31st December
1989, and (b) 31st December 1990.
130 CHAPTER 7. APPLICATIONS OF RESERVES

(ii) Find the profit or loss from mortality for the year ending 31st December 1990 for a group of
such policies, all effected on 1st January 1978 by lives then aged 40 exactly, given that one
policyholder dies in 1990 and 120 policies remained in force at 31st December 1990. There
were no withdrawals in 1990.
(iii) The office found that it made neither profit nor loss in 1990 in respect of the policies of (ii)
above. What rate of interest did it earn in 1990?
(Expenses are negligible.)

Solution.
(i) Let initial premium be P .
D60
P [ä40:20 + 0.2510 |ä40:10 ] = 20000A40:20 + 500010 |A40:10 + 5000
D40

20000M40 + 5000M50 − 25000M60 + 30000D60


∴P =
N40 + 0.25N50 − 1.25N60
= £907.00

(a)
D60
12 V = 25000A52:8 + 5000 − 1.25P ä52:8
D52
= £14085

(b)

[(12 V + 1.25P )(1.04) − 25000q52 ]


13 V =
1 − q52
= £15772 (this may also be calculated directly)

(ii)

E.D.S. = 121q52 (25000 − 13 V)


= £6734
A.D.S. = 25000 − 13 V = £9228
∴ mortality profit = 6374 − 9228 = −£2494

(iii) Let interest rate earned be i0 . The interest profit is

(i0 − 0.04)[121](12 V + 1.25P ) = 2494

∴ i0 = 0.0414, or 4.14%

Surrender profits. Assume that surrenders may take place only at the end of a policy year (just
before payment of any premium then due). The surrender profit is
X
(t+1 V − S.V.) (7.6.2)
surrenders
7.6. OTHER SOURCES OF PROFIT AND LOSS 131

where S.V. is the surrender value paid.

Summary of profit and loss

Profit or loss for year = accumulation of funds at end of year


− cost of death claims
− cost of surrenders
− cost of setting
up reserves for
remaining policies
= Mortality profit
+ interest profit
+ surrender profit (7.6.3)

Example 7.6.2. On 1st January 1977 an office issued a block of without profits endowment assur-
ance policies and a block of without profits pure endowment policies to men aged exactly 40. Each
policy was for a term of 20 years and a sum assured of £1000. Premiums are payable annually in
advance during the term of the policies and death claims are paid on 31st December each year.
On 1st January 1987 there were 1000 endowment assurance policies and 5000 pure endowment
policies in force. During the year to 31st December 1987 sixty endowment assurance policies became
claims by death and twenty pure endowment policies were terminated, without payment of any sum
assured, by the deaths of the assured lives.
On 31st December 1987 ten endowment assurance policies and five pure endowment policies were
surrendered.
Calculate the profit or loss arising during the year 1987, showing separately the contributions
from mortality and from surrender if the office uses the following basis:

Premiums and reserves: A1967-70 ultimate at 4% p.a. interest

Surrender values: Net premium reserve on A1967-70 ultimate at


6% p.a. interest

Ignore expenses.

Solution. Let us consider each type of policy separately.

Endowment Assurances.
Reserve per policy at end of 1987 = 100011 V40:20
µ ¶

= 1000 1 − 51:9
ä40:20
µ ¶
7531
= 1000 1 −
13.764
= 452.85

∴ Death strain at risk per policy in force at start of 1987 is 1000 − 452.85 = 547.15
∴ E.D.S. = 1000q50 × 547.15 = 26202
A.D.S. = 60 × 547.15 = 32829
132 CHAPTER 7. APPLICATIONS OF RESERVES

∴ mortality profit = E.D.S. − A.D.S. = −6627


Surrender profit=10(Reserve-S.V.), where

S.V. = 100011 V40:20 at 6 %


µ ¶
7.029
= 1000 1 − = 407.88
11.871

∴ Surrender profit = 449.70


Pure Endowments. Annual premium is
D60
1000 D
P = 40
= 29.70
ä40:20

Reserve per policy at end of 1987 is


µ ¶ µ ¶
N40 − N51 132002 − 68970
P = 29.70 = 425.55
D51 4399.08

Death strain at risk is −425.55 per policy.

E.D.S. = 5000q50 (−425.55) = −10189

A.D.S. = −20 × 425.55 = −8511

∴ mortality profit = E.D.S. − A.D.S.


= −1678

S.V.= net premium reserve at 6%. It is easier to find the reserve prospectively.
1000v 20 ll60 279.25
Define P 0 = net premium at 6% p.a. interest = 40
= = 23.52
ä40:20 11.871

l60
∴ Reserve = 1000v 9 − P 0 ä51:9 at 6% p.a. interest
l51
= 546.87 − 165.32 = 381.55

∴ Surrender profit is 5(425.55-381.55)=220.00

Summary

Mortality profit = −£8305


Surrender profit = £670
Total profit = −£7635 (a loss of £7,635)
7.7. EXERCISES 133

Exercises

7.1 Explain briefly how a life office calculates the new sum assured or annual premium on the
alteration or conversion of an existing policy.

7.2 A life office issued a whole life without profits policy to a woman aged 30, with sum assured of
£110,000. Premiums were payable annually in advance throughout life, and the sum assured
was payable at the end of the year of death. Immediately before payment of the 15th premium,
the policyholder requests that the policy be converted to a without profits endowment assurance
for the same sum assured, payable on maturity at age 60 or at the end of the year of death, if
before age 60. The office calculates premiums and maintains reserves on the basis of A1967-70
ultimate mortality and 4% per annum interest, with expenses of 4% of each premium. Expenses
of alteration may be ignored.
Find the revised annual premium.

7.3 On 1 January 1972 an office issued a large number of 40-year endowment assurance policies,
each with sum assured £1000 (payable at the end of the year of death, if this occurs within the
term) to a group of lives all then aged 25. On 31 December 1988 there were 8567 policies still
in force. During 1989 there were 13 deaths among the policyholders.
Find the actual death strain, the expected death strain and the mortality profit or loss for the
business in 1989, using the following basis for all calculations:

mortality: A1967-70 select


interest: 4% p.a.
expenses: none

7.4 On 1 January 1991, a life office issued a number of identical special endowment assurances to
lives then aged 45. Each policy had a term of 15 years and level annual premiums were payable
throughout the term. On survival to the end of the term, a sum assured of £5000 is payable.
On death before the end of the term, a sum assured of £1000 is payable at the end of the year
of death and, in addition, all premiums paid are returned without interest.
The premium basis for these policies was as follows:

mortality: A1967-70 select


interest: 4% p.a.
expenses: Nil

(i) Show that the annual premium for each of these policies is £242.27.
(ii) Calculate the reserve, on the premium basis, on 1 January 1996 for one of these policies,
assuming that the premium then due is unpaid.
(iii) On 1 January 1995, a total of 10,000 of these policies were still in force. In 1995, 47
of these lives died, ten policies were surrendered, and the life office earned 4% interest
on its investments. Expenses were negligible in 1995. The surrenders all took place at
the end of 1995, and the office gave surrender values equal to 95% of the reserve on the
premium basis.
Calculate the profit or loss to the office in 1995 from this block of business in respect
of
(a) mortality , and
134 CHAPTER 7. APPLICATIONS OF RESERVES

(b) surrenders.
7.5 A life office has issued a 5-year decreasing temporary assurance to a life aged 60. The sum
assured, payable at the end of the year of death, is £100,000 if the life dies in the first year,
£90,000 if the life dies in the second year, and so on, decreasing by £10,000 each year. Level
annual premiums are payable throughout the term of the policy.
The premium and reserve basis for this policy is as follows:
mortality: A1967-70 Ultimate
interest: 4% p.a.
expenses: Nil

(i) Calculate the annual premium for the policy.


(ii) Write down a relationship between the reserve at integer duration t and the reserve at
duration t + 1.
7.8. SOLUTIONS 135

Solutions

7.1 The office calculates V1 , the reserve for the original policy at the date of conversion. The office
then sets up the equation
V1 − C = V2

where
C = cost of alteration (if any)

and
V2 = (prospective) reserve of new policy.

Solve for unknown quantity (usually a sum assured or premium.)

7.2 Let
110000
P = original A.P. = P30
1 − 0.04

Reserve before conversion = V1 = 110, 000A44 − 0.96P ä44


= 110, 00014 V30
· ¸
ä44
= 110, 000 1 −
ä30
= 16, 923

Let new A.P. be P 0 . We solve for P 0 in the equation V1 = V2 , i.e.

V1 = 110, 000A44:16 − 0.96P 0 ä44:16

110, 000 × .54703 − 16, 923


∴ P0 =
0.96 × 11.777
= £3, 825

7.3 On 31/12/88 the duration is 17 years. The reserve at the end of 1989 (per unit sum assured)
therefore equals
18 V[25]:40 = 1 − ä43:22 /ä[25]:40 = .28397

Hence in 1989 we have, per policy in force at the start of the year, a death strain at risk of
(1000-283.97)=£716.03.
The expected death strain per policy is 716.03 × q42 = 1.31105.
For the block of business, therefore, we have:

total expected death strain = 8567 × 1.31105 = 11, 234.52


total actual death strain = 13 × 716.03 = 9, 308.4

The 1989 mortality profit is therefore (11,234.52−9,308.4)=£1,926.14.


136 CHAPTER 7. APPLICATIONS OF RESERVES

7.4 (i) Let A.P. be P .


P ä[45]:15 (= 11.250) = 1, 000A[45]:15 (= 0.56732)
µ ¶
D60
+ 4000 (= 0.50271)
D[45]
1
+ P (IA) [45] :15 (= 0.60855)
∴ P = £242.27

(ii)
D60 1 1
5V = 1000A50:10 + 4000 + 5P A50 :10 + P (IA)50:10 − P ä50:10
D50
= £1, 349

(iii) (a)
D.S.A.R. in 1995 in respect of 1 policy = (1000 + 5P ) − 5 V
= £861.97
EDS = 10, 000q49 × DSAR = £36, 714
ADS = 47 × DSAR = £40, 513
∴ mortality profit = EDS − ADS = −£3, 798 (loss of £3,798)

(b)
Surrender profit per policy = 0.05 × reserve
∴ Total surrender profit = 10 × 0.05 × 5 V
= £675.

7.5 (i) Let the Annual Premium be P .


1 1
P ä60:5 = 110, 000A60 :5 − 10, 000(IA)60:5

which gives
P = £1, 347.03

(ii)
(t V + P )(1.04) = q60+t (100, 000 − 10, 000t) + (1 − q60+t )t+1 V
(since the sum assured on death in year t + 1 is 100, 000 − 10, 000t).
Chapter 8

EXTRA RISKS

8.1 Introduction
It is usual in practice for a life office to accept the majority of proposers for life assurance policies on
“normal” (or “standard”) terms, the remainder being either declined or accepted on special terms,
i.e. they are accepted but with higher premiums or reduced benefits. Those accepted on special
terms are considered to be subject to an extra mortality risk due to a health impairment, a dangerous
occupation or a dangerous spare-time activity (such as motor racing.) The assessment of extra risks
is usually carried out by experienced underwriters using a “numerical rating system”; we shall not
go into this, but shall assume that the addition to the normal rates or force of mortality is known.
Some offices make extensive use of “rating up”: that is impaired lives are treated as normal
lives a number of years older than actual age. (Females are sometimes treated as males 5, say, years
younger, although this rule is very inaccurate at certain young ages.) Smokers are sometimes treated
as non-smokers 6 years older.

8.2 A constant addition to the force of mortality


Let µ[x]+t refer to the force of mortality at duration t years for a life who was selected for assurance
to age x on the office’s normal terms. (If a non-select table is used, omit [ ].) Let n years be the
duration of the policy, and let µ∗[x]+t be the corresponding force of mortality for a life accepted on
special terms. We now assume that there is k > 0 such that:

µ∗[x]+t = µ[x]+t + k (0 ≤ t ≤ n) (8.2.1)

Hence
· Z t ¸
∗ ∗
p
t [x] = exp − µ [x]+r dr
0
· Z t ¸
= exp − (µ[x]+r + k) dr
0
= e−kt t p[x] (8.2.2)

137
138 CHAPTER 8. EXTRA RISKS

Also,


t E[x] = v t t p∗[x]
= e−(δ+k)t t p[x]
0
= e−δ t t p[x] (8.2.3)

where
δ0 = δ + k (8.2.4)
0
and hence i0 = eδ − 1 = eδ+k − 1.
Since an annuity factor is a sum or integral of pure endowment factors, we have (for example)
ä∗[x] = ä[x]
on normal mortality but with force of interest δ 0 = δ + k (8.2.5) This rule does not
apply to assurances and premiums, but one may sometimes use conversion relationships to express
these in terms of annuity factors, as in the following example.

Example 8.2.1. A certain life office uses the following basis for calculating premiums for assurance
policies on lives accepted at normal rates:

A1967-70 ultimate

4% interest

expenses are ignored

A certain proposer, aged 50, for a 15-year endowment assurance without profits by annual premiums
is considered by the office to be subject to the mortality of the office’s normal table with an addition
of 0.019048 to the force of mortality for the next 15 years. The sum assured is £10,000, payable
at the end of the year of death, or on survival for 15 years. Calculate the addition to the normal
annual premium.

Solution. Let ∗ indicate special mortality basis.


Extra annual premium is

10, 000[P50:15 − P50:15 ]

Note that

∗ 1
P50:15 = − d by a conversion relationship
ä∗50:15
1
= − d since δ.04 + 0.019048 = δ.06
ä50:15 0.06

But d is still at 4% interest.


Hence the extra premium is
· ¸
1 1
10, 000 − = £108.51
ä50:15 .06 ä50:15 .04
8.3. A VARIABLE ADDITION TO THE FORCE OF MORTALITY 139

In some problems we must evaluate assurance factors directly, without the use of conversion rela-
tionships; e.g.

(I¯Ā)∗ x1:n = M.P.V. of an increasing temporary assurance


for an impaired life aged x at entry
Z n
= tv t t p∗x µ∗x+t dt
0
Z n
= tv t (t px e−kt )(µx+t + k) dt
Z0 n
= t (v 0 )t t px (µx+t + k) dt
0
| {z }
v 0 is at rate i0
(with δ 0 = δ + k)

= (I¯Ā)x1:n at rate of interest i0


¯ x:n
+ k(Iā) at rate of interest i0 (8.2.6)

A formula for the expectation of life of an impaired life


We have
Z ∞
o
e∗x = E(T ∗ ) = ∗
t px dt (by formula (1.4.3))
0
Z ∞
−kt
= t px e dt
0
= āx at force of interest k p.a. (8.2.7)

Note also that


Z ∞
∗ 2
E[(T ) ] = 2 t.t p∗x dt (by integration by parts)
0
Z ∞
=2 t.t px e−kt dt
0
¯ x
= 2(Iā) at force of interest k p.a. (8.2.8)

Hence

¯ x − (āx )2 at force of interest k p.a.


Var(T ∗ ) = 2(Iā) (8.2.9)

Remark. To find the median future lifetime of the impaired life, we solve the equation

t qx = 0.5
∗ lx+t e−kt
i.e. t px = = 0.5
lx

8.3 A variable addition to the force of mortality


Let us now suppose that there is ξ(t) ≥ 0 such that

µ∗[x]+t = µ[x]+t + ξ(t) (0 ≤ t ≤ n) (8.3.1)


140 CHAPTER 8. EXTRA RISKS

We obtain the formula µ Z t ¶



t p[x] = t p[x] exp − ξ(r) dr (8.3.2)
0

Z t
In some cases ξ(r) dr may be calculated analytically, as in the next example.
0

Example 8.3.1. A certain impaired life aged 75 experiences mortality according to a(55) males
ultimate with an addition to the force of mortality. The addition is 0.005 at age 75, increasing
linearly to 0.020 at age 90.

(i) Find a formula for the probability that this life will be alive at age 75 + t years (0 ≤ t ≤ 15).

(ii) Estimate, by approximate integration using the trapezoidal rule (not repeated), the single
premium for a temporary assurance of £50,000 payable immediately on death of (75), if this
occurs within 15 years, on the following basis:

mortality: as described above


interest: 10% p.a.
expenses: 2% of the single premium.

Solution.
(i)
µ∗75+t = µ75+t + 0.005 + 0.001t (0 ≤ t ≤ 15)
· Z t ¸
∴ t p∗75 = t p75 exp − (0.005 + 0.001r) dr
0
£ ¤
= t p75 exp −(0.005t + 0.0005t2 )

(ii) Let single premium be P .


Z 15
0.98P = 50, 000 v t t p∗75 µ∗75+t dt
0
15 £ ∗ ¤
; 50, 000 × µ75 + v 15 15 p∗75 µ∗90
2
= 50, 000 × 7.5 [0.06550 + 0.1104 × 0.23939 × 0.2636]
∴ P = £27, 730

8.4 Rating up
In practice, impaired lives are often considered to experience the mortality of a “normal” life r years
older (where r depends on the severity of the impairment.) This allows the life office to base all its
premiums on one set of tables.
8.4. RATING UP 141

A practical method of finding “r” is as follows. Suppose that the normal mortality table follows
Gompertz’ law, so there are constants B, c such that

µx+t = Bcx+t (0 ≤ t ≤ n)

(This is usually a fairly accurate representation of mortality for all ages over about 30.) Suppose
that the impairment is such that

µ∗x+t = (1 + θ)µx+t (0 ≤ t ≤ n) (8.4.1)

for some constant, θ. (This formula is usually suitable for “medical” impairments; for accident risks
it is generally better to add a constant to the normal force of mortality.) Hence

µ∗x+t = (1 + θ)Bcx+t
= Bc(x+r)+t (0 ≤ t ≤ n)

where cr = 1 + θ, i.e.
log(1 + θ)
r= (8.4.2)
log c
In practice c is usually about 1.08, so we have the approximate rule

log(1 + θ)
r= (8.4.3)
log 1.08

For example, if θ = 0.4 (corresponding to a 40% increase in the force of mortality),

log 1.4
r; = 4.37
log 1.08

For practical purposes this value of r would be rounded to 4 or 5 years. This rule is not precise,
but it may be sufficiently accurate in practice since the value of θ may in general be estimated only
rather approximately.

Example 8.4.1. A certain life office’s premium basis for lives accepted at normal rates is as follows:

Mortality: A1967-70 ultimate

Interest: 4% per annum

Expenses: none

A certain impaired life, aged 50, is considered to be subject to the office’s normal mortality rates
for a life 10 years older. This life requires a contract providing the following benefits:

(i) £30, 000 at the end of the year of death, if within 10 years;

(ii) on survival for 10 years, the sum of £(30, 000 + S).

The impaired life finds that the level annual premium for this policy is the same for a policy providing
identical benefits for a life his age accepted at normal rates. Find S.
142 CHAPTER 8. EXTRA RISKS

Solution.
30, 000A50:10 + S D60
D50
normal premium =
ä50:10
µ Á ¶
D60
= 30, 000P50:10 + S ä50:10
D50
µ Á ¶
D70
“impaired life” = 30, 000P60:10 +S ä60:10
D60

Equate these to give

30, 000(P60:10 − P50:10 )


S=³ . ´ ³ . ´
D60
D50 ä50:10 − D 70
D60 ä 60:10

30, 000(0.08986 − 0.08339)


=
0.075689 − 0.068168
= £25, 809

8.5 Debts
Instead of paying higher premiums, a proposer accepted on special terms may elect to pay the normal
premiums with reduced death benefits. Such reductions in the death benefits are called “debts” or
“liens”, and may be either

(a) a level amount; or

(b) diminishing in arithmetic progression to zero in the final policy year.

We first give an example involving a level debt. (Note that a debt is a reduction in the death benefit
only: the maturity benefit is not reduced.)

Example 8.5.1. A life office calculates annual premiums for without profits endowment assurances
using the following basis:

Mortality: A1967-70 select

Interest: 4%

Initial expenses: 50% of the first premium

Renewal expenses: 5% of each premium after the first

(i) Calculate the annual premium payable for a 25 year endowment assurance taken out by a male
life aged exactly 45 for a sum assured of £50,000. The death benefit is payable at the end of
the year of death.

(ii) A man aged exactly 45 effects a policy identical to that in (i) above but rated up 7 years.

(a) Calculate the level extra premium, payable throughout the term of the policy.
(b) Alternatively, the life office offers to charge the standard premium but to impose a level
debt. Calculate the amount of the debt.
8.5. DEBTS 143

Solution.
(i) Let A.P. be P .
0.95P ä[45]:25 = 50, 000A[45]:25 + 0.45P
∴ P = £1, 492.41

(ii) (a) Consider a “normal” (select) life aged 52. Let P 0 be premium.
0.95P 0 ä[52]:25 = 50, 000A[52]:25 + 0.45P 0

∴ P 0 = 1, 752.32
∴ Extra premium = £259.91
(b) Let death benefit be reduced by D. The equation of value is
1
0.95P ä[52]:25 = 50, 000A[52]:25 − DA[52] :25 + 0.45P
∴ D = £12, 649

Now suppose that we are dealing with an n-year endowment assurance, and that the debt diminishes
in arithmetical progression to zero in the final policy year; that is, the debts run as follows:

(n − 1)D in year 1

(n − 2)D in year 2

..................

D in year n − 1

0 in year n

Consequently the M.P.V. of the debts, at the issue date, is

D{nA∗ [x]
1 ∗ 1
:n − (IA) [x]:n } (8.5.1)

where ∗ indicates impaired lives’ mortality; the death benefits are taken to be payable at the end of
the year of death (if these benefits are payable immediately on death, a bar should be placed over
the A symbols.) The calculations are illustrated in the next example.

Example 8.5.2. A life office issues 20-year without-profit endowment assurance policies providing
a sum assured of £8,000 on maturity or at the end of the year of earlier death. Level monthly
premiums are payable in advance throughout the term or until earlier death.

A certain impaired life aged 45 is considered by the office to have the mortality rates of a “nor-
mal” (select) life 5 years older.

Assuming that the ordinary premium appropriate to this life’s actual age is payable, calculate the
initial amount of the debt (i.e. the deduction from the sum assured) such that the debt reduces
uniformly each year to nil in the last policy year. Basis:

Mortality of “normal” lives: A1967-70 select

Interest: 4%
144 CHAPTER 8. EXTRA RISKS

expenses: 30% of premiums in first year


5% of later premiums

P
Solution. Let be “normal” monthly premium. The equation of value for P is
12

(12) (12)
0.95P ä[45]:20 − 0.25P ä[45]:1 = 8000A[45]:20

8000 × [0.48051]
P =n h ³ ´i h ³
D[45]+1
´io
.95 ä[45]:20 − 11
24 1 − DD[45]
65
− 0.25 1 − 11
24 1− D[45]

3844.08
=
{0.95[13.507 − 0.45833 × 0.62253] − 0.25[1 − 0.45833 × 0.04014]}
3, 844.08
=
12.5606 − 0.24541
= £312.14 (£26.01 per month)

Let the initial debt be 19D. The equation of value for D is


(12) (12) £ 1 1
¤
0.95 × 312.14ä[50]:20 − 0.25 × 312.14ä[50]:1 = 8000A[50]:20 − 20DA[50] :20 − D(IA)[50]:20

½ · µ ¶¸
D70
D= 8000 × 0.49664 − 312.14 × 0.95 × ä[50]:20 − 0.45833 1 −
D[50]
· µ ¶¸¾
D[50]+1
+0.25 × 312.14 1 − 0.45833 1 −
D[50]
. ½ 20(M ¾
[50] − M70 ) − (R[50] − R70 − 20M70 )
D[50]
3973.12 − 296.53[13.087 − 0.45833 × 0.66887] + 78.035[0.98111]
= 1
4581.322 [20 × 758.289 − 9360.22]

259.898
= = 205.09
1.2672
∴ Initial Debt is 19D = £3, 896.71
8.6. EXERCISES 145

Exercises

8.1 An explorer aged exactly 57 has just made a proposal to a life office for a whole life assurance
with a sum assured of £10,000 payable at the end of the year of death. For lives accepted at
normal rate, level annual premiums are payable until death under this policy.
The explorer is about to undertake a hazardous expedition which will last three years. The life
office estimates that during these three years the explorer will experience a constant addition of
0.02871 to the normal force of mortality, but after three years will experience normal mortality.
The life office quotes a level extra premium payable for the first three years.
Calculate this level extra premium on the following basis:

normal mortality: A1967-70 ultimate


interest: 3% per annum
expenses: none

8.2 An impaired life aged exactly 55 wishes to effect a without profit endowment assurance for a
sum assured of £1,000 payable at the end of 10 years or at the end of the year of earlier death.
Level annual premiums are payable throughout the term of the policy.
Special terms are offered on the assumption that the life will experience mortality which can
be represented by:

(a) for the first five years, a constant addition of 0.009569 to the normal force of mortality,
and
(b) for the remaining five years, the mortality of a life 8 years older.

The life office quotes a level extra premium payable throughout the term. Calculate this level
extra premium. Basis:

normal mortality: A1967-70 ultimate


interest: 3% per annum
expenses: none

8.3 A group of impaired lives now aged 40 experience mortality according to A1967-70 ultimate
with an addition to the force of mortality. The addition is 0.0005 at age 40, increasing linearly
to 0.0025 at age 60, at which level the addition remains constant.
Find the probability that an impaired life aged exactly 40

(i) will die within 20 years;


(ii) will die within 30 years;
(iii) will die between 20 and 30 years from the present time.

8.4 (i) Can you envisage circumstances under which an office could offer an impaired life who
wishes to pay the “normal” premiums a level debt but not a diminishing debt?
Hint Consider a life who is very severely impaired, and think of an approximate rela-
tionship between
(a) the level debt and
(b) the initial debt when debts decrease linearly to zero.
146 CHAPTER 8. EXTRA RISKS

(ii) A certain proposer, aged 40, for a 20-year endowment assurance by annual premiums
with sum assured £40,000 without profits (payable at the end of the year of death) is
considered by your office to be subject to the mortality of the normal table with an
addition of 10 years to the age. Your office’s basis for calculating premiums for “normal”
lives is
A1967-70 ultimate
4% p.a. interest
expenses of 6% of all premiums.

(a) The proposer asks to pay the annual premium for a “normal” life aged 40, and
suggests that the office should make the normal death benefits subject to a debt
which decreases in arithmetical progression to zero in the final year. Calculate the
initial debt.
(b) Suppose that the proposer also asks your office to quote a level debt. Calculate the
level debt.
8.5 The mortality of a certain impaired life aged exactly 60 may be represented as follows:
(1) at ages up to exact age 65, there is a constant addition of 0.009569 to the “normal” force
of mortality, which is that of A1967-70 select; and
(2) at ages between exact age 65 and exact age 70, mortality follows that of a “normal”
(ultimate) life 4 years older.

Suppose that this life requires a 10-year endowment assurance without profits with sum assured
£10, 000 payable on maturity or at the end of the year of earlier death. The life wishes, however,
to pay the “normal” level annual premium for his age. Calculate the level debt which would
be deducted from the normal death benefit, using the basis given below:
mortality: as given above
interest: 4% p.a.
expenses: 2 12 % of all premiums, with an additional initial expense of
1% of the sum assured (before deducting any debt.)
8.7. SOLUTIONS 147

Solutions

8.1 Let P be the normal annual premium. Then

ä57 × P = 10, 000A57 at 3%

P ; 377.41
Let E be the extra premium charged for 3 years. Then, the equation of value will be (* denotes
impaired mortality):
(P + E)ä∗57:3 + P 3 |ä∗57 = 10, 000A∗57
where
ä∗57:3 3% = ä57:3 6% = 2.804

∗ l60
3 |ä57 = 1.06−3 ä60 at 3% = 11.076
l57

A∗57 = A∗ 57
1 ∗
:3 + 3 E57 A60 at 3%
= A∗57:3 − 3 E57
∗ ∗
+ 3 E57 A60 at 3%
l60
= 1 − d3% ä57:3 6% − 1.06−3 (1 − A60 at 3%)
l57
= 0.59572

Hence
10, 000A∗57 − P (ä∗57:3 + 3 |ä∗57 )
E= = 256.3207
ä57:3

8.2 PNORMAL = 1000P55:10 = £85.84


" #
∗ 1
P = 1000 − d
ä∗55:10
| {z } | {z }

at 4%
ä∗55:10 = ä∗55:5 + [5 E55

]ä∗60:5
l60
= ä55:5 at 5% + v 5 at 5% [ä68:5 at 4%]
l55
= 7.677

∴ P ∗ = £91.79 so extra annual premium = £5.95

8.3 Let µx refer to A67-70 ult., µ∗x to impaired lives.


(
µ40+t + 0.0005 + 0.0001t (0 ≤ t ≤ 20)
µ∗40+t =
µ40+t + 0.0025 (t > 20)
148 CHAPTER 8. EXTRA RISKS

(i)
½ Z 20 ¾

20 p40 = exp − (µ40+t + 0.0005 + 0.0001t) dt
0
( · ¸20 )
1 2
= 20 p40 exp − 0.0005t + (0.0001t )
2 0
l60
= × exp {−(0.01 + 0.02)}
l40
= 0.895579 × 0.9704455 = 0.86911
∴ ans = 0.1309
(ii)

30 p40 = 0.86911 × 10 p∗60
l70
= 0.86911 × × exp[−10 × 0.0025]
l60
= 0.66656
∴ Ans = 0.3334
(iii) 0.3334−0.1309=0.2025
8.4 (i) The debt cannot, of course, exceed the “normal” sum assured on death. The level debt
must be (very roughly) about one-half of the initial debt (since the average dim. debt is
one-half of the initial debt), so we may have cases in which:

initial debt > S.A. > level debt


(when debts diminish) (in cases of serious impairment)
(ii) (a)
40, 000P40:20
Normal A.P. = = 1454.89
0.94
If initial debt is 19D, we have the equation of value
1 1
0.94(1454.89)ä50:20 = 40, 000A50:20 − D{20A50 :20 − (IA)50:20 }
i.e.
40, 000 × 0.49818 − 0.94 × 1454.89 × 13.047
D=
20 × 0.16819 − 2.04074
= 1, 575.26
∴ Initial debt = £29, 929.94 (£29, 930)
1 1 1
(b) Replace D{20A50 :20 − (IA)50:20 } in above by the term DA50:20 , to give
2084.17
D= = £12, 392
0.16819

8.5 Let P be the normal A.P.


0.975P ä[60]:10 = 10, 000 A[60]:10 + 100
| {z } | {z }
7.884 0.69677
∴ P = £919.45
8.7. SOLUTIONS 149

Let ∗ indicate special mortality.


l65
ä∗[60]:10 = ä[60]:5 0.05 + v0.05
5
ä69:5 0.04
l[60]
| {z } | {z } | {z }
4.452 0.72603 4.29496
= 7.570

Let level debt be D.


1
0.975P ä∗[60]:10 = 10, 000 A∗[60]:10 − D A∗ [60] :10 + 100
| {z } | {z } | {z }

7.570 1 − dä∗[60]:10 = 0.708835


5 5 l65 l74
0.708835 − [v.05 v.04 ]
l[60] l69
= 0.708835 − 0.478114
= 0.230719

402.119
∴D=
0.230719
= £1, 743
150 CHAPTER 8. EXTRA RISKS
Chapter 9

PROFIT-TESTING

9.1 Principles of profit-testing


In this book we shall consider “conventional” life assurance business only, not unit-linked policies
(which are considered in part D1.) The main new ideas in profit-testing are as follows:

(1) The sale of a without-profits policy is considered by the life office as an “investment” on
the part of the shareholders and/or with-profits policyholders. (In a mutual life office there
are no shareholders.) Similarly, the sale of a with-profits policy may be considered to be an
“investment” on the part of the office’s shareholders, though the position is complicated by the
fact that future profits are shared between the with-profits policyholders and the shareholders.

(2) The office calculates the expected net cash flow, and the expected profit, in each policy year in
two stages.
Stage A. The net cash flow in each policy year, and the profit after allowing for appropriate
reserves, of a given contract is calculated on the assumption that the policy is still in force at
the start of that year. The resulting net cash flow is called the in force net cash flow, and the
vector of profits (indexed by the policy year, t) is called the profit vector, which is written as
{(PROt )}.
Stage B. The estimated net cash flow per policy sold in policy year t is called the initial
net cash flow for that year, and is found by multiplying the “in force” net cash flow by the
probability that the policy is still in force at the start of the tth policy year, t−1 px . (We assume
here that the age at entry to assurance is x, and that a non-select mortality table is used.) The
corresponding vector of profits (after allowing for reserves) is called the profit signature, which
is written as {σt }.

(3) All cash flows are considered at the end of the policy year, so premiums less expenses are
accumulated to the end of the appropriate policy year at the rate of interest which the office
assumes it will earn on the life funds.

(4) Random fluctuations in mortality are ignored; that is, each policy is considered as if it were one
of a large number of identical contracts whose experience exactly follows the mortality table
used by the office in its projections.

151
152 CHAPTER 9. PROFIT-TESTING

9.2 Cash flow calculations


We consider an n-year “generalised endowment assurance”, issued to (x), and use the following
notation:
Pt = premium payable in policy year t (at the start of that year, i.e. payable at time t − 1 years
from the issue date);
et = expenses assumed to be incurred in policy year t (at the start of that year);
Dt = death benefit in policy year t (paid at the end of the policy year);
St = survival benefit at the end of policy year t (St is usually zero for t < n);
i = rate of interest p.a. which the life office assumes it will earn on its life funds;
The symbols qx+t , t−1 px , etc, refer to the mortality table which the life office assumes will apply to
the policyholders under consideration.

Note. In general, three distinct bases are used in profit-testing:


Basis 1: the premium basis, which is used only to calculate the premiums;
Basis 2: the reserving basis, which is used only to calculate the reserves;
Basis 3: the experience basis, which is that which, the office assumes, will be experienced.
In some cases all three of these bases agree, or two of them agree. Note that the symbols et , i and
qx+t given above all refer to Basis 3, while Pt is calculated using Basis 1.
We now define, for 1 ≤ t ≤ n,
(CF)t = the “in force” expected net cash flow in policy year t (9.2.1)
We clearly have
(CF)t = (Pt − et )(1 + i) − qx+t−1 Dt − px+t−1 St (9.2.2)
| {z } | {z } | {z }
accumulation expected expected
of premium cost of cost of
less expenses to death survival
end of year benefits benefits
The “initial” expected net cash flow in year t, i.e. the net cash flow in year t per policy sold, is
found from the important equation:
“initial” net cash flow in year t = t−1 px (CF)t (9.2.3)
| {z }
prob. that policy will
still be in force at
time t − 1, i.e. at
start of policy year t

Example 9.2.1. An endowment assurance policy, with sum assured £5000, term five years and level
annual premiums, is issued to a life aged 55.

The annual premium is calculated on the following basis:


9.2. CASH FLOW CALCULATIONS 153

Mortality: A1967-70 ultimate


Interest: 6% per annum

Initial expenses: £250


Renewal expenses (associated with the payment of the second and each subsequent premium):
£42 at the time of payment of the second premium, increasing thereafter by 5% per annum
(compound).
The death benefit is payable at the end of the year of death.

(i) Show that the annual premium is £948.74


(ii) Assume that in calculating cash flows and profit signature for the policy the office uses the
premium basis.
On this basis determine for each year of the policy’s duration
(a) the ‘in force’ expected cash flow, and
(b) the ‘initial’ expected cash flow.

Solution.
(i) Let the annual premium be P . Then
4
X
P ä55:5 = 5000A55:5 + 250 + 42(1.05)t−1 t p55 v t
t=1

(where all functions are on A1967-70 ultimate, 6%) , from which it follows that
5000 × 0.75171 + 250 + 152.62
P =
4.386
= £948.74
154 CHAPTER 9. PROFIT-TESTING

(ii) We draw up the following table:


(1) (2) (3) (4) (5) (6)
Death Survival
Year Premium Expenses Interest
Cost Cost
t Pt et .06(Pt − et ) 5000q54+t St p54+t
1 948.74 250.00 41.92 42.20 0.00
2 948.74 42.00 54.40 47.10 0.00
3 948.74 44.10 54.28 52.50 0.00
4 948.74 46.30 54.14 58.45 0.00
5 948.74 48.62 54.01 64.95 4935.05

(7) (8) (9)


In force
Prob. in Initial expected
expected
force cash flow
cash flow
(CF)t t−1 p55

698.46 1.00000 698.46


914.04 .99156 906.33
906.42 .98222 890.30
898.13 .97191 872.90
−4045.87 .96054 −3886.22

Column(7) = (2) − (3) + (4) − (5) − (6)


Column(9) = (7) × (8),

where column (8) gives the probability that the policy will be in force at the start of year t.
9.3. THE PROFIT VECTOR AND THE PROFIT SIGNATURE 155

9.3 The profit vector and the profit signature


We now suppose that the office maintains suitable reserves t V at the end of each policy year. We
assume that 0 V = 0 and n V = 0 (i.e., any survival benefit payable when the contract matures at
time n years has been paid.) Corresponding to the “in force” and “initial” net cash flows, we have
the profit vector and profit signature; that is,

(PRO)t = cash which will, according to office’s (9.3.1)


projections, be transferred from the
life fund to the with-profits policyhold-
ers/shareholders at the end of policy year
t, per policy in force at the start of that
year
σt = as for (PRO)t , but per policy sold (9.3.2)

The relationship between these quantities is therefore

σt = t−1 px (PRO)t (9.3.3)

Calculation of (PRO)t
We observe that
(PRO)t = t−1 V(1 + i) + (CF)t − px+t−1 .t V (9.3.4)
| {z } | {z } | {z }
accum. of “in force” money needed
reserves net cash for reserves
until end flow in at end of
of year year t the year

(since the chance that a given policy in force at the start of the policy year remains in force at the
end is px+t−1 .) This formula is sometimes written in the form

(PRO)t = (CF)t + it−1 V − (IR)t (9.3.5)


| {z } | {z }
interest increase
on in reserves
reserve required

where

(IR)t = px+t−1 .t V − t−1 V (9.3.6)


= money needed at end of the year
for reserves, less money held at
the start
We may then calculate {σt } by formula (9.3.3).

Example 9.3.1. Consider the policy described in example 9.2.1. Assuming that the premium and
experience basis are as described in that question and that the reserves held are as follows:

0 V=0
156 CHAPTER 9. PROFIT-TESTING

1 V=919

2 V=1,876

3 V=2,873

4 V=3,914

5 V=0

Calculate the profit vector and profit signature.

Solution. We draw up the following table:


(1) (2) (3) (4) (5) (6)
Reserve Increase Interest In force
Survival
Year at start in on expected
prob.
of year reserve reserve cash flow
t t−1 V p54+t (IR)t .06t−1 V (CF)t
1 0.00 .99156 911.24 0.00 698.46
2 919.00 .99058 939.33 55.14 914.04
3 1876.00 .98950 966.83 112.56 906.42
4 2873.00 .98831 995.25 172.38 898.13
5 3914.00 .98701 −3914.00 234.84 −4045.87

(7) (8) (9)


In force
Prob. Profit
profit
in force signature
for year
(PRO)t t−1 p55 σt
−212.78 1.00000 −212.78
29.85 .99156 29.60
52.15 .98222 51.22
75.26 .97191 73.15
102.97 .96054 98.91

Column(4) = p54+t .t V − t−1 V


Column(7) = (6) + (5) − (4)
Column(9) = (7) × (8),

where column (8) is the probability that the policy will be in force at the start of year t.
9.4. THE ASSESSMENT OF PROFITS 157

Example 9.3.2. A life office issues a 3-year without profits endowment assurance policy to a life
aged 62. The sum assured of £1, 500 is payable on maturity or at end of the year of death, if within
3 years, and there are level annual premiums of £472.50 payable in advance.
The office uses the following “experience” basis:

mortality: A1967-70 ultimate

interest: 6% per annum

initial expenses: £20

renewal expenses: £5 at the beginning of the second and third policy years.

The office’s reserve basis is as follows:

net premium method, using A1967-70 ultimate mortality


and 3% p.a. interest.

Determine the profit signature of this policy.

Solution. We first work out the “in force” net cash flows, (CF)t .

(1) (2) (3) (4) (5) (6)


t Prem. Exp. Interest Death cost Maturity (CF)t = (1) + (2)
= 0.06((1) − (2)) = 15, 000q61+t Cost +(3) − (4) − (5)

1 472.50 20 27.15 26.62 0 453.03


2 472.50 5 28.05 29.48 0 466.07
3 472.50 5 28.05 ←−−−−− 1, 500 −−−−−→ −1,004.45
Now we find the profit vector, (PRO)t , and the profit signature, σt .

(1) (2) (3) (4) (5) (6) (7)


Reserve at Interest
start on p61+t (IR)t (PRO)t = t−1 p62 σt
of year Reserve
t = 0.06(1) (CF)t + (2) − (4) = (5) × (6)

1 0 0 0.98225 468.12 −15.09 1 −15.09


2 476.58 28.60 0.98035 466.07 14.71 0.98225 14.45
3 975.71 58.54 0.97826 −975.71 29.80 0.96295 28.70
| {z }
by
net premium method
do not use office
premium in valuation

9.4 The assessment of profits


The profit signature {σt } can be assessed in one or more of the following ways.
158 CHAPTER 9. PROFIT-TESTING

(1) One could work out the Internal Rate of Return (or yield) by solving the equation
n
X
v t σt = 0 (9.4.1)
t=1

(the internal rate of return, i0 , being the solution of this equation.)


(2) The shareholders may value the net profits at a certain rate of interest, j per annum. This
rate is called the Risk Discount Rate, and may reflect uncertainties in {σt }, with j normally
higher than i.
The net present value of the profits is thus
n
X
NPV(j) = v t σt at rate j (9.4.2)
t=1

(3) The Profit Margin is defined as

n.p.v. of profits
, both at some rate of interest, im , say
n.p.v. of premiums
Pn
v t σt
= Pn−1t=1 , at rate im (9.4.3)
t
t=0 Pt+1t px v

(im is often equal to the risk discount rate, j.)

Example 9.4.1. A life office issues a three-year non-profit endowment assurance policy to a man
aged 30. The sum assured is £60,000 on maturity or at the end of the year of earlier death. Level
premiums of £19,000 are payable annually in advance.
The office maintains reserves as follows:

reserve at end
policy year
of policy year
1 £19,000
2 £38,000

The office expects that its life funds will earn interest at 7% p.a. over the next 3 years.
The office expects expenses to be as follows:
initial expenses: 10% of the first year’s premium,
renewal expenses: 2% of later premiums.
Mortality is expected to follow A1967-70 ultimate.
Calculate
(i) the profit signature;
(ii) the net present value at the issue date of the profit to the office, using a risk discount rate of
10% p.a.
9.5. SOME THEORETICAL RESULTS ABOUT {σT } 159

Solution.

(i) We work out the net cash flows, (CF)t , per policy in force at the start of the policy year.
(1) (2) (CF)t
t (Pt − et ) (Pt − et )(1 + i) death + mat. costs = (1) − (2)
1 17,100 18,297 39.22 18,258
2 18,620 19,923 40.26 19,833
3 18,620 19,923 60,000 −40,077

[ death cost = 60, 000q30+t−1


mat.cost (in final year only) = 60, 000p30+t−1 ]

We now work out the profit vector.

(1) (2) (3) (4)


t tV (CF)t i.t−1 V p30+t−1 .t V − t−1 V (PRO)t = (1) + (2) − (3)
1 19,000 18,258 0 18,987 −729
2 38,000 19,883 1330 18,975 2,238
3 0 −40,077 2660 −38,000 583
Hence profit signature is:

−729
2, 238 × 0.999346 = 2, 237
583 × 0.998676 = 582

(ii)
−729v + 2, 237v 2 + 582v 3 at 10% = £1, 623

9.5 Some theoretical results about {σt }


Theorem 9.5.1. If all three bases defined in section 9.2 agree,

σt = (PRO)t = 0 for all t

Proof. This follows from the formula

(PRO)t = t−1 V(1 + i) + (CF)t − px+t−1 .t V = 0

which is merely a restatement of the equation

(t−1 V + Pt − et )(1 + i) = qx+t−1 Dt + px+t−1 St + px+t−1 .t V

which holds (cf chapter 7) since all three bases agree.

Theorem 9.5.2. If the premium basis and experience basis agree, the internal rate of return earned
by the office on a sale of a policy is equal to the rate i p.a. earned by the office’s life funds.
160 CHAPTER 9. PROFIT-TESTING

Proof. We must show that


n
X
v t σt = 0 at rate i p.a.
t=1

Now we have
n
X n
X
t
v σt = v t [t−1 px (CF)t + t−1 px .t−1 V(1 + i) − t px .t V]
t=1 t=1

But
n
X
v t t−1 px (CF)t = 0
t=1

as this is merely a restatement of the equation of value for calculating premiums. Hence
n
X n
X n
X
v t σt = v t t−1 px (1 + i)t−1 V − v t t px .t V
t=1 t=1 t=1
n−1
X n
X
= v r r px .r V − v t t px .t V
r=1 t=1
= 0, using the fact that 0 V = n V = 0

Corollary. If the life office wishes to obtain an I.R.R. on the sale of a policy in excess of the rate
of interest it believes it will earn on the life funds, premiums must be higher than those calculated
on the “experience” basis.

Example 9.5.1. What is the I.R.R. associated with the profit signature of example 9.3.1?

Solution. Since the premium and reserving bases agree, the I.R.R. must be i = 6% p.a.

9.6 Withdrawals
So far we have ignored the possibility of surrender. We shall now assume that surrenders may occur,
but only at the end of a policy year (just before payment of the premium then due). Let wt denote
the chance that a policy will be surrendered at the end of year t (t = 1, 2, . . . , n − 1). We may
assume that wn = 0, since the policyholder will receive the maturity benefits (if any) at that time.
Let (SV)t denote surrender value at time t. We observe that (PRO)t is as before, except that
there are additional profits due to the surrender of

(px+t−1 )wt [t V − (SV)t ] (9.6.1)


| {z }| {z }
prob. that a profit at time t
policy in force per surrender
at time t − 1 at that time
will be surrendered
at time t
9.6. WITHDRAWALS 161

Hence

(PRO)0t = profit vector at time t, allowing for surrenders


= (PRO)t + px+t−1 wt [t V − (SV)t ] (9.6.2)

Note that (PRO)0t = (PRO) if t V = (SV)t for t = 1, 2, . . . , n − 1.


We must also adjust σt , to allow for
(i) the change from (PRO) to (PRO)0 , and
(ii) the changed probability of the policy being in force at time t − 1.
We now have
0
t−1 px = probability that policy is in force at time t − 1
(9.6.3)
= t−1 px (1 − w1 )(1 − w2 ) . . . (1 − wt−1 )
Hence
σt0 = revised profit signature = t−1 p0x (PRO)0t (9.6.4)

Example 9.6.1. Consider the policy of Example 9.3.1, but now assume that at each of the durations
1,2,3,4 years, 5% of the surviving policyholders will surrender, and that the S.V. is 98% of the office’s
reserve. Find the new profit signature.

Solution. We work out the revised profit vector, (PRO)0t :

t (PRO)t + p55+t−1 (0.05)(0.02 × t V)∗


1 −212.78 + 0.99156 × .05 × .02 × 919 = −211.87
2 29.85 + 0.99058 × .05 × .02 × 1, 876 = 31.71
3 52.15 + 0.98950 × .05 × .02 × 2, 873 = 54.99
4 72.26 + 0.98831 × .05 × .02 × 3, 914 = 79.13
5 102.97 + 0 = 102.97
* when t = 5, (PRO)0t = (PRO)t .

t (PRO)0t 0
t−1 px σt0
1 −211.87 1 −211.87
2 31.71 .99156 × .95 29.87
Hence we have:
3 54.99 .98222 × .952 48.75
4 79.13 .97191 × .953 65.93
5 102.97 .96054 × .954 80.56
Notes
1. The I.R.R. allowing for withdrawals may be found by solving the equation
n
X
v t σt0 = 0 (9.6.5)
t=1

2. Even if (PRO)0t = (PRO)t for t = 1, 2, . . . , n, {σt0 } is not the same as {σt } because the factors
{t−1 p0x } are not the same as {t−1 px }.
3. Unlike mortality rates, which are fairly predictable by actuaries, withdrawal rates depend to a
great extent on economic and commercial factors which are less easy to forecast accurately.
162 CHAPTER 9. PROFIT-TESTING

9.7 The actual emergence of profits


The actual profits emerging from the sale of a given policy depend, of course, on whether the
policyholder dies (or surrenders the policy) during the term, and the year in which such an event
occurs.
Let us suppose here that there are no withdrawals, and use the symbols it and et to denote the actual
interest rate earned by the life funds in year t, and the actual expenses incurred at the beginning of
that policy year. The actual year-end profits emerging in the n years (maximum) of the policy are
as follows:

(1) If the policy is no longer in force at the beginning of policy year t, the profit emerging in that
year is zero;

(2) If the policy is still in force at the beginning of the year, the profit is

(t−1 V + Pt − et )(1 + it ) − Dt (9.7.1)

if the life dies during the year, and

(t−1 V + Pt − et )(1 + it ) − St − t V (9.7.2)

if the life survives the year.

These formulae may be easily modified if the policy is altered, as in the following example.

Example 9.7.1. A life office issues a 5-year with-profits endowment assurance policy to a life aged
exactly 60. The policy has a basic sum assured of £10,000 payable at the end of the year of death
or at the maturity date. Level premiums are payable annually is advance throughout the term of
the policy. Simple reversionary bonuses vest at the start of each year, including the first.
The premium is calculated according to the following basis:

mortality: A1967-70 select

interest: 4% per annum

simple reversionary bonuses at the rate of 4% per annum are assumed

initial expenses: 60% of the first premium

renewal expenses: 5% of each premium after the first

(i) Show that the premium is equal to £2,627.

(ii) The office holds net premium reserves using a rate of interest of 3% per annum and A1967-70
ultimate mortality .
Calculate the profit signature for this policy, assuming that the office will earn interest at
7% per annum on its assets, mortality follows the A1967-70 ultimate table, and expenses and
bonuses will follow the premium basis.
9.7. THE ACTUAL EMERGENCE OF PROFITS 163

(iii) Immediately before the fourth premium was due (before the fourth bonus declaration) the
policy was made paid-up, with no entitlement to further bonuses. The paid up sum assured
was 60% of the benefits immediately before the alteration, including declared bonuses.
The policyholder survived to the maturity date. Interest was earned on the life funds was at
6% per annum over the period of the contract, and bonuses in the first three years followed
the premium assumptions. Expenses followed the premium assumptions up to the alteration
date, and no expenses were incurred after the policy was made paid-up.
For each of the five years of the policy term, calculate the actual year-end profit earned on
the policy.

Solution.
(i) Let P be the annual premium. M.P.V. of premiums less expenses is

0.95P ä[60]:5 − 0.55P = 3.7563P

M.P.V. of benefits is
· ¸
R[60] − R65 − 5M65 + 5D65
10, 000A[60]:5 + 400(IA)[60]:5 = 10, 000 × 0.82565 + 400
D[60]
= 9867.88

Hence
P = £2, 627.02

(ii) The reserves held are:

1V = 10, 400A61:4 − 10, 000P60:5 ä61:4 at 3% interest


µ ¶

= 10, 000 1 − 61:4 + 400A61:4
ä60:5
= 2191.60
µ ¶
ä62:3
2 V = 10, 000 1 − + 800A62:3 = 4, 475.67
ä60:5
µ ¶
ä62:3
3 V = 10, 000 1 − + 1200A63:2 = 6, 862.31
ä60:5
µ ¶
ä64:1
4 V = 10, 000 1 − + 1600A64:1 = 9, 366.17
ä60:5
164 CHAPTER 9. PROFIT-TESTING

This gives the following table:


Year Profit per
Premiums Expenses Reserve Interest Cost of Cost of policy in force
t−1 V Claims Reserves at start of
at start at End year

t (1) (2) (3) (4) (5) (6) (7)


1 2627.02 1576.21 0 73.56 150.10 2159.97 −1185.71
2 2627.02 131.35 2191.60 328.11 172.95 4404.00 438.43
3 2627.02 131.35 4475.67 487.99 198.80 6740.51 520.02
4 2627.02 131.35 6862.31 655.06 227.99 9182.08 602.97
5 2627.02 131.35 9366.17 830.33 12,000 0 692.17

Col. (4) = 0.07[(1) − (2) + (3)]


Col. (5) = cost of death and maturity claims
(including bonuses)
Col. (6) = p60+t−1 .t V
Col. (7) = (1) − (2) + (3) + (4) − (5) − (6)

Hence the profit signature for the contract is

(−1185.71, 483.43p60 . . . 692.174 p60 ) = (−1185.71, 432.10, 504.31, 574.37, 646.38)

(iii) We draw up the following table:

Year Reserve Interest Reserve Survival Profit


at start at end benefits
Pt − et t−1 V tV
t (1) (2) (3) (4) (5) (6)
1050.81 0 63.05 2191.60 0 −1077.74
2 2495.67 2191.60 281.24 4475.67 0 492.84
3 2495.67 4475.67 418.28 6862.31 0 527.31
4 0 6862.31 411.74 6524.25∗ 0 749.80
5 0 6524.25 391.46 0 6720 195.71
∗6720A64:1 at 3% = 6524.25; the sum assured after alteration is 0.6[10, 000 + 3 × 400] = 6720,
and the office calculates reserves by the net premium method.

Col. (3) = 0.06[(1) + (2)]


Col. (6) = (1) + (2) + (3) − (4) − (5)
9.8. EXERCISES 165

Exercises

9.1 (i) In the context of profit-testing, explain the difference between the “profit vector” and
the “profit signature”.
(ii) A certain life office sells assurance policies with term 3 years to lives aged 70. For each
policy, the profit vector is estimated to be (−50, 30, 30). Given that the mortality of the
policyholder is expected to follow A1967-70 ultimate, calculate
(a) the profit signature per policy sold;
(b) the net present value of the profit to the office on the basis of a risk discount rate of
8% per annum.

9.2 A life office issues a 5-year guaranteed bonus endowment assurance policy to a life aged 60,
with basic sum assured £30,000. The sum assured, with attaching bonuses, is payable at the
end of the year of death or at maturity. Level premiums are payable annually in advance.
The office holds net premium reserves, using the basis A1967-70 ult at 3% p.a. Interest on
premiums and reserves is expected to be earned at an effective rate of 8% p.a. Bonuses will be
declared annually at a rate of 3% of the basic sum assured.
Bonuses vest at the start of each policy year. Expenses of 40% of the first year’s premiums and
5% of subsequent years’ premiums will be incurred. Mortality is expected to follow A1967-70
ultimate. Withdrawals may be ignored.
(i) For each policy year, calculate the total sum assured.
(ii) For each duration t = 1, 2, 3, 4 years, calculate the reserve (t V) immediately before pay-
ment of the premium then due, given the following values on A1967-70 ultimate, 3% p.a.
interest:
(IA)60:5 = 4.1853
(IA)61:4 = 3.4681
(IA)62:3 = 2.6973
(IA)63:2 = 1.8672
(IA)64:1 = v = 0.97087
(iii) (Difficult.) Calculate the annual premium (P ) required for the shareholders/with profits
policyholders to achieve an internal rate of return of 12% p.a. on the sale of this contract.
9.3 Ten years ago a life office issued a large block of 10-year without profits endowment assurances
to lives then aged 30. Each policy was effected by annual premiums, and had a sum assured
of £40,000, payable on survival or at the end of the year of death. The office’s premium basis
was
A1967-70 ultimate
5% interest
expenses of 2% of all premiums with additional initial expenses of 0.5% of the sum assured.
It was found that mortality , interest and expenses followed these assumptions, but there were
surrenders just before payment of the premiums due at durations 1,2 and 3 years. At each of
these times, 3% of the surviving policyholders surrendered their contracts, and were given a
surrender value equal to the office’s reserve, which was calculated on the premium basis, minus
a surrender penalty of £40, which the office transferred to the surplus account. By using a
profit-testing approach, or otherwise, calculate the surplus accruing to the office at the end of
each of the first three policy years, per policy sold.
Hint. Note that (PRO)t = 0, so profits arise only from surrenders.
166 CHAPTER 9. PROFIT-TESTING

9.4 Your office is considering the issue of 3-year annual-premium endowment assurance policies
without profits to lives aged 62. In respect of a policy with sum assured £10, 000, payable at
the end of the year of death (if within 3 years) or on maturity, calculate the net present value
of the profit signature on the following assumptions:
premium basis: mortality: A1967-70 ultimate
interest: 6% p.a.
expenses: 3% of all premiums.
reserve basis: net premium method using A1967-70 ultimate,
4% p.a. interest
rate of interest to
be earned in life fund: 8% p.a.
expenses: 3% of office premiums
mortality: A1967-70 ultimate
risk discount rate: 10% p.a.
9.5 A life office issues 3-year term assurance policy to a man aged exactly 59. The sum assured
is £15, 000, payable at the end of the year of death. Level premiums are payable annually in
advance. Expenses are expected to be as follows:
initial expenses: £10
renewal expenses: £2 incurred at the beginning of the 2nd and each subsequent policy year.

It is assumed that interest of 7% per annum will be earned on the life funds, and that mortality
follows the A1967-70 ultimate table. The risk discount rate used by the office is 15% per annum.
The office calculates the annual premium by requiring that the net present value of the expected
profit on each policy is equal to 20% of one office premium. Calculate the office premium on
each of the following reserving bases:
(i) The office holds zero reserves at each year-end.
(ii) The office sets up a reserve at each year-end (except the last) equal to 80% of one office
premium.
9.9. SOLUTIONS 167

Solutions

9.1 (i) The profit vector refers to expected profits per policy in force at start of year, whilst the
profit signature refers to profits per policy sold. Relationship is
σt = (PRO)t × prob. that policy is in force at time t − 1

(ii) (a) σt = t−1 p70 (PRO)t , so


σ1 = −50
µ ¶
l71
σ2 = 30 = 28.83
l70
µ ¶
l72
σ3 = 30 = 27.59
l70

(b) N.P.V. = σ1 v + σ2 v 2 + σ3 v 3 at 8%

= −50v + 28.83v 2 + 27.59v 3

= −46.30 + 24.72 + 21.90 = £0.32

9.2 Note: In (ii), we need the increasing assurance factors (A65+t:5−t and ä65+t:5−t are directly
tabulated)

(i) policy year t total S.A.


1 30,900
2 31,800
3 32,700
4 33,600
5 34,500
(ii) The formula is
t V = (30, 000 + 900t)A60+t:5−t + 900(IA)60+t:5−t − P1 ä60+t:5−t on A67-70 ult. 3%

where
P1 = net premium on A67-70 ult. 3%
30, 000A60:5 + 900(IA)60:5
=
ä60:5
30, 000 × 0.86682 + 900 × 4.1853
= = £6511.67
4.572

This leads to the table (note that 0 V = 5 V = 0):


t tV
1 30, 900A61:4 + 900(IA)61:4 − P1 ä61:4 = 6, 353
2 31, 80062:3 + 900(IA)62:3 − P1 ä62:3 = 12, 947
3 32, 70063:2 + 900(IA)63:2 − P1 ä63:2 = 19, 811
4 33, 60064:1 + 900(IA)64:1 − P1 ä64:1 = 26, 983
(iii) We first work out (CF)t (in terms of P ):
168 CHAPTER 9. PROFIT-TESTING

(1)prs. less exp., with interest (2)death costs (3)survival costs (1)−(2)−(3)
t (P − et )(1.08) q59+t Dt p59+t St (CF)t
1 0.648P 445.96 0 0.648P − 445.96
2 1.026P 509.23 0 1.026P − 509.23
3 1.026P 580.42 0 1.026P − 580.42
4 1.026P 660.39 0 1.026P − 660.39
5 1.026P 34500 1.026P − 34, 500
| {z }
since sum paid on
death or survival
We now work out (PRO)t :

(1)interest on reserve (2)increase in reserve (3)cash flow (1)-(2)+(3)


t 0.08t−1 V (IR)t = p59+t .t V − t−1 V (CF)t (PRO)t
1 0 6261.31 0.648P − 445.96 0.648P − 6707
2 508.24 6386.67 1.026P − 509.23 1.026P − 6388
3 1035.76 6512.36 1.026P − 580.42 1.026P − 6057
4 1584.88 6641.66 1.026P − 660.39 1.026P − 5717
5 2158.64 −26983.00 1.026P − 34, 500 1.026P − 5358
We have the equation
X5
v t t−1 p60 (PRO)t = 0 at 12% interest
t=1

i.e.
(0.648P − 6707)v
+(1.026P − 6388)(0.985568)v 2
+(1.026P − 6057)(0.969785)v 3
+(1.026P − 5717)(0.952572)v 4
+(1.026P − 5358)(0.93385)v 5
=0

∴ 3.2577P = 21, 488


∴ P = £6, 596

(Rough check: right order of magnitude:- P =' P1 , and 5P ' 34, 500 (maturity benefit)).

9.3 Profits arise only in respect of surrenders at times 1,2,3. The profit at time t years (t = 1, 2, 3)
per policy in force at start of year is
(PRO)0t = (1 − q30+t−1 )(0.03) (40) = 1.2p30+t−1
| {z }
surrender
penalty

The probability of being in force at the start of year t is


0 t−1
t−1 p30 = t−1 p30 (1 − 0.03) t=1,2,3

∴ Expected surplus arising at time t, per policy sold, is


σt0 = (1.2)0.97t−1 t p30
9.9. SOLUTIONS 169

t 1.2(0.97)t−1 t p30 = σt0


1 1.2 × 0.99935 = £1.20
2 1.2 × 0.97 × 0.99868 = £1.16
3 1.2 × ×0.972 × 0.99798 = £1.13

9.4 Let P = annual premium

10, 000P62:3 .06 10, 000 × 0.30263


P = =
0.97 0.97
= £3, 119.90
µ ¶
ä63:2
Reserves: 0 V = 0, 1 V = 10, 0001 V62:3 .04 = 10, 000 1 − at 4%
ä62:3

= 3, 146.38
µ ¶
5.138
2 V = 10, 000 1 −
6.519

= 6, 472.66, 3V =0
Net cash flows (ignoring reserves)

t (CF)t
1 0.97P (1.08) − 10, 000q62 = 3, 090.91
2 0.97P (1.08) − 10, 000q63 = 3, 071.86
3 0.97P (1.08) − 10, 000 = −6, 731.59

Profit vector and signature


t tV (PRO)t = (CF)t + i.t−1 V − [p62+t−1 .t V − t−1 V] t−1 p62 σt
1 3,146.38 3, 090.91 + 0 − 3, 090.53 = 0.38 1 0.38
2 6,472.66 3, 071.86 + 251.71 − 3, 199.06 = 124.51 0.98225 122.30
3 0 −6, 731.59 + 517.81 + 6, 472.66 = 258.88 0.96294 249.29
∴ N.P.V. = 0.38v + 122.30v 2 + 249.29v 3 at 10%
= £288.71
170 CHAPTER 9. PROFIT-TESTING

9.5 (i) Let A.P. be P


(1) (2) (3) (4) (5) (6)
Year Premiums Interest Expected (CF)t = (PRO)t t−1 p59 σt = (4) × (5)
less expenses = 0.07 × (1) death costs = (1) + (2) − (3)
1 P−LO 0.07P − 0.7 194.91 1.07P−205.61 1 1.07 P−205.61
2 P−2 0.07P − 0.14 216.49 1.07P−218.63 0.987006 1.0561P−215.79
3 P−2 0.07P − 0.14 240.20 1.07P−242.34 0.972761 1.0409P−235.74
We solve the equation:
0.2P = vσ1 + v 2 σ2 + v 3 σ3 at 15%
205.61 × 1.15−1 + 215.79 × 1.15−2 + 235.74 × 1.15−3
∴ P =
−0.2 + 1.07 × 1.15−1 + 1.0561 × 1.15−2 + 1.0409 × 1.15−3
496.96
= = £224.53
2.2133
(ii)
(1) (2) (3) (4) (5) (6)
Year Reserve Interest (CF)t Reserve Increase in (PRO)t =
at start on reserve at end reserve
of year = 0.07 × (1) of year = p59+t−1 .t V − t−1 V = (2) + (3) − (5)
1 0 0 1.07P−205.61 0.8P 0.7896 0.28039P−205.61
2 0.8P 0.056P 1.07P−218.63 0.8P −0.01155 1.13755P−218.63
3 0.8P 0.056P 1.07P−242.34 0 −0.8 1.926 P−242.34
(7)
σt =
t−1 p59 .(PRO)t
0.28039P−205.61
1.12277P−215.76
1.87354P−235.74
We solve the equation:
0.2P = vσ1 + v 2 σ2 + v 3 σ3 at 15%
∴ P = £233.90
Chapter 10

STATIONARY POPULATIONS

10.1 Some Definitions


We recall that

ex = complete expectation of life at age x
= E(T )

where T refers to the future lifetime of (x) in years, including fractions.


Note that
Z ∞

ex = t.t px µx+t dt (10.1.1)
0
Integrating by parts gives
Z ∞

ex = [t(−t px )]∞
0 + t px dt
0
Z ∞
= t px dt
Z0 ∞
lx+t
= dt (10.1.2)
0 lx

(as it may be shown that limt→∞ (t.t px ) = 0 when ex is finite.)
We recall also that

ex = curtate expectation of life at age x


= E(K)

where K is the integer part of T .


We find that

X
ex = t px (10.1.3)
t=1

and, using the Euler–Maclaurin formula,


◦ 1
ex 'ex −
2

171
172 CHAPTER 10. STATIONARY POPULATIONS

Define

Z ∞
Tx = lx+t dt
Z0 ∞
= ly dy (on setting y = x + t)
x

from which we obtain

R∞
◦ x
ly dy
ex =
lx
Tx
=
lx
and hence

Tx = lx ex (10.1.4)
A stationary population is an idealised large population such that (in an old-fashioned mathe-
matical notation) the number of lives in the population between ages x and x + dx is always equal
to
klx dx
where k is called the scaling factor and lx is according to some life table.
By integrating over all ages greater than or equal to x, we can see that

Z ∞
Total population aged ≥ x = kly dy
x
= k.Tx (10.1.5)

The position is illustrated in Figure (10.1.1).

Figure 10.1.1: a stationary population

(It is nearly always advisable to draw a diagram in stationary population questions.) In any time-
interval of length t years, the number of lives attaining a specified exact age x is klx .t, so the rate at
which lives attain age x is klx per annum, i.e. klx lives ‘flow’ continuously over exact age x each year.

Note that (if lives are considered at all ages) there will be kl0 births each year. Also, the num-
ber of deaths each year between ages x1 and x2 (x1 < x2 ) is

k(lx1 − lx2 ) (10.1.6)

The number of lives in the population between any two ages x1 and x2 (x1 < x2 ) is

k(Tx1 − Tx2 ) (10.1.7)


10.1. SOME DEFINITIONS 173

Example 10.1.1. The membership of a certain learned society is stationary at 11,500. Members
enter only at exact age 50. They are subject to the mortality of English Life Table No.12 - Males,
and there are no withdrawals.

What is the annual number of entrants ?

Solution
One knows that
k.T50 = 11, 500

So

kl50 e50 = 11, 500

The annual number of entrants is


11, 500
kl50 = ◦ = 507.05 or 507.
e50

Example 10.1.2. A large company has for many years maintained a staff in a stationary condition
by recruiting 500 annual entrants at exact age 20, uniformly over the year. If the staff retire at age
60, there are no withdrawals, and English Life No. 12 – Males mortality is experienced, find:
(a) the size of the staff,
(b) the number of staff who retire each year,
(c) the number of pensioners.

Solution

(a) We have
kl20 = 500.

The total number of staff is


500 ◦ ◦
k(T20 − T60 ) = (l20 e20 −l60 e60 )
l20
= 19, 115.

(b) Number who retire each year

= kl60
µ ¶
500
= l60
l20
= 410

(c) Number of pensioners

= kT60
µ ¶
500 ◦
= l60 e60
l20
= 6, 172.
174 CHAPTER 10. STATIONARY POPULATIONS

Some more symbols


We let
Lx = the number of lives between ages x
and x + 1 in a stationary population with scaling factor k = 1.
= Tx − Tx+1
Z 1
= lx+t dt (10.1.8)
0

We may use the approximations


1
Lx ' lx+ 12 ' (lx + lx+1 )
2
Another definition is


ex:n = average lifetime lived by (x) between ages x and x + n
= E(T ∗ )
where the random variable T ∗ is defined as follows:
(
∗ T if (x) dies within n years
T =
n if (x) lives for n years.
The variable T ∗ has p.d.f. t px µx+t for 0 < t < n, and there is discrete probability n px that T ∗ = n.
Hence

Z n

E(T ) = t.t px µx+t dt + n.n px
0
Z n
n
= [−t.t px ]0 + t px dt + n.n px (by integration by parts)
0
Z n
= t px dt
0

Hence
◦ Tx − Tx+n
ex:n =
lx
◦ lx+n ◦
=ex − ex+n (10.1.9)
lx

(Notice that āx:n =ex:n if i = 0 .)

Similarly,

ex:n = average number of complete years lived by


(x) between ages x and x + n
X n
lx+1 + lx+2 + ... + lx+n
= t px =
t=1
lx
lx+n
=ex − ex+n (10.1.10)
lx
10.2. THE CENTRAL DEATH RATE 175

Example 10.1.3. You are given that


· ¸
log x
lx = l1 1 − (1 ≤ x ≤ 11)
9 log 10

Find e1:10 .
Solution

Z 10 Z 11
◦ lt+1 lx
e1:10 = dt = dx
0 l 1 1 l1
Z 11 · ¸
log x
= 1− dx
1 9 log 10
1 x=11
= 10 − [x log x − x]x=1
9 log 10
11 log 11 − 10
= 10 −
9 log 10
= 9.2097.

The average age at death of those who die between ages x and x + n may be worked out as
follows. We have

E(T ∗ ) = E(T ∗ |T ∗ < n)P r{T ∗ < n} + E(T ∗ |T ∗ ≥ n)P r{T ∗ ≥ n}

so that

ex:n = E(T ∗ |T ∗ < n)n q x + n.n px
Hence the average age at death of those who die between ages x and x + n is

x + E(T ∗ |T ∗ < n)

ex:n −n.n px
=x +
n qx
Tx − Tx+n − nlx+n
=x + (10.1.11)
lx − lx+n
Note that this result refers to a randomly-chosen life aged x: we need not assume that there is a
stationary population.

10.2 The Central Death Rate


Define

h mx = the central death rate between ages x and x + h


lx − lx+h
= R x+h
x
ly dy
number of deaths each year between ages x and x + h
=
population aged between x and x + h in a stationary population
If h = 1, it is omitted, giving
dx
mx = (10.2.1)
Lx
176 CHAPTER 10. STATIONARY POPULATIONS

Note that
R1
0
lx+t µx+t dt
mx = R1 ' µx+ 12 (10.2.2)
0
lx+t dt

10.3 Relationships Between mx and qx


Assume that there is a Uniform Distribution of Deaths (U.D.D.) between ages x and x + 1, i.e. lx+t
is linear for 0 ≤ t ≤ 1.
We have
Z 1
1
Lx = lx+t dt = (lx + lx+1 ) (10.3.1)
0 2
(as the trapezoidal rule is exactly correct, not just an approximation).
Since lx+1 = lx − dx , we have 1
Lx = lx − dx (10.3.2)
2
Hence
dx
mx =
Lx
dx
=
lx − 12 dx
qx
= (10.3.3)
1 − 12 qx

We may rearrange this equation to get


mx
qx = (10.3.4)
1 + 12 mx
If U.D.D. does not hold, these results may be used as approximations.

10.4 Stationary Funds


Consider a pension scheme, etc., such that
B = annual benefit outgo for scheme (plus expenses, if any)
and
C = annual contribution income
are constant.
If the funds, F , are such that the interest on them pays B − C, then the funds will also remain
constant.
There are two cases:
(a) If income and outgo are received and paid continuously, then

δ.F = B − C (10.4.1)

(B, C are rates of payment per annum).


(b) If income and outgo are received and paid only at the end of each year, then

iF = B − C (10.4.2)

Note: In case (b), F denotes the fund at the start of the year.
10.4. STATIONARY FUNDS 177

Example 10.4.1. Each year for many years a life office has issued 10,000 temporary assurance
policies each with a term of ten years and a sum assured of £5,000 to lives aged 25 exactly.

One third of those who survive to age 35 then effect a without profits whole life policy for the
same sum assured as the term policy, and one quarter effect a 25-year without profits endowment
assurance for twice that sum assured. All premiums are payable annually in advance and death
claims are paid at the end of the year of death. Policies are issued uniformly throughout the year.

The office calculates premiums on A1967-70 ultimate 4%, ignoring expenses. If the office’s ex-
perience follows this basis, calculate the size of the fund held for these contracts.
Solution
The term assurance premium = 5, 000 M25 −M35
N25 −N35 = £3.29.
Whole life premium = 5, 000P35 = £56.85.
Endowment assurance premium = 10, 000P35:25 = £245.30.
Annual premium income is

· ¸
l35
10, 000 1 + e25 − (1 + e35 ) × 3.29
l25
l35 1
+ 10, 000 × (1 + e35 ) × 56.85
l25 3
· ¸
l35 1 l60
+ 10, 000 × 1 + e35 − (1 + e60 ) × 245.30
l25 4 l35
= 328, 003 + 7, 634, 065 + 14, 769, 057
= £22, 731, 125.

Annual rate of payment of claims is


· µ ¶ µ ¶¸
l25 − l35 l35 1 1
10000 5000 + × 5000 + × 10, 000
l25 l25 3 4
= £41, 723, 716.

Hence δ.F = 41, 723, 716 − 22, 731, 125


= 18, 992, 591

where i = 0.04.
18992591
Therefore Fund = δ = £484, 249, 000.
178 CHAPTER 10. STATIONARY POPULATIONS

Exercises

10.1 The staff of a large company is maintained as a stationary population by 500 new entrants
each year at exact age 20. One third of those reaching age 30 leave immediately. Of the
remainder, 41 of those attaining age 60 retire immediately and the survivors retire at age 65.
The only other decrement is death.
Calculate
(i) the number of staff,
(ii) the number of deaths in service each year.
Basis: English Life Table No. 12 - Males
10.2 A certain country’s school system provides education for all children between the ages of 5
and 16 exactly. The country’s population is stationary, there being 100,000 births per year,
uniformly distributed over the year. The population of the country is subject to the mortality
of English Life Table No. 12 - Males.
(i) Find the number of pupils at any given time.
(ii) The country’s teacher training colleges are such that a constant flow of new entrants to
the profession is maintained. Teachers are recruited uniformly over the year, and the ratio of
pupils to teachers is 20 to 1. All teachers enter the profession at age 21 and retire at age 60,
there being no withdrawals. Find the annual number of new teachers recruited.
10.3 For many years a company has recruited, uniformly over each year, 200 employees on their
20th birthdays and a fixed number of additional employees on their 25th birthdays. Mor-
tality has followed English Life Table No. 12 - Males. Employees may retire on their 60th
or 65th birthdays, and one third of employees reaching their 60th birthdays retire on that
date. Employees leave the company only through death or retirement, and the total number
of employees is 10,000.

Find the total number of new recruits each year.


10.4 The male population of a certain country has been stationary for many years, there being
100,000 male births per annum, spread uniformly over the year; the mortality of males follows
English Life Table No.12 - Males, and migration may be ignored.
(a) Calculate the size of the country’s male population at any time.
(b) The government of the country has decided to introduce a social security plan, under
which all employed men between ages 15 and 65 must contribute a fixed sum every week, the
same sum being also payable by their employers. Men over age 65 will receive a pension of 100
units of currency per week, and those sick or unemployed between ages 15 and 65 also receive
this amount. If it may be assumed that at any time 95% of men between ages 15 and 65 are
employed, while the remaining 5% are sick or unemployed, calculate the weekly contribution
payable by each employed worker. The scheme is to be financed on a pay-as-you-go basis (no
fund is built up) and administrative costs are to be ignored.
10.5 A large manufacturing company has for many years staffed one of its divisions by the recruit-
ment, uniformly over each year, of 1,000 staff at exact age 20. At the end of one year in the
job, new staff are examined for suitability, and 20% are dismissed. All employees are assessed
at age 35, and 50% are immediately moved out of the division. At age 40, all remaining em-
ployees are moved out of the division. Death is the only other reason for leaving the division.
Staff experience mortality according to English Life Table No. 12 - Males.
Calculate the number of staff in the division.
10.5. EXERCISES 179

10.6 In a certain country, all civil servants are recruited on their 25th birthdays. (Birthdays are
assumed to occur uniformly over the calendar year). Of those who reach exact age 40, 10%
obtain employment in private companies and leave the civil service immediately. All remaining
civil servants retire when they reach age 60 or die in service before this age. Civil servants
have for many years experienced the mortality of English Life Table No. 12 - Males, and will
do so for the indefinite future. The population of civil servants has been stationary for many
years, and civil servants are recruited at the rate of 3,000 each year.
(i) How many civil servants are in service at any given date?
(ii) The government of the country has just decided to reduce the size of the civil service by
10%. This is to be done by immediately lowering the retirement age of civil servants. Find
the new retirement age (to the nearest month), assuming that there are no other changes.
10.7 In a mortality table with a one-year select period, the following relationships apply at age x:
1
l[x] = (lx + lx+1 )
2
t q[x] = t.q[x] (0 ≤ t ≤ 1)

(i) Express p[x] in terms of px .


(ii) Express e[x] in terms of ex and px .
◦ ◦
(iii) Express e[x] in terms of ex+1 and px .
(iv) Express m[x] in terms of px .
10.8 For many years a life office has issued a steady flow of 10-year endowment assurances without
profits to lives aged 55. Premiums are payable continuously and the sum assured is payable
immediately on death, if death occurs before age 65. Premiums are calculated on the following
basis:
mortality: English Life Table No. 12 – Males
interest: 4%
expenses: 5% of all office premiums, plus 25% of office premiums in the first year.
Mortality, interest and expenses have always been in accordance with this basis. The sums
assured issued each year have always been £5,000,000, spread uniformly over the year. No
policies terminate except by death or maturity.
In respect of this business, calculate
(i) the annual rate of gross premium income,
(ii) the annual rate of payment of maturity claims,
(iii) the annual rate of payment of death claims, and
(iv) the size of the fund which has been built up.
180 CHAPTER 10. STATIONARY POPULATIONS

Solutions

10.1 (i) kl20 = 500.


µ ¶
1 1 1
Number of Staff = k T20 − T30 − T60 − T65
3 6 2
µ ¶
500 ◦ 1 ◦ 1 ◦ 1 ◦
= e e e e
l20 20 − l30 30 − l60 60 − l65 65
l20 3 6 2
= 15, 361

(ii) Number of deaths is µ ¶


500 1 1 1
500 − l30 + l60 + l65
l20 3 6 2
= 89.

10.2 (i) kl0 = number of births each year = 100,000


As l0 = 100, 000, k = 1.

Number of pupils = k(T5 − T16 )


◦ ◦
= l5 e5 −l16 e16
= 1, 066, 409

1
(ii) Number of teachers = 20 × 1, 066, 409 = 53, 320.
Let annual number of recruits be c.
Then the equation is
c
(T21 − T60 ) = 53, 320
l21
Hence c = 1, 431 teachers per annum.
10.3 kl20 = 200 employees enter at age 20 per annum. Let cl25 be the number who enter at age 25
per annum.
Then the number µof employees is ¶ µ ¶
1 2 1 2
k T20 − T60 − T65 + c T25 − T60 − T65 = 10, 000
3 3 3 3

where k = 200
l20 .
Solving this for c gives

10, 000 − 8157.81


c=
3, 447, 483
= 0.00053436

Hence number recruited each year = 200 + cl25


= 251

10.4 (a) kl0 = 100, 000


Hence k = 1.
10.6. SOLUTIONS 181

Total male population = kT0



= 100, 000 e0
= 6, 809, 000

(b) Number of pensioners = kT65


Number aged between 15 and 65 who receive benefit = 0.05k(T15 − T65 ).
Number of contributors = 0.95k(T15 − T65 ).

Let the employee s contribution be c per week.


Hence 2c × (number of contributors) = 100 × (no. of beneficiaries)
Therefore

100[T65 + 0.05(T15 − T65 )]


c=
2 × 0.95(T15 − T65 )
T15 = 5, 352, 735 and T65 = 818, 455 giving
c = 12.13 units.
10.5 kl20 = number of staff recruited per annum

= 1, 000
Number of staff = k(T20 − T21 ) + 0.8k(T21 − T35 ) + 0.4k(T35 − T40 )
1000
= (T20 − 0.2T21 − 0.4T35 − 0.4T40 )
l20
= 14, 060

10.6 (i) kl25 = 3, 000


Number of civil servants is

k(T25 − 0.1T40 − 0.9T60 )


3000 ◦ ◦ ◦
= (l25 e25 −0.1l40 e40 −0.9l60 e60 )
l25
= 94713

(ii) Number of civil servants required is


0.9 × 94713 = 85, 242

Let x be the new age of retirement.


Then
3000
(T25 − 0.1T40 − 0.9Tx ) = 85, 242
l25
Hence Tx = 1, 524, 475

T55 = 1, 602, 333


T56 = 1, 516, 571

So using linear interpolation gives


x ' 55.91
' 55 years and 11 months
182 CHAPTER 10. STATIONARY POPULATIONS

10.7 (i)
lx+1 lx+1 2px
p[x] = = 1 =
l[x] 2 (lx + lx+1 ) 1 + px

(ii)
lx+1 + lx+2 + ...
e[x] =
l[x]
µ ¶
lx p[x] 2
= · ex = · ex = ex
l[x] px 1 + px

(iii) Z ∞
◦ 1
e[x] = [ l[x]+t dt]
l[x] 0
Z 1 Z ∞
1
= [ l[x]+t dt +
lx+t dt]
l[x] 0 1
1 1 ◦
= [ (l[x] + lx+1 ) + lx+1 . ex+1 ]
l[x] 2

(as t q[x] = t.q[x] , l[x]+t is linear for 0 ≤ t ≤ 1)


1 ◦
= (1 + p[x] ) + p[x] . ex+1
2µ ¶ µ ¶
1 1 + 3px 2px ◦
= + ex+1
2 1 + px 1 + px

(iv)
R1
l[x]+t µ[x]+t dt
0
m[x] = R1
l
0 [x]+t
dt
l[x] − lx+1
= 1
2 (l[x] + lx+1 )
1 − p[x]
= 1
2 (1 + p[x] )
2(1 − px )
= (using part (i))
1 + 3px

10.8 (i) Let P 0 be the office annual premium per £1 sum assured.
We have
0.95P 0 ā55:10 = A55:10 + 0.25P 0 ā55:1
Hence P 0 = 0.100663.
The total sums assured in force at any given time are
5, 000, 000
(T55 − T65 )
l55
5, 000, 000
(£5,000,000 “flows across” age 55, so the scaling factor is .)
l55
µ ¶
◦ l65 ◦
Total sums assured = 5, 000, 000 e55 − e65
l55
= £45, 618, 854.
10.6. SOLUTIONS 183

Hence rate of gross premium income = £4, 592, 130 p.a.

l65
(ii) 5, 000, 000 = £3, 985, 870
l55

(iii) 5, 000, 000 − 3, 985, 870 = £1, 014, 130

(iv) Annual rate of payment of benefits = £5, 000, 000

Annual rate of payment of expenses = 5% of gross premium income


+ 25% of gross premium income in first policy year
= 0.05 × 4, 592, 130
· µ ¶ ¸
◦ l56 ◦
+ 0.25 5, 000, 000 e55 − e56 × 0.100663
l55
= £355, 209

Let funds be F . We have


δF = 5, 000, 000 + 355, 209 − 4, 592, 130
Hence F = £19, 456, 000 (final figures are unreliable.)
184 CHAPTER 10. STATIONARY POPULATIONS
Chapter 11

JOINT-LIFE FUNCTIONS

11.1 Joint-Life Mortality Tables


Consider 2 independent lives aged x and y respectively, subject to the mortality of Tables I and II
respectively.
Let T = min{T1 , T2 },
where T1 = future lifetime of (x)
and T2 = future lifetime of (y).
That is, T is the “joint future lifetime” of (x), (y), i.e. the time until the first of them dies.

Remark
If (x) and (y) are a married couple, or business partners, the assumption of independence of T1 , T2
is questionable: consider, for example, a car accident in which both lives are killed. But in practice
independence is normally assumed.
Define
I II
t pxy = P r{both (x), (y) will survive for t years at least}
= t pIx · t pII
y (by independence of T1 , T2 ) (11.1.1)

and
I II
t q xy = P r{T ≤ t}
= the distribution function of T (11.1.2)
I II
=1− t pxy (11.1.3)

Note A common error is to write


I II
t q xy = t q Ix · t q II
y

If Table I = Table II, we may omit the superscripts, so (for example)

t pxy = t px · t py

on the table under consideration.


Note In the a(55) tables, the notation t pxy , etc. refers to a male life aged x and a female life
aged y (the male life comes first.) That is, Table I is a(55) males ultimate, Table II is a(55) females
ultimate.

185
186 CHAPTER 11. JOINT-LIFE FUNCTIONS

From now on we will omit “I, II” (unless needed.) Define

lxy = lx ly

(x ≥ α1 , y ≥ α2 , where α1 , α2 are the youngest ages in tables I, II respectively). By equation 11.1.1,


lx+t:y+t
t pxy =
lxy
Note that

t q xy = 1 − t pxy
= 1 − t px .t py
= 1 − (1 − t q x )(1 − t q y )
= t q x + t q y − t q x .t q y (11.1.4)

Using equation 11.1.3,


lxy − lx+t:y+t
t q xy =
lxy
and
dxy = lxy − lx+1:y+1 .

The force of mortality is defined as

h qxy
µxy = lim
h→0+ h
h q ³ q ´ ³ q ´ i
h x h qy h x h y
= lim + − · h
h→0+ h h h h
= µx + µy (11.1.5)
h qx q
(since h → µx , hhy → µy as h → 0+ ).

Example 11.1.1.

Given lxy = 1, 000


lx+10:y = 960
lx:y+10 = 920

calculate the probability that, of two lives aged x and y, only one will survive for 10 years.
Solution
lx+10:y
10 px = = 0.96
lx:y
lx:y+10
10 py = = 0.92
lxy

Required probability =10 px (1 −10 py ) +10 py (1 −10 px )


= 0.1136.
11.1. JOINT-LIFE MORTALITY TABLES 187

Theorem
Z t
t pxy = exp(− µx+r:y+r dr) (11.1.6)
0

Proof. From (11.1.1),


Z t Z t
t px t py = exp[− µx+r dr] · exp[− µy+r dr]
0 0
Z t
= exp[− (µx+r + µy+r ) dr]
0
Z t
= exp[− µx+r:y+r dr] by (11.1.5)
0

The probability density function (pdf ) of T.


Note that
F (t) = P r{T ≤ t} = t q xy

is the distribution function of T .


(When t < 0, the distribution function of T is zero).
Now

f (t) = p.d.f. of T
= F 0 (t)
d
= [1 − t pxy ] (for t > 0)
dt
· µ Z t ¶¸
d
=− exp − µx+r:y+r dr (from (11.1.6)).
dt 0
· Z t ¸
= exp − µx+r:y+r dr µx+t:y+t (using Chain Rule)
0

(
t pxy µx+t:y+t (t > 0)
Thus f (t) = (11.1.7)
0 (t < 0)

In particular, we must have


Z ∞ Z ∞
t pxy µx+r:y+r dr = t pxy (µx+t + µy+t ) dt
0 0
=1

since the integral of a p.d.f. is always equal to 1.

The discrete variable K = integer part of T


This is the number of complete years to be lived in the future by the “joint life status” of (x, y).
Note that
K = min{K1 , K2 }.
188 CHAPTER 11. JOINT-LIFE FUNCTIONS

where K1 = integer part of T1 , K2 = integer part of T2 . This variable is discrete, with probabilities

P r{K = k} = P r{1st death occurs between times k and k + 1}


= k |qxy
=k pxy −k+1 pxy (by an argument similar to that for a single life)
lx+k:y+k − lx+k+1:y+k+1
=
lxy
dx+k:y+k
= (k = 0, 1, 2 . . .) (11.1.8)
lxy

11.2 Select Tables


We replace (x), (y) by [x] + r, [y] + u (referring to lives aged x + r, y + u respectively who were
selected r, u years ago respectively). Define

l[x]+r:[y]+u = l[x]+r · l[y]+u

which leads to
l[x]+r+t:[y]+u+t
t p[x]+r:[y]+u =
l[x]+r:[y]+u

Example 11.2.1. On A67 − 70 calculate 3 p[59]+1:[69]+1 .


Solution
l[59]+4:[69]+4
3 p[59]+1:[69]+1 =
l[59]+1:[69]+1
l63 .l73
=
l[59]+1:[69]+1
= 0.856.

We also define ³ q ´
h [x]+r:[y]+u
µ[x]+r:[y]+u = lim = µ[x]+r + µ[y]+u
h→0+ h
The p.d.f. of T = future lifetime of ([x] + r, [y] + u) is
(
t p[x]+r:[y]+u .µ[x]+r+t:[y]+u+t (t ≥ 0)
f (t) =
0 (t < 0).

11.3 Extensions to More than 2 Lives


We extend the definitions of T , t pxy , etc., as follows: consider 3 lives aged x, y, z respectively, whose
future lifetimes are T1 , T2 , T3 respectively (assumed to be independent).

t pxyz = P r{(x), (y) and (z) all survive for at least t years}
= t px .t py .t pz
lx+t:y+t:z+t
=
lxyz
11.3. EXTENSIONS TO MORE THAN 2 LIVES 189

where lxyz = lx ly lz for all x, y, z ≥ α1 , α2 , α3 (α1 , α2 , α3 being the youngest ages in tables I, II, III).
Other functions follow similarly; in particular

h qxyz
µxyz = lim+ = µx + µy + µz
h→0 h
and T = min{T1 , T2 , T3 } has p.d.f.
(
t pxyz µx+t:y+t:z+t = t pxyz (µx+t + µy+t + µz+t ) (t ≥ 0)
f (t) =
0 (t < 0)

Example 11.3.1. Evaluate Z ∞


t pxxxx µx+t:x+t dt
0
Solution
Z ∞ Z ∞
t pxxxx µx+t:x+t dt = 0.5 t pxxxx (4µx+t ) dt.
0 0
= 0.5.
To find the probability that exactly one life out of three survives for t years, use the formula
P r{(x) survives but (y), (z) die}
+P r{(y) survives but (x), (z) die}
+P r{(z) survives but (x), (y) die}
(note that these events are mutually exclusive)
= t px (1 − t py )(1 − t pz ) + t py (1 − t px )(1 − t pz ) + t pz (1 − t px )(1 − t py )
Similar calculations apply to
P r{exactly one life dies}
and other possibilities.
Also
P r{at least 1 dies within t years} = t q xyz
= 1 − P r{all 3 survive}
= 1 − t pxyz
= 1 − t px t py t pz
= 1 − (1 − t q x )(1 − t q y )(1 − t q z ).

Example 11.3.2. The probability that exactly one of three lives aged x will survive for n years is
27 times the probability that all three will die within n years. Find the probabilities that
(a) at least two will survive n years
(b) at least one will die within n years
Solution
(a) 3n px (1 −n px )2 = 27(1 −n px )3
thus n px = 9(1 −n px )
hence n px = 0.9, so n qx = 0.1.
190 CHAPTER 11. JOINT-LIFE FUNCTIONS

By the binomial distribution, the probability of at least two survivors


µ ¶
3
= (n px )2 (1 −n px ) + (n px )3
2
= 3(n px )2 − 2(n px )3 = 0.972

(b) P r{at least one will die} = 1 − (n px )3


= 1 − (0.9)3
= 0.271

11.4 The Joint Expectation of Life


Define

exy = the complete joint expectation of life for the pair aged (x, y)
= E(T ), where T = min{T1 , T2 } as before.

Hence Z ∞

exy = t.t pxy .µx+t:y+t dt
0

Proceeding as in single-life case, using integration by parts, we get


Z ∞

exy = t pxy dt (11.4.1)
0

We also have

exy = curtate joint expectation of life of (x, y)


= E(K) where K = integer part of T

X
= t pxy (11.4.2)
t=1

The Euler–Maclaurin formula gives the approximation

◦ 1
exy 'exy −
2
The variance of T is calculated as follows:

Var(T ) = E(T 2 ) − [E(T )]2


Z ∞

= t2 .t pxy µx+t:y+t dt − (exy )2
0
Z ∞ ·Z ∞ ¸2
= 2t.t pxy dt − t pxy dt
0 0

(Use integration by parts; similar to the single-life case).


11.5. MONETARY FUNCTIONS 191

11.5 Monetary Functions


Notation is very similar to single-life case (changing ‘x’ to ‘xy’). See “International Actuarial Nota-
tion” in “Formulae and Tables for Actuarial Examinations”.
(a) Annuities
We define

āxy = mean present value (m.p.v.) of an annuity


of £1 per annum (p.a.) payable continuously
so long as both (x), (y) are alive
= E[āT ] where T = future lifetime of joint-life status
Z ∞
= āt̄| .t pxy µx+t:y+t dt
0
Z ∞
= v t t pxy dt (11.5.1)
0

1
(the proof is similar to single-life case; note that v = 1+i ). Note also that

äxy = m.p.v. of an annuity of £1 p.a. payable at the


start of each year so long as both (x), (y) are alive
= E[äK+1 ] where K = integer part of T
X∞
= v t t pxy . (11.5.2)
t=0

Other joint-life annuity functions are


X
axy = v t .t pxy = äxy − 1
t=1
äxy:n = äxy −n pxy v n äx+n:y+n

Woolhouse’s approximation for annuities payable mthly hold for joint lives, i.e.

m−1
ä(m)
xy ' äxy −
2m
(m) m−1
äxy:n ' äxy:n − (1 − n pxy v n )
2m
The Euler–MacLaurin formula gives

1 1
āxy ' äxy − ' axy + .
2 2

Evaluation of annuities using commutation functions.


Commutation functions are not often used, and the only functions available in the examination
192 CHAPTER 11. JOINT-LIFE FUNCTIONS

tables are
1
Dxy = v 2 (x+y) lxy (11.5.3)
X∞
Nxy = Dx+t:y+t (11.5.4)
t=0

From these we get

n Exy = m.p.v. of pure endowment of £1 payable


at time n if (x), (y) are both alive
= v n .n pxy
Dx+n:y+n
=
Dxy

and

X lx+t:y+t
äxy = vt
t=0
lxy
Nxy
= .
Dxy

Note: In A67 − 70 we only find Dxx and Nxx (i.e. equal ages only.)

(b) Assurances

Āxy = m.p.v. of £1 payable immediately on the failure of the joint-life status


= E(v T ) where T = future lifetime of joint-life status
Z ∞
= v t .t pxy µx+t:y+t dt (11.5.5)
0

This can be evaluated numerically by approximate integration. Note that


µ ¶
1 − vT 1 − Āxy
āxy = E =
δ δ
so we have the “conversion” relationship

Āxy = 1 − δāxy .

We also have

X
Axy = E[v K+1 ] = v t+1 t |qxy (11.5.6)
t=0

and it will now be shown that


Axy = 1 − däxy
Proof
µ ¶
1 − v K+1
äxy = E[äK+1 ] = E
d
1
= (1 − Axy )
d
11.5. MONETARY FUNCTIONS 193

So Axy = 1 − d.äxy

An approximation which can often be used is


1
Axy ' (1 + i)− 2 Āxy
Write Ā 1 to indicate that a sum assumed is payable on the first death of (x), (y) if within n
z}|{
xy :n
years.

Example 11.5.1. Show how one can evaluate Ā 1 on the a(55) tables (male/female.)
z}|{
xy :n
Solution. Method 1:
1
Ā 1 ' (1 + i) 2 A 1
z}|{ z}|{
xy :n xy :n
1
= (1 + i) 2 [Axy:n − v n ·n pxy ] (as in single-life case).
½ ¾
1
n lx+n ly+n
= (1 + i) 2 1 − däxy:n − v ·
lx ly

lx+n ly+n
Now use äxy:n = äxy − v n · äx+n:y+n
lx ly
and finally use äxy = axy + 1.

Method 2:

Ā 1 = Āxy:n − v n ·n pxy
z}|{
xy :n
lx+n ly+n
= (1 − δāxy:n ) − v n · ·
lx ly
· ¸
1 lx+n ly+n 1
' 1 − δ(axy + ) − v n · · 1 − δ(ax+n:y+n + ) .
2 lx ly 2

(c) Premiums
We employ equations of value in the same way as for single-life policies. If there are no expenses we
obtain net premiums, e.g., in the International Actuarial Notation,

Āxy 1
P̄ (Āxy ) = = −δ
āxy āxy

If there are expenses, calculate office premiums as for single-life policies by setting up an equation
of value. The general symbols P and P 0 may be used for any net or gross premium.
194 CHAPTER 11. JOINT-LIFE FUNCTIONS

(d) Reserves
Calculated as for single-life policies, e.g.,

t V̄ (Āxy ) = net premium reserve at duration t years for


the joint-life assurance of £1, payable immediately
on the first death of two lives aged x and y
at the issue date, affected by annual
premiums payable continuously during the joint-lifetime.
= Āx+t:y+t − P̄ (Āxy )āx+t:y+t

(using prospective method).


Note: The formula for retrospective reserve in this example is
Dxy
[P āxy:t − A 1 ].
Dx+t:y+t z}|{
xy :t
Also, one may show that
āx+t:y+t
tV (Axy ) = 1 −
āxy
using conversion relationships. We use the general symbol t V for the reserve at duration t, just
before payment of any premium then due.

Exam tables
(i) A67-70 gives:

äxx , Nxx , Dxx , ä[xx] (= ä[x]:[x] ), äxxx , äxxxx

all at 4% p.a. interest.


(ii) a(55) gives axy at 2-year intervals for male aged x and female aged y (x, y ≥ 60 only), at 4%,
6% and 8% interest.
Interpolation is needed to get (for example) a61:69 , as follows:
1
a61:69 ' [a60:68 + a62:68 + a60:70 + a62:70 ].
4

Example 11.5.2. Two lives, aged 50 and 60, effect a 10-year temporary joint life assurance with
sum assured £60,000. The sum assured is payable immediately on the first death, provided that
this occurs within 10 years of the issue date. The policy has annual premiums, payable during the
joint lifetime of both lives for at most 10 years.
1
(i) Express Āz }| as an integral. Estimate the value of the integral by the following form of
{
50 : 60:10
Simpson’s rule: Z 10
10
f (t) dt = · [f (0) + 4f (5) + f (10)]
0 6
(ii) Hence, or otherwise, estimate the values of A50:60:10 and ä50:60:10 , and calculate the annual
premium for the policy.
Basis: A1967 − 70 ultimate mortality
4% p.a. interest
No expenses.
11.6. LAST SURVIVOR PROBABILITIES (TWO LIVES ONLY) 195

Solution R 10
1
(i) Āz }| { = 0 v t .t p50:60 (µ50+t + µ60+t ) dt.
50 : 60:10
Simpson’s rule gives an approximate answer of

Āz 1 = 0.2238.
}| {
50 : 60:10

(ii)

l60 l70
A50:60:10 = Az 1
}| { + v 10 ·
50 : 60:10 l50 l60
1 l70
' (1.04)− 2 .Āz 1
}| { + v 10
50 : 60:10 l50
= 0.70792.
1 − A50:60:10
ä50:60:10 = = 7.594.
d
The premium P p.a. is
Āz 1
}| {
50 : 60:10
P = 60000 = £1, 768.24
ä50:60:10

11.6 Last Survivor Probabilities (two lives only)


Define,

t pxy = P r{at least 1 of (x), (y) will be alive in t years’ time}


= 1 − t q xy .

where t q xy = P r{last survivor of (x), (y) dies within t years}.


By elementary probability theory,

t pxy = t px + t py − t pxy

and hence
t q xy = t q x + t q y − t q xy
These formulae are true even if (x), (y) are not independent. If they are independent, we may write

t pxy = t px · t py

giving
t pxy = t px + t py − t px · t py (11.6.1)
and
t q xy = t qx · t qy . (11.6.2)
Note that t q xy is the distribution function of the variable

Tmax = max{T1 , T2 }.

and the p.d.f. of Tmax = time to death of last survivor is

d d d d
f (t) = (t q ) = (t q x ) + (t q y ) − (t q xy )
dt xy dt dt dt
196 CHAPTER 11. JOINT-LIFE FUNCTIONS

Assume independence of T1 , T2 . We get:

f (t) = t px µx+t + t py µy+t − t px t py (µx+t + µy+t ) (t > 0)

Now the “force of mortality” has to be defined as follows:

µxy (t) = hazard rate at time t


f (t)
=
1 − F (t)

where

f (t) = the p.d.f. of Tmax ,


F (t) = the distribution function of Tmax .

This gives
t px µx+t + t py µy+t − t px .t py (µx+t + µy+t )
µxy (t) =
t pxy

Note This is not often used in practice.

11.7 Last Survivor Monetary Functions


(a) Annuities
Define, for example,

äxy = m.p.v. of an annuity of £1 p.a. payable annually


in advance so long as at least one life is alive.

On multiplying the equation


t pxy = t px + t py − t pxy
by v t , and summing or integrating over t, one obtains the following relationships for annuity functions

äxy = äx + äy − äxy


āxy = āx + āy − āxy (11.7.1)
(m)
äxy = ä(m)
x + ä(m)
y − ä(m)
xy

We may also define temporary last survivor annuity functions, e.g.


n−1
X
äxy:n = v t .t pxy
t=0
n−1
X
= v t [t px + t py − t pxy ]
t=0
= äx:n + äy:n − äxy:n

Note: these formulae hold even without the assumption of independence of the lives.
(b) Assurances
Taking expected values on each side of the relationship

v T1 + v T2 = v min{T1 ,T2 } + v max{T1 ,T2 }


11.7. LAST SURVIVOR MONETARY FUNCTIONS 197

one obtains

Āxy = m.p.v. of a last survivor assurance of £1


payable immediately on the death of (x), (y),
= Āx + Āy − Āxy (11.7.2)

Similarly, when the sum assured is payable at the end of the year of death, we have

v K1 +1 + v K2 +1 = v min{K1 +1,K2 +1} + v max{K1 +1,K2 +1}

and hence
Axy = Ax + Ay − Axy (11.7.3)
(which is still true even when the lives are not independent.)
Note also the conversion relationships

Axy = Ax + Ay − Axy
= (1 − däx ) + (1 − däy ) − (1 − däxy )
= 1 − däxy

and, similarly, Āxy = 1 − δāxy , etc.

Example 11.7.1. Evaluate A40:40 on A1967 − 70 ultimate, 4% interest.

Solution

A40:40 = 1 − dä40:40
= 1 − d[2ä40 − ä40:40 ]
= 1 − d[2 × 18.894 − 17.052]
= 0.20246

(c) Premiums.
We may use an equation of value to calculate net or gross premiums. For example, using the
International Actuarial Notation, we have

Pxy = net annual premium payable annually in advance for a policy providing
£1 at the end of the year of the death of the second to die of (x), (y)
Axy
=
äxy

Using the conversion relationships given above, the following equations can be derived:

Āxy 1
P̄ (Āxy ) = = −δ
āxy āxy

and
1
Pxy = − d.
äxy
198 CHAPTER 11. JOINT-LIFE FUNCTIONS

11.8 Reserves for Last Survivor Assurances


Consider a whole-life assurance providing £S at the end of the year of death of the last survivor of
(x), (y). We ignore expenses and assume that the premium and reserve bases are the same. Then
· ¸
1
Annual premium, P = SPxy = S −d
äxy
The value of the prospective reserve at duration t, just before payment of the premium then due,
depends on whether or not both lives are still alive at time t.
Let

t V1 = reserve assuming both lives are still alive


t V2 = reserve assuming (x) now alive but (y) has died

t V3 = reserve assuming (y) now alive but (x) has died

We find that
£ ¤
t V1 = S Ax+t:y+t − Pxy äx+t:y+t
t V2 = S [Ax+t − Pxy äx+t ]
t V3 = S [Ay+t − Pxy äy+t ]

Theorem When the first life dies, the prospective reserve will increase (from t V1 to t V2 or t V3 ).
Proof
£ ¤
t V1 = S (1 − däx+t:y+t ) − Pxy äx+t:y+t
£ ¤
= S 1 − (Pxy + d)äx+t:y+t
< S [1 − (Pxy + d)äx+t ] (as äx+t < äx+t:y+t )
= S [(1 − däx+t ) − Pxy äx+t ]
= S [Ax+t − Pxy äx+t ]
= t V 2 ( and similarly for t V3 )
If it is not known whether one or both of (x), (y) survive at time t (and in practice this may be
the case since one death does not result in a claim or a change in the premium), one may value the
policy as a ‘weighted average’ of t V 1 , t V 2 and t V 3 , with weights proportional to the probabilities
t pxy , t px (1 − t py ) and t py (1 − t px ) respectively. Hence the weights are

t pxy t px (1 − t py ) t py (1 − t px )
, , respectively
t pxy t pxy t pxy

in order for them to add up to 1. Therefore the weighted average reserve is


t pxy t px (1− t py ) t py (1 − t px )
V = · tV 1 + · tV 2 + · tV 3
t pxy p
t xy t pxy

Finally, it can be shown that the retrospective reserve is equal to the weighted average reserve,
i.e.  
1  
V = S Pxy äxy:t − A 1 .
v t .t pxy z}|{
xy :t
11.8. RESERVES FOR LAST SURVIVOR ASSURANCES 199

Example 11.8.1. A last-survivor policy provided £10, 000 immediately on the death of the sec-
ond to die of a man aged 65 and his wife aged 60. Premiums were payable monthly in advance so
long as at least one of the couple survived. The office which issued the policy used the following basis:

mortality : a(55)ultimate, males/females as appropriate


interest : 8% per annum ,
expenses : 2.5% of all office premiums .

(i) Find the monthly premium.


(ii) Just after the payment of the first premium, the man died. Find the office’s prospective
reserve just after this event, on the premium basis.
Solution
(i) Let annual premium, payable monthly, be P 0 .
(12) 1
0.975P 0 ä m f
= 10, 000(1 + i) 2 A m f
65:60 65:60

(12) 11
ä m ' ä m f − = 7.228
65:60
f
65:60 24
(12) 11
ä m ' ä m − = 7.942
65 65 24
(12) 11
ä f ' ä f − = 9.836
60 60 24
(12) 11 11
Hence ä m ' ä m f − = 11.0075 − = 10.549
65:60
f
65:60 24 24
Am f
= 1 − dä m f
= 0.18463
65:60 65:60

1
10, 000 × (1.08) 2 × 0.18463
0
Hence P = = £186.55
0.975 × 10.549
Therefore monthly premium = £15.55 .

1 (12)
(ii) Reserve = 10000(1 + i) 2 Af60 − 0.975P 0 a f
60
= 2, 476.96 − 1, 773.82
(12) 11
(using Af60 = 1 − d.äf60 , and a f = af + )
60 60 24
Hence Reserve = £694.15.
200 CHAPTER 11. JOINT-LIFE FUNCTIONS

Exercises

11.1 Given that n px = 0.3, n py = 0.4, n pz = 0.6, find the probability that, of the lives (x), (y) and
(z),
(a) none will survive n years
(b) exactly one will survive n years
(c) at least one will survive n years.
11.2 Prove that
(IA)xy = äxy − d.(Iä)xy
11.3 Express in terms of px , py , and pz the probabilities that, of three lives (x), (y) and (z),
(a) all three will survive one year
(b) at least one will survive one year
(c) exactly two will survive one year
(d) at least two will survive one year
11.4 Derive the formula

X
exy = E(K) = t pxy where K = integer part of T (T = min{T1 , T2 })
t=1

using P r{K = k} = k |qxy .


11.5 Evaluate A75:75 on the basis of A1967 − 70 ultimate at 4% interest.
11.6 The probability that at least one of three lives aged 60 will survive to age 65 is eight times the
probability that exactly one will survive to age 65. Assuming that the 3 lives are independent
and subject to the same table of mortality, find the probability that exactly one life will
survive to age 65.
11.7 (i) Define t pxy and show that t pxy = t px + t py − t pxy
(ii) Hence, or otherwise, show that äxy = äx + äy − äxy
11.8 12 years ago a man then aged 48 effected a without profits whole life assurance for £10,000
(payable at the end of the year of death) by annual premiums. The premium now due is
unpaid. He now wishes to alter the policy so that the same sum assured will be payable at
the end of the year of the first death of himself and his wife, who is 4 years older than himself.
Calculate the revised office annual premium, ceasing on the first death, if the office uses the
following basis for premiums and reserves.

mortality: A1967-70 ultimate, rated down 4 years for female lives,


interest: 4% per annum,
expenses: 3% of all office premiums including the first, with additional initial expenses of 1 21 %
of the sum assured. (This additional initial expense is not charged again on the conversion of
an existing policy, providing that the sum assured does not increase.)
11.9 Consider the random variable L equal to the present value of £1 payable immediately on
(i) the first death of (x) and (y), and
(ii) the second death, in each case at a given rate of interest i p.a.

Show that, in case (i),


var(L) = Ā∗xy − (Āxy )2
where ∗ indicates a rate of interest of 2i + i2 p.a., and give a corresponding result for case (ii).
11.10. SOLUTIONS 201

Solutions

11.1 (a) (1 − n px )(1 − n py )(1 − n pz ) = 0.7 × 0.6 × 0.4


= 0.168
(b) n px (1 − n py )(1 − n pz ) + n py (1 − n px )(1 − n pz ) + n pz (1 − n px )(1 − n py )

= 0.3(0.6)(0.4) + 0.4(0.7)(0.4) + 0.6(0.7)(0.6) = 0.436

(c) 1 − (1 − n px )(1 − n py )(1 − n pz ) = 1 − 0.168


= 0.832.
11.2 Let K = curtate future lifetime
£ of (x,¤y)
(Iä)xy = E (Iä)K+1
" #
äK+1 − (K + 1)v K+1
=E
d
1
= [äxy − (IA)xy ] (as (IA)xy = E[(K + 1)v K+1 ])
d
Hence (IA)xy = äxy − d(Iä)xy .
11.3 (a) px . py . pz
(b) 1 − (1 − px )(1 − py )(1 − pz )
(c) px py (1 − pz ) + px pz (1 − py ) + py pz (1 − px )
(d) px py (1 − pz ) + px pz (1 − py ) + py pz (1 − px ) + px py pz
11.4
exy = curtate joint expectation of life
X∞
= k.k |qxy
k=1

1 X
= k.dx+k:y+k
lxy
k=1
1
= [dx+1:y+1 + 2dx+2:y+2 + 3dx+3:y+3 + · · · ]
lxy
1
= [(dx+1:y+1 + dx+2:y+2 + · · · ) + (dx+2:y+2 + dx+3:y+3 + · · · ) + · · · ]
lxy
1
= [lx+1:y+1 + lx+2:y+2 + · · · ]
lxy

1 X
= lx+t:y+t
lxy t=1

X
= t pxy
t=1

11.5
Ā75:75 = 2Ā75 − Ā75:75
1 1
' 2(1.04) 2 A75 − (1.04) 2 [1 − d.ä75:75 ]
= 0.64676.
202 CHAPTER 11. JOINT-LIFE FUNCTIONS

11.6 Let 5 p60 = p. We have (using binomial distribution)

1 − (1 − p)3 = 8 × 3p(1 − p)2


Hence 3p − 3p2 + p3 = 24p(1 − p)2
3 − 3p + p2 = 24 − 48p + 24p2
So 23p2 − 45p + 21 = 0
p
45 ± (−45)2 − 4 × 23 × 21
p=
√ 46
45 ± 93
= = 0.76862 (ignoring root > 1)
46
Hence probability of exactly one survivor = 3p(1 − p)2 = 0.12345
11.7 (i)
t pxy = P r{ the last survivor of (x) and (y) survives for t years}
= P r{(x) survives for t years but (y) dies}
+ P r{(y) survives for t years but (x) dies}
+ P r{(x) and (y) both survive for t years}
= t px (1 − t py ) + t py (1 − t px ) + t px t py
= t px + t py − t px t py

X
(ii) t
äxy = t pxy v
t=0

X
= v t (t px + t py − t px t py )
t=0
X∞ ∞
X ∞
X
= v t t px + v t t py − v t t pxy
t=0 t=0 t=0
= äx + äy − äxy
11.8
V1 = reserve of original policy
= 10, 000[1.01512 V48 − 0.015] (using Zillmer’s formula).
ä60
= 10, 000[1.015(1 − ) − 0.015]
ä48
= £2, 341.89
(there is no need to find the original premium)

Let P be the revised annual premium.


Equation: V1 = 10, 000A60:60 − 0.97P ä60:60
= 10, 000(1 − d.ä60:60 ) − 0.97P ä60:60
= 6175.77 − 9.6447P
Hence P = £397.51
11.9 In case (i), L = v T where T = min{T1 , T2 }. We have
var (L) = E(L2 ) − [E(L)]2
= E[(v ∗ )T ] − (Āxy )2
= Ā∗xy − (Āxy )2 .
11.10. SOLUTIONS 203

The corresponding result for case (ii) is



var (L) = Āxy − (Āxy )2 .
204 CHAPTER 11. JOINT-LIFE FUNCTIONS
Chapter 12

CONTINGENT ASSURANCES

12.1 Contingent Probabilities


Suppose we have 2 independent lives (x), (y), subject to the same mortality table. (If these tables
are different, it is easy to make the necessary adjustments in the formulae.) Define
t q1 = P r{(x) dies within t years, and before (y)}
xy
i.e. (x) dies within t years and (y) dies after this event (not necessarily within t years).

Let
T1 = future lifetime of (x) with pdf f1 (t1 ) = t1 px µx+t1 (t1 > 0)
T2 = future lifetime of (y) with pdf f2 (t2 ) = t2 py µy+t2 (t2 > 0)
Then

t q1 = P r{T1 ≤ t and T1 ≤ T2 }
xy
ZZ
= f1 (t1 )f2 (t2 ) dt1 dt2
0<t1 ≤t
t1 ≤t2
Z t ·Z ∞ ¸
= t1 px µx+t1 · t2 py µy+t2 dt2 dt1
0 t1
Z t
= t1 px µx+t1 · t1 py dt1
0
Z t
= r pxy µx+r dr (12.1.1)
0

If t = 1 we may omit it, giving Z 1


q1 = t pxy µx+t dt
xy
Z ∞
0
Note also that ∞ q1 = t pxy µx+t dt = P r{(x) dies before (y)}
xy 0
If x = y, we clearly have Z t
t q1 = r pxx µx+r dr
x:x 0
Z
1 t
= r pxx µx+r:x+r dr
2 0
1
= t q xx (12.1.2)
2

205
206 CHAPTER 12. CONTINGENT ASSURANCES

By general reasoning, the following result holds:

t q1 + tq 1 = t q xy (12.1.3)
xy xy

Note also that ∞ q1 + ∞q 1 = ∞ q xy = 1


xy xy

One may also define deferred contingent probabilities, and we find that

t |q 1 = t pxy .q 1
x:y x+t:y+t

and t |∞ q 1 = t pxy .∞ q 1
x:y x+t:y+t

Consideration of the second death


We also have the definition,

t q2 = P r{(x) dies after (y) and within t years}


xy

By calculations similar to those for t q1 , we have


xy

Z t
t q2 = (1 − r py )r px µx+r dr
xy 0
= t qx − t q1
xy

Thus, as expected by general reasoning,

t qx = t q1 + t q2 (12.1.4)
xy xy

which follows from


P r{T1 ≤ t} = P r{T1 ≤ t and T1 ≤ T2 }
+ P r{T1 ≤ t and T1 > T2 }
(the two terms on the R.H.S. are the probabilities of mutually exclusive events.)

12.2 Contingent Assurances


Consider a benefit of £1 payable immediately on the death of (x) if this occurs before the death of
(y). The M.P.V. (at a specified rate of interest) is:
· T ¸
v 1 if T1 ≤ T2
Ā 1 = E
xy 0 if T1 > T2
ZZ
= v t1 (t1 px µx+t1 )(t2 py µy+t2 ) dt2 dt1
t1 ≤t2
Z ∞ µZ ∞ ¶
= v t1 t1 px µx+t1 t2 py µy+t2 dt2 dt1
0 t1
Z ∞
= v t1 t1 px µx+t1 (t1 py ) dt1
0
Z ∞
= v t t pxy µx+t dt (12.2.1)
0

This is often evaluated by approximate integration.


12.2. CONTINGENT ASSURANCES 207

Example 12.2.1. Using A1967-70 mortality and 4% interest estimate, by approximate integration,
the value of
Ā 1
50:60

[Assume l108 = 0, so that the integral is over a range of 48 years. Break this into 4 sub-intervals and
use Simpson’s rule over each.]
Solution

Z ∞
Ā 1 = v t t p50 µ50+t t p60 dt (at 4% interest)
50:60 0
Z 48
= f (t) dt
0

where f (t) = v t t p50:60 µ50+t .

Using Simpson’s rule

Ā 1 = 2 [f (0) + 4f (6) + 2f (12) + 4f (18) + 2f (24)


50:60
4f (30) + 2f (36) + 4f (42) + f (48)]
= 0.1546.

Temporary contingent assurance functions

A1 = m.p.v. of £1 payable immediately on death of (x)


x:y:n

if this occurs before the death of (y) and within n years


Z n
= v t t pxy µx+t dt (12.2.2)
0

Note also that


1
Āxx Ā 1 = (12.2.3)
2 xx

(since there is a 50% chance that the first “x” of (x, x) will be the first to die).
We may therefore evaluate contingent assurances on 2 equal ages from tables of Āxx . As expected
by general reasoning, we also have
Axy = A 1 + A 1 (12.2.4)
xy xy

Example 12.2.2. (a) Estimate, by approximate integration, the value of a contingent assurance of
£100,000 payable immediately on the death of a female aged 60, provided a male aged 50 is still alive
and provided her death occurs within 15 years. The mortality of the female follows a(55) ultimate
(females), and that of the male follows A1967-70 ultimate. The interest rate is 7 21 % per annum and
expenses are ignored.
Note. A very accurate approximation to the value is not expected.

(b) It has been suggested that a policy providing the above benefit should be issued by annual
premiums ceasing on the death of the female life or after 15 years, whichever is earlier. State with
reasons whether you agree with this suggestion. If you do not, suggest a more suitable premium-
paying term, other than issuing the contract by a single premium.
208 CHAPTER 12. CONTINGENT ASSURANCES

Solution

(a) Value = 100, 000Ā 1


m f
50:60:15
Z 15
= 100, 000 v t t p50:60 µ60+t dt
0

Evaluate integral by (for example) the three-eighths rule:


Z 15
15
f (t) dt ' [f (0) + 3f (5) + 3f (10) + f (15)]
0 8
= 0.12657.
Hence value = £12, 657

(b) No: premiums must also cease on death of (50), otherwise they may still be payable with no
prospect of claim: negative prospective reserve. Restrict premium-paying period so that premiums
cease on first death or after 15 years, if earlier.

We may also encounter contingent assurances payable on the second death.


Define

Ā 2 =m.p.v. of £1 payable immediately on death


xy

of (x) if this occurs after the death of (y).


By a similar argument to that for Ā1xy ,
Z ∞
2
Āxy = v t t px µx+t (1 − t py ) dt
0

Hence

Āx = Ā1xy + Ā2xy (12.2.5)


as expected.

We may also encounter temporary contingent assurances payable on the second death, e.g. the
m.p.v. of £1 payable immediately on death of (x) within n years, provided that (x) dies after (y),
equals
Z n
v t t px µx+t (1 − t py ) dt
0
= A1 − A1
x:n xy:n

Note also that


1
Ā2xx =
Āxx (12.2.6)
2
since there is a fifty per cent chance that the first “x” will be the last survivor.

Example 12.2.3. Using the fact that Ā1 + Ā 1 = Āxy , find on A1967 − 70 ultimate at 4% interest
xy xy
the values of (i) Ā 1 , (ii) Ā 2 and (iii) Ā 1 .
60:60 60:60 60:60:10
12.2. CONTINGENT ASSURANCES 209

Solution.

(i) Ā 1 = 0.5Ā60:60
60:60
1
' 0.5(1.04) 2 [1 − dä60:60 ]
' 0.31491

(ii) Use A 2 = A60 − A 1 to obtain


60:60 60:60

Ā 2 = 0.21259
60:60

1 1
(iii) Ā 1 = Āz }| {
60:60:10 2 60 : 60:10
· ¸
1 1 D70:70
' (1.04) A60:60 −
2 A70:70
2 D60:60
= 0.15513

The symbols A1 , etc.


xy

These are the same as Ā1 , etc., but with the benefit payable at the end of the year of death.
xy
An exact expression is

X
A1 = v t+1 t pxy q 1 (12.2.7)
xy x+t:y+t
t=0

In practice, we may use the approximations


1
A1 ' (1 + i)− 2 Ā1 , etc. (12.2.8)
xy xy

Note the following results, similar to those given above for A 1 , etc.:
xy

Axy = A 1 + A 1
xy xy
Ax = A1 + A2
xy xy
1
A1 = Axx
xx 2
1
A2 = Axx
xx 2

The variance of the present value of a contingent assurance


Let

Z = present value of £1 payable on death of (x), if this occurs before the death of (y)
(
1
v T1 if T1 ≤ T2 where v = 1+i
=
0 if T1 > T2

As we have shown above, this has mean

E(Z) = Ā1
xy
210 CHAPTER 12. CONTINGENT ASSURANCES

The variance of Z is
E(Z 2 ) − [E(Z)]2
where
· 2 T ¸
(v ) 1 if T1 ≤ T2
E(Z 2 ) = E
0 if T1 > T2
· ∗ T ¸
(v ) 1 if T1 ≤ T2 1
=E where v ∗ = with i∗ = i2 + 2i
0 if T1 > T2 1 + i∗
= Ā∗1 at rate of interest i2 + 2i (or force of interest 2δ)
xy

Hence
Var(Z) = Ā∗1 − (Ā1 )2 (12.2.9)
xy xy

12.3 Premiums and Reserves for Contingent Assurances


Premiums are calculated by the usual equation of value (including expenses if necessary). Reserves
are usually calculated prospectively, making allowance for any deaths which have already occurred.

Example 12.3.1. Two lives (A and B) are both aged 30. Calculate, on the basis of A1967 − 70
ultimate mortality and 4% p.a. interest, the annual premium, payable during the lifetime of A, to
provide an insurance of £1000 payable at the end of the year of death of A, provided A dies after
B.
Find the policy value (on the premium basis) after 10 years (before the premium then due is
paid) if

(i) only A is then alive;

(ii) both lives are then alive.

Solution

A2 = A30 − A 1
30:30 30:30
1 1
= A30 − A30:30 = (1 − dä30 ) − (1 − dä30:30 )
2 2
1 1
= − d(ä30 − ä30:30 )
2 2
Hence
A2 · µ ¶¸
30:30 1 1 ä30:30
premium = · 1000 = 1000 −d 1−
ä30 2ä30 2 ä30
= 1000.(0.00327) = 3.27 = P, say.

Hence the reserves are as follows:

(i) 10 V = 1000A40 − P ä40 if only A is alive = 211.56


(ii) 10 V = 1000A 2 − P ä40 if both are alive = 39.49
40:40
12.4. A PRACTICAL APPLICATION – THE PURCHASE OF REVERSIONS 211

Lapse options
One should avoid having a negative prospective reserve at any duration, as this may lead to a lapse
option against the office. In particular, one should ensure that premiums cease as soon as there is
no possibility of future benefits. (See Example 12.2.2 above.)

12.4 A Practical Application – The Purchase of Reversions

Definitions
A reversion is a contract providing a sum of money payable on the death of a certain life, (y).
An absolute reversion is a reversion in which the sum is paid under all circumstances, whilst in a
contingent reversion the sum is paid only if certain other lives are (or are not) then alive.

Example 12.4.1. J. Brown(50) will receive £1,000,000 on the death of his mother (aged 80),
provided that he is then alive. He wishes to sell his “interest”. What is it worth?

Solution
Suppose that a purchaser uses a certain rate of interest, i p.a., and assumes that a certain mortality
table applies to both J. Brown and to his mother. (Note that the purchaser might assume different
tables for the lives). The interest is contingent since it requires (50) to be alive when (80) dies.
Hence

M.P.V. = 1, 000, 000Ā 1


50:80

In practice, the purchaser will want to be sure of getting the money when (80) dies, and will
“plug the gap” with an insurance policy (which pays out on the death of (80) if she dies second.)
Suppose that the life office issuing the insurance policy uses the same mortality and interest basis
as the purchaser, and also ignores expenses.

Single premium to buy insurance = 1, 000, 000Ā 2


50:80

Hence purchase price = 1, 000, 000Ā80 − cost of premium for insurance policy
h i
= 1, 000, 000 Ā80 − Ā 2
50:80
= 1, 000, 000Ā 1 (as found before)
50:80

Note:- The life office issuing the policy would examine the health (and any other risk factors) of
(50) very carefully.
212 CHAPTER 12. CONTINGENT ASSURANCES

12.5 Extension to Three Lives


We briefly mention the following symbols and formulae:

∞q 1 = P r{(x) dies first of (x), (y), (z)}


xyz
Z ∞
= t px µx+t · t py · t pz dt (12.5.1)
0
∞q 2 = P r{(x) dies second, (y) having died first}
xy z
1
Z ∞
= t px µx+t (1 − t py )t pz dt
0
= ∞q 1 − ∞q 1 (12.5.2)
xz xyz

∞q 2 = P r{(x) dies second of (x), (y), (z)}


xyz
= ∞q 2 + ∞q 2 (since either (y) or (z) dies first)
xy z xyz
1 1

= ∞ q 1 + ∞ q 1 − 2∞ q 1 using formula (12.5.2)


xy xz xyz

Similar definitions and relationships apply for contingent assurances, e.g.

A2 = m.p.v. of £1 payable immediately


xyz

on the death of (x) if he dies second


=A 1 + A 1 − 2A 1
xy xz xyz

More complicated functions may be defined and evaluated, but we do not pursue this topic.
12.6. EXERCISES 213

Exercises

12.1 Which (if any) of the following statements are correct?


A. Āxx = 2Ā1
xx
B. Āxx = 2Āx
C. Āxx = 2Āx − Ā1
xx
D. Āxx = Ā1 + Ā2
xx xx

12.2 Adams (aged 40) and Brown (aged 50) are two business partners. Adams wishes to provide
for the sum of £80,000 to be paid immediately on Brown’s death if Brown predeceases him
within ten years, and effects a policy providing this benefit by single premium. The life office
issuing the contract employs the following basis:
Mortality (both lives) : A1967 − 70 ultimate
Interest : 6%
Expenses: 2% of the single premium.

Using Simpson’s rule, or otherwise, estimate the single premium payable by Adams.
12.3 Estimate the value of ∞ q1 on the basis of A1967 − 70 ultimate mortality.
74:84
(Assume l108 = 0, so that the integral is over a range of 24 years. Break this into 3 sub-intervals
and use Simpson’s Rule over each.)
12.4 (i) Express A2 in terms of äx , äxx and the rate of interest.
xx

(ii) Smith and Jones are both aged 60. A life office has been asked to issue a special joint-life
assurance policy providing £10,000 at the end of the year of death of the first to die of these
two lives. In addition, if Smith is the second to die, a further £5,000 will be payable at the
end of the year of his death. The policy is to have annual premiums payable during the joint
lifetime of Smith and Jones.

Calculate the annual premium on the following basis:


A1967 − 70 ultimate mortality
4% interest
expenses are 5% of all premiums, with an additional initial expense of £100.

(iii) Write down (but do NOT evaluate) formulae for the reserve at duration 10 years (imme-
diately before payment of the premium then due) on the premium basis, if
(a) both Smith and Jones are alive; and
(b) Jones has died but Smith is alive.
12.5 A policy providing the sum of £100,000 immediately on the death of (x) if she dies before (y)
is to be issued by a life office to a group of trustees.
(i) Ignoring expenses, write down an expression for the single premium in terms of an integral.
(ii) The trustees suggest that level annual premiums should be payable in advance until the
death of the last survivor of (x) and (y).
214 CHAPTER 12. CONTINGENT ASSURANCES

(a) Ignoring expenses, give a formula for the annual premium.

(b) Would you advise the life office to issue the policy with premiums payable as suggested?
Give reasons for your answer.
12.6 Estimate, by the trapezoidal rule or another suitable rule for approximate integration, the
single premium for a temporary contingent assurance of £50,000 payable immediately on the
death of Mrs Smith (aged 60), provided that this event occurs within 5 years and that her
husband (aged 50) is alive at the date of her death. Mrs Smith is subject to the mortality of
a(55) ultimate (females) and Mr Smith is subject to the mortality of A1967 − 70 ultimate.
An interest rate of 7.5% p.a. is to be used, and allowance is to be made for expenses of 6% of
the single premium.
(Note A very accurate answer is not expected.)
12.7 Define the following functions in words, and give an expression for each of them in terms of
an integral.
(i) ∞ q1
xy
(ii) Ā2
xy
(iii) Ā1
x:y:n
12.8 The chief of a certain tribe holds that office until age 50 or earlier death, and may be succeeded
only by a person aged from 36 to 45. (A person aged exactly 36 is eligible, but a person aged
exactly 45 is not.) The customs of the tribe require that a chief’s successor be his oldest
eligible brother; if there is no eligible brother, the position of chief is given to someone from
outside the previous chief’s family, who are then permanently debarred from becoming chief.
The present chief is aged exactly 47 and has two brothers, aged exactly 37 and 33 respectively.
The chief and his brothers may be regarded as independent lives subject to the mortality of
a given table.
Obtain an expression, in terms of quantities of the form n px , n q1 only, for the probability
xy
that (33) will become chief.
12.9 Your life office has been asked to quote a single premium for a contingent assurance policy
providing £300,000 immediately on the death of a woman now aged 80 within 15 years,
provided that at the date of her death a man now aged 60 has died. Your office uses the
following basis:

mortality : males - a(55)ultimate (males)


females - a(55) ultimate (females)
interest : 8% per annum
expenses : 10% of the single premium.

(i) Assuming that the two lives are independent, write down a formula for the single premium
in terms of an integral.

(ii) State a suitable non-repeated rule of approximate integration for evaluating this integral.
(You are NOT required to carry out the evaluation.)

(iii) Would you subject the male life to stringent underwriting procedures? Give brief reasons
for your answer.
12.7. SOLUTIONS 215

Solutions

12.1 Only A is correct.


12.2 Let single premium be P . Z
10
P = 80000 v t t p50 µ50+t · t p40 dt + 0.02P at 6% interest
0
Z
80000 10
= f (t) dt where f (t) = v t t p50 µ50+t .t p40
0.98 0
80000 10
' [f (0) + 4 · f (5) + f (10)] using Simpson’s rule
0.98 6
= £4, 686.

12.3 Z ∞
∞ q1 = t p74 µ74+t .t p84 dt
74:84 0
Z 24
= f (t) dt
0
where f (t) = t p74 µ74+t .t p84
4
' [f (0) + 4f (4) + 2f (8) + 4f (12) + 2f (16) + 4f (20) + f (24)]
3
= 0.2907.

12.4
1 1
(i) A2 = Axx = Ax − Axx
xx 2 2
1
(or use A2 = Ax − A1 = Ax − Axx );
xx xx 2
1
= (1 − däx ) − (1 − däxx )
µ ¶2
1 i 1
= − (äx − äxx )
2 1+i 2

(ii) Let the annual premium be P, and set x = 60.


0.95P äxx = 10, 000Axx + 5, 000A2 + 100
xx
= 10, 000[1 − däxx ]
· ¸
1 1
+ 5, 000 − d(äx − äxx ) + 100
2 2

Putting x = 60 gives ¸ ·
0.04
0.95P · (9.943) − 100 =10, 000 1 − · 9.943
1.04
0.04
+ 5, 000[0.5 − (12.551 − 0.5 × 9.943)]
1.04
=6175.77 + 1042.40 = 7218.17

7318.17
Hence P = = £774.75
9.44585
216 CHAPTER 12. CONTINGENT ASSURANCES

(iii) (a) Reserve = 10, 000A70:70 + 5, 000A 2


70:70
− 0.95P ä70:70
(b) Reserve = 5, 000A70
R∞
12.5 (i) 100, 000 0
v t t pxy µx+t dt
R∞
100000 0
v t t pxy µx+t dt
(ii) (a) Annual premium, P = .
äxy
(b) No, because premiums should not continue after first death. If (x) dies first, the benefit
is paid and the policy may be lapsed, and if (y) dies first there is no possibility of benefit and
the policy be lapsed.
There is thus a lapse option on the first death.
12.6 Let single premium = P .
Then
1
0.94P = 50, 000Ā1f m at 7 % interest
60:50:5 2
Z 5
= 50, 000 v t t p f µ f .t p m dt
0 60 60+t 50

By the trapezoidal rule. Z 5


vt t p f µ f .t p m dt ' 0.041726
0 60 60+t 50

Hence

50000 × 0.041726
P ' = £2, 219
0.94
12.7 (i)
∞ q1 = P r{(x) will die before (y)}
xy
Z ∞
= t pxy µx+t dt
0

(ii)
Ā2 = m.p.v. of a contingent assurance of £1
x:y
payable immediately on death of (x),
provided this occurs after the death of (y).
Z ∞
= v t t px µx+t (1 − t py ) dt
0

(iii)
Ā 1 = m.p.v. of a temporary contingent assurance of £1 payable
x:y:n

immediately on death of (x), provided that this occurs within


n years, and before (y) dies.
Z n
= v t .t pxy µx+t dt
0
12.7. SOLUTIONS 217

12.8 Let chief = (y) = (47), brothers (x1 ) = (37), (x2 ) = (33).
(x2 ) becomes chief if and only if
(1) he survives for 3 years and (y) “retires” then, (x1 ) having died.
(2) (x1 ) becomes chief (by succeeding (y) on his death or “retirement”) and dies after time 3
and before time 12, leaving (x2 ) alive.
Probability of event (1) is
3 py (1 − 3 px1 )3 px2

Probability of event (2) is


P r{(x1 ) dies between times 3 and 12, leaving (x2 ) alive}
Z 12
= t px1 µx1 +t · t px2 dt
3
Z 9
= r+3 px1 µx1 +r+3 · r+3 px2 dr
0
Z 9
= 3 px1 x2 r px1 +3:x2 +3 µx1 +r+3 dr
0
= 3 px1 · 3 px2 .9 q 1
x1 +3:x2 +3

12.9 R 15
300, 000 0
vt t p f µ f (1 − t p m ) dt
80 80+t 60
(i) annual premium =
0.90
(ii) The three-eighths rule would be suitable, since it avoids evaluation of the integrand when
t = 7 12 .
(iii) Yes. If (60) dies soon, there is a high chance that (80) will die before age 95 and so give
rise to a claim. The office must check:
(1) the health of (60);
(2) whether he has any occupational or other risks, e.g. participation in a dangerous sport.
Note. The sum assured is large enough to justify the costs of a medical examination.
218 CHAPTER 12. CONTINGENT ASSURANCES
Chapter 13

REVERSIONARY ANNUITIES

13.1 Reversionary Annuities Payable Continuously


Consider an annuity of £1 p.a. payable continuously to (y) after the death of (x). The present value
of this reversionary annuity is
(
āU − āT if U > T
Z=
0 if U ≤ T
where
T = future lifetime of (x), with p.d.f. = t px µx+t (t > 0);
U = future lifetime of (y), with p.d.f. = u py µy+u (u > 0).
Note. If (x) and (y)’s mortality rates follow different tables, this should be indicated.
Define

āx|y = m.p.v. of Z
ZZ
= (au − āt )(u py µy+u )(t px µx+t ) du dt
u>t
Z ∞ ·Z ∞ ¸
= (āu − āt )u py µy+u du t px µx+t dt (13.1.1)
0 t

Theorem
āx|y = āy − āxy (13.1.2)
Proof It is easiest to proceed indirectly, as follows. Observe that
(
(āU − āT ) + āT if U > T
Z + āmin(T,U ) = = aU
āU if U ≤ T

Taking expected values on both sides gives

E(Z) + āxy = āy

Another important formula


Z ∞
āx|y = v t t px µx+tt py āy+t dt
0

219
220 CHAPTER 13. REVERSIONARY ANNUITIES

Proof
Z ∞
āx|y = v t t py (1 − t px ) dt
0
Z ∞
= f (t)g 0 (t) dt
0

where
f (t) = 1 − t px = tRq x (which is such that f 0 (t) = t px µx+t ) and

g(t) = −t |āy = − t v r r py dr (which is such that g 0 (t) = v t t py )
Using integration by parts
Z ∞
āx|y = [f (t)g(t)]∞ 0 − f 0 (t)g(t) dt
0
Z ∞

= [(−t |āy )t q x ]0 + t px µx+t (t |āy ) dt
0
Z ∞
t
= t px µx+t v t py āy+t dt.
0

Evaluation of reversionary annuities


Using the Euler–Maclaurin formula we have
āx|y = āy − āxy
1 1
' (ay + ) − (axy + )
2 2
= ay − axy
= ax|y

Example 13.1.1. Calculate an appoximate value of ā m f on the a(55) tables at 8% interest.


65|69

Solution
ā m f ' a f − am f
65|69 69 65:69

= 7.533 − 5.877 (using interpolation)


= 1.656

13.2 Reversionary Annuities Payable Annually or mthly


1
We first assume that payments are made mthly , the first payment being at the end of the m year
(m)
(measured from the issue date) following the death of (x), and use the symbol ax|y to refer to this
case. Using the equation
(m)
ax|y + a(m) (m)
xy = ay

it can be seen that


(m)
ax|y = m.p.v. of reversionary annuity of £1 p.a.
payable monthly to (y) after the death
of (x)
= a(m)
y − a(m)
xy
13.3. WIDOW’S (OR SPOUSE’S) PENSION ON DEATH AFTER RETIREMENT 221

When m = 1, it may be omitted, giving

ax|y = ay − axy

By Woolhouse’s formula
(m) m−1 m−1
ax|y ' (ay + ) − (axy + )
2m 2m
= ay − axy
= ax|y .

An alternative approach is to regard this reversionary annuity as a collection of “pure endow-


1 2
ments” (payable at times m , m , · · · ) which are paid if (x) has died and (y) is alive, i.e.

X∞
(m) 1 t t
ax|y = v m P r{(x) has died but (y) is alive at time }
t=1
m m

1 X t
= v m (1 − mt px ). mt py
m t=1
= a(m)
y − a(m)
xy (as before).

Suppose now that a mthly reversionary annuity (of £1 p.a.) begins immediately on the death of
1
(x). Since the payments begin on average 2m year earlier than in the case discussed previously, the
m.p.v. is approximately
1 (m)
(1 + i) 2m ax|y (13.2.1)
An exact formula is
Z ∞
(m)
v t t px µx+t·t py äy+t dt
0
Z ∞ µ ¶
1
' v t t pxy µx+t āy+t + dt
0 2m
1
=āx|y + Ā 1
2m xy
1
'ax|y + Ā 1 (13.2.2)
2m xy
In practice this gives results similar to (13.2.1).

13.3 Widow’s (or Spouse’s) Pension on Death after Retire-


ment
Many pension schemes provide a spouse’s pension on the death of the member in service (D.I.S. =
Death In Service) and/or on death after retirement (D.A.R. = Death After Retirement)
We consider D.A.R. only at this stage and suppose that a male employee retires at age 65.
Now consider 2 different cases:-
Case 1
A widow’s pension of £1 p.a. is payable on the death of (65) only if he was married to the same
woman at retirement. In this case, the m.p.v. at age 65 per married man is

ā m f (assuming pension is payable continuously)


65|y
222 CHAPTER 13. REVERSIONARY ANNUITIES

where

y = average age of wife of member aged 65


= 65 − d, where d = age difference between husband and wife
(approximately 3 years in practice)

Hence the value of widow’s pension for each member retiring at age 65 (marital status unknown)
is
h65 ā m f (13.3.1)
65|65−d

where hx = the probability that a man aged x is married.


Note. We ignore the possibility of divorce (or assume that the ex-wife still gets pension).

Case 2 (now more common)


The widow’s pension is payable to any widow: we use the “collective” approach.
Suppose, for example, that the widow’s pension of £1 p.a. is payable monthly in advance, beginning
immediately on death of her husband.
The m.p.v. for a man retiring at age 65 (marital status then unknown) is
Z ∞
(12)
v t t pm m
65 µ65+t h65+t ä f
dt (13.3.2)
0 65−d+t

where
(12)f
ä65−d+t = m.p.v. of widow’s annuity to the widow of a
man dying at age 65 + t, if widow is
assumed to be d years younger than husband

Example 13.3.1. A life office sells “personal pensions” policies under which the benefits for men
on retirement at age 65 consist of:

(a) a member’s pension (payable monthly in advance for 5 years certain and for life thereafter),
and (for men married at age 65 only)

(b) a spouse’s pension (payable monthly in advance, beginning immediately on the death of the
member) of half the member’s pension. The possibility of divorce of men aged over 65 may be
ignored, and post-retirement marriages do not give rise to spouse’s pension.

The member’s contributions are invested in certain unitised with-profits funds, and it is assumed
that, in respect of a certain Mr Brown’s policy, the fund available to purchase pension at age 65 will
be £100,000.

Suppose that the life office uses the following basis to calculate the amount of pension which may
be purchased at retirement in respect of a given fund:
mortality of males: a(55) males ultimate
mortality of females: a(55) females ultimate
interest: 8% per annum
expenses: 1% of the fund (at age 65)
13.3. WIDOW’S (OR SPOUSE’S) PENSION ON DEATH AFTER RETIREMENT 223

Men who are not married at age 65 need not buy spouse’s pension, but married men must buy
this. Calculate Mr Brown’s expected monthly pension if

(i) he is assumed to be single at age 65; and

(ii) he is assumed to be married at age 65, and his wife is 3 years younger.
Solution. (i) Let P be annual pension, payable monthly in advance.
(12) (12)
0.99 × 100, 000 = P (ä5 +5 |ä65 ) on males’ table
· ¸
i l70 11
= P ( (12) a5 + v 5 ä70 − )
d l65 24
= P (4.1637 + 0.68058 × 0.86621 × 6.8097)
= 8.1782P
Hence P = £12, 105, so monthly pension = £1, 008.75

(ii) The equation of value is now


1 1
³ ´
99, 000 ' P (8.1782) + P (1.08) 24 a f − a m f
2 62 65:62
' P (8.1782 + 1.2026)
Hence P = £10, 553, so monthly pension = £879.42

Notes
(1) There may be rules to try to exclude benefits for widows of “deathbed marriages”.
(2) There may be problems if the man was married more than once (and his ex-wives are still
alive). In the U.K. the last wife receives all the pension.
(3) If a wife is very much younger than her husband, there may be an “actuarial reduction” (see
later).
(4) Widow’s pensions might cease on remarriage, but this rule is no longer common.
(5) h65+t is sometimes assumed to be “piecewise continuous”, e.g.


 0.85 for 0 ≤ t < 1

0.87 for 1 ≤ t < 2
h65+t =

 0.89 for 2 ≤ t < 3


0.90 for t ≥ 3.

The integral in (13.3.2 ) may be evaluated approximately by a sum, i.e.


Z ∞
(12)
v t t pm
65 µ m (h65+t )ä f
dt
0 65+t
65−d+t

1
m (12) 1 (12)
' 0.85v 2 q65 ä f
+ 0.87v 1 2 1 |q m ä f
65
65−d+ 21 65−d+1 21
1 (12)
+ 0.89v 2 2 2 |q m ä f
65
65−d+2 21
" #
3 12 (12) 4 12 (12)
+ 0.9 v ·3 |q m äf +v ·4 |q m äf + ···
65 66
65−d+3 21 65−d+4 12

(assuming that the man dies on average half-way through each year of age).
224 CHAPTER 13. REVERSIONARY ANNUITIES

13.4 Actuarial Reduction Factors


Suppose that, in a pension policy or scheme, the rules state that a reduction applies to the normal
widow’s pension if wife is more than 10 years younger than her husband. (This rule may also apply
to widow’s D.I.S. pensions).
Suppose a member dies aged x, leaving widow aged y (y < x − 10).
The value of this widow’s pension is calculated to be the same (actuarially speaking) as for a
widow aged x − 10 (whose pension is not, of course, reduced.) Let the widow aged y get £R p.a.
for each £1 p.a. of ‘normal’ widow’s pension. Then
(12) (12)
(Full widow’s pension).Räf = (Full widow’s pension).ä f
y x−10

This gives
(12)
ä f
x−10
R = actuarial reduction factor = (12)
äf
y
13.5. EXERCISES 225

Exercises

13.1 (i) Define the following symbols in words, and give a formula in terms of an integral for each
of them:
(a) A 1
xy

(b) A 2
xy

(c) ay|x

(ii) Consider the following sets of payments:


(1) £1 immediately on the death of (y) if (y) dies before (x), and
(2) an income of £δ p.a. payable continuously to (x) after the death of (y), plus £1 immedi-
ately on the death of (x) if this occurs after that of (y).
Prove that the present values (at force of interest δ p.a.) of (1) and (2) are equal. Hence write
down a relationship involving A 1 , A 2 and ay|x .
xy xy
13.2 A special life policy on 2 lives aged x and y respectively provides cash sums of £10,000 and
£20,000 immediately on the first and second deaths respectively. In addition, an annuity
at the rate of £1,000 per annum will be paid continuously, commencing immediately on the
first death and ceasing immediately on the second death. Obtain an expression for the mean
present value of the benefits in terms of joint-life and single- life annuity functions and the
force of interest. Ignore expenses.
13.3 An office issues a policy on the lives of a woman aged 60 and her husband aged 64. Under this
policy, level premiums are payable annually in advance for 20 years or until the first death of
the couple, if earlier.
On the first death of the couple, the survivor will receive an annuity of £10,000 per annum,
payable weekly, beginning immediately on the first death.
Calculate the annual premium if the office uses the basis given below:
Mortality males: a(55) males ultimate
females: a(55) females ultimate

Expenses: 20% of the first premium


5% of each premium after the first

Interest: 6% per annum.


13.4 A special annuity, payable yearly in arrear, is effected on the lives of a man aged x and his
wife aged y. The conditions of payment are:
(a) so long as both survive the rate of payment will be £3,000 per annum;
(b) if the wife dies first, the rate of payment will be £2,000 per annum until the man’s death;
(c) payments at the rate of £3,000 per annum will continue for six years certain after the
death of the husband, the first payment being at the end of the year of his death, and will be
reduced thereafter to £1,500 per annum during the lifetime of the wife.

Obtain an expression for the present value of this annuity in terms of single and joint-life
annuity factors, life table and compound interest functions. Assume that the same (non-
select) table of mortality is appropriate for the two lives.
226 CHAPTER 13. REVERSIONARY ANNUITIES

13.5 A single-premium policy provides the following benefits to a husband and wife each aged 40.
(1) An annuity of £5,000 per annum, payable continuously, commencing on the husband’s
death within 25 years, or on his survival for 25 years, and continuing so long as either husband
or wife is alive.
(2) A return of half the single premium without interest immediately on the death of the
husband within 25 years, provided that his wife has already died.

The office issuing the contract uses the following basis:

mortality : A1967-70 ultimate


interest : 4% per annum
expenses are ignored.
Calculate the single premium.
13.6 A husband and wife, aged 70 and 64 respectively, effect a policy under which the benefits are
(1) a lump sum of £10,000 payable immediately on the first death, and (2) a reversionary
annuity of £5,000 p.a. payable continuously throughout the lifetime of the surviving spouse
after the death of the first. Level premiums are payable annually in advance until the first
death.
Calculate the annual premium on the undernoted basis:

Males’ Mortality: a(55) males ultimate


Females’ Mortality: a(55) females ultimate
Interest: 8% p.a.
Expenses: 10% of all premiums
Ignore the possibility of divorce.
13.6. SOLUTIONS 227

Solutions

13.1 (i) (a) The m.p.v. of £1 payable immediately on the death of (y), if this occurs before that
of (x). Z ∞
A 1= v t t pxy µy+t dt
xy 0

(b) The m.p.v. pf £1 payable immediately


Z ∞ on the death of (x) if this occurs after that of (y).
A2 = v t t px µx+t (1 − t py ) dt
xy
0

(c) The m.p.v. of a reversionary annuity of £1 p.a. payable continuously to (x) after the
death of (y). Z ∞
ay|x = v t t py µy+t · t px · ax+t dt
0

(other expressions also possible)


(ii) Consider payment (1). Suppose it is invested (at force of interest δ p.a.) to give income
so long as (x) lives (assuming (y) dies first, otherwise there is no payment (1)). On the death
of (x), if after that of (y), the capital (£1) is paid immediately. Payments (2) thus have the
same present value as payment (1) (both being random variables depending on the future
lifetimes of (x) and (y)). Take means of these present values to get
A 1 = δ.āy|x + A 2
xy xy

13.2
Benefit = 10, 000Axy + 20, 000Axy + 1000(āx|y + āy|x )
= 10, 000(Āxy + 2Ax + 2Ay − 2Axy ) + 1000(āx + āy − 2āxy )
= 10, 000(2Ax + 2Ay − Axy ) + 1000(āx + āy − 2āxy )
= 10, 000(2 − 2δāx + 2 − 2δāy − 1 + δāxy ) + 1000(āx + āy − 2āxy )
= (1000 − 20, 000δ)(āx + āy ) + (10, 000δ − 2000)āxy + 30, 000

13.3 Let annual premium be P . Then


0.95P ä m f − 0.15P = 10000[ā m f + ā f m ]
64:60:20 64|60 60|64

= 10000[ā f − ā m f + ā m − ā m f ]
60 64:60 64 64:60
' 10000[a f + a m − 2a m f ]
60 64 64:60
= 42230

lm lf
ä m f = ä m f − v 20 84
· 80
· ä m f = 8.5709
64:60:20 64:60 lm lf 84:80
64 60

42230
Hence P = = £5, 284 .
0.95 × 8.5709 − 0.15
13.4 (a) value of benefit = 3000axy .
(b) value of benefit = 2000(ax − axy )
228 CHAPTER 13. REVERSIONARY ANNUITIES

(c) value of benefit is value of


(1) an annuity-certain (of 3000 p.a., for 6 years) beginning at end of year of death of (x), plus
(2) 1500 p.a., payable at times t (t ≥ 7) if (y) alive and (x) dead 6 years previously

X
= 3000ä6 · Ax + 1500 v t t py (1 − t−6 px )
t=7
= 3000ä6 (1 − däx ) + 1500v 6 6 py (ay+6 − ax:y+6 )
Add (a), (b), (c) to find total value of the annuity.
13.5 Let single premium be P .
Consider benefit (1) with payment at rate £1 p.a. There are two cases:
(a) husband dies within 25 years;
(b) husband survives for 25 years.
(Indicate “m,f” to clarify which life is which.)

In case (a), M.P.V. is


Z 25
v t t pm
40 µ m pf ā f dt
0 40+t·t 40 40+t

In case (b), M.P.V. is · ¸


v 25 25 pm
40 25 p f a m f
+ (1 − 25 p f )a m
40 65:65 40 65
25 25
=v 25 p m · 25 p f (ā f − ā m f ) + v 25 p m ā m
40 40 65 40 65
Z ∞ 65:65
=v 25 25 p m · 25 p f · v t t p m f µ m ā f dt + v 25 25 p m ā m
40 40 0 65:65 65+t 65+t 40 65

Therefore total M.P.V. is Z ∞


vt t p m f µ m ā f dt + v 25 25 p m ā m
0 40:40 40+t 40+t 40 65

=ā m f + v 25 25 p m ā m
40|40 40 65

Benefit (1) = 5000(ā40|40 ) + 5000v 25 25 p40 ā65


= 5000(ā40 − ā40:40 ) + 5000v 25 25 p40 ā65
1
' 5000[ä40 − ä40:40 + v 25 25 p40 (ä65 − )]
2
= 5000(1.842 + 3.1418) = 24, 919.

Z
P 25 t
Benefit (2) = v t p40 µ40+t (1 − t p40 ) dt
2 0
P
= [A 1 −A1 ]
2 40:25 40:40:25
P 1 1
= [A 1 − Az }| { ]
2 40:25 2 40 : 40:25
· µ ¶¸
P 1 D65 1 D65:65
= (1.04) 2 A40:25 − − 1 − d.ä40:40:25 −
2 D40 2 D40:40
P
= (0.094993 − 0.5 × 0.17511) = 0.003719P
2
13.6. SOLUTIONS 229

Hence equation of value is


P = 24919 + 0.003719P

Therefore P = £25, 012.


13.6 Let annual premium be P .
Value of benefits = 10, 000A m f + 5000(ā m f + ā f m )
70:64 70|64 64|70

= 10, 000(1 − δā m f ) + 5000(ā m + ā f − 2ā m f )


70:64 70 64 70:64
1
' 10, 000[1 − δ(a m f + )] + 5000[a m + a f − 2a m f ]
70:64 2 70 64 70:64
= 5, 323.85 + 18, 450
= 23, 773.85

Value of premiums less expenses = 0.9P ä m f = 5.9184P .


70:64
23773.85
Hence P = = £4, 017 .
5.9184
230 CHAPTER 13. REVERSIONARY ANNUITIES
Chapter 14

PROFIT TESTING FOR


UNIT-LINKED POLICIES

14.1 Unit-Linked Policies


Most of the money paid in premiums by the policyholders is used to purchase “units”: that is,
money is placed in a unitised investment of some kind (U.K. equities, property, etc.). The assets
underlying each policyholder’s units form the Unit Fund of the policy, the value of which may be
calculated from the unit price and the number of units held. Unit prices are quoted at two levels,
the “Bid Price” (at which the units can be sold) and the “Offer Price” (at which the units must be
bought).
The proceeds on maturity (at policy duration n) or earlier surrender are usually equal to the bid
value of the units. On death, the benefits are usually equal to the value of the units (at bid price),
subject to a minimum death benefit.
In addition, the office holds a balancing account, called the Sterling Fund (or Sterling Reserves),
into which are paid deductions from the premiums for expenses and fund management charges, and
from which it pays the actual office expenses and death guarantee costs.
The office may also transfer profits/losses from the sterling reserves to the shareholders or with-
profits policyholders (who may be considered to be “investing” in the sale of the unit-linked policies).
They may not take money from the unit fund as this belongs entirely to the policyholders.

14.2 Mechanics of the Unit Fund


Unit prices are quoted at 2 prices, the bid price and the offer price, which is an artificial higher
price. Define
Bid Price
1−λ=
Offer Price

where λ is called the Bid/Offer Spread (λ being perhaps 0.05).


The bid price is the “real” price and all valuation calculations use the bid price.
We use the following notation:

231
232 CHAPTER 14. PROFIT TESTING FOR UNIT-LINKED POLICIES

Pt = the office premium in year t (t = 1, 2, ..., n) (which is actually paid at time t − 1)

at = the allocation proportion in year t


= the proportion of the premium Pt which is allocated to buying units
(at the offer price)

The cost of allocation in year t is the money actually used by the office to buy units, that is

(1 − λ)at Pt .

Hence Pt − (1 − λ)at Pt may be transferred to the sterling reserves as a deduction for expenses.
Define

ct = the fund management charge in year t


= a charge made at the end of year t, usually a percentage
of the value of the unit fund, which is transferred to the sterling fund.

The accumulation of the unit fund


Let iu be the assumed rate of growth per annum of the unit fund.
Define

Ft =the value of the unit fund at time t (after


deduction of the fund management charge,
but before payment of any premium
then due) asssuming that the policy is
still in force.

Define
F0 = 0
Then
Ft = [Ft−1 + (1 − λ)at Pt ] (1 + iu ) − ct (14.2.1)
Suppose that the fund management charge is a proportion m of the unit fund; we have

ct = m [Ft−1 + (1 − λ)at Pt ] (1 + iu ) (14.2.2)

and thus
Ft = (1 − m) [Ft−1 + (1 − λ)at Pt ] (1 + iu ) (14.2.3)
Note. In some cases ct may be a fixed sum rather than a proportion of the fund.

Example 14.2.1. A life office issues a large block of 3-year unit-linked endowment assurances under
which 80% of the first year’s premium and 101% of subsequent premiums are invested in units at
the offer price. The bid price of the units is 95% of the offer price. The units are subject to an
annual management charge of 0.75% of the bid value of the fund at the end of each policy year.
The annual premium is £1,000 and unit prices are assumed to grow at 9% per annum.
Calculate the bid value of the units at the end of each year, according to the office’s projections.
14.3. THE STERLING FUND (OR STERLING RESERVES) 233

Solution
(1) (2) (3) (4) (5)
Cost of Fund brought Fund at end
Allocation forward from start of year
t Pt (1 − λ)at Ft−1 + Pt (1 − λ)at before F.M.C. F.M.C. Ft

1 760.00 760.00 828.40 6.21 822.19


2 959.50 1,781.69 1,942.04 14.57 1,927.47
3 959.50 2,886.97 3,146.80 23.60 3,123.20

(3) = (1.09) × (2)


(4) = 0.0075 × (3)
(5) = (3) − (4).

14.3 The Sterling Fund (or Sterling Reserves)


Money is assumed to earn interest at rate is p.a. in the sterling reserves. Define

et = projected expenses for office in year t (payable at the start of year t)


ct = fund management charge (as before)
(DG)t = death guarantee cost in year t
(
qx+t−1 (St − Ft ) if St > Ft
=
0 if St ≤ Ft

where x = the age of the policyholder at the start of the policy,


and St = the guaranteed minimum death benefit in year t. We suppose that the policy is one
of a large number of similar unit-linked contracts on lives whose mortality follows a specified table,
random variations being ignored.

Define

(SCF )t = the expected net cash flow in the sterling


fund in year t per policy in force at the start of the year
= the “in force” net cash flow

The “initial” expected net cash flow in the sterling fund in year t; that is the expected net cash
flow per policy sold, is given by

t−1 px .(SCF )t (t = 1, 2, ..., n) (14.3.1)

The formula for (SCF )t is

(SCF )t = [Pt − (1 − λ)at Pt − et ](1 + is ) + ct − (DG)t (14.3.2)


Maturity Bonuses
In some policies, there may be a maturity bonus in the form of a fixed sum, or a proportion of
the bid value of the fund at maturity. This money must come from the sterling fund, so we must
adjust (SCF )n as follows:

(SCF )n = “normal” (SCF )n (as in (14.3.2)) − px+n−1 × (Maturity Bonus) (14.3.3)


234 CHAPTER 14. PROFIT TESTING FOR UNIT-LINKED POLICIES

The Profit Vector and Profit Signature


Define

(P RO)t = the profit vector


= the expected net profit to the office in
year t per policy in force at the start of the year
σt = the profit signature
= the expected net profit to the office in
year t per policy sold
=t−1 px .(P RO)t (14.3.4)

There are 2 cases to consider.


Case 1
If there is no need (or desire) to maintain sterling reserves at the end of each policy year, we
have
(P RO)t = (SCF )t for t = 1, 2, ..., n

Case 2
Suppose now that the office wishes to maintain sterling reserves of t V at the end of year t
(t = 1, 2, ..., n − 1). It is assumed that 0 V = n V = 0.
The calculations are very similar to those for conventional profit-testing.
The profit vector is
(P RO)t = (SCF )t + is ·t−1 V − (IR)t (14.3.5)
where

(IR)t = increase in reserves


= px+t−1 ·t V −t−1 V

Therefore

(P RO)t = (SCF )t + (1 + is )t−1 V − px+t−1 ·t V (14.3.6)

In both cases the profit signature is found by equation 4.3.4, i.e.

σt =t−1 px · (P RO)t

Example 14.3.1. (continued from Example 14.2.1)


Suppose that, in addition to the information in example 14.2.1, the death benefit, payable at the
end of the year of death, is the greater of twice the annual premium and the bid value of the units.
The office expects to incur initial expenses on these policies of 20% of the first premium. Renewal
expenses are expected to be £20, payable at the beginning of each policy year after the first. The
mortality rate at each age is assumed to be 0.003. Sterling reserves are assumed to earn interest at
4% per annum. Ignore the possibility of withdrawal.
Assuming that the office holds zero sterling reserves at the end of each policy year, calculate the
profit signature.
14.4. THE ASSESSMENT OF PROFITS 235

Solution
(1) (2) (3) (4) (5) (6)
Premium less cost Accumulation Death Guarantee In Force
of allocation Expenses of Sterling Costs FMC cash flow
t Pt − Pt (1 − λ)at et Fund in year (DG)t ct (SCF )t

1 240.00 200 41.60 3.53 6.21 44.28


2 40.50 20 21.32 0.22 14.57 35.67
3 40.50 20 21.32 0 23.60 44.92

(3) = [(1) − (2)] × (1.04)


(4) = 0.003(2000 − Ft )
(6) = (3) − (4) + (5)

(7) (8) (9)


Profit Vector Profit signature
t (P RO)t t−1 px σt

1 44.28 1 44.28
2 35.67 0.997 35.56
3 44.92 0.994 44.65

(7) = (6) (as no reserves held)


(8) = (0.997)t−1
(9) = (7) × (8)

14.4 The Assessment of Profits


The profit signature {σt } can be assessed in one or more of the following ways.
(1) One could work out the Internal Rate of Return (or yield) by solving
n
X 1
v t σt = 0 where v =
t=1
1+j

(the internal rate of return, j, being the solution of this equation.)

(2) The shareholders may value the net profits at a certain rate of interest, j per annum. This
rate is called the Risk Discount Rate, and may reflect uncertainties in {σt }, with j normally higher
than is .
The net present value of the profits is thus
n
X
N P V (j) = v t σt at rate j.
t=1

(3) The Profit Margin is defined as


n.p.v. of profits
, both at some rate of interest, im say
n.p.v. of premiums
Pn
v t σt
= Pn−1 t=1 , at rate im .
t
t=0 Pt+1 · t px v
236 CHAPTER 14. PROFIT TESTING FOR UNIT-LINKED POLICIES

Example 14.4.1. Find the net present value of the profit signature in example 4.3.1 at a risk
discount rate of 10%.

Solution
3
X
n.p.v. of profit = v t σt at 10%
t=1
= 44.58v + 35.56v 2 + 44.65v 3
= 103.46.

14.5 Zeroisation of the Profit Signature


Suppose that the profit signature, σt , with no provision for sterling reserves at the end of each year
(i.e. t V = 0 for all t), is of the following form:
 
26.22
−8.05
 
(σt ) = 
−5.18
 (14.5.1)
−2.29
0.62
It may be considered undesirable for the shareholders to take a profit of £26.22 at time 1, as
there may not be enough money to cover the “negative profits” at times 2, 3 and 4. Thus the profit
taken by the shareholders in year 1 should be reduced in order to avoid negative sterling fund profits
in years 2, 3 and 4. This process is called Zeroisation.
Let X be the sum needed at time 1 to pay the negative cash flows at times 2, 3 and 4 when the
Sterling Reserves earn interest at rate is = 4 12 % p.a. (for example).
Then
1
X = 8.05v + 5.18v 2 + 2.29v 3 at rate 4 %
2
= £14.46

Note
X is the amount required at time 1, and hence the negative cash flows are discounted to time 1 (not
the start of the policy).
Hence the shareholders may take a profit at time 1 of

26.22 − 14.46 = £11.76.

There will now be no need for capital injections (from the shareholders to the Sterling Reserves)
at times 2, 3 and 4. The shareholders may still take a profit of £0.62 at time 5.
The zeroised profit signature {σt0 } is thus
 
11.76
 0 
 
(σt0 ) = 
 0 

 0 
0.62

Notes
14.6. WITHDRAWALS 237

(1) This process may be carried out even if σ1 < 0.

(2) We may calculate the revised profit vector using the equation

σt0
(P RO)0t =
t−1 px

(3) We may also calculate the sterling reserves, t V , implied by zeroisation.

Example 14.5.1. Using the profit signature (14.5.1), and supposing that qx+t−1 = 0.01 for t =
1, 2, 3, 4, 5, calculate the sterling reserves in each year that are implied by zeroisation of the profit
signature.

Solution
(1) (2) (3) (4) (5) (6)
Original profit Zeroised profit Remainder of Accum. Prob. that
signature signature net-cash flow of (3) policy is in force
t σt σt0 σt − σt0 at is t px tV

1 26.22 11.76 14.46 14.46 0.99 14.61


2 -8.05 0 -8.05 7.06 0.980 7.20
3 -5.18 0 -5.18 2.19 0.970 2.26
4 -2.29 0 -2.29 0 0.961 0
5 0.62 0.62 0 0 0.951 0

(4)
(6) =
(5)
Observe that column (4) gives the sterling fund per policy sold, and the probability of being in
force at time t is t px . Thus

Funds needed at time t per policy sold = t V × P r{ policy is still in force at time t}.

14.6 Withdrawals
So far we have ignored the possibility of surrender. Now assume that the surrender of a policy may
occur only at the end of a policy year (just before payment of the premium then due).
Define

wt = the probability that a policy will be


surrendered at the end of year t (t = 1, 2, ..., n − 1)

Assume that wn = 0 as the policyholder will receive the maturity benefit at that time.
The chance that a policy in force at the start of year t is surrendered at the end of year t is
therefore,
px+t−1 · wt
Define
(SV )t = the surrender value at time t
which is usually equal to Ft , the bid value of the policy’s units, in some cases minus a surrender
penalty of (say) £10 or 1% of Ft .
238 CHAPTER 14. PROFIT TESTING FOR UNIT-LINKED POLICIES

The revised profit vector, (P RO)0t , allowing for withdrawals is

(P RO)0t = (P RO)t + px+t−1 · wt [Ft − (SV )t +t V ] (14.6.1)

where (P RO)t is as before.


If t V = 0 for all t, then obviously

(P RO)0t = (P RO)t + px+t−1 · wt [Ft − (SV )t ]. (14.6.2)

Note that if there is no surrender penalty (i.e. (SV )t = Ft ),

(P RO)0t = (P RO)t .

The profit signature, σt , must also be adjusted to allow for surrenders; we have

σt0 =t−1 p0x · (P RO)0t (14.6.3)

where

0
t−1 px = P r{ policy is in force at time t − 1, allowing for withdrawals}
(
t−1 px (1 − w1 )(1 − w2 )...(1 − wt−1 ) if t ≥ 2
=
1 if t = 1

Note
Even if (SV )t = Ft , and hence (P RO)0t = (P RO)t , σt0 will still differ from σt as the probability of
the policy still being in force in each year will be different, when t > 1.
The profit signature allowing for withdrawals, {σt0 }, may be zeroised in the same way as before.
When calculating the reserves, t V , implied by zeroisation, the probability of the policy being in force
at time t should be changed from t px to

t px (1 − w1 )(1 − w2 )...(1 − wt )

Example 14.6.1. A life office issues a large number of 3-year unit-linked endowment policies to men
aged 65, under each of which level annual premiums of £1,000 are paid. 80% of the first premium
and 105% of each subsequent premium is invested in units at the offer price. There is a bid/offer
spread in unit values, the bid price being 95% of the offer price.

A fund management charge of 0.5% of the bid value of the policyholder’s fund is deducted at the
end of each policy year, before payment of any benefits then due.

The death benefit, which is payable at the end of the year of death, is £3,000 or the bid value
of the units if greater. The maturity value is equal to the bid value of the units.

The office incurs expenses of £100 at the start of the first year and £20 at the start of each of
the second and third years.

Mortality is assumed to follow A1967-70 ultimate. It is assumed that, at the end of each of the
first two policy years, 2% of the surviving policyholders withdraw. The withdrawal benefit is 98%
of the bid value of the units, after deducting the management charge.
14.6. WITHDRAWALS 239

(a) Assuming that the growth in the unit value is 7% p.a. and that the office holds unit reserves
equal to the bid value of units and zero Sterling Reserves at the end of each year, calculate the
profit emerging at the end of each policy year per policy sold. Sterling Reserves are assumed to earn
interest at 6% p.a.

(b) Calculate the revised profit emerging at the end of each year if the office takes a smaller
profit in year 1 in order to ensure that the profit emerging in the second and third policy years is
zero.
Solution
(a) We first work out the unit fund, Ft , per policy sold, assuming it remains in force.
Policy Cost of Funds brought Funds on end Unit fund
Year allocation forward from start before deduction F.M.C. at end of year
t Pt (1 − λ)at Pt (1 − λ)at + Ft−1 of charge ct Ft

1 760.00 760.00 813.20 4.07 809.13


2 997.50 1806.63 1933.09 9.67 1923.42
3 997.50 2920.92 3125.38 15.63 3109.75
We now calculate (P RO)t = (SCF )t and then find (P RO)t + px+t−1 wt (0.02Ft ), where x = 65.

Policy Prem. less cost Accumulation D. G. In force


Year of allocation Expenses in Sterling Fund costs F.M.C. cash flow
t Pt − Pt (1 − λ)at et at 6% interest (DG)t ct (SCF )t

1 240 100 148.40 -52.65 4.07 99.82


2 2.50 20 -18.55 -28.57 9.67 -37.45
3 2.50 20 -18.55 0 15.63 -2.92
(P RO)t px+t−1 wt (0.02Ft ) (P RO)0t 0
t−1 px σt0

99.82 0.32 100.14 1 100.14


-37.45 0.75 -36.70 0.95645 -35.10
-2.92 0 -2.92 0.91245 -2.66
0
where t−1 px =t−1 px (1 − w1 )...(1 − wt−1 ) with w3 = 0.

(b) Let reduction in profit at time 1 be X. Then

X = 35.10v + 2.66v 2 at is = 6%
= 35.48.

Hence profit taken at time 1 is

100.14 − 35.48 = £64.66


240 CHAPTER 14. PROFIT TESTING FOR UNIT-LINKED POLICIES

Exercises

14.1 A life office issues a three-year unit-linked endowment policy to a life aged exactly 60. The
annual premium is £2,000, payable at the start of each year. The allocation proportion is
90% in year 1 and 97% thereafter. At the end of year of death during the term, the policy
pays the higher of £10,000 and the bid value of units allocated to the policy, after deduction
of the fund management charge. A bonus of 2% of the (bid) value of the unit fund is payable
at maturity. The life office makes the following assumptions in projecting future cash flows:

Mortality A1967-70 ultimate


Initial expenses: £300
Renewal expenses: £50, incurred at the start of the
second and the third years
Fund management charge: 2% per annum, taken at the end of each year
before payment of any benefits
Sterling fund interest rate: 4% per annum
Bid/offer spread: 6%
Unit fund growth rate: 10% per annum.
Construct tables to show the following:
(i) the growth of the unit fund;
(ii) the profit signature, assuming that no sterling reserves are held;
(iii) the profit signature after taking into account sterling reserves, given that the sterling
reserves per policy are to be £36.48 before receipt of the premium due at time 1 year and
£78.64 before receipt of the premium due at time 2 years.
In each case, indicate clearly how you calculate your table entries. Ignore the possibility of
surrenders.
14.2 (a) In the context of profit-testing of unit-linked business, define the following terms briefly:
(i) the Unit Fund,
(ii) the Sterling Reserves,
(iii) the profit vector of a policy,
(iv) the profit signature of a policy,
(v) the risk discount rate, and
(vi) zeroisation of Sterling Reserves.
(b) A life office issues a large number of identical 4-year annual premium unit-linked endow-
ment assurances to lives aged 65. According to the office’s calculations, the profit vector per
policy sold, ignoring withdrawals and assuming that no Sterling reserves are maintained at
the end of each year, is as follows (£):
191.12
−111.45
−3.28
10.95

The office’s mortality basis is A1967-70 ultimate, and Sterling Reserves earn interest at 5%
per annum.
Calculate
(i) the profit signature per policy sold, ignoring any need to maintain Sterling
Reserves at the end of each year, and
14.7. EXERCISES 241

(ii) the profit signature per policy sold if Sterling reserves are zeroised.
(c) The office now wishes to make an allowance for surrenders. It assumes that, at the end
of the first and the second policy years, 3% of the surviving policyholders will surrender (just
before payment of the second and third annual premiums respectively.) Surrender values are
equal to the value of the policyholder’s units (after deduction of fund management charges),
with a surrender penalty of £10. Calculate
(i) the revised profit signature per policy sold, ignoring any need to maintain Sterling Reserves
at the end of each year,
(ii) the revised profit signature per policy sold if the Sterling Reserves are zeroised, and
(iii) the net present value, at a risk discount rate of 15% per annum, of the revised profit
signature per policy sold, assuming that the Sterling Reserves are zeroised.
14.3 (a) If a profit test for a unit-linked policy reveals negative cash flows in the second or any
subsequent policy year, it is customary to eliminate these negative values by setting up sterling
reserves at the end of each year.
Describe briefly the technique (“zeroisation”) by which these reserves are calculated.
(b) An office issues a 3-year unit-linked policy with a yearly premium of £500. The death
benefit, payable at the end of the year of death, is £1,000 or the bid value of units if greater.
The maturity value is the bid value of the units at maturity.
95% of each premium is invested in units at the offer price. The bid price of units is 95% of
the offer price. Management charges of 41 % of the bid value of the units are deducted at the
end of each year (before payment of death and maturity claims).
The office expects to incur expenses of £75 at the start of the first year and £25 at the start
of each subsequent year.
Using a profit testing analysis, calculate for a life aged 60 at entry
(i) the expected profit in each of the 3 years per policy in force at the beginning of the year,
(ii) the net present value at the issue date of the expected profit from one policy assuming a
risk discount rate of 10% per annum.
Assume that the unit fund grows at 8% per annum (before deduction of management charges),
that sterling reserves need not be maintained at the end of each year, and that the possibility
of surrender may be ignored. The mortality of policyholders follows A1967-70 ultimate and
sterling reserves earn interest at 6% per annum during each policy year.
14.4 If a profit test for a unit-linked policy reveals negative cash flows in the second or any sub-
sequent policy year, it is customary to eliminate these negative values by setting up sterling
reserves at the end of each year.
Calculate the sterling reserves required at the end of each policy year, per policy then in force,
for a 3- year policy for which the profit signature (with no allowance for sterling reserves at
the end of each year) is (250, - 100, - 50), given that the rate of mortality is 0.01 per annum
at each age and sterling reserves earn interest at 8% p.a.
14.5 An office issues a unit-linked endowment assurance with annual premium £400 and term five
years to a life aged 60 who is subject to A1967-70 ultimate mortality.
The sum assured, payable at the end of year of death or at the maturity date, is the bid value
of the units held, subject to a guaranteed minimum death benefit of £2,000. The allocation
proportion is 70% for the first annual premium and 98% for all subsequent annual premiums.
For units the bid/offer spread is 5% and the annual rate of management charge is 0.75%.
In determining the sterling reserves necessary for the policy the office makes the following
assumptions:
242 CHAPTER 14. PROFIT TESTING FOR UNIT-LINKED POLICIES

Initial expenses: £125


Renewal expenses (associated with the payment of the second and each subsequent premium)
£20 increased by 7% p.a. compound from the outset of the policy.
Growth rate for units: 7% p.a.
Interest rate for sterling fund: 4% p.a.
(a) On this basis
(i) construct a table showing the growth of the unit fund over the duration of the policy, and
(ii) construct a table showing the growth of the sterling fund in the absence of reserves.
(b) Hence determine the sterling reserves which should be held by the office to eliminate the
sterling fund negative cash flows in the second and subsequent years of the policy’s duration.
(c) Consider the unit-linked policy described above.
Suppose, however, that the growth rate for units will be 10% p.a., that the sterling fund
interest rate will be 6% p.a., and that the inflationary growth rate for renewal expenses will
be 4% p.a. (from the outset of the policy).
(i) Construct a table showing the growth of the unit fund over the duration of the policy
and
(ii) Construct a table showing the growth of the sterling fund in the absence of reserves.
(iii)Assuming that the office sets up the sterling fund reserves found above, determine the
resulting sterling fund profit vector and signature. Find also the internal rate of return
corresponding to the profit signature.
14.8. SOLUTIONS 243

Solutions

14.1 (i)
(1)
t Pt (1 − λ)at Pt (1 − λ)at + Ft−1 (1) × 1.1 F.M.C. Ft
1 1692 1692 1861.20 37.22 1823.98
2 1823.60 3647.58 4012.34 80.25 3932.09
3 1823.60 5755.69 6331.26 126.63 6204.63
(ii)
(2) (3) Maturity
t Pt − Pt (1 − λ)at et [(2) − (3)] × 1.04 (DG)t FMC Cost (SCF )t
1 308 300 8.32 118.00 37.22 0 -72.46
2 176.40 50 131.46 97.17 80.25 0 114.54
3 176.40 50 131.46 67.37 126.63 121.89 68.83
(Maturity cost in year 3 = 0.02(6204.63) × p62 .)

t (P RO)t t−1 p60 σt


1 -72.46 1 -72.46
2 114.54 0.98557 112.89
3 68.83 0.96979 66.75
(iii) (P RO)t = (SCF )t + (1 + is )t−1 V −t V.px+t−1 .

t (SCF )t (1.04)t−1 V p59+t .t V (P RO)t t−1 p60 σt


1 -72.46 0 35.95 -108.41 1 -108.41
2 114.54 37.94 77.38 75.10 0.98557 74.02
3 68.83 81.79 0 150.62 0.96979 146.07
14.2 (a) Simple definition of each term required. Check definitions with text.
(b) (i)

t (P RO)t t−1 p65 σt


1 191.12 1 191.12
2 -111.45 0.97597 -108.77
3 -3.28 0.95007 -3.12
4 10.95 0.92226 10.10

(ii) Let X be the amount withheld at time 1 to cover negative cash flows at times 2 and 3.
X = 108.77v + 3.12v 2 at 5% interest
= 106.42

 
84.70
 0 
Hence σt = 
 0 

10.10
(c) (i)
Notice that Ft − (SV )t = 10 for all t.
Hence (P RO)0t = (P RO)t + 10wt p64+t
244 CHAPTER 14. PROFIT TESTING FOR UNIT-LINKED POLICIES

t (P RO)t 10wt p64+t (P RO)0t 0


t−1 p65 σt0
1 191.12 0.29 191.41 1 191.41
2 -111.45 0.29 -111.16 0.94669 -105.23
3 -3.28 0 -3.28 0.89392 -2.93
4 10.95 0 10.95 0.86775 9.50
(
0.03 for t = 1, 2
Notice that wt =
0 for t = 3, 4
(ii) Retain X at time 1.
X = 105.23v + 2.93v 2 at 5% interest
= 102.88
 
88.53
 0 
Hence zeroised profit signature is 
 0 

9.50
(iii) NPV = 88.53v + 9.50v 4 at 15% interest = £82.41.
14.3 (a) Zeroisation is the process whereby the profit in the first year is reduced to pay for any
future negative cash flows (as explained in text).
(b) (i)

(1)
t Pt (1 − λ)at Pt (1 − λ)at + Ft−1 (1) × 1.08 FMC Ft
1 451.25 451.25 487.35 1.22 486.13
2 451.25 937.38 1012.37 2.53 1009.84
3 451.25 1461.09 1577.98 3.94 1574.04

(2) (3)
t Pt − Pt (1 − λ)at et [(2) − (3)] × 1.06 (DG)t FMC (SCF )t = (P RO)t
1 48.75 75 -27.82 7.42 1.22 -34.02
2 48.75 25 25.17 0 2.53 27.70
3 48.75 25 25.17 0 3.94 29.11
(ii)
t (P RO)t t−1 p60 σt
1 -34.02 1 -34.02
2 27.70 0.98557 27.30
3 29.11 0.96979 28.23
3
X
Hence net present value = σt v t at 10% interest
t=1
= −34.02v + 27.30v 2 + 28.23v 3
= £12.84.

14.4  
250
σt = −100
−50
14.8. SOLUTIONS 245

Let X be the sum retained in year 1 to cover the later negative cash flows. Then
X = 100v + 50v 2 at 8% interest
= £135.46


114.54
Hence the zeroised profit signature is σt0 =  0 
0

(1) Accumulation
t σt σt0 σt − σt0 of (1) t px tV
1 250 114.54 135.46 135.46 0.99 136.83
2 -100 0 -100 46.30 0.980 47.24
3 -50 0 -50 0 0.970 0
14.5 (a) (i)
(1)
t Pt (1 − λ)at Pt (1 − λ)at + Ft−1 (1) × 1.07 FMC Ft
1 266.00 266.00 284.62 2.13 282.49
2 372.40 654.89 700.73 5.26 695.47
3 372.40 1067.87 1142.62 8.57 1134.05
4 372.40 1506.45 1611.90 12.09 1599.81
5 372.40 1972.21 2110.26 15.83 2094.43
(ii)
(2) (3)
t Pt − Pt (1 − λ)at et [(2) − (3)] × 1.04 (DG)t FMC (SCF )t
1 134 125 9.36 24.79 2.13 -13.30
2 27.60 21.40 6.45 20.89 5.26 -9.18
3 27.60 22.90 4.89 15.37 8.57 -1.91
4 27.60 24.50 3.22 7.87 12.09 7.44
5 27.60 26.22 1.44 0 15.83 17.27

t (P RO)t t−1 p60 σt


1 -13.30 1 -13.30
2 -9.18 0.98557 -9.05
3 -1.91 0.96979 -1.85
4 7.44 0.95257 7.09
5 17.27 0.93385 16.13
(b) Let X be the amount that has to be withheld at time 1 to pay for the negative cash flows
at times 2 and 3.
Hence X = 9.05v + 1.85v 2 at 4% interest
= £10.41

So thezeroisedprofit signature is
−23.71
 0 
 
σt0 = 
 0 

 7.09 
16.13
246 CHAPTER 14. PROFIT TESTING FOR UNIT-LINKED POLICIES

(4) Accumulation of
t σt − σt0 (4) at 4% interest t px tV
1 10.41 10.41 0.98557 10.56
2 -9.05 1.78 0.96979 1.84
3 -1.85 0 0.95257 0
4 0 - 0
5 0 - 0

(c) (i)

t Pt (1 − λ)at Pt (1 − λ)at + Ft−1 (4)×1 · 1 FMC Ft


1 266.00 266.00 292.60 2.19 290.41
2 372.40 662.81 729.09 5.47 723.62
3 372.40 1096.02 1205.62 9.04 1196.58
4 372.40 1568.98 1725.88 12.94 1712.94
5 372.40 2085.34 2293.87 17.20 2276.67
(ii)
(5) (6)
t Pt − Pt (1 − λ)at et [(5) − (6)] × 1.06 (DG)t FMC (SCF )t
1 134.00 125.00 9.54 24.67 2.19 -12.94
2 27.60 20.80 7.21 20.44 5.47 -7.76
3 27.60 21.63 6.33 14.26 9.04 1.11
4 27.60 22.50 5.41 5.64 12.94 12.71
5 27.60 23.40 4.45 0 17.20 21.65

(iii)
t (SCF )t tV (1.06)t−1 V p59+t · t V (P RO)t t−1 p60 σt
1 -12.94 10.56 0 10.41 -23.35 1 -23.35
2 -7.76 1.84 11.19 1.81 1.62 0.98557 1.60
3 1.11 0 1.95 0 3.06 0.96979 2.97
4 12.71 0 0 0 12.71 0.95257 12.11
5 21.65 0 0 0 21.65 0.93385 20.22
This uses
(P RO)t = (SCF )t + (1 + is )t−1 V − px+t−1 .t V

Let the internal rate of return be j p.a. Then j solves


X5
1
σt v t = 0 where v = .
t=1
1+j

i.e. −23.35v + 1.60v 2 + 2.97v 3 + 12.11v 4 + 20.22v 5 = 0.


Solving this equation by trials and interpolation gives
v = 0.87173.

Hence j = 0.147 = 14.7% p.a.


Chapter 15

MULTIPLE-DECREMENT
TABLES

15.1 Introduction
Consider a body of lives subject to two “modes of decrement”, α and β. For example, consider
a group of bachelor employees of a large company, from which men can leave by either mode α,
marriage, or by mode β, leaving the company (mortality being ignored).
Take a bachelor employee aged x, and let

T1 =time to marriage of (x), whether


or not he is still an employee of the company

and

T2 =time until (x) leaves the service of the company, whether


he is then married or not

(At this point it is assumed that all bachelors eventually marry.)

Let T = min{T1 , T2 } =time until exit from the group of


bachelor employees, by either mode α
(marriage) or mode β (withdrawal from service),
whichever comes first, of a
bachelor employee aged x

Note This is similar to the joint-life situation in Chapter 11.

Define

t (ap)x =the probability that (x) will “survive” for at


least t years with respect to both modes
of decrement. (In the above example, he will not
marry or leave the service within t years.)
=P r{T ≥ t}

247
248 CHAPTER 15. MULTIPLE-DECREMENT TABLES

and

t (aq)x =P r{T < t}


=P r{(x) leaves before time t, either by
mode α or mode β}

In the example above

t (aq)x = the probability that (x) gets married


or leaves service (or does both
these things) within t years

We may proceed through the development of “life tables”, by merely putting “a” in front of the
various functions. For example, the function, (al)x , x ≥ x0 , is constructed to be such that
(al)x+t
t (ap)x = , (x ≥ x0 , t ≥ 0)
(al)x
When t = 1 it may be omitted, giving
(ad)x (al)x − (al)x+1
(aq)x = =
(al)x (al)x
and

(ad)x+t
t |(aq)x =
(al)x
Also, define the “force of exit” from the double-decrement table (by whichever mode occurs first)
by
h (aq)x
(aµ)x = lim+
h→0 h
It follows as for ordinary life tables that
· Z t ¸
t (ap)x = exp − (aµ)x+r dr
0
The probability density function of T is
(
t (ap)x (aµ)x+t , t≥0
0 , t<0

and its distribution function is


Z t
t (aq)x = r (ap)x (aµ)x+r dr (t ≥ 0)
0
We also define

(am)x =central rate of “total decrement” (modes


α and β together)
R1
(ad)x (al)x+t (aµ)x+t dt
= = 0 R1
(aL)x (al)x+t dt 0
15.2. THE ASSOCIATED SINGLE-DECREMENT TABLES 249

15.2 The Associated Single-Decrement Tables


Each mode of decrement may be thought of as possessing its own “life table”, denoted by, for
example, lxα or lxβ . Thus

α
t px = P r{(x) does not exit by mode α before time t,
whether exit by mode β happens or not}
( = P r{(x) does not marry within t years } in our example.)

β
t px = P r{(x) does not exit by mode β before time
t, whether exit by mode α happens or not}
( = P r{(x) does not leave service within t years} in our example.)

Note that
α
t px = P r{T1 ≥ t} and t pβx = pr {T2 ≥ t}.
We also have

α
t qx = P r{(x) exits by mode α before time t,
whether exit by mode β happens or not}
= P r{T1 ≤ t}
= 1 − t pα
x.

Similarly

β
t qx = P r{T2 ≤ t}
= 1 − t pβx .

The functions, t q α β
x , t q x are called the independent rates (or probabilities) of exit within t years
at age x in their respective single-decrement tables.

15.3 The Relationships between the Multiple-Decrement Ta-


ble and its Associated Single-Decrement Tables
It is usually assumed that the variables T1 and T2 are independent. This gives

t (ap)x = P r{T1 ≥ t and T2 ≥ t}


= P r{T1 ≥ t}P r{T2 ≥ t}
= t pα β
x t px (15.3.1)

Hence we may construct the function (al)x from lxα , lxβ . In particular
β
(al)x+t lα lx+t
t (ap)x = = x+t · for all x ≥ x0 , t ≥ 0
(al)x lxα lxβ
250 CHAPTER 15. MULTIPLE-DECREMENT TABLES

Put x = x0 , y = x0 + t to obtain

(al)y = k.lyα .lyβ (y ≥ x0 )

where k is a constant. If we choose the radix (al)x0 to be equal to lxα0 .lxβ0 , then

(al)y = lyα .lyβ for all y ≥ x0 ( as k = 1)

Some Important Formulae

t (aq)x = 1 − t (ap)x
= 1 − (1 − t q α β
x )(1 − t q x ), assuming independence of T1 and T2 ,

= t qα β α β
x + t q x − t q x .t q x

If t = 1 ,
(aq)x = qxα + qxβ − qxα qxβ (15.3.2)
Also

h (aq)x
(aµ)x = lim
h→0+ h
· α β α β
¸
h q x + h q x − h q x .h ax
= lim+
h→0 h
α β
µ α
¶µ β

h qx h qx h qx h qx
= lim + lim + lim h
h→0+ h h→0+ h h→0+ h h
= µα β
x + µx (similar to joint-life argument).

15.4 Dependent Rates of Exit


Define
α
t (aq)x = P r{(x) exits by mode α within time t,
exit by mode β not having previously occurred}

In terms of our example, this is the probability that (x) gets married within t years while still
an employee of the company.
We have
α
t (aq)x = P r{T1 < t and T1 < T2 }

where T1 , T2 have p.d.f.’s

f1 (t1 ) = t1 pα α
x µx+t1 , t1 > 0,
β β
f2 (t2 ) = t2 px µx+t2 , t2 > 0, respectively .

So
15.4. DEPENDENT RATES OF EXIT 251

ZZ
α
t (aq)x = f1 (t1 ).f2 (t2 ) dt1 dt2
t1 <t2
t1 <t
Z tZ ∞
α α β β
= t1 px µx+t1 .t2 px µx+t2 dt2 dt1
0 t1
Z t µZ ∞ ¶
α α β β
= t1 px µx+t1 t2 px µx+t2 dt2 dt1
0 t1
Z t
α α β
= t1 px µx+t1 .t1 px dt1
0

Thus

Z t
α α α β
t (aq)x = r px .µx+r .r px dr (15.4.1)
0
Z t
α
= r (ap)x µx+r dr
0

When t = 1, Z 1
(aq)α
x = α α β
t px µx+t .t px dt.
0

(aq)α β
x , (aq)x are called the dependent rates (or probabilities) of exit at age x by mode α, β respec-
tively. (The word ‘dependent’ indicates that (aq)α x depends not only on mode α but also on mode β).

Example 15.4.1. A population is subject to 2 modes of decrement, α and β. For 30 ≤ x ≤ 32, it


is known that

µα
x = 0.05

and µβx = (35 − x)−1

For an individual in this population at exact age 30, calculate


(a) the probability that he will still be in the population at age 32.
(b) the probability that he will leave by mode α before age 32.
Solution
µ Z t ¶
(a) t pα
30 = exp − µα
30+r dr = e−0.05t
0
µ Z t ¶
β −1
p
t 30 = exp − [35 − (30 + r)] dr
0
µ Z t ¶
1
= exp − dr
0 5−r
5−t
= exp[log(5 − r)]t0 = .
5
β 3
Thus 2 (ap)30 =2 pα
30 .2 p30 = e
−0.1
· 5 = 0.5429
252 CHAPTER 15. MULTIPLE-DECREMENT TABLES

Z 2
β
(b) 2 (aq)α
30 =
α α
t p30 µ30+t .t p30 dt
0
Z 2 µ ¶
5−t
= 0.05e−0.05t dt
0 5
Z 2 Z 2
−0.05t
= 0.05 e dt − 0.01 t.e−0.05t dt
0 0
2
= (1 − e−0.1 ) + e−0.1 − 4(1 − e−0.1 ) (using integration by parts)
5
= 0.07645.

Theorem If there is a Uniform Distribution of Decrements between ages x and x + 1 in each


single-decrement table, then µ ¶
1 β
(aq)αx = q α
x 1 − q
2 x
and µ ¶
1
(aq)βx = qxβ 1 − qxα
2
Proof Under a Uniform Distribution of Decrements (U.D. of D.) for a life table,
(a) lx+t is linear (0 ≤ t ≤ 1)
(b) t q x = t.qx (0 ≤ t ≤ 1)
(c) t px µx+t = qx (0 ≤ t ≤ 1)

Hence
Z 1
(aq)α
x =
α α β
t px .µx+t .t px dt
0
Z 1
= qxα (1 − t q βx ) dt (using U.D. of D. for mode α)
0
Z 1
= qxα (1 − t.qxβ ) dt (using U.D. of D. for mode β)
0
· ¸t=1
t2 β
= qxα
t − qx
2 t=0
1
= qxα (1 − qxβ ).
2
Note
It is often assumed that U.D. of D. holds approximately in each single-decrement table, so the above
results may be used as approximations.

Example 15.4.2. In a double-decrement table with 2 causes of decrement, death (d) and withdrawal
(w), the central rates of decrement at a certain age x are
mdx = 0.01
mwx = 0.2
Show that if the central rate of withdrawal was doubled, the dependent rate of mortality would be
reduced by approximately 0.00076.
15.5. PRACTICAL CONSTRUCTION OF MULTIPLE-DECREMENT TABLES 253

Solution Assuming U.D. of D.,

mdx
qxd ' = 0.009950
1 + 21 mdx
mw x
qxw ' = 0.18182
1 + 21 mwx
1
Hence (aq)dx ' qxd (1 − .qxw ) = 0.00905
2

Now suppose mw
x is increased to 0.4.
Then
0.4
qxw ' = 0.3333.
1 + 0.2
Hence

1
(aq)dx ' qxd (1 − × 0.3333) = 0.00829
2
i.e. a reduction of 0.00076.

Getting qxα , qxβ from (aq)α β


x , (aq)x
Assuming U.D. of D., we have shown that
1 β
(aq)α α
x = qx (1 − qx ), (i)
2
β β 1 α
(aq)x = qx (1 − qx ) (ii)
2
(aq)β
Substituting qxβ = x
1− 12 qx
α from (ii) into (i) gives a quadratic equation for qxα , i.e.

(qxα )2 + qxα [(aq)βx − (aq)α α


x − 2] + 2(aq)x = 0.

This can be solved (rejecting any solution outside the range 0 and 1) to find qxα , and hence qxβ
may be found from (i) or (ii).

15.5 Practical Construction of Multiple-Decrement Tables


We define

(ad)α
x =the expected number of exits by mode α
between ages x and x + 1 in a
multiple-decrement table with
(al)x lives at age x
=(al)x (aq)α
x

Note also that

(aq)x = (aq)α β
x + (aq)x
254 CHAPTER 15. MULTIPLE-DECREMENT TABLES

To prove these results, we note that


α β
t (aq)x + t (aq)x = P r{T1 ≤ t and T1 ≤ T2 } + P r{T2 ≤ t and T2 ≤ T1 }
= P r{min{T1 , T2 } ≤ t}
= t (aq)x .

and hence (putting t = 1, which can be omitted)

(ad)x = (al)x (aq)x


= (ad)α β
x + (ad)x

Also,

α
t |(aq)x = P r{(x) leaves by mode α in the
multiple-decrement table between
ages x + t and x + t + 1}
= t (ap)x (aq)α
x+t
(ad)α
x+t
=
(al)x

Procedure
There are 3 basic steps in the construction of a multiple-decrement table.
Step 1
Choose a radix (al)x0 where x0 = the youngest age considered.
(For example, (al)x0 may be 10,000 or 100,000).
Step 2
Calculate qxα , qxβ (x = x0 , x0 + 1, ...) and hence evaluate (aq)α β
x , (aq)x (x = x0 , x0 + 1, ...), assuming
U.D. of D.
Step 3
Calculate
(ad)α α
x0 = (al)x0 (aq)x0

and

(ad)βx0 = (al)x0 (aq)βx0


Then find

(al)x0 +1 = (al)x0 − [(ad)α β


x0 + (ad)x0 ]
= (al)x0 − (ad)x0

Repeat this procedure for x0 + 1, x0 + 2 and so on.

Example 15.5.1. A certain population is subject to 2 modes of decrement,

d = death
i = permanent disability
15.5. PRACTICAL CONSTRUCTION OF MULTIPLE-DECREMENT TABLES 255

The independent rates of mortality are in accordance with A67-70 ult., and there is an indepen-
dent rate of disablement of 0.01 at each age x (60 ≤ x ≤ 62).
Construct a multiple-decrement table for ages x = 60, 61, 62 (and include the number of survivors
at age 63.)

Solution
Use U.D. of D. to find
1 i
(aq)dx = qxd (1 − ·q )
2 x
and
1 d
(aq)ix = qxi (1 − ·q )
2 x
Choose a radix of (al)x0 = 100, 000 (for example)

x qxd qxi (aq)dx (aq)ix (al)x (ad)dx (ad)ix

60 0.01443 0.01 0.01436 0.00993 100,000 1,436 993


61 0.01601 0.01 0.01593 0.00992 97,571 1,554 968
62 0.01775 0.01 0.01776 0.00991 95,049 1,679 942
63 92,428

Applications of Multiple-Decrement Tables

1. The probabilities of various events can be calculated.

Example 15.5.2. Using the multiple-decrement table constructed in example 15.5.1, calculate
(a) The probability that a person aged exactly 60 is alive and not disabled
at age 63.
(b) The probability that a person aged 60 becomes disabled within 3 years.

Solution
(a)

(al)63
3 (ap)60 =
(al)60
92428
= = 0.92428
100000
(b)

i (ad)i60 + (ad)i61 + (ad)i62


3 (aq)60 =
(al)60
993 + 968 + 942
=
100, 000
= 0.02903

2. Financial calculations can be carried out using multiple-decrement tables, as will be shown in the
next chapter.
256 CHAPTER 15. MULTIPLE-DECREMENT TABLES

15.6 Further Formulae


The Identity of the forces
Define
α
h (aq)x
(aµ)α
x = lim
h→0+ h
Theorem
(aµ)α α
x = µx for all x (15.6.1)

Proof
Assume that µα β
x and µx are continuous.
Then
α
h (aq)x
(aµ)α
x = lim+
h→0 h
Rh
0 t
(ap)x µα
x+t dt
= lim+
h→0 h

Now, by L’Hôpital’s rule,

d
Rh α
dh [ 0 t (ap)x µx+t dt]
(aµ)α
x = lim d
dh (h)
h→0+
α
h (ap)x µx+h
= lim+
h→0 1
= µα
x (as lim h (ap)x = 1)
h→0+

This result is called the identity of the forces. Note that

h (aq)x
(aµ)x = lim+
h→0 h
· α β
¸
h (aq) x h (aq)x
= lim +
h→0+ h h
= (aµ)α β
x + (aµ)x

= µα β
x + µx

Central Rates of Decrement


The dependent central rate of decrement by mode α at age x is given by

(ad)α
x
(am)α
x =
(aL)x
Rl
(al)x+t (aµ)αx+t dt
= 0 R1
0
(al)x+t dt
R1
(al)x+t µαx+t dt
= 0R 1 (from (15.6.1))
0
(al)x+t dt
' µα
x+ 1 2

Compare this to the corresponding independent central rate of decrement:


15.7. GENERALIZATION TO 3 MODES OF DECREMENT 257

R1 α
lx+t µα
x+t dt

x = 0
R1 α
l
0 x+t
dt
' µα
x+ 1 2

Therefore it is normally assumed that

(am)α α
x ' mx

Note
We may also construct tables using select rates of decrement, using

qxd = q[x] , qx+1


d
= q[x]+1 , ...

for example.

15.7 Generalization to 3 Modes of Decrement


Suppose there are now 3 modes of decrement, α, β and γ. By generalising the results for 2 modes,
it can be shown that
α β γ
t (ap)x = t px .t px .t px

and Z 1
(aq)α
x =
α α β γ
t px µx+t .t px .t px dt
0
Theorem
Under U.D. of D. of each mode of decrement in its single-decrement table
1 β 1 β γ
(aq)α α γ
x = qx [1 − (qx + qx ) + qx · qx ]
2 3
Proof
Z 1
(aq)α
x = α α β γ
t px µx+t .t px .t px dt
0
Z 1
= qxα (1 − t q βx )(1 − t q γx ) dt (using U.D. of D. on α)
0
Z 1
= qxα (1 − t.qxβ )(1 − t.qxγ ) dt (using U.D. of D. on β and γ)
0
Z 1 £ ¤
= qxα 1 − t(qxβ + qxγ ) + t2 qxβ .qxγ dt
0
· ¸t=1
t2 β t3 β γ
= qxα γ
t − (qx + qx ) + qx .qx
2 3
· ¸ t=0
1 1
= qxα 1 − (qxβ + qxγ ) + qxβ .qxγ .
2 3

Note also that the identity of the forces remains true, and

(aµ)x = µα β γ
x + µx + µx .
258 CHAPTER 15. MULTIPLE-DECREMENT TABLES

In the practical construction of a multiple-decrement table, we find the values of (al)x , (ad)α
x,
(ad)βx , (ad)γx and use
(al)x+1 = (al)x − [(ad)α β γ
x + (ad)x + (ad)x ].

Example 15.7.1. The members of a large company’s manual workforce are subject to three modes
of decrement, death, withdrawal and promotion to supervisor. It is known that these workers’
independent rates of mortality are those of English Life Table No. 12 - Males, the independent
withdrawal rate is 0.04 at each age, and their independent promotion rate is 0.02 at age 50 and 0.03
at age 51.

(a) Draw up a service table for manual workers from age 50 to age 51 with a radix of 100,000
at age 50, including the value of (al)52 .

(b) Calculate the probability that a life aged exactly 50 will gain promotion within 2 years.
Solution

(a) Let d = death, p = promotion, w = withdrawal.


x qxd qxw qxp (aq)dx (aq)w
x (aq)px (al)x (ad)dx (ad)w
x (ad)px

50 0.00728 0.04 0.02 0.00706 0.03946 0.01953 100,000 706 3946 1953
51 0.00823 0.04 0.03 0.00795 0.03924 0.02928 93,395 742 3665 2735
52 86,253
This table assumes U.D. of D., i.e.
1 1
(aq)dx = qxd [1 − (qxw + qxp ) + qxw .qxp ].
2 3
with similar formulae for withdrawal and promotion rates.

(b)

p (ad)p50 + (ad)p51
2 (aq)50 =
(al)50
1953 + 2735
= = 0.04688.
100, 000

15.8 “Abnormal” Incidence of Decrement


Suppose that there are 2 modes of decrement, α and β operating between ages x and x + 1. Assume
mode β operates “smoothly” (i.e. µβx exists and is continuous) between ages x and x + 1. However,
mode α only operates at a particular age, x + k (0 ≤ k ≤ 1).
That is (
α 1 for t < k
t px = α
1 − qx for t > k.
It follows that µα
x+t is not defined for t = k and hence formulae such as
Z 1
α α β α
(aq)x = t px .t px µx+t dt
0
15.8. “ABNORMAL” INCIDENCE OF DECREMENT 259

cannot be used.

To deal with this “abnormal” mode of decrement, we argue that from age x to x + k, mode β
operates by itself. Then, at age x + k, there is a chance qxα of exit by mode α, so

(aq)α
x = P r{(x) survives to age x + k under mode β only}
× P r{(x) leaves at age x + k by mode α, given
survival until then}
= k pβx .qxα
= qxα (1 − k q βx )

If (as usually the case) U.D. of D. for mode β is assumed

(aq)α α β
x = qx (1 − k.qx ) (15.8.1)
To obtain (aq)βx , we may use

(aq)βx = (aq)x − (aq)α


x

= [qxα + qxβ − qxα .qxβ ] − qxα (1 − k.qxβ )

So

(aq)βx = qxβ [1 − (1 − k)qxα ] (15.8.2)


Special Cases
1. If k = 12 , we obtain the familiar formulae

1 β
(aq)α α
x = qx (1 − ·q )
2 x
and
1 α
(aq)βx = qxβ (1 − ·q )
2 x
2. If k = 0
(aq)α α
x = qx

and
(aq)βx = qxβ (1 − qxα )
3. If k = 1, (i.e. we consider exits by mode α to occur just before reaching age x + 1)

(aq)α α β
x = qx (1 − qx )

and
(aq)βx = qxβ .

Example 15.8.1. Military personnel attend a training camp for an intensive course which involves
3 weeks of continual exercises.

Each soldier remains in the camp for exactly 3 weeks, provided he is not hospitalised because
260 CHAPTER 15. MULTIPLE-DECREMENT TABLES

of injury or “failed” by one of the instructors and sent back to his base. The independent weekly
Hospitalised
Week through injury Being failed
rates of decrement are as follows:
1 0.078 0.132
2 0.102 0.092
3 0.058 0.043
(i) Of a group of 1,000 soldiers who start the course, calculate the number who will successfully
complete the course, the number who will be hospitalised and the number who will be failed.
Assume a uniform distribution over each week of hospitalisation and failure. Anyone sent to
hospital or failed leaves the camp immediately and does not return. There are no other modes of
decrement.

(ii) Some time later the course is altered so that in the first week the instructors test the sol-
diers only at the start of the sixth day, and the independent weekly rate of being failed in week 1
alters to 0.160. During weeks 2 and 3, testing takes the form of continuous assessment as previously,
with no change to the independent weekly rates of being failed.
Calculate the revised numbers of soldiers successfully completing the course, being hospitalised
and being failed, assuming nothing else changes.
Solution

(i) Consider “age” as time (in weeks) since entry to camp. Assume U.D. of D., which gives
1
(aq)hx = qxh (1 − qxf )
2
1
and (aq)x = qx (1 − · qxh )
f f
2
where h = hospitalised, f = failed.

x qxh qxf (aq)hx (aq)fx (al)x (ad)hx (ad)fx


0 0.078 0.132 0.07285 0.12685 1000 73 127
1 0.102 0.092 0.09731 0.08731 800 78 70
2 0.058 0.043 0.05675 0.04175 652 37 27
3 588
Hence number who complete the course = 588.

5
(ii) Here, k = 7 (test is at end of 5th day), so
2 f
(aq)xh = qxh (1 − ·q )
7 x
and
5 h
(aq)fx = qxf (1 − ·q )
7 x
for x = 0 only.
q0f is now 0.16, but all the other values of qxf and qxh are unchanged.
x qxh qxf (aq)hx (aq)fx (al)x (ad)hx (ad)fx
0 0.078 0.16 0.07443 0.15109 1000 74 151
1 0.102 0.092 0.09731 0.08731 775 75 68
2 0.058 0.043 0.05675 0.04175 632 37 26
3 570
Hence now the number who complete the course = 570.
15.8. “ABNORMAL” INCIDENCE OF DECREMENT 261

Application to Profit-Testing
Let α = withdrawal, and β = mortality. It is assumed that withdrawals occur only at the end of a
policy year. That is, mode α only operates at time k = 1.
In chapter 14, the following results were used:

P r{ policy in force at time t − 1 will be surrendered at time t}


mortality withdrawal
= (1 − qx+t−1 ).qx+t−1
withdrawal
where qx+t−1 was denoted by wt
and
mortality
P r{ death occurs in policy year given it is in force at start of year} = qx+t−1

These formulae correspond exactly to those given in this section when k = 1.

Extension to 3 modes of decrement


With 3 modes of decrement, formulae similar to (15.8.1) and (15.8.2) may be awkward to derive.
These problems are best handled from first principles. Often one mode (α, say) operates at exact
age x and the others (β and γ) operate uniformly over the year of age from x to x + 1.
This can be dealt with by defining (al)x = number of lives at age x before mode α operates, and
(al)+
x = number of lives at age x after mode α operates.
Then the numbers of exits by modes β and γ between ages x and x + 1 can be worked out as if
there were no other modes of decrement. Note however that

(ad)βx = (al)+ β
x (aq)x
1
where (aq)βx = qxβ (1 − qxγ )
2
Similar calculations apply to “abnormal” exits at the end of the year.
262 CHAPTER 15. MULTIPLE-DECREMENT TABLES

Example 15.8.2. Among the employees of a certain firm retirement may take place at or after age
57, but is compulsory at age 60. The independent rates of mortality of the employees are those of
A1967-70 ultimate. 20% of those attaining age 57 retire at once. Leaving aside these retirements,
the central rates of retirement are as follows:
Central Rate
Age of Retirement
57 0.09
58 0.08
59 0.05
There are no withdrawals or ill-health retirements after age 50.
Construct a service table for employees from age 57 to age 60, with a radix of 100,000 employees
attaining age 57.
Solution
Create a special mode of decrement, r0 , to refer to retirement at exact age x. Then,
0
(ad)r57 = 20, 000,

leaving (al)+
57 lives who are subject to ‘normal’ retirements and death, both of which are approxi-
mately “U.D. of D.”
So construct a table of (aq)dx and (aq)rx (ignoring mode r0 ).
mrx
x qxd qxr ' 1+ 21 mrx
(aq)dx (aq)rx

57 0.01050 0.08612 0.01004 0.08567


58 0.01169 0.07692 0.01124 0.07647
59 0.01299 0.04878 0.01267 0.04846
Now construct the multiple-decrement table, noting that

(ad)rx = (al)+ r
x (aq)x .
0
x (al)x (ad)rx (al)+
x (ad)rx (ad)dx

57 100,000 20,000 80,000 6,854 803


58 72,343 0 72,343 5,532 813
59 65,998 0 65,998 3,198 836
60 61,964 61,964 0
Note
0
We may take (al)60 = (ad)r60 since all survivors at age 60 retire at once.
15.9. EXERCISES 263

Exercises

15.1 Let α1 , α2 be the modes of decrement in a double-decrement table. Suppose that α1 is


uniformly distributed over the year of age from x to x + 1 in its associated single- decrement
table, and µα
x+t = c for 0 ≤ t ≤ 1.
2

Find formulae for (aq)α α2 α1


x and (aq)x in terms of qx and qx .
1 α2

15.2 For a certain group of married women, for whom remarriage is not permitted, the dependent q-
type rate of widowhood at each integer age x from 70 to 72 inclusive is twice the corresponding
dependent q-type rate of mortality. The independent rates of mortality of wives follow a(55)
ultimate (females).
(i) Using a radix of 100,000 and assuming a uniform distribution of each mode of decrement in
its associated single-decrement table, construct a double-decrement table for married women
from age 70 to age 72 inclusive, giving also the value of (al)73 , the number of wives at age 73.
(ii) Find the probabilities that a wife now aged 70 will
(a) be alive and married at age 73; and
(b) be widowed within 3 years.
15.3 In a certain country, widowed and divorced men are subject to the following independent
q-type rates of decrement:
mortality: English Life Table No. 12 - Males
remarriage: rates depend on the age at, and the duration since, the end of former marriage;
the following table is an extract from these rates:
exact
age at end duration duration
of former 0 1
marriage year year

50 0.050 0.025
51 0.045 0.023
52 0.042 0.020
Calculate the probability that a man aged exactly 50 whose marriage has just ended will
remarry within 2 years.
15.4 A large industrial company recruits a constant number of school leavers aged exactly 18 years
on 1 July each year.
Upon joining, workers undergo training for one year. Of those who complete this period of
training, ten per cent fail a final test of competence and are dismissed. Employees may also
leave service voluntarily at any time. The central rate of voluntary withdrawal from service
is 0.15 for trainees and 0.10 at each age for fully trained employees.
The occupation is hazardous and all workers, including trainees, are exposed to the risk of
injury. The independent q-type rate of injury is 0.051219 at age 18 and 0.050030 at ages 19
and above. An employee who is injured is transferred to alternative work with a subsidiary
company, at a relocation cost of £1,000.
The mortality of all employees follows English Life Tables No.12 - Males. The number of
employees attaining age 21 each year is 500.

(i) Construct a service table covering the first 3 years of employment with the original com-
pany, distinguishing between those about to take the final test of competence and those who
pass it. [Regard failing the test as a special mode of decrement ]
(ii) How many people are recruited on each 1st July?
264 CHAPTER 15. MULTIPLE-DECREMENT TABLES

Solutions

15.1 α1 is uniformly distributed, so


α1 α1 α1
t px µx+t = qx for 0≤t≤1

µα
x+t = c, so
2
Z t
α2
t px = exp[− µα
x+t dt] = e
2 −ct
for 0 ≤ t ≤ 1
0

Z 1
α1 α1
(aq)α
x =
1 α2
t px µx+t .t px dt
0
Z t
= qxα1 e−ct dt
0
· ¸t=1
1 −ct
= qxα1 − e
c t=0
(1 − e−c )
= qxα1
· c α2 ¸
α1 −qx
= qx as c = − log(1 − qxα2 )
log(1 − qxα2 )

(aq)α α1
x = (aq)x − (aq)x
2

· ¸
−qxα2
= [qxα1 + qxα2 − qxα1 qxα2 ] − qxα1 (from above)
log(1 − qxα2 )

15.2 (i) (aq)w d


x = 2(aq)x for x = 70, 71, 72
where
w = widowhood
d = death.
Using U.D. of D.

1 d 1 w
(aq)w w d d
x = qx (1 − qx ) = 2qx (1 − qx ) = 2(aq)x
2 2
d
2qx
Hence qxw = 1+ 21 qx
d

d d
qx (1− 21 qx )
Therefore (aq)dx = 1+ 21 qxd , and (aq)w d
x = 2(aq)x .

x qxd (aq)dx (aq)w


x (al)x (ad)dx (ad)w
x
70 0.02307 0.02254 0.04509 100,000 2,254 4,509
71 0.02559 0.02494 0.04989 93,237 2,326 4,651
72 0.02839 0.02760 0.05519 86,260 2,380 4,761
73 79,119
79119
(ii)(a) 3 (ap)70 = 100,000 = 0.79119
(ad)w w w
70 +(ad)71 +(ad)72
(b) 3 (aq)w
70 = (al)70 = 0.1392.
15.3 Denote
d = death
15.10. SOLUTIONS 265

r = remarriage
r
Notice that q51 = q[50]+1 , i.e. duration 1 year from age 50 at end of former marriage.
x qxd qxr (aq)dx (aq)rx (al)x (ad)dx (ad)rx
50 0.00728 0.050 0.00710 0.04982 100,000 710 4,982
51 0.00823 0.025 0.00813 0.02490 94,308 767 2,348
91,193
r (ad)r50 + (ad)r51
2 (aq)50 =
(al)50
= 0.0733.

15.4 (i) Denote


d = death
w = withdrawal (excluding those who fail test)
f = those who fail test
i = injury

Use
mw x
qxw '
1 + 21 mw
x

and in view of U.D. of D. use formulae such as

1 1
(aq)dx = qxd [1 − (qxw + qxi ) + qxw qxi ]
2 3
for deaths, injuries and withdrawals, and treat the test as an “abnormal” mode of exit.
x qxd qxi qxw (aq)dx (aq)ix (aq)w
x
18 0.00112 0.05122 0.13953 0.001016 0.04762 0.13588
19 0.00117 0.05003 0.09524 0.001087 0.04762 0.09280
20 0.00119 0.05003 0.09524 0.001105 0.04762 0.09280
Define
(al)19 = number of people before test (at age 19 exactly)
(al)+
19 = number of people who pass test.
with
(al)+
19 = 0.9(al)19

Let (al)18 = 10, 000 (for example)


x (al)x (ad)fx (al)+
x (ad)dx (ad)ix (ad)wx
18 10,000 0 10,000 10 476 1359
19 8,155 816 7,339 8 349 681
20 6,301 0 6,301 7 300 585
21 5,409
Notice that (ad)fx = (al)x − (al)+
x
(ii) 10,000 entrants give 5,409 employees after 3 years. Hence 924 entrants are needed to get,
on average, 500 employees aged exactly 21.
266 CHAPTER 15. MULTIPLE-DECREMENT TABLES
Chapter 16

FINANCIAL CALCULATIONS
USING
MULTIPLE-DECREMENT
TABLES

16.1 Principles
One may value cash flows, calculate premiums and find reserves using the same ideas as are used
when there is just one mode of decrement (death).
Notation is best considered from first principles, and there are no commutation functions (except
for pensions, which are covered later).
The procedure is generally as follows:
Stage 1
Construct a multiple-decrement table.
Stage 2
Write down an equation of value of the form
m.p.v. of premiums = m.p.v. of benefits + m.p.v. of expenses
(This assumes that the expected present value of the profits to the office is zero. If it is not, add
a suitable profit term to the equation.)
Then solve the equation of value for the unknown quantity (for example, the annual premium or
the sum assured payable on death)
Note
It should normally be assumed that exits occur, on average, in the middle of each policy year, and
that premiums cease if a life exits by any mode of decrement.

16.2 The Use of “Defective” Variables


A defective random variable, T , is such that
lim F (t) = k < 1
t→∞

where F (t) is the distribution function of T . For example, let


T = time of exit of (x) by mode α in a double-decrement table, the other mode of exit being β.
Then

267
268CHAPTER 16. FINANCIAL CALCULATIONS USING MULTIPLE-DECREMENT TABLES

F (t) = P r{T ≤ t}
α
= t (aq)x
Z t
α α β
= r px µx+r .r px dr
0

Since
α β
∞ (aq)x + ∞ (aq)x = 1,
we have
α
lim F (t) = lim t (aq)x < 1
t→∞ t→∞

A defective variable T may have a probability density function, f (t) = F 0 (t). In the above
example, (
α α β
t px µx+t .t px , t > 0
f (t) =
0, t<0
With defective variables, we still have results such as
Z ∞
E[g(T )] = g(t)f (t) dt
−∞

For example, consider a benefit of £1 payable immediately on exit by mode α to a life now aged
x. The m.p.v. of this benefit is
Z ∞
E[v T ] = v t .t p α α β
x µx+t .t px dt
0

Notice the similarities to the joint-life and contingent assurance functions in chapters 11 and 12.

16.3 Evaluation of Mean Present Values


This can either be done by integrals or sums.

(A) Integrals
The theory of ‘defective’ variables is used to value the m.p.v. of benefits on exit by a given mode of
decrement. We also consider the m.p.v. of premium payments.
(i) Consider a double-decrement table with exits by mode α and mode β, and suppose that there is
a benefit of £S payable immediately on the exit of (x) by mode α within n years.
The present value of this benefit, as a random variable, is
(
Sv T if T < n
Z=
0 if T > n.

where T = time to exit by mode α in the double-decrement table.


T is a defective variable with p.d.f.
α α β
t px .µx+t .t px (t > 0).

Hence the m.p.v. of this benefit is


Z n
S v t .t pα α β
x µx+t .t px dt (16.3.1)
0
16.3. EVALUATION OF MEAN PRESENT VALUES 269

(ii) Consider premiums of £P per annum, payable continuously for at most n years while (x) remains
a member of the group under consideration. The mean present value is
Z n Z n
(al)x+t
P vt · dt = P v t .t (ap)x dt
0 (al)x 0
Z n
=P v t .t pα β
x .t px dt (16.3.2)
0

(assuming 2 modes of decrement, α and β)

Example 16.3.1. Suppose there are 2 modes of decrement, death (d) and withdrawal (w), and
there is a constant force of withdrawal of k per annum.
Calculate the value of premiums of £P per annum payable continuously for at most n years while
(x) remains a member of the group.
Solution
µw
x+t = k for all t.
Hence t pw
x =e
−kt
for all t.

Z n
m.p.v. of premiums = P v t .t pdx .t pw
x dt
0
Z n
=P e−δt .t pdx .e−kt dt
0

(as v t = e−δt where δ is the force of interest)

Z n
=P e−(δ+k)t .t pdx dt
0
= P āx:n at force of interest δ 0 = δ + k

(B) Sums
Sums can be used to value death and other benefits and premium payments either exactly (if financial
transactions occur at the end of policy years) or by approximating integrals (16.3.1) and (16.3.2) if
benefits are payable immediately or premiums are paid continuously.
(i) Consider a benefit of £S payable at the end of the year of exit of (x) by mode α within n years.
The m.p.v. of this benefit is
· α α ¸
(ad)α
x 2 (ad)x+1 n (ad)x+n−1
S v +v + ··· + v .
(al)x (al)x (al)x
If the benefit is payable immediately, then exits can be assumed to occur, on average, mid-way
through year. The m.p.v. of the benefit is
· α α α ¸
1 (ad) 3 (ad)x+1 1 (ad)x+n−1
x
S v2 + v2 + · · · + v n− 2
(al)x (al)x (al)x

(ii) Consider an annual premium of £P per annum, payable in advance while (x) is still a member
of the group, for at most n years. The m.p.v. is
· ¸
(al)x+1 2 (al)x+2 n−1 (al)x+n−1
P 1+v +v + ··· + v
(al)x (al)x (al)x
270CHAPTER 16. FINANCIAL CALCULATIONS USING MULTIPLE-DECREMENT TABLES

If the premiums are payable continuously, expression (16.3.2) can be approximated to a sum.
The m.p.v. is now " #
1 (al)x+ 1 3 (al)x+ 3 1 (al)x+n− 1
P v2 2
+ v2 2
+ · · · + v n− 2 2

(al)x (al)x (al)x

Note
(al)x+ 12 , (al)x+ 23 , etc. must often be found by interpolation.

Example 16.3.2. A multiple-decrement table referring to mortality (d) and withdrawal (w) from
a life assurance contract is
x (al)x (ad)w
x (ad)dx

60 10,000 64 126
61 9,810 61 131
62 9,618 48 134
63 9,436
Suppose a 3-year term assurance is issued to a life aged 60, providing £20,000 at the end of the
year of death. If expenses consist of 5% of each premium, calculate the annual premium, P , payable
in advance while a policyholder, for at most 3 years. Interest is at 4% per annum.

Solution

20, 000
m.p.v. of benefits = [v.(ad)60 + v 2 (ad)61 + v 3 (ad)62 ]
(al)60
= 722.79

P
m.p.v. of premiums = [(al)60 + v(al)61 + v 2 (al)62 ]
(al)60
= 2.8325P.

Hence P solves
0.95 × (2.8325P ) = 722.79
Therefore P = £268.61 per annum.

Benefits other than cash sums


In some cases, the benefits on exit by a certain mode of decrement consists not of a cash sum but
an annuity or some other benefit. In such cases, treat the m.p.v. of the annuity (or other benefit)
at the date of exit as if it were paid out at that time. (The annuity will often have to be evaluated
by interpolation if it begins mid-way through a year of age.)

Example 16.3.3. A life office issues policies to lives aged under 60 providing the following benefits:
(i) on becoming permanently disabled before age 60, an annuity of £2,000 per annum payable weekly
for life and £20,000 immediately on death, and
(ii) immediately on death before age 60 while not permanently disabled, £20,000.

Calculate the office annual premium, payable weekly and ceasing on death, on permanent disability
or on reaching age 60, for a life aged 58 if the office uses the following basis:
16.3. EVALUATION OF MEAN PRESENT VALUES 271

Mortality: the independent rates of mortality of those not permanently disabled are those of
A1967-70 ultimate; the permanently disabled are subject to the mortality of English Life Table
No.12 - Males with the age rated up by 6 12 years;
Permanent disability: a constant independent rate of 0.006;
Interest: 4% per annum;
Expenses: 2 12 % of all office premiums, plus £50 at the issue date.
Solution

Construct service table with d = death and i = disability, assuming U.D. of D.


x qxd qxi (aq)dx (aq)ix (al)x (ad)dx (ad)ix

58 0.01169 0.006 0.01165 0.00596 100,000 1,165 596


59 0.01299 0.006 0.01295 0.00596 98,238 1,272 585
60 96,381
Value of benefits:
1(ad)i58 h i
(i) v 2 2, 000āih
58 1 + 20, 000Ā
ih
58 1
(al)58 2 2

i h i
3 (ad)
59
+ v2 2, 000āih
59 1 + 20, 000Ā
ih
59 1
(al)58 2 2

· d d
¸
1 (ad) 3 (ad)
58 59
(ii) 20, 000 v 2 +v 2 = 468.34
(ad)58 (al)58
For benefit (i)

āih
58 1 = ā65 on E.L.T. 12 - Males = 8.918
2

Āih
58 1 = Ā65 on E.L.T. 12 - Males = 0.65023
2

Similarly āih
59 1 = ā66 = 8.587
2

Āih
59 1 = Ā66 = 0.66323.
2

So value of (i) = 180.24 + 167.89 = 348.13.

Let P be the annual premium. The value of premiums less expenses is


" #
1 (al)58 1 3 (al)59 1
0.975P v 2 2
+ v2 2
− 50
(al)58 (al)58

1
(al)58 12 ' [(al)58 + (al)59 ] = 99119
2
(al)59 12 ' 97310

Hence value of premiums less expenses is 1.8422P − 50.


Therefore
1.8422P − 50 = 468.34 + 348.13
.
Consequently P = £470.35.
272CHAPTER 16. FINANCIAL CALCULATIONS USING MULTIPLE-DECREMENT TABLES

16.4 Benefits on Death by a Particular Cause


Sometimes a policy will provide death benefits if (x) dies from a particular cause. One may write
µαx+t = force of mortality from cause α, and
µβx+t = force of mortality from all other causes.

When these are added together to give (aµ)x+t , this will often be the force of mortality for a
given life table.

Example 16.4.1. A national newspaper recently advertised “free insurance for subscribers”, whereby
a benefit of £10,000 would be paid immediately on
accidental death (mode α) within n years. Assume µα x+t = 0.0005 for t ≥ 0 (for all x) and mortality
due to all causes follows A1967-70 ult.
Assuming a given rate of interest and ignoring expenses, calculate the annual premium payable
continuously by the newspaper to provide this “free” policy.
Solution
Let the annual premium be P .
Z n
m.p.v. of benefits = 10000 v t .t pα α β
x µx+t .t px dt
0
Z n
= 10000 × 0.0005 v t .t pA67−70ult
x dt
0
= 5āx:n on A67-70 ult.
m.p.v. of premiums = P ax:n

Thus P = £5 per annum (or £0.42 per month.)

Note:
Observe that P does not depend on the mortality table (for all causes), nor the rate of interest, nor
the term of n years. Also notice that £0.42 is a net premium, whereas an office premium (allowing
for expenses) would be larger.

16.5 Extra Risks Treated as an Additional Mode of Decre-


ment
Suppose that in addition to “normal” mortality (mode α), a group of lives are subject to certain
additional hazards (mode β). Mode β may refer to:
(a) Certain occupational hazards;
(b) Risks associated with a leisure activity, such as motor racing;
(c) Some medical conditions.

One may wish to calculate premiums for policies which pay out only on death from the “addi-
tional” cause, in which case the formulae in Section 16.3 may be used. If the death benefit is paid
on any cause of death, the problem can be treated as an “extra risk” question, as covered in the
Actuarial Subject A2.

Example 16.5.1. A certain life office’s premium basis for policies accepted at normal rates is:
16.5. EXTRA RISKS TREATED AS AN ADDITIONAL MODE OF DECREMENT 273

A1967-70 select,
4% interest,
expenses are ignored.

A proposer, aged 45, for temporary assurance ceasing at age 65 is subject to an extra occupational
hazard which is considered to be equivalent to an addition of 0.009569 to the force of mortality at
all ages. The sum assured, which is payable immediately on death, is £10,000.
(a) Calculate the level annual premium, payable throughout the term of the policy.
(b) The proposer requests that, in the event of death occurring as a result of the special occupa-
tional hazard, the sum assured should be doubled, and offers to pay an additional single premium
at the outset for this extra cover.
Calculate this single premium.

Solution
(a) Consider α = normal mortality, β = extra occupational mortality
Value of benefits is
Z 20
β β
10000 v t t pα α
[45] .t p[45] (µ[45]+t + µ[45]+t ) dt
0
Z 20
= 10, 000 v t e−kt .t p[45] (µ[45]+t + k) dt (where k = 0.009569)
0

= 10, 000Ā 1 where * indicates normal plus extra mortality
[45]:20
1
= 10, 000(1.04) 2 A∗ 1
[45]:20
1
= 10, 000(1.04) [(1 − d.ä∗[45]:20 ) − A∗
2
1 ]
[45]:20

Now use the rule (as in extra risks) that the rate of interest (in annuity and pure endowment
functions) may be altered to allow for the addition to the force of mortality.

ä∗[45]:20 is at force of interest δ 0 = δ + k


where δ = log(1.04) = 0.03922 (rate of interest = 4%)
0
So δ 0 = 0.048790 and hence, i0 = eδ − 1 = 0.05 = 5%.
Hence m.p.v. of benefits is
1
10, 000(1.04) 2 [1 − d.ä[45]:20 0.05 −A 1 ]
[45]:20 0.05
l65 20
= £2, 102.87 (using A 1 = v )
[45]:20 0.05 l[45] 0.05

2102.87
So annual premium =
ä∗[45]:20
2102.87
=
ä[45]:20 0.05
= £167.78

(b) The m.p.v. of this benefit is found by considering exit by mode β only. This gives a single
274CHAPTER 16. FINANCIAL CALCULATIONS USING MULTIPLE-DECREMENT TABLES

premium of
Z 20
β β
10, 000 v t .t pα
[45] .t p[45] .µ[45]+t dt
0
Z 20
= 10, 000 e−δt .e−kt .t p[45] .k dt (k = 0.009569)
0
= 10, 000kā∗[45]:20
= 10, 000kā[45]:20 0.05
· ¸
1 l65 20
' 95.69 ä[45]:20 0.05 − (1 − v0.05 )
2 l[45]
' 95.69 × 12.1899 = £1, 166.45

16.6 Calculations Involving a Change of State


Often there is interest in the probabilities of survival for a life who leaves a multiple-decrement table
and continues in another state. These problems may often be treated from first principles, rather
than by the construction of a multi-state model.

Example 16.6.1. Manual workers between the ages of 50 and 60 are subject to 2 modes of decre-
ment, d = death, and p = promotion to foreman.
Foremen are subject to the mortality of another table, T 0 , with functions lx0 , etc...
Find the probabilities that
(a) A manual worker aged 50 will be alive and a foreman at age 52.
(b) A manual worker aged 50 will be promoted to foreman but then die before age 52.

Solution
(a) The exact formula is
Z 2 µ 0

d p p l52
t p50 .t p50 µ50+t 0 dt
0 l50+t
(the expression in brackets giving survival as a foreman to age 52), which
approximates to

(ad)p50 l520
(ad)p51 l52
0
· 0 + 0
(al)50 l50 1 (al)50 l51 1
2 2

(b) The probability is approximately


" # " #
(ad)p50 0
l52 (ad)p51 0
l52
1− 0 + 1− 0
(al)50 l50 1 (ad)50 l51 1
2 2
16.7. EXERCISES 275

Exercises

16.1 The following is an extract from a multiple-decrement table referring to mortality and with-
drawal from certain life assurance contracts, these modes being referred to as ‘d’ and ‘w’
respectively.
age,
x (al)x (ad)w
x (ad)dx

50 15,490 24 51
51 15,415 21 58
52 15,336 15 60
(i) What is the probability that a policyholder aged 51 will withdraw before attaining age 53?
(ii) Suppose that a single-premium three-year term assurance contract is to be issued to a life
aged 50, subject to the mortality and withdrawal rates shown in the above table. The sum
assured is £50,000, payable immediately on death, and the benefit on withdrawal is zero. On
the basis of rate of interest of 6% p.a., and allowing for expenses of 10% of the single premium,
calculate the single premium for the above policy.
(iii) Suppose now that the office issuing the policy of (ii) above wishes to introduce surrender
values for these three-year contracts. The surrender value is to be equal to 0.5 per cent of
the single premium for each week between the date of withdrawal and the end of the term.
Assuming that surrenders in each policy year take place on average half-way through the year,
write down an equation of value from which you could calculate the revised single premium.
(Do NOT proceed to the evaluation of this premium.)
16.2 Employees of a certain company are given the opportunity of early retirement immediately
after they complete a 3-year overseas assignment.
Those who undertake this assignment effect a 3-year policy providing the following benefits
payable at the end of the year of claim:
(a) on death or ill-health retirement during the term, the sum of £6,000;
(b) on survival as an employee of the company to the end of the term, a lump sum;
(c) on withdrawal from the company during the second year an amount equal to 1 41 times the
annual premium, and on withdrawal from the company during the third year an amount equal
to 2 12 times the annual premium. No benefit is payable on withdrawal from the company in
the first year.
The following multiple-decrement table is applicable to employees going overseas at exact age
47.
(d = death, w = withdrawal, i = ill-health retirement)
x (al)x (ad)dx (ad)ix (ad)w
x

47 100,000 853 13,059 23,007


48 63,081 616 11,604 8,468
49 42,393 478 9,035 1,875
50 31,005
Using an interest rate of 4% p.a., calculate the lump sum for an employee going overseas at
exact age 47 who pays level annual premiums of £2,000 in advance. Ignore expenses.
16.3 An organization recruits new employees at exact age 20. The employees serve an apprentice-
ship during the first two years of employment. The central rate of withdrawal of apprentices
276CHAPTER 16. FINANCIAL CALCULATIONS USING MULTIPLE-DECREMENT TABLES

at ages 20 and 21 is 0.05 per annum, and withdrawals are uniformly spread over the year
of age (in the single-decrement table for withdrawals). At exact age 22, apprentices join the
permanent staff, who retire at exact age 60. The force of withdrawal of permanent staff is
0.0094787 per annum. Both apprentices and permanent staff experience mortality according
to A1967-70 ultimate. The employer pays an immediate benefit of £2,000 on withdrawal and
£8,000 on death in service to permanent staff. The death benefit (but not the withdrawal
benefit) is paid to apprentices.
(a) Construct a double-decrement table for apprentices between ages 20 and 22 with a radix
of 100,000 at age 20.
(b) Calculate, using an interest rate of 5% per annum, the value of the benefits in respect of
a new employee aged 20.
16.4 A life office sells policies to lives aged 63 which provide a benefit of £50 per week, ceasing at
age 65 or earlier death, to those becoming permanently disabled before age 65. The following
basis is used to calculate the single premium for this contract:
Independent rates of
mortality of policyholders
(not disabled): A1967-70 select (at entry)
Force of disablement: 0.01 p.a.
Mortality of disabled lives: A1967-70 ultimate, rated up by 7 years
Rate of interest: 4% p.a.
Expenses are ignored.
Weekly benefits may be taken as payable continuously.
Evaluate the single premium for this contract.
16.5 In a certain country, married men are subject to the following independent q-type rates of
decrement:
Mortality: A1967-70 ultimate
Widowhood: wives of married men are subject to A1967-70 ultimate
Divorce: 0.02 per annum at all ages
Widowed and divorced men are subject to the following independent q-type rates of decrement:
Mortality: English Life Table No. 12 - Males;
Remarriage: rates depend on the age at, and the duration since, the end of the former
marriage; the following table is an extract from these rates:
age at end duration duration
of former 0 1
Marriage year year

50 21 0.050 0.025
51 12 0.045 0.023
52 12 0.042 0.020
53 12 0.040 0.018
Calculate the probability that a married man aged 50, whose wife is also aged 50, will attain
age 52 as a widower who has not remarried between ages 50 and 52.
16.8. SOLUTIONS 277

Solutions

16.1 (i)
w (ad)w w
51 + (ad)52
2 (aq)51 =
(al)51
= 0.002335

(ii) Let single premium be P". Then P solves #


1 1 1
(ad)d50 v 2 + (ad)d51 v 1 2 + (ad)d52 v 2 2
0.9P = 50, 000 at 6% interest
(al)50
= 50, 000 × 0.009977
= 498.86

Hence P = £554.29.
(iii) Let P 0 be the premium allowing for surrender values. There are on average 52.18 weeks
in 1 year. So, the surrender value in year 1 is on average 52.18 × 0.005 × 2.5P 0 .
The average surrender value in year 2 is 52.18 × 0.005 × 1.5P 0 , and so on.
The equation of value is

0.9P 0 = value of death benefit (as above)


52.18 × 0.005 0 1 1 12 2 21
+ P [v 2 (ad)w
50 (2.5) + v (ad)w
51 (1.5) + v (ad)w
52 (0.5)]
(al)50

16.2 Let the lump sum be X.


· ¸
(al)48 (al)49 2
m.p.v. premiums = 2000 1 + v+ v
(al)47 (al)47
= 3996.99.

m.p.v. benefits:
(a)
6, 000
[v((ad)d47 + (ad)i47 ) + v 2 ((ad)d48 + (ad)i48 ) + v 3 ((ad)d49 + (ad)i49 )]
(al)47
= 1987.92

(al)50
(b) X.v 3 = 0.27563(X)
(al)47
2000 w 2
(c) (al)47 [1.25(ad)48 .v + 2.5(ad)w 3
49 .v ]
= 279.07
So X solves
3996.99 = 2266.99 + 0.27563(X)
Hence X = £6276.53

16.3 (a)
w mw 20 0.05 w
q20 ' = = 0.04878 = q21
1 + 12 mw20 1.025
278CHAPTER 16. FINANCIAL CALCULATIONS USING MULTIPLE-DECREMENT TABLES

Using (aq)dx = qxw (1 − 21 qxd ), the double-decrement table is,


x qxd qxw (aq)dx (aq)w
x (al)x (ad)dx (ad)w
x
20 0.000889 0.04878 0.000868 0.04876 100,000 87 4876
21 0.000841 0.04878 0.000821 0.04876 95,037 78 4634
22 90,325
(b) Consider value of benefits for employee attaining age 22. Let k = 0.0094787.
Value of death benefit is
Z 38
8000 v t e−kt .t pd22 µd22+t dt
0
= 8000Ā∗1 at force of interest δ 0 = δ + k = 0.058269
22:38
= 8000Ā 1
22:38 0.06
· ¸
1 l60 38
= 8000(1.06) A22:38 2
0.06 − v
l22 0.06
= 206.80

Value of withdrawal benefitZis


38
2000 v t e−kt .t pd22 .k dt
0
= 2000.kā22:38 ,0.06
· ¸
1 l60 38
= 2000 × 0.0094787 ä22:38 − (1 − v )
2 l22 0.06
= 285.64

Value of benefit in first 2 years, valued at age 20, is


8000 1 1
[v 2 (ad)d20 + v 1 2 (ad)d21 ]
(al)20
= 12.59

Hence value of benefits for new employee is


(al)22
12.59 + v 2 [206.80 + 285.64]
(al)20
= £416.03

16.4 Let d = death and i = disablement.


d d
Note that q63 = q[63] and q64 = q[63]+1 on A67-70 tables.
i i −0.01
Also, q63 = q64 = 1 − e = 0.00995
x qxd qxi (aq)dx (aq)ix (al)x (ad)dx (ad)ix
63 0.00839 0.00995 0.00835 0.00991 100,000 835 991
64 0.01245 0.00995 0.01239 0.00989 98,174 1216 971
65 95,987
50 × 52.18 h 1 1
i
m.p.v. benefits ' v 2 (ad)i63 .ā∗ 1 1 + v 1 2 (ad)i64 ā∗ 1 1
(al)63 63 2 :1 2 64 2 : 2

where
16.8. SOLUTIONS 279

ā∗ = ā70 1 :1 1 on A67-70 ultimate


63 21 :1 12 2 2

1
' [ā70:2 + ā71:1 ]
2
1
= [1.84904 + 0.96010] = 1.4046
2

ā∗ = ā71 1 : 1 on A67-70 ultimate


64 12 :1 12 2 2

1
' (0.96010) = 0.48005
2
Hence single premium = £47.08.
16.5 Denote
α = mortality of married men
β = mortality of wives
γ = divorce.
Use formula of the form
1 β 1 β γ
(aq)α α γ
x = qx [1 − (qx + qx ) + qx qx ]
2 3

x qxα qxβ qxγ (aq)βx


50 0.004789 0.004789 0.02 0.004730
51 0.005377 0.005377 0.02 0.005309
A multi-decrement table can be used, but short cuts can be taken.
α β γ
(ap)50 = (1 − q50 )(1 − q50 )(1 − q50 ) = 0.97064.

So, P r{ married man aged 50 becomes widower between ages 51 and 52}
= 1 |(aq)β50 = (ap)50 (aq)β51 = 0.005154

1
P r{ widower aged 50 attains age 52 without remarrying}
2
l52 1
= (1 − 0.050)(1 − × 0.025) (using E.L.T. 12-Males)
l50 12 2
= 0.92674. (i)

1
P r{ widower aged 51 attains age 52 without remarrying}
2
l52 1
= (1 − × 0.045) = 0.97322. (ii)
l51 12 2

Hence required probability is


(aq)β50 × (i) + (ap)50 (aq)β51 × (ii)
= 0.004730 × 0.92674 + 0.005154 × 0.97322
= 0.009399
280CHAPTER 16. FINANCIAL CALCULATIONS USING MULTIPLE-DECREMENT TABLES
Chapter 17

MULTIPLE-STATE MODELS

17.1 Two Points of View


There are essentially two approaches to the theory of multiple-decrement tables:
(a) Traditionalist - uses the underlying single-decrement tables;
(b) Modernist - uses continuous-time stochastic processes with a finite number of states.
Both view-points have advantages and disadvantages, and both produce the same practical re-
sults. In chapters 15 and 16, the traditional approach and notation was used. This chapter shall
briefly take a more modernist approach, and show the differences between the two methods.
A development of the modernist approach for multiple-state models is covered in Actuarial
Subject D2.

17.2 Kolmogorov’s Forward Equations


Suppose there are n possible states. For example, n = 3 in the following system, which refers to a
double-decrement table:
Define

pij (x, x + t) =P r{(x) will be in state j at age x + t


given that he is in state i at age x}

Figure 17.2.1: a double-decrement model

281
282 CHAPTER 17. MULTIPLE-STATE MODELS

Note that the initial conditions are


(
1 if i = j
pij (x, x) =
0 if i =
6 j.

The forces of transition, µij (y), are defined as

pij (y, y + h)
µij (y) = lim+ (i 6= j)
h→0 h
Also,

X
µi (y) = µij (y)
i6=j

= the force of transition from state i


to any other state at age y

Note
If all these forces are constant, this is a homogeneous chain. If the forces of transition vary with
age, it is called inhomogeneous.
Kolmogorov’s Forward Equations (for the general case in which there are n states) give the
following system for each fixed x:

d
pij (x, x + t) = −µj (x + t)pij (x, x + t)
dt X
+ µνj (x + t)piν (x, x + t) (1 ≤ i, j ≤ n, t ≥ 0) (17.2.1)
ν6=j

The three-state model of Figure 17.2.1 is such that

µ1 (y) = µ12 (y) + µ13 (y)


µ2 (y) = 0
µ3 (y) = 0 (as no one leaves states 2 or 3)

so Kolmogorov’s Forward Equations give

d
p12 (x, x + t) = p11 (x, x + t)µ12 (x + t) (17.2.2)
dt
and

d
p13 (x, x + t) = p11 (x, x + t)µ13 (x + t) (17.2.3)
dt
Note also that

p11 (x, x + t) = 1 − p12 (x, x + t) − p13 (x, x + t) (17.2.4)


and hence

d d d
p11 (x, x + t) = − p12 (x, x + t) − p13 (x, x + t)
dt dt dt
17.3. LIFE TABLES AS STOCHASTIC PROCESSES 283

Adding equations (17.2.2) and (17.2.3) gives


d
[p12 (x, x + t) + p13 (x, x + t)] = p11 (x, x + t)[µ12 (x + t) + µ13 (x + t)]
dt
= p11 (x, x + t)µ1 (x + t)

And hence, using equation (17.2.4),


d
p11 (x, x + t) = −p11 (x, x + t)µ1 (x + t) (17.2.5)
dt
(Equation (17.2.5) also follows directly from equation (17.2.1) with i = j = 1.)

This is a straightforward first-order differential equation. Using the initial condition p11 (x, x) = 1,
(17.2.5) can be solved to give the unique solution
Z t
p11 (x, x + t) = exp[− µ1 (x + r) dr] (17.2.6)
0

This corresponds exactly to a result derived in chapter 15, i.e.


Z t
t (ap)x = exp[− (aµ)x+r dr]
0
Z t
β
= exp[− (µα
x+r + µx+r ) dr]
0
β
where µα
x+r corresponds to µ12 (x + r), and µx+r corresponds to µ13 (x + r) .

Returning to the differential equation (17.2.2) and solving with the initial condition p12 (x, x) = 0
gives Z t
p12 (x, x + t) = p11 (x, x + r)µ12 (x + r) dr
0
Again this corresponds to a “traditional” formula in chapter 15, i.e.
Z t
α α
t (aq)x = r (ap)x µx+r dr
0

17.3 Life Tables as Stochastic Processes


Life tables just contain one mode of decrement (death) and hence the continuous-time model is
Note that

µ12 (y) = the force of mortality at age y


= µy in “traditional” notation

Proceeding as in section 17.2, but with only one mode of decrement, we find that Kolmogorov’s
Forward Equations may be solved to give
· Z t ¸
p11 (x, x + t) = exp − µ12 (x + r) dr
0

In “traditional” notation, this is just the familiar result that


· Z t ¸
t px = exp − µx+r dr
0
284 CHAPTER 17. MULTIPLE-STATE MODELS

Figure 17.3.1: the life table model

Remark
In the stochastic processes approach, it is assumed that the Chapman–Kolmogorov equations hold.
That is,
Xn
pij (x, z) = pik (x, y)pkj (y, z) (17.3.1)
k=1

for all x ≤ y ≤ z, and for the n states in the continuous-time model.


In Figure 17.3.1 there are only 2 states (alive and dead), and clearly

p21 (x1 , x2 ) = 0 for all x1 ≤ x2

and
p22 (x1 , x2 ) = 1 for all x1 ≤ x2
Equation (17.3.1) with i = 1 and j = 1 is just a re-statement of the following axiom of the
traditional life table:
z−x px =y−x px .z−y py

17.4 Sickness Models


The Continuous Mortality Investigation Bureau (C.M.I.B.) has used the following multiple-state
model for sickness rates:
Define

σy = the force of sickness at age y,


py,z = the force of recovery at age y and duration of sickness z,
µy = the force of mortality of healthy lives at age y,
νy,z = the force of mortality of sick lives at age y and duration of sickness z

The various forces of transition have been graduated using Permanent Health Insurance data.
Suppose one wishes to calculate probabilities such as

p12 (x, x + t) = P r{ a healthy life aged x will be sick at time t}.

The existence of the parameter z complicates the multiple-state model: Kolmogorov’s Forward
Equations have to be modified, as the forces of recovery and mortality of sick lives depend on
17.4. SICKNESS MODELS 285

Figure 17.4.1: a sickness model

duration of sickness as well as age. This leads to “semi-Markov” processes which are more difficult
to handle than the cases so far discussed.
A further complication is caused by the need (in practical applications) to consider “deferred
periods”. For example, one may wish to calculate

P r{ a healthy life aged x will be sick at time t and has been sick for
at least d weeks (d is the deferred period)}

A possible simplification
One possible way to simplify this model is to initially ignore mortality and to consider only forward
transitions between “healthy” and “sick” states. Define T1 , T2 , T3 , ... to be the times (in years) spent
as
1. a healthy life (who has not been sick since age x),
2. a sick life (who has been sick exactly once),
3. a healthy life (who has been sick exactly once),
and so on.
A pictorial representation of this is as follows:
In this set-up, one can work out
P r{ aXhealthy life aged x will be sick at age x + t}
= P r{ a life in state 1 at age x will be in state j at time t}
j=2,4,6,...

Each term may be evaluated, using the joint density function f (t1 , t2 , ..., tj ) of the variable
(T1 , T2 , ..., Tj ). That is,

P r{ a life aged x in state 1 will be in state j at time t}


=P r{T1 + ... + Tj−1 < t, but T1 + ... + Tj > t}
= a multiple integral (we omit the details)

Advantages of this method:


1. The mathematics is not so difficult to follow.
2. The deferred period may be easily handled by modifying the range of the multiple integrals.

Disadvantages:
1. There can be difficulties in evaluating the multiple integrals for large values of j.
286 CHAPTER 17. MULTIPLE-STATE MODELS

Figure 17.4.2: sick and healthy states

In practice, the Manchester Unity System is often used to calculate sickness functions; this will
be covered in Chapter 18.
Chapter 18

SICKNESS FUNCTIONS

18.1 Rates of Sickness


The force of sickness, z̄x , at age x is the probability that a life aged exactly x is “sick” (according
to the rules of the scheme).
In practice, sickness benefit must be claimed for short periods (e.g. days or weeks) during which
time the life is either “sick” or “not sick”.
Consider these periods to be of length h years.
Define

zx(h) = P r{(x) is entitled to sickness benefit for the time period until age x + h}

It is supposed that, uniformly on any bounded age interval,

lim zx(h) = z̄x


h→0+

If benefit is paid at the rate of £1 per week, this amounts to £52.18 per year on average. Suppose
that benefit is payable in advance over intervals of length h years, where h = n1 , i.e. each year is
divided into n intervals. The expected cash to be paid in sickness benefit between ages x and x + 1
for a life now aged x is
n−1
X (h)
jh px (52.18h)zx+jh
j=0

Letting n → ∞ (or h → 0+ ) gives

sx = expected cash paid out in sickness benefit


Z 1
= 52.18 t px z̄x+t dt (18.1.1)
0

sx is called the annual rate of sickness at age x: it is the expected number of weeks of sickness
between ages x and x + 1 for a life now aged x.
We also define

zx = the central rate of sickness at age x


R1
52.18 0 t px z̄x+t dt
= R1 (18.1.2)
p dt
0 t x

287
288 CHAPTER 18. SICKNESS FUNCTIONS

Equations (18.1.1) and (18.1.2) give the approximations:

sx ' 52.18. 12 px z̄x+ 21 (18.1.3)


zx ' 52.18z̄x+ 12 (18.1.4)

and hence
sx ' 12 p zx (18.1.5)
x
“Formulae and Tables for Actuarial Examinations” makes use of the Manchester Unity Expe-
rience 1893-97, Occupational Groups AHJ. Fuller details of this experience can be found in the
Appendix to this chapter.

Sickness by Duration
Benefits may depend on the duration of sickness. Define

z̄xm/n = P r{(x) is “sick” and has been sick for more than m weeks
but not more than m + n weeks.}
m/n m/n
Similar modifications are used to define zx and sx . For example,

z̄x13/13 = P r{(x) is “sick” with duration of sickness more


than 13 weeks but less than 26 weeks}.

Note that

z̄x13 = z̄x0/13 = P r{(x) is “sick” and has been sick for less than 13 weeks}

Also,

z̄x104/all = P r{(x) is sick with duration of sickness greater than 104 weeks, or 2 years}.

The Tables give the following values:


13/13 26/26 52/52 104/all
zx13 , zx , zx , zx , zx

Furthermore,
13/13 26/26 52/52 104/all
zx = zxall = zx13 + zx + zx + zx + zx

Some More Definitions


(a) The Deferred Period
This is the time between falling sick and being able to claim sickness benefits.
In Permanent Health Insurance, one encounters the notation D1, D4, etc. which refers to the length
of the deferred period in weeks. For example, in a D1 policy, a member must be sick for 1 week
before being allowed to receive sickness benefit.
(b) The Waiting Period
This is the time between joining a friendly society or sickness benefit scheme and being able to
claim sickness benefits. This time-period is often 6 months or a year.
(c) The Off-Period
This is the minimum time that must elapse between 2 bouts of sickness in order for them not to be
considered as the same bout of sickness for benefit calculations.
18.2. VALUING SICKNESS BENEFITS 289

If the sickness benefit falls with duration of sickness, there may be a temptation for people to
temporarily “recover” and then to start claiming benefit again at the higher initial rate. To prevent
this, an off-period is specified so that if 2 spells of sickness are separated by less than the off-period,
the later spell is treated, for benefit purposes, as a continuation of the first spell. In the Manchester
Unity experience, the off-period is 1 year. If benefits do not fall as duration of sickness increases
there is no need for an off-period rule.

18.2 Valuing Sickness Benefits


Consider a life aged x, subject to a certain mortality table, and suppose that sickness benefit will
be payable at the rate of £1 (per week) during all sickness within the next T years. Consider the
age-range from x to x + T to be split into nT short intervals, each of length h years, and suppose
that sickness benefit is payable in advance on sickness during any of these short-age intervals. The
mean present value of the sickness benefit is thus
nT
X −1
(h)
v jh jh px (52.18h)zx+jh
j=0

Letting n → ∞ gives
Z T
m.p.v. of sickness benefit = 52.18 v t .t px z̄x+t dt (18.2.1)
0

This can be approximated by a sum, i.e.

T
X −1
1
m.p.v. of sickness benefit ' v t+ 2 .t+ 12 p (52.18z̄x+t+ 12 )
x
t=0
T
X −1
1
= v t+ 2 .t+ 12 px zx+t (18.2.2)
t=0

Notes
1. If sickness benefits continue throughout life (although this is unusual as it is not easy to define
“sickness” among the very old), we may let T → ∞, giving
Z ∞
m.p.v. of sickness benefit = 52.18 v t .t px z̄x+t dt (18.2.3)
0

X 1
' v t+ 2 .t+ 12 px zx+t (18.2.4)
t=0

2. If the sickness benefit is payable for sickness of duration greater than m weeks but less than m + n
m/n m/n
weeks, replace z̄x and zx by z̄x and zx , respectively.
The M.P.V. of a sickness benefit may be evaluated by direct evaluation of a sum, by approximate
integration or by commutation functions.

Commutation functions
Define
Z 1
Hx = 52.18 v x+t lx+t z̄x+t dt (18.2.5)
0
290 CHAPTER 18. SICKNESS FUNCTIONS

This can be approximated to give


1
Hx ' 52.18v x+ 2 lx+ 21 z̄x+ 12 (18.2.6)
and so

Hx ' Dx+ 12 zx (18.2.7)


where Dx = lx v x , as usual.
We also define

X
Kx = Hx+t (18.2.8)
t=0

so that the m.p.v. of a sickness benefit of £1 per week for life to (x) is approximately


X Dx+t+ 1 2
zx+t (from (18.2.4))
t=0
Dx
P∞
t=0 Hx+t
=
Dx
Kx
= (18.2.9)
Dx

Note
13/13 26/26 52/52 104/all
Kx = Kxall = Kx13 + Kx + Kx + Kx + Kx , where Hx13 and Kx13 , etc., are defined by
13
replacing zx by zx , etc., in the formulae for Hx and Kx .

Commutation functions are available in “Formulae and Tables” on the following basis only:

• English Life Tables, No.12 - Males

• 4% interest

• Manchester Unity 1893-97 (A, H, J)

Temporary and Deferred benefits


If sickness benefit ceases at a certain age, say 65, then the m.p.v. of sickness benefit of £1 per week
to (x) is
Kx − K65
(18.2.10)
Dx
If benefits are deferred for n years, we obtain

Kx+n
Dx

rather than

Kx
Dx
This is used in connection with waiting periods. If there is a waiting period of 6 months for sickness
benefits, replace Kx by Kx+ 12 (interpolate in the Tables.)
18.2. VALUING SICKNESS BENEFITS 291

Example 18.2.1. A life office is proposing to issue 3-year sickness benefit policies to lives aged 30.
The benefits are £50 per week during sickness within the next three years. There is no waiting
period and the off-period is as in the Tables provided. Find the single premium on each of the
following bases:
mortality: English Life Table No.12 - Males
interest: (i) 4% p.a.,
(ii) 5% p.a.
sickness: Manchester Unity 1893-97 (AHJ)
expenses: none.
Solution
(i)
K30 − K33
m.p.v. of benefits = 50 ·
D30
· ¸
1784760 − 1706624
= 50
29372
= £133.01

(ii) Calculate value of benefits from first principles. The value is


50 h 12 1 12 2 12
i
v0.05 l30 12 z30 + v0.05 l33 12 z31 + v0.05 l32 21 z32
l30
The value of l30 12 , l31 12 , l32 21 can be found by interpolation using English Life Tables, No.12 -
Males, to give
50
m.p.v. of benefits = × 249682
95265
= £131.05

If sickness benefit is payable for sickness lasting more than m weeks but less than m + n weeks,
m/n m/n
replace Kx and Hx by Kx and Hx respectively.
For example,
13/all 13/all
Kx − K65
= m.p.v. of a sickness benefit of £1 per week
Dx
payable up to age 65 on sickness lasting more than 13 weeks
13/all
Since Kx is not given directly in the Tables, one must use
13/all 13/13 26/26 52/52 104/all
Kx = Kx + Kx + Kx + Kx

Example 18.2.2. A friendly society issued a policy providing the following benefits to a man aged
exactly 25 at entry:
(a) on death at any time before age 60, the sum of £4,000 payable immediately;
(b) on survival to age 60, an annuity of £8 per week payable weekly in advance for as long as he
survives;
(c) on sickness, an income benefit to be payable during sickness of £32 per week for the first 6
months reducing to £16 per week for the next 18 months and to £8 per week thereafter. Sickness
292 CHAPTER 18. SICKNESS FUNCTIONS

benefit is not payable after age 60. There is no waiting period.


Premiums are payable monthly in advance for at most 35 years, and are not waived during periods
of sickness.

The society uses the following basis to calculate premiums. Find the monthly premium.

mortality: English Life Table No.12 - Males


sickness: Manchester Unity Sickness Experience 1893-97, Occupation Group AHJ
interest: 4% p.a.
expenses: none
Solution
Value of benefits: ³ ´
(a) 4000Ā 1 = 4000 M 25D−M
25
60
= 268.05
25:35
(b) 52.18 × 8 · N 60
= 926.65
D
³ 26 26 ´25 µ ¶ µ ¶
26/78 26/78 104/all 104/all
K25 −K60 K25 −K60 K25 −K60
(c) 32 D25 + 16 D25 +8 D25
= 660.92
Let P = annual premium payable monthly. Then the equation of value is
(12)
P ä25:35 = 268.05 + 926.65 + 660.92
= 1855.62

(12) (12) D60 (12)


ä25:35 = ä25 − ä
D25 60
µ ¶
1 D60 1
= (ā25 + ) − ā60 + = 18.495.
24 D25 24
Hence P = £100.33 and monthly premium = £8.36.

18.3 Various Other Points


(a) A possible adjustment
Consider a policy providing a sickness benefit of £10 per week on sickness lasting more than 2 years
for an entrant aged x, with all benefits ceasing at age 65. One might want to adjust the formula
" #
104/all 104/all
Kx − K65
10 (18.3.1)
Dx

to
" 104/all 104/all
#
Kx+2 − K65
10 (18.3.2)
Dx
since the benefit cannot be received in the first 2 years.

This point is discussed in the Appendix to this Chapter, and it is concluded that neither formula is
exactly right, so the use of adjusted version (18.3.2) is usually optional.
The only exception (in which (18.3.2) is accurate and (18.3.1) is not) occurs when the term of
the policy is very short. For example when x = 63, formula (18.3.2) gives zero, which is correct.
18.3. VARIOUS OTHER POINTS 293

(b) Waiver of Premium Benefits


Suppose that a policy has a weekly premium of £P but this is waived during sickness (or during
sickness of a certain duration).
This is handled by assuming that all premiums are paid by the policyholder, but there is an
additional sickness benefit of £P per week.
Also, as a general rule, expenses will apply even when premiums are waived. If this is not the
case, one should adjust the equation of value appropriately.

Example 18.3.1. A friendly society issues sickness insurance policies which provide income during
periods of sickness as follows:
(a) £100 per week while a sickness has duration in excess of 13 weeks but less than 1 year;
(b) £75 per week while a sickness has duration in excess of 1 year but less than 2 years;
(c) £50 per week while a sickness has duration in excess of 2 years.

All benefits cease at age 65. Premiums are payable weekly until age 65 and are waived when
sickness benefit is being paid. There is no waiting period.
Calculate the weekly premium for a life aged 38 at entry. Basis:
mortality: E.L.T. No.12 (Males)
sickness: Manchester Unity Sickness Experience 1893-97, Occupation Group
AHJ.
interest: 4% p.a.
expenses: 50% of all premiums payable in the first year, plus 10% of all
premiums payable after the first year.
Solution
Let the weekly premium be P .
m.p.v. of (premiums-expenses) is
0.9 × 52.18P ā38:27 − 0.4 × 52.18P a38:1
= 713.98P (i)
m.p.v. of benefits and premiums waived is
13/39 13/39 52/52 52/52 104/all 104/all
K38 − K65 K38 − K65 K38 − K65
100[ ] + 75[ ] + 50[ ]
D38 D38 D38
13/all 13/all
K38 − K65
+ P[ ]
D38
= 784.80 + 267.63 + 543.00 + P × 22.276
= 1595.43 + 22.276P (ii)
Setting (i) = (ii) gives
P (713.98 − 22.276) = 1595.43
Hence P = £2.31

(c) Reserves
These are calculated prospectively or retrospectively (usually the former) in a way similar to that
used for life policies.
If the premium and reserve bases agree, the prospective and retrospective reserves are equal. If
the bases differ, one must specify how to find the reserve.
294 CHAPTER 18. SICKNESS FUNCTIONS

Example 18.3.2. Consider a policy issued to (x) providing a sickness benefit of £10 per week
ceasing at age 65.
(a) Give a formula for the weekly premium, P , ceasing at age 65 (ignoring expenses).
(b) Calculate the reserve at duration t years by
(i) the prospective method, and
(ii) the retrospective method,
on the premium basis.
Solution h i
(a) 52.18P āx:65−x = 10 KxD −K65
h i x
Kx+t −K65
(b) (i) t V = 10 − 52.18P āx+t:65−x−t
h Dx+t ³ ´i
(ii) t V = DDx+t
x
52.18P āx:t − 10 Kx −KDx
x+t

Note that (i) and (ii) are equal, by an argument similar to that used for life policies.

(d) Understanding the Tables for Actuarial Exams


1. Pages 82-83 give
zx13 , zx13/13 , zx26/26 , zx52/52 , zx104/all , zx (= zxall )
2. Pages 84-85 give
13/13 26/26 52/52 104/all
Kx13 Kx Kx Kx Kx Kx
, , , , ,
Dx Dx Dx Dx Dx Dx
3. Pages 86-87 give

Dx , Kx13 , Kx13/13 , Kx26/26 , Kx52/52 , Kx104/all


18.4. EXERCISES 295

Exercises

18.1 (i) In a combined sickness and mortality table

Kx+1 = 554, 405


zx = 1.129
Dx = 24, 510
Dx+1 = 23, 425

Estimate Kx .
(ii) An office offers an optional waiver of premium benefit on sickness of any duration in respect
of a 25-year with or without profits endowment assurance policy with weekly premiums payable
for 25 years or until earlier death. There is a waiting period of 12 months for the waiver of
premium benefit, and, during the second year of the policy, only half the premium (including
the extra premium for the waiver benefit) is waived. The sum assured under the endowment
policy is payable immediately on death, or on survival until the end of the term.
Using the basis given below, calculate the percentage by which the normal weekly premium
(i.e. the premium for a policy without the waiver benefit) for a life aged exactly 30 at entry
should be increased in order to provide the waiver benefit.

mortality: English Life Table No.12-Males;


sickness: Manchester Unity 1893-97, Occupation Group AHJ;
interest: 4% per annum;
expenses: 15% of each extra premium for the waiver benefit.
18.2 A policy issued by a life office to a male life aged exactly 35 is subject to level weekly premiums
ceasing at exact age 65. If the man has been sick for 6 months or more when a premium falls
due, the premium is waived. The policy provides the following benefits:
(a) on survival to exact age 65, an annuity of £5,000 per annum payable monthly in advance,
(b) on death before age 65, a return of all premiums paid (including those waived during
sickness) together with compound interest at 4% per annum to the date of death.
There is no waiting period and the off periods are the same as those underlying the tables in
Formulae and Tables for Actuarial Examinations. Calculate the weekly premium.
Basis: English Life Table No.12-Males, Manchester Unity Sickness Experience 1893-97, Oc-
cupation Group AHJ, interest 4% per annum, no expenses.
18.3 (a) Ten years ago, a man then aged exactly 30 effected an insurance policy providing sickness
benefits of £100 per week for the first six months of sickness, £50 per week for the remainder
of the first year and £30 per week thereafter, with benefit ceasing at age 60. Calculate the
weekly premium payable to age 50 on the following basis:
Mortality: English Life Table No. 12 - Males;
Sickness: Manchester Unity 1893-97, Occupation Group AHJ;
Interest: 4% per annum;
Expenses: 10% of each premium.
(b) The man now wishes to alter his policy so that premiums will in future be waived during
all periods of sickness. Calculate the revised premium payable assuming that the alteration
basis follows the premium basis above.
Note. Expenses are incurred even when premiums are waived.
296 CHAPTER 18. SICKNESS FUNCTIONS

18.4 A certain friendly society recruits only married men aged under 55. The society provides the
following benefits:
(i) immediately on the death of the member at any age, £2,000,
(ii) during the first 10 years of membership, immediately on the death of the member’s wife
before her husband, £500,
(iii) on survival of the member to age 65, an annuity of £10 per week for life, and
(iv) during any spell of sickness of the member before age 65, £10 per week reducing to £6
per week after 3 months’ sickness.
The basis for all calculations is:
mortality of members: E.L.T. No.12 - Males
mortality of wives: E.L.T. No. 12 - Males with an age-deduction of 5 years
interest: 4% per annum
sickness: Manchester Unity 1893-97 AHJ
expenses: 5% of all contributions, including those waived during sickness
Wives of members are taken as being of the same age as their husbands; there is no benefit
on the death of wives of marriages taking place after entry to the society, and the possibility
of divorce is to be ignored. There is no waiting period and the off-period is as in the Tables
provided. The possibility of withdrawal is ignored.
(a) Calculate the weekly contribution rate, waived during sickness and ceasing at age 65 or
the previous death of the member, for an entrant aged 25.
(b) Calculate the reserve for a member aged 35 who joined at age 25.
Note Use Simpson’s rule for approximate integration, i.e.
Z b
b−a a+b
f (t) dt ' [f (a) + 4f ( ) + f (b)]
a 6 2

18.5 A Friendly Society issues policies providing the following benefits:


(i) A sickness benefit of £25 per week for the first 13 weeks of sickness and £12.50 per week
thereafter, benefit ceasing at age 60. Contributions are waived during sickness.
(ii) On death before age 60, a lump sum of £1,000 plus a return of contributions (including
any waived) without interest.
(iii) On survival to age 60, a lump sum of £2,000.
Contributions are payable by level weekly amounts until age 60. There is a six-month waiting
period for the sickness benefit (including the premium waiver) and the off-period may be
assumed to be the same as that underlying the tables in “Formulae and Tables for Actuarial
Examinations”.
(a) Calculate the weekly contribution payable by a new member aged 35 on the basis given
below.
(b) Calculate the reserve (on the basis given below) to be held for this member five years after
he joins the Society.

Basis: English Life Table No. 12 - Males


Manchester Unity Sickness Experience 1893-97, Occupation Group AHJ
interest 4% per annum.
expenses are 5% of all premiums (including those waived).
18.4. EXERCISES 297

18.6 A friendly society provides the following benefits:


(i) On sickness, £40 per week for the first 26 weeks and £50 per week for the next 26 weeks.
No sickness benefit is payable after age 65.
(ii) On attaining age 65 or immediately on earlier death, the sum of £3,000.
Members contribute £1 per week, ceasing at age 65 or earlier death, but this is waived during
periods of sickness (whether benefit is payable or not.) Calculate the reserve which should be
held for a member aged 50, using the following basis:
English Life Table No.12 - Males,
4% per annum interest,
Manchester Unity Sickness Experience 1893-97 (AHJ).
There is no waiting period, and off-periods are as in “Formulae and Tables for Actuarial
Examinations”.
18.7 On 1 January 1995 a friendly society had in force a large group of policies providing certain
sickness and life assurance benefits. These policies had all been issued 10 years ago to lives
now aged 45. The benefits cease when the policyholders reach age 65, and the policies have
level weekly premiums payable throughout the term of the policy. Each policy provides an
income of £20 per week during any period of sickness which has lasted more than 6 months,
and a lump sum of £20,000 immediately on death before age 65. Premiums are waived during
sickness (even when no benefit is payable), and the off-period is as in the Manchester Unity
sickness tables. The basis for the calculation of premiums and reserves is:
Mortality: E.L.T. 12 - Males
Interest: 4% p.a.
Sickness: Manchester Unity Experience 1893-97 (Occupation Group AHJ)
Expenses: 20% of all premiums, including those waived
(a) Calculate the weekly premium for each of these policies.
(b) Calculate the reserve per policy in force on 1 January 1995.
298 CHAPTER 18. SICKNESS FUNCTIONS

Solutions

18.1 (i)
Kx = Hx + Kx+1
' Dx+ 12 .zx + Kx+1
1
' (Dx + Dx+1 )zx + Kx+1
2
= 581, 464

(ii) Let weekly premium for the basic policy be P , and let k.P be the extra premium for the
waiver benefit.
The equation of value is ·1 ¸
2 (K31 − K32 ) + (K32 − K55 )
52.18 × 0.85kP ā30:25 = P (1 + k)
D30

(note the waiting period of 1 year).


Hence 690.07k = (1 + k)20.81
Therefore k = 0.0311 = 3.11%.

Note
One does not need to calculate the actual premium P in this example.
18.2 Let weekly premium be P .
The equation of value is Z 30
D65 (12)
52.18P.ā35:30 = 5000 ä65 + v t .t p35 µ35+t (52.18P s̄t̄| ) dt
D35 0
h i
26/all 26/all
+ P K35 − K65 /Dx (i)

Z 30
Note that v t .t p35 µ35+t (52.18P s̄t̄| ) dt
0
Z 30 µ ¶
1 − vt
= 52.18P t p35 µ35+t dt
0 δ
52.18P h i
= 30 q35 − Ā 1
δ 35:30

Hence (i) gives


869.13P = 9, 994.55 + 203.26P + 16.8815P
Hence P = £15.40.
18.3 (a) Let weekly premium be P . The equation
µ 26of value ¶is
K30 − K6026 ³ ´
26/26 26/26
52.18 × 0.9P ā30:20 = 100 + 50 K30 − K60
D30
à !
52/all 52/all
K30 − K60
+ 30
D30

Hence 640.51P = 1711.35 + 125.21 + 242.74


and so
P = £3.25 per week.
18.5. SOLUTIONS 299

(b) Let extra premium be E per week.


µ The equation
¶ of value for E is
K40 − K50
(E + 3.25) = 52.18 × 0.9Eā40:10
D40

Therefore (E + 3.25) × 13.077 = 382.67E


and so
E = 0.115, say £0.12.
Hence revised premium is £3.37 per week.
18.4 (a) Let weekly contribution be P .
m.p.v. benefits:
(i) 2000Ā25 = 377.64
(ii)
Z 10
500 v t .t p20 µ20+t .t p25 dt ' 500 × 0.008638
0
= 4.319 (using Simpson’s Rule)

(iii) 10 × 52.18 · D
D25 ā65 = 693.28
65

³ ´ ³ 13 13 ´
K25 −K65
(iv) (6 + P ) K25D−K25
65
+ 4 D25 = 254.23 + 31.853P
Hence the equation of value is
52.18 × 0.95P ā25:40 = 377.64 + 4.319 + 693.28 + 254.23 + 31.853P

Hence 927.54P = 1329.46


and so, P = £1.43.
(b) µ ¶
N̄65 K35 − K65
10 V = 2000Ā35 + 52.18 × 10 + (6 + 1.43)
D35 D35
µ 13 13

K35 − K65
+4 − 0.95 × 1.43 × 52.18ā35:30
D35
= 537.38 + 1038.18 + 270.88 + 62.55 − 1180.71
= £728.28

The reserve may also be calculated by the retrospective method.


18.5 (a) Let the weekly contribution be P . The equation of value is
0.95 × 52.18P ā35:25 = 1, 000Ā 1 + 52.18P (I¯Ā) 1
35:25 35:25
µ ¶
D60 K35 12 − K60
+ 2000 + (12.5 + P )
D35 D
à 13 13 !
K35 1 − K60
+ 12.5 2
(i)
D35

Use,
1
(I¯Ā) 1 ' (I Ā) 1 − Ā 1
35:25 35:25 2 35:25
13
and find K 35 21 and K35 1 by linear interpolation.
2
300 CHAPTER 18. SICKNESS FUNCTIONS

Equation (i) gives


759.52P = 86.28 + 71.38P + 625.57 + (12.5 + P )28.066 + 165.92

and so
P = £1.86 per week

(b)
5V = (1000 + 5 × 52.18P )Ā1 + (52.18 × 1.86)(I¯Ā)1
40:20 40:20
µ ¶ µ 13 13

D60 K40 − K60 K40 − K60
+ 2000 + (12.5 + P ) + 12.5
D40 D40 D40
− 0.95 × 52.18P ā40:20 (Prospectively)

Using P = 1.86, this gives


5 V = 142.35 + 113.99 + 768.11 + 413.06 + 59.53 − 1222.68
= £374.36

18.6 Using the prospective method, Ã ! µ


µ 26 26
¶ 26/26 26/26 ¶
K50 − K65 K50 − K65 K50 − K65
Reserve = 40 + 50 +
D50 D50 D50
· ¸
D65
+ 3000 Ā1 + − 52.18ā50:15
50:15 D50
= 681.85 + 209.36 + 39.08 + 1775.84 − 542.88
= £2163.25

18.7 (a) Let the weekly premium be P . The equation of valueÃis !


26/all 26/all
K35 − K65
0.8 × 52.18P ā35:30 = 20000Ā1 + 20
35:30 D35
µ ¶
K35 − K65
+P
D35

This gives 695.30P = 2472.44 + 337.63 + 36.46P


Hence P = £4.27 per week.
(b) Using the prospective method à !
26/all 26/all µ ¶
K45 − K65 K45 − K65
10 V = 20000Ā1 + 20 +P
45:20 D45 D45
− 0.8 × 52.18P ā45:20

With P = 4.27, this gives


10 V = 3176.82 + 416.27 + 168.42 − 2285.91
= £1475.60
18.5. SOLUTIONS 301

Appendix: The Manchester Unity Experience 1893-97


1. Until fairly recently, when the C.M.I. published statistics concerning P.H.I. (Permanent Health
Insurance), the principal published set of sickness rates were those derived from the experience
in 1893-97 of the Independent Order of Oddfellows, Manchester Unity (a large “affiliated
order” friendly society, i.e. an association of lodges with some measure of central control.)
The members were all male.

2. Friendly societies existed largely to protect wage-earners in times of adversity, there being
no State benefits before 1908 (unless one counts the Workmen’s Compensation Acts of 1897,
which covered injuries at work.) There was (and to some extent still is) a wide variety of
societies ranging in size and financial stability, and not all societies provided sickness benefits.
From 1911 to 1946, sickness benefits for wage-earners were provided by Approved Societies
(friendly societies or life offices approved under the National Insurance Act, 1911). A part
of their income was provided by a Government grant, and they had to use a valuation basis
specified by the Government Actuary. Societies with favourable experience were able to pro-
vide additional (in practice, usually dental and ophthalmic) benefits. Since 1946 the State has
provided welfare benefits (including sickness benefits) directly, either in return for national
insurance contributions or under a means test. The friendly societies have generally declined
in importance; many have disappeared, and others concentrate on social activities. A certain
number (particularly of centralized societies) continue, and indeed prosper, in modern con-
ditions: under the Friendly Societies Act 1992, they are able to conduct a greater variety of
business.

3. The Manchester Unity 1893-97 investigation was carried out by Alfred W. Watson, and pub-
lished in 1903. A calendar year system was used, lives being classified according to age nearest
birthday at the start of the calendar year. The unadjusted sickness rates are of the form

no. of weeks of sickness at age x last birthday


ẑx =
Exc

These rates were graduated by an adjusted-average formula to produce the published rates, zx .
(The statistical basis of the graduation of sickness rates is more complex than for mortality
rates, and we do not attempt a discussion.) The rates were also subdivided by periods of
13/13 26/26 52/52 104/all
sickness, giving zx13 , zx , zx , zx , zx . The off-period assumed in the investigation
was 1 year, this being the actual off-period for most of the lodges.

4. Members were also subdivided according to the following occupational groups:


A: Agricultural Workers
B: Outdoor Tradesmen and Labourers
C: Railwaymen
D: Seamen and Fishermen
E: Quarry Workers
F: Iron and Steel Workers
G: Miners
H: Rural workers not included in A-G
J: Urban workers not included in A-G
Sickness rates were calculated for the four groups AHJ, BCD, EF, G, it being found that
sickness rates were lowest in the first group and increased as one moved from one group to the
302 CHAPTER 18. SICKNESS FUNCTIONS

next. The experience of occupation group AHJ is given in Tables for Actuarial Examinations.
Since the middle classes and well-to-do did not (in general) join friendly societies providing
sickness benefits, this represents the experience of wage-earners in 1893-97, excluding those in
the more hazardous occupations. (In some cases, however, sickness rates were high because
the jobs demanded a high level of physical fitness.)
5. Mortality investigations were also conducted, with separate tables for 3 geographical areas
and for urban and rural areas, but these have long been out of date. It is usual to combine
the M.U. sickness rates with a more modern mortality table (e.g. English Life Table No. 12-
Males, as used in Tables for Actuarial Examinations.)
6. The sickness rates of M.U. (whole society) were found to be remarkably similar to those for
employed men in the national insurance scheme in 1953-58.
7. Strictly speaking, sickness rates should be “select”, i.e. they should depend on duration of
membership as well as on attained age, since new entrants are not normally accepted unless
they are reasonably healthy. Another point is illustrated by the example of a new entrant aged
x who is to receive £20 per week for the first 26 weeks of sickness, £10 for the second 26 weeks
and £5 per week for the remainder of sickness, subject to a waiting period of 6 months. The
value of the benefits might be adjusted from
26 26/26 52/all
20Kx+ 1 + 10K
x+ 1
+ 5Kx+ 1
2 2 2
(i)
Dx
to
26 26/26 52/all
20Kx+ 1 + 10Kx+1 + 5Kx+1.5
2
(ii)
Dx
to allow for the fact that no “26/26” benefit can be received before age x + 1 and no “52/all”
benefit can be received before age x + 1.5. But formula (ii) is not quite right; consider, for
example, the “26/26” benefits, which are now valued as

26/26 26/26
10[Dx+1.5 zx+1 + Dx+2.5 zx+2 + ...]
Dx
26/26
Now zx+1 is based on an experience (M.U.) in which the exposed to risk included some recent
entrants who could not claim “26/26” benefit because they had not been eligible for sickness
26/26
benefit for more than 6 months. That is, zx+1 should be slightly adjusted upwards relative
26/26
to that of M.U. Similarly, zx+2 should be slightly increased, so formula (ii) understates the
expected value of the benefits. It is therefore better to use the formula

26/26 26/26
10[0.5Dx+0.5 zx + Dx+1.5 zx+1 + ...]
Dx
since the “extra” first term allows approximately for the fact that the later terms are too low.
This argument leads us to use sickness rates without deferment (except for the waiting period),
as in formula (i), except perhaps when benefits cease soon after the entry age.
In practice, the uncertain definition of “sickness” and other factors are likely to be of much
greater importance in the estimation of sickness rates. But when comparing the actual weeks
of sickness claim at the later durations in a given experience with that expected on the basis of
Manchester Unity (say), one must bear in mind any differences in the proportions of members
whose durations of membership are so short that they cannot claim. Thus, for example, an
experience consisting mainly of recent entrants may be expected to have low rates of sickness
at the later durations.
Chapter 19

PENSION FUNDS

19.1 General Introduction


Pension schemes may be described (in broad terms) as either:
(a) defined - benefit schemes,
or
(b) defined - contribution schemes.

Defined - benefit schemes


These are pension schemes whereby the pension and other benefits are set out in the rules of the
scheme. Most schemes of this type provide benefits which depend on the ‘final salary’. For example,
1
in a typical U.K. public sector final salary scheme, the annual pension is 80 × the final salary per
year of service. There may be various other benefits such as death benefits and a lump sum on
retirement.
The benefits are paid for by a combination of
(i) the employee’s contributions (e.g. a fixed percentage of salary)
and
(ii) the employer’s contributions, which may vary from time to time according to actuarial advice.

Remarks
1. A “non-contributory” pension scheme is one in which the employees do not contribute (e.g. U.K.
civil service).
2. An “insured” pension scheme is one in which benefits are secured by contracts with a life office.

Defined - contribution schemes


These are also called “money purchase” schemes; the contributions of both employees and em-
ployer are fixed (often as percentages of the salary). The pension benefit is what these combined
contributions can buy, usually after investment (either directly by the scheme or by life office con-
tracts) until retirement age. Obviously, the annual pension depends on the rate of return obtained
on the employer’s and member’s contributions and the terms on which pension may be purchased
at retirement (unless pension is purchased directly when the contributions are received).

19.2 Valuation Principles


Only defined-benefit schemes shall be considered, and the mean present value of the future benefits
and future contributions of an “active” member aged x will be calculated. (“Active” refers to a

303
304 CHAPTER 19. PENSION FUNDS

member who has not yet retired).


The reserve for each member is calculated prospectively. That is,

reserve = mean present value of future benefits - mean present


value of future contributions (of both employee and employer)

Notes
1. The rate of interest, i per annum, used in valuing the benefits and contributions is normally gross
(free of tax). i is called the valuation interest rate.
2. Expenses are usually ignored, as they are paid for separately by the employer.
3. A service table is required. This is a multiple-decrement table with various modes of decrement:
death, ill-health retirement, etc.
4. If benefits or contributions depend on the employee’s salary, a salary scale is required to estimate
future salaries from current salaries.
5. Sometimes one may require to find the employer’s contribution rate by setting the reserve equal
to zero at entry to the scheme of a new entrant, or for a group of new entrants.

19.3 Service Tables


The following notation is used, as in “Formulae and Tables in Actuarial Examinations”:
lx = number of members at exact age x
dx = number of deaths at age x last birthday
ix = number of “ill-health” retirements at age x last birthday
rx = number of “age” retirements at age x last birthday
wx = number of withdrawals at age x last birthday

Notes
1. “Age” retirements are the retirements at or above the minimum normal retirement age (NRA) of
the scheme. “Age” retirements may be concentrated at an exact age (for example 60 or 65) or may
be spread uniformly between ages 60 and 65, or both.
2. Members often have to retire by a certain age (often 65). In the “Formulae and Tables”, “age”
retirements occur only from age 60 onwards, and all members must retire at age 65 at the latest.
3. To calculate the value of benefits one may require separate mortality tables for “age” retirement
pensioners and ill-health retirement pensioners. We may also have to deal with benefits for people
with “deferred” pensions in the scheme who no longer work for the company, so a mortality table
for them is sometimes required (see later.)

19.4 Salary Scales


Assume that salaries are revised continuously.
Define {s̄x }, the salary scale function, to be such that the salary rate per annum at age x + t of a
life now aged x with current salary rate £(SAL) per annum is
µ ¶
s̄x+t
(SAL) × (19.4.1)
s̄x

Also define
Z x+1 Z 1
sx = s̄y dy = s̄x+t dt (19.4.2)
x 0
19.4. SALARY SCALES 305

Using approximate integration in (19.4.2), we have

1
sx ' s̄x+ 12 ' [s̄x + s̄x+1 ] (19.4.3)
2
“Formulae and Tables” gives values of sx (not s̄x ), so we find s̄x by linear interpolation:

1
s̄x ' [sx−1 + sx ] ' sx− 12 (19.4.4)
2
Note
To estimate s̄65 , one must use s̄64 12 ' s64 and s̄63 12 ' s63 and then employ linear extrapolation: see
Example 19.4.1 below.
The salary to be earned by (x) between ages x + t and x + t + 1, given that he is an active
member during this age-interval, is estimated as:
sx+t
(SAL) (19.4.5)
s̄x

where SAL is the current salary rate (in £) earned by (x).

We also find that:

s̄x+t
(SAL) = assumed salary rate per annum
s̄x
at exact age x + t (19.4.6)

Adjustments
1. If SAL refers to the earnings received in the past year (i.e. between ages x − 1 and x), adjust the
denominator from s̄x to sx−1 .
2. If SAL refers to the expected earnings in the coming year (i.e. between ages x and x + 1), adjust
the denominator from s̄x to sx .

Example 19.4.1. (a) Consider a person now aged exactly 25 whose annual salary rate is currently
£9,192. Estimate
(i) his annual salary rate at exact age 53,
(ii) his earnings between exact ages 64 and 65,
(iii) the average amount earned by him each year between exact ages 60 and 65,
(iv) his annual salary rate at exact age 65.
If he dies at age 57 last birthday, determine the average values of
(v) his annual salary rate at the moment of death,
(vi) the total amount earned over his last year of life.
(b) Calculate revised answers for (i) to (vi) assuming that the person aged exactly 25 is expected
to earn £9,192 in the coming year.
Assume that salaries are revised continuously, and use the pension table in “Formulae and Tables
for Actuarial Exams” with 4% p.a. interest.

Solution
(a) (i) 9192
s̄25 · s̄53 '
9192
s24 1 s52 2
1 = 9192 × 4.77
1.80 = £24, 359
2
9192 9192
(ii) s̄25 s64' s24 1 s64 = £27, 576
£ 2
s60 +s61 +s62 +s63 +s64
¤
(iii) 9192
s̄25 5 ' £27, 198.
306 CHAPTER 19. PENSION FUNDS

(iv) Annual salary rate at 64 is 9192 s̄s̄64


25
' 27, 499
s̄64 1
Annual salary rate at 64 21 is 9192 s̄252 ' 27, 576
Hence by linear extrapolation the annual salary rate at age 65 is approximately
27, 576 + (27, 576 − 27, 499) = £27, 653
(v) On average he will die at exact age 57 12 . Hence answer is
9192 9192
· s̄57 12 ' · s57 = £25, 891
s̄25 s̄25
(vi) 9192
s̄25 s56 2 ' £25, 738.
1

(b) Adjusting the denominator from s̄25 to s25 results in multiplying each answer by a factor of
s̄25 s24 12
' ' 0.962567.
s25 s25
This gives answers of:
(i) £23, 447 (ii) £26, 544 (iii) £26, 180
(iv) £26, 618 (v) £24, 922 (vi) £24, 775
Estimation of s̄x and sx
s̄x is usually estimated by a product of 2 factors, one to allow for future inflationary increases and
one to take account of “career progression”. The inflation factor is usually of the form (1 + e)x ,
where e is the assumed annual rate of future salary escalation.
It is often assumed that

i − e ' the “real” rate of interest (assuming


wage inflation and price
inflation are about the same)
'2% (or some similar figure)

where i is the valuation interest rate and future salary inflation is assumed to be e per annum.

19.5 The Value of Future Contributions


There are 2 main ways of calculating contributions:
(a) contributions are at the rate of k% of salary;
(b) contributions are at a fixed rate of £F per annum (payable continuously).

(a) The mean present value of the future contributions (of employee, employer or both) at rate k%
of salary for a member age x with current salary rate of £SAL per annum is
Z
k SAL 65−x t lx+t
· v s̄x+t dt
100 s̄x 0 lx
This is usually approximated to
64−x
k SAL X t+ 1 lx+t+ 12
· v 2 sx+t (using s̄x+ 21 ' sx ) (19.5.1)
100 s̄x t=0 lx

(Note the summation is up to “64 − x”).


Now define
Dx = v x lx , (as previously defined)
19.5. THE VALUE OF FUTURE CONTRIBUTIONS 307

R1
D̄x = 0 Dx+t dt ' Dx+ 21 ' 12 [Dx + Dx+1 ]
s
D̄x = sx D̄x
and P
s 64−x
N̄x = t=0 s D̄x+t

One can now evaluate (19.5.1) by means of commutation functions, giving a mean present value
of

1
64−x x+t+ 2
k SAL X v .lx+t+ 12
· x
sx+t
100 s̄x t=0 v lx
64−x
k SAL X Dx+t+ 12
= sx+t
100 s̄x t=0 Dx
64−x
k SAL X s
' D̄x+t
100 s̄x Dx t=0
k SAL s
= · N̄ x .
100 s̄x Dx

So, the m.p.v. of future contributions is

k SAL s
· N̄ x (19.5.2)
100 s̄x Dx

Notes
1. If SAL refers to the past year, change s̄x to sx−1 in the denominator.
2. If SAL refers to the coming year, change s̄x to sx in the denominator.

Example 19.5.1. Consider a life aged 35 with current salary rate £10,000 who contributes 5% of
his salary to a pension scheme. Using the Examination Tables, calculate the mean present value of
the employee’s future contributions.

Solution
m.p.v. of contributions is

5 10000 s 500 × 417, 224


· · N̄35 ' 1 = £9520.
100 s̄35 D35 2 (2.98 + 3.08) × 7232

(b) Suppose contributions are independent of salary and that these are a fixed annual sum of £F,
308 CHAPTER 19. PENSION FUNDS

payable continuously. Then the mean present value of future contributions is


Z 65−x
lx+t
F vt dt
0 lx
64−x
X 1 lx+t+ 1
'F v t+ 2 2

t=0
lx
64−x
X Dx+t+ 12
=F
t=0
Dx
64−x
X
1
'F · D̄x+t
Dx t=0
µ ¶
N̄x
=F (19.5.3)
Dx
P64−x
where N̄x = t=0 D̄x+t .

Example 19.5.2. If a life aged 35 contributes £500 each year to his pension scheme, calculate the
value of his future contributions.
Solution

N̄35
m.p.v. = 500 · = £7, 096.
D35
Suppose now that contributions are k% of the “pensionable salary”, where pensionable salary
is defined as “salary - A”, A being a fixed amount. Assuming that the current salary rate, SAL,
already exceeds A, the m.p.v. of these contributions is
µ ¶ µ ¶
k SAL s N̄x k N̄x
· − ·A (19.5.4)
100 s̄x Dx 100 Dx

Example 19.5.3. Employees contribute to a pension scheme at a rate of 6% of “pensionable salary”,


where “pensionable salary” equals actual annual salary rate less £2000 (to allow for other pension
arrangements). Find the present value of the future contributions payable by a member aged exactly
30 whose current annual salary rate is £15,500.
Solution

15500 × 0.06 s N̄30


m.p.v. ' · N̄30 − 2000 × 0.06
s29 12 · D30 D30
= 18, 377.7 − 1, 606.5 = £16, 771

19.6 The Value of Pension Benefits


Pension benefits are usually of one of the following forms:
(a) A fixed pension of £P per year of service;
(b) A fraction of the average salary per year of service;
19.7. FIXED PENSION SCHEMES 309

(c) A fraction of the final salary per year of service.

Notes
1. Years of service normally include fractions counting pro rata.
2. The benefits on the date of age retirement or ill-health retirement are valued by multiplying
the annual pension by an appropriate annuity function. For example, the benefit on age retirement
at age 65 is
annual pension × ār65
where r indicates age retirement mortality rates.
Similarly, āix refers to a life retiring at age x due to ill-health. Ill-health retirement mortality is
usually heavier than that for age retirements.
3. Although the symbol ār65 is usually used, the benefit may be payable in various ways, e.g.
monthly in advance, and is possibly subject to a guarantee that at least 5 years’ payments will be
made. In this example,
(12) (12)
ār65 = ä5 + 5 |ä65
on a suitable mortality table.

19.7 Fixed Pension Schemes


Consider a fixed pension of £P per year of service, including fractions pro rata, for a life now aged
x with n years’ past service.
Ill-health and age retirements will be considered separately.

Ill-health retirements
The m.p.v. of the benefit is
64−x
X 1 ix+t 1
v t+ 2 (n + t + )P āix+t+ 1
t=0
lx 2 2

This can be divided into 2 terms:


1. The value of the Past Service Pension (P.S.P.), which is the pension earned in respect of past
service with the employer; in this case the P.S.P. is nP ;
2. The value of the Future Service Pension (F.S.P.), which is the pension which will be earned in
future; in this case the F.S.P. is (t + 12 )P if retirement occurs (due to ill-health) at age x + t + 12 .
So the m.p.v. is
64−x
X 64−x
X
1 ix+t i 1 ix+t 1
nP v t+ 2 āx+t+ 1 + P v t+ 2 (t + )āix+t+ 1 (19.7.1)
t=0
lx 2
t=0
lx 2 2

Define the following commutation functions:


1
Cxia = v x+ 2 ix āix+ 1 ,
2
64−x
X
Mxia = ia
Cx+t
t=0
1
M̄xia = Mxia − Cxia ,
2
and
64−x
X
R̄xia = ia
M̄x+t
t=0
310 CHAPTER 19. PENSION FUNDS

It follows that
64−x
X 1 ia
R̄xia = (t + )Cx+t
t=0
2
Proof.
R̄xia = M̄xia + M̄x+1
ia ia
+ ... + M̄64
1
= ( Cxia + Cx+1ia
+ ... + C64ia
)
2
1 ia ia ia
+ ( Cx+1 + Cx+2 + ... + C64 )
2
+ ...
1 ia
+ C64
2
1 ia 1 ia 1
= Cx + 1 Cx+1 + ... + (64 − x + )C64
2 2 2
64−x
X 1 ia
= (t + )Cx+t .
t=0
2

These commutation functions can now be used to calculate (19.7.1).


The value of the P.S.P. is
64−x
X 1
v x+t+ 2
nP ix+i āix+t+ 1
t=0
v x lx 2

64−x
1 X ia
= nP · C
Dx t=0 x+t
µ ia ¶
Mx
= nP
Dx
The value of the F.S.P. is
64−x
X 1
v x+t+ 2 ix+t 1
P x
(t + )āix+t+ 1
t=0
v lx 2 2

64−x
1 X 1 ia
=P· (t + )Cx+t
Dx t=0 2
µ ia ¶
R̄x
=P
Dx
Hence the value of all ill-health pension benefits is
· ¸
n.Mxia + R̄xia
P (19.7.2)
Dx
Age retirements
The valuation of benefits caused by age retirements is very similar to that for ill-health retirements,
but with a final term corresponding to age retirement at exact age 65. The m.p.v. of age retirement
benefits is
64−x
X µ ¶
t+ 12 rx+t 1 r65
v (n + t + )P ārx+t+ 1 + v 65−x (n + 65 − x)P ār65 (19.7.3)
t=0
lx 2 2 lx
19.8. AVERAGE SALARY SCHEMES 311

Again, this may be separated into the P.S.P. and the F.S.P. terms.
Define the commutations ( 1
ra
v x+ 2 rx ārx+ 1 , x < 65
Cx = 2

v 65 r65 ār65 , x = 65
P 65−x ra
Mxra = t=0 Cx+t (note the summation is up to “65 − x”)

M̄xra = Mxra − 21 Cxra


and
64−x
X 64−x
X 1 ra
R̄xra = ra
M̄x+t = (t + )Cx+t ra
+ (65 − x)C65
t=0 t=0
2
(using a very similar proof to that for ill-health retirements).
Then the value of the P.S.P. is
"64−x #
1 X 1
nP · v x+t+ 2 rx+t ārx+t+ 1 + v 65 r65 ār65
Dx t=0 2

µ ra ¶
Mx
= nP ·
Dx
The value of the F.S.P. is
"64−x #
1 X 1 x+t+ 1 r 65 r
P· (t + )v 2r
x+t āx+t+ 1 + (65 − x)v r65 ā65
Dx t=0 2 2

"64−x #
1 X 1 ra ra
=P· (t + )Cx+t + (65 − x)C65
Dx t=0 2
µ ra ¶
R̄x
=P
Dx
Hence the value of all age retirement pension benefits is
· ¸
n.Mxra + R̄xra
P .
Dx
(i+r)a
So if one defines Mx = Mxia + Mxra , and similarly for other commutation functions, then the
value of a fixed pension of £P per annum payable for any retirement is
" #
(i+r)a (i+r)a
n.Mx + R̄x
P (19.7.4)
Dx
for a member aged x with n years’ past service.

19.8 Average Salary Schemes


Suppose that the annual pension on retirement is
1
× (total salary in service of company)
60
1
= × (total past salary, T.P.S.)
60
1
+ × (total future salary).
60
312 CHAPTER 19. PENSION FUNDS

Thus the benefits can be separated into the Past Service Pension and the Future Service Pension.
Value of P.S.P.
1
This is just the value of a fixed pension of 60 (T.P.S.) so, using the functions defined in the
previous section, Ã !
(i+r)a
T.P.S. Mx
m.p.v. of past service benefits = (19.8.1)
60 Dx
Value of F.S.P.
1
Consider ill-health retirement first. The salary to be earned between ages x + t and x + t + 2 is
estimated as
1 sx+t
(SAL)
2 s̄x
The m.p.v. of benefits is therefore
64−x
1 X t+ 1 ix+t SAL 1
v 2 (sx + sx+1 + ... + sx+t−1 + sx+t )āix+t+ 1
60 t=0 lx s̄x 2 2

64−x
SAL X 1 ia
= (sx + sx+1 + ... + sx+t−1 + sx+t )Cx+t (19.8.2)
60s̄x Dx t=0 2

Define the commutation functions


s
M̄xia = sx M̄xia
and
64−x
X
s ia
R̄xia = s
M̄ x+t
t=0

The ‘summation’ in (19.8.2) can be re-written as


1
sx ( Cxia + Cx+1
ia ia
+ ... + C64 )
2
1 ia ia ia
+ sx+1 ( Cx+1 + Cx+2 + ... + C64 )
2
µ ¶
1 ia
+ ...s64 C
2 64
ia
= sx M̄xia + sx+1 M̄x+1 ia
+ ... + s64 M̄64

So the value of the F.S.P. is


64−x
SAL X ia
sx+t M̄x+t
60s̄x Dx t=0
64−x
SAL X s ia
= M̄ x+t
60s̄x Dx t=0
SAL s ia
= · R̄x
60s̄x Dx
Consider age retirements now. The m.p.v. of the future service pension is very similar to the
summation in (19.8.2), with all ‘i’ terms being replaced by ‘r’ terms and with an extra term relating
to retirements at exactly age 65; this means that a final term of
SAL ra
· (sx + sx+1 + ... + s64 )C65
60s̄x Dx
19.9. FINAL SALARY SCHEMES 313

has to be added.
On defining the commutation functions
P64−x
s ra
M̄xra = sx M̄xra and s R̄xra = t=0 s M̄ x+t

and using a similar argument to that used for ill-health retirements, the value of the F.S.P. is
found to be
SAL s ra
· R̄x
60s̄x Dx
Thus the value of all benefits for an average salary scheme is
à ! à !
(i+r)a (i+r)a
T P S Mx SAL s R̄x
+ (19.8.3)
60 Dx 60s̄x Dx

Example 19.8.1. A pension scheme provides each member who retires (whether for “age” or “ill-
1
health” reasons) with an annual pension of 60 th of his average annual income over a member’s
entire service, for each year of service. Fractions of years of service are included when calculating
the amount of pension payable.
If contributions are paid entirely by the employer, calculate the appropriate contribution rate
(as a percentage of salary) for a new entrant aged 20.

Solution
Let k% be the contribution rate. Then k must solve
µ ¶ Ã !
(i+r)a
k SAL s N̄20 SAL s R̄20
· =
100 s̄20 D20 60s̄20 D20

s (i+r)a
R̄20
Hence, k = = 5.25%.
0.6s N̄20

19.9 Final Salary Schemes


Suppose that the annual pension is given by the formula
1
× “final salary” per year of service.
80
“Final salary” is usually defined as the average annual salary in the last m years of service.
Define
1
zx = (sx−m + sx−m+1 + ... + sx−1 ).
m
In “Formulae and Tables”, m = 3, and hence
1
zx = (sx−3 + sx−2 + sx−1 )
3
The method of valuing benefits is similar to that in section 19.7, allowing for salary factors.

Ill-health retirements
The m.p.v. of benefits for a member aged x with n years past service is
64−x
X SAL 1 1 ix+t
zx+t+ 12 (n + t + )v t+ 2 āi 1 (19.9.1)
t=0
80s̄x 2 lx x+t+ 2
314 CHAPTER 19. PENSION FUNDS

Define the commutation functions


z
Cxia = zx+ 12 Cxia ,

64−x
X
z
Mxia = z
C ia
x+t ,
t=0
1 z ia
z
M̄xia = z M ia
x − C ,
2 x
and
64−x
X 64−x
X
ia 1
z
R̄xia = z
M̄ x+t = (t + )z C ia
x+t
t=0 t=0
2
(note similarity to definitions in section 19.7)

Expression (19.9.1) becomes


64−x
SAL X 1
(n + t + )z Cx+t
ia
80s̄x Dx t=0 2
and, following the usual steps, we find that the m.p.v. of the benefits is equal to

SAL n.z Mxia SAL z R̄xia


+ · (19.9.2)
80s̄x Dx 80s̄x Dx
This shows the terms for past and future service pensions separately.

Age retirements
Again this is similar to the fixed pension case, but with salary factors inserted. The new commutation
functions are
(
z ra zx+ 21 Cxra if x < 65,
Cx = ra
z65 C65 if x = 65,
65−x
X
z
Mxra = z
C ra
x+t (note the upper limit of 65 − x)
t=0
1 z ra
z
M̄xra = z M ra
x − C
2 x
64−x
X
z ra
R̄xra = z
M̄ x+t
t=0
We find that the m.p.v. of benefits due to age retirements is

"64−x #
SAL X 1
(n + t + )z Cx+t
ra
+ (n + 65 − x)z C65
ra
80s̄x Dx t=0
2
SAL n.z Mxra SAL z R̄xra
= + (19.9.3)
80s̄x Dx 80s̄x Dx
Thus the value of all pension benefits for a final salary scheme is
SAL h z (i+r)a z (i+r)a i
n. M x + R̄x (19.9.4)
80s̄x Dx
19.9. FINAL SALARY SCHEMES 315

Example 19.9.1. Three members of a pension scheme whose age nearest birthday is 45 have the
following annual rates of salary and exact numbers of years of past service:
A : £15,000 20 years
B : £12,000 10 years
C : £14,000 5 years
For these members find the present value of a pension, payable on age-retirement or on ill-health
1
retirement, of 100 th of the average salary in the final 3 years before retirement for each year of service,
including fractions. Give separately the values of the past-service and future-service benefits.
Solution

P3
1 i=1 ni (SAL)i z (i+r)a
Value of P.S.P. = · M 45
100 s̄45 D45
1 490000(z M45 ia
+ z M ra45 )
' 1
100 (s
2 44 + s 45 )D 45

= 29, 071

P3
1 (SAL)i z (i+r)a
Value of F.S.P. = · i=1 · R̄45
100 s̄45 D45
1 41000 ia ra
' (z R̄45 + z R̄45 )
100 12 (s44 + s45 )D45
= 42, 929.

Hence total benefits have m.p.v. £72,000.

Remarks
1. If retirement is possible within m years, the factor zx should be adjusted to allow for the actual
past salary progression. Also, if someone retires with less than m years’ service, the final salary
must be redefined as the average over a shorter period. (In calculations these points are sometimes
ignored.)
2. If “final salary” means the salary rate at the date of retirement, then we change zx to s̄x . Also,
(i+r)a (i+r)a
if m = 1, zx = sx−1 . But one must not change z R̄x to s R̄x , as the latter refers to average
salary schemes.
3. If fractions of a year are not included when calculating the pension value, it is usually sufficiently
accurate to adjust the n years’ past service to n − 12 . If there is no past service, consider the member
as having “− 12 ” year past service. So one must substitute

1
(n − ).z Mx(i+r)a for n.z Mx(i+r)a
2
in formula (19.9.4) (even when n = 0).
4. Benefits may depend on a “pensionable salary” equal to salary minus some fixed sum, say £A. If
so, then the m.p.v. of the pension benefit in a final salary scheme is now
à !
SAL ³ z (i+r)a z (i+r)a ´ A n.Mx
(i+r)a (i+r)a
− R̄x
n. Mx + R̄x −
80s̄x Dx 80 Dx
5. It may be the case that the number of years of service are restricted to, say, 40 for pension
purposes. This will affect lives joining at ages under 25, assuming a latest retirement age of 65.
There are 2 cases to consider:
316 CHAPTER 19. PENSION FUNDS

(a) If n < 40, then leave the P.S.P. unchanged but restrict the F.S.P. . In the final salary scheme
mentioned above, the value of the F.S.P. is altered to

SAL ³z (i+r)a z (i+r)a ´


R̄x − R̄x+40−n (19.9.5)
80s̄x Dx

(b) If n ≥ 40, then restrict the past service to 40 years and make the F.S.P. zero. Hence the
value of pension benefits is
40 SAL z (i+r)a
· Mx (19.9.6)
80 s̄x Dx

1
Example 19.9.2. An executive pension scheme provides a pension of 45 of final salary for each
2
year of scheme service, with a maximum of 3 of final salary, upon retirement due to age between
the ages of 60 and 65.
Final salary is defined as salary in the 3 years prior to retirement.
A director, now aged 47 exactly has 14 years of past service and expects to earn £80,000 over
the coming year.
Using the symbols defined in the Formulae and Tables for Actuarial Examinations, what is the
expected present value of the future service pension on age retirement for this member?

Solution As the maximum pension is 23 = 45 30


of the final salary, service is restricted to a maximum
of 30 years for pension purposes. Hence, as n is 14,
ra ra
80000 z R̄47 − z R̄63
m.p.v. of F.S.P. = ·
45 s47 D47
(notice that £80,000 is the salary next year and not the salary rate, and hence we put s47 in the
denominator).

19.10 Lump Sums on Retirement


Suppose there is a cash payment on retirement of 3 times the annual pension, where the pension is
1
80 × final salary per year of service (as in section 19.9).
Define the commutation functions
1
Cxi = ix v x+ 2
z
Cxi = zx+ 21 Cxi ,

and so on, changing the annuity factor in all commutation functions (both “age” or “ill-health”) to
1.
These functions are tabulated (on the same basis as in “Formulae and Tables”) in the Supplement
to this book.
Using nearly the same arguments as before, the m.p.v. of the lump sum on retirement is

SAL i+r
3· (n.z M i+r
x + z R̄x ) (19.10.1)
80s̄x Dx
If the annual pension is a fixed sum of £P per year of service, then a lump sum of 3P on
retirement has m.p.v. µ ¶
n.Mxi+r + R̄xi+r
3P (19.10.2)
Dx
19.11. DEATH AND WITHDRAWAL BENEFITS 317

19.11 Death and Withdrawal Benefits


The benefits on death in service usually consist of one or more of:
1. a fixed sum, or a certain multiple of salary;
2. a return of the employee’s contributions, accumulated at a rate of interest
j (where j may be zero);
3. a spouse’s pension.

We begin by considering the first type of benefit.


Suppose the death benefit is 2× the salary rate at date of death. The benefit for a person aged x,
with current salary rate SAL, has m.p.v.
64−x
X 1 dx+t sx+t
2(SAL) v t+ 2 (19.11.1)
t=0
lx s̄x

Define the commutation functions


1
Cxd = v x+ 2 dx ,
s
Cxd = sx Cxd
64−x
X
Mxd = d
Cx+t
t=0
64−x
X
s
Mxd = s
C dx+t
t=0

Expression (19.11.1) can be written in the form


64−x
SAL X d
2 sx+t Cx+t
s̄x Dx t=0

and hence the value of the benefit is


SAL s Mxd
2 · (19.11.2)
s̄x Dx
If the benefit on death is just a fixed amount, £B, then the m.p.v. is
64−x
X 1 dx+t
B v t+ 2
t=0
lx
µ ¶
Mxd
B· (19.11.3)
Dx

Withdrawals
Similar formulae may be developed (replacing the ‘d’ with ‘w’) but withdrawal benefits are usually
in the form of a return of contributions or a deferred pension. (We shall discuss the formulae for
valuing these benefits later.)

Note
The commutation functions Mxd , Mxw , etc. are tabulated in the Supplement to this book. Note that
Mxd corresponds to j Mxd when j = 0 (page v of the Supplement) and Mxw corresponds to j Mxw when
j = 0 (page vi of Supplement).
318 CHAPTER 19. PENSION FUNDS

Example 19.11.1. A company has a pension scheme which provides a pension upon retirement
1
of 80 th of the final salary per year of service. In addition the sum of £10,000 is paid on death in
service of a member. If all contributions are paid by the employer, find the contribution rate as a
percentage of salary required for a new entrant aged 40 with a salary rate of £10,000.
Use the basis of “Formulae and Tables” (with the supplement); final salary is the average annual
salary in the 3 years prior to retirement.
Solution
Let k be the contribution rate per cent. Then k solves

10, 000 s 10, 000 (i+r)a Md


0.01k · · N̄ 40 = × z R̄40 + 10, 000 40
s̄40 D40 80s̄40 D40 D40
Thus,

0.01k × 172628.7 = 14827.60 + 895.6


Hence k = 9.11%.

19.12 Return of Contributions on Death or Withdrawal


Suppose that the employee’s contributions are returned with compound interest at rate j per annum
if the member leaves service.
Let TPC be the member’s total past contributions to the scheme.
The following additional commutation functions are defined:
1 1
j
Cxw = (1 + j)x+ 2 v x+ 2 wx ,
64−x
X
j w
Mxw = j
C x+t
t=0

j w 1j w
M̄xw = j M x − C
2 x
sj w
M̄xw = sx j M̄ x
64−x
X
sj 1 w
R̄xw = (1 + j)−(x+t+ 2 )sj M̄ x+t
t=0

Similar functions for returns of contributions on death are defined by substituting ‘d’ for ‘w’.
Suppose that the employees contribute at rate k% of salary. Then the m.p.v. of the return of
contributions on withdrawal for a member aged x is
j
Mxw k SAL sj w
(T P C)(1 + j)−x · + · · R̄x (19.12.1)
Dx 100 s̄x Dx
Proof. See the Appendix to this Chapter.

Note
w w
The values of j M x , sj
R̄x , etc. are given for j = 0.03 and j = 0 in the Supplement to this book.

If the employee’s contributions are fixed at £A per annum, payable


continuously, the m.p.v. of the return of contributions on withdrawal is
j
Mxw j w
R̄x
(T P C)(1 + j)−x +A· (19.12.2)
Dx Dx
19.12. RETURN OF CONTRIBUTIONS ON DEATH OR WITHDRAWAL 319

(see the Appendix to this Chapter.)


Note
When j = 0 one may omit the ‘j’ from the commutation functions, so if contributions are returned
without interest their mean present value is

Mxw k SAL s w
(T P C) + · R̄x (19.12.3)
Dx 100 s̄x Dx

if the employee contributes k% of salary, and

Mxw R̄w
(T P C) +A· x (19.12.4)
Dx Dx

if he contributes at the fixed rate of £A p.a.

If employee’s contributions are returned on death (with or without interest), formulae (19.12.1)
to (19.12.4) are changed by just substituting ‘d’ for ‘w’.
Furthermore, if contributions are returned on either death or withdrawal, one may adjust formula
(19.12.1) to
j
Mxd+w k SAL sj d+w
(T P C)(1 + j)−x + · · R̄x (19.12.5)
Dx 100 s̄x Dx
d w
where j Mxd+w = j M x + j M x
d w
and sj R̄xd+w = sj R̄x + sj R̄x , as expected.
(Similar remarks apply to formulae (19.12.2) to (19.12.4).)

Example 19.12.1. You are actuary to a pension scheme which provides the following benefits:
1
(a) a pension of 80 × final salary for each year of service (including fractions pro rata) on retire-
ment for “age” or “ill-health” reasons.
(b) a lump sum on retirement of 3 times the annual pension.

Your actuarial basis is that given in the “Formulae and Tables”, and you calculate reserves prospec-
tively, ignoring expenses. Final salary is the average annual salary in the 3 years before retirement.
Members pay contributions at the rate of 2% of salary. The total contribution rate is assessed
for each member separately, and is the proportion of salary which will pay for the benefits, when
the member joins, i.e. the prospective reserve at the entry date is zero.
1. Find the employer’s contribution rate for a member joining at age 20.
2. Find the reserve held for a member aged 45, with salary at the rate of £20,000 per annum,
who joined the employer at age 20.
3. Your pension scheme accepts transfer values from other pension schemes. Suppose that a life
aged 45 joins the scheme, bringing a transfer value of £10,000. The member’s current salary rate is
£15,000 p.a., and he asks for his transfer value to be used to give him “added years” of service, i.e.
to credit him with n years of past service. Find n, given that, in the event of death in service, the
transfer value will be returned with compound interest at 3% p.a.

Note
There are no withdrawals over age 45 in the Tables, so we need not specify what happens to the
transfer value in that event.
320 CHAPTER 19. PENSION FUNDS

Solution
1. Let k be total contribution rate per cent for a new member aged 20. Then
k SAL s SAL z (i+r)a i+r
· N̄20 = [ R̄20 + 3.z R̄20 ]
100 s̄20 D20 80s̄20 D20
Thus
" #
z (i+r)a i+r
100 R̄20 + 3.z R̄20
k= s N̄
= 7.443.
80 20

So employer contributes at rate 7.443% − 2% = 5.443% of salary.


2. Reserve is

20, 000 (i+r)a (i+r)a i+r i+r 20, 000


[25.z M 45 +z R̄45 + 3(25.z M45 +z R̄45 )] − (0.0744)s N̄45
80s̄45 D45 s̄45 D45
= £59, 585.

3. Let n years of past service be credited. The equation of value is


15000n z (i+r)a i+r
10000 = ( M45 + 3.z M45 )
80s̄45 D45
µj d ¶
M45
+ 10000(1.03)−45 where j = 0.03
D45
Hence 10, 000 = 1420.47.n + 1394.42
Therefore n = 6.06 years

(The transfer value is treated as if it were the T.P.C.)

19.13 Spouse’s Benefits


A pension scheme may provide the following benefits (among others):
(a) a spouse’s death in service (D.I.S.) pension,
and/or
(b) a spouse’s death after retirement (D.A.R.) pension.

If spouse’s pension ceases on remarriage, a double-decrement table (for death and remarriage)
should be constructed and used in place of the mortality table for spouses.

(a) Spouse’s D.I.S.


Consider (for example) a male member aged x of a pension scheme providing an annuity payable
(say) monthly in advance to his widow if he dies in service. As in chapter 13, we define
hx = probability that a man is married at age x
and d = the average age difference (in years) between husband and wife, i.e. average value of
(husband’s age - wife’s age).
The m.p.v. of this benefit is
64−x
X 1 dx+t (12)
v t+ 2 h 1 × (spouse’s pension p.a.)ä (19.13.1)
t=0
lx x+t+ 2 f
x+t+ 1 −d 2
19.13. SPOUSE’S BENEFITS 321

The size of spouse’s pension may depend on the member’s salary at or near the date of death
and the number of years of service.

Note. Commutation functions are complicated and are usually ignored.

(b) Spouse’s D.A.R.


One must consider the following 2 cases:
(i) Widow’s pension is only payable if the widow was married to the scheme member when he
retired. (Post-retirement marriages do not count for benefit purposes.)
(ii) Any widow may receive the pension.
The spouse’s D.A.R. pension is valued by first considering the appropriate value at the retirement
date of the member, and then allowing for survivorship and interest before retirement.
For example, consider a man retiring at age 65 in normal health. Assuming that spouse’s pensions
are payable continuously, the value at that age of the spouse’s D.A.R. benefit is as follows:
Case (i) : (annual widow’s pension).h65 ā m f (19.13.2)
65|65−d

Case (ii) : R∞
(annual widow’s pension). 0 v t .t pm
65 µ m h65+t ā f dt (19.13.3)
65+t 65−d+t

Example 19.13.1. A pension fund provides the following benefits for widows of male members:
(a) on death of the member in service, a widow’s pension of annual amount equal to one-third
of the member’s salary rate at the time of his death; and
1
(b) on death of the member after age or ill-health retirement, a widow’s pension of 120 th of the
member’s average salary in the 3 years immediately preceding retirement for each year of service,
fractions of a year counting pro rata.

Develop formulae for valuing the widow’s benefits for a male member aged x with n years’ past
service (including fractions). You may assume that all age-retirements take place at exact age 60,
between ages 60 and 65, or at exact age 65, and that ill-health retirements may take place at any age
between 35 and 60. Widows’ pensions on death after retirement are payable to any widow (not just
to a widow who was married to the member when he retired.) Widows’ pensions are payable monthly
in advance and do not cease on remarriage. You may assume that a service table has already been
constructed, and that the member’s current salary rate per annum is SAL. Commutation functions
need not be developed.
Solution
(a) Define s̄x , sx , hx as previously. Let
1
zx = (sx−3 + sx−2 + sx−1 ) as before.
3
Let

y = average age of wife of (x)


=x−d
Then value of benefit is
64−x µ ¶
SAL X t+ 1 dx+t s̄x+t+ 12 (12)
v 2 hx+t+ 12 ä f
3 t=0 lx s̄x y+ t+ 21
322 CHAPTER 19. PENSION FUNDS

(b) Consider ill-health retirement first.


59−x µ ¶
SAL X t+ 1 ix+t zx+t+ 21 1
m.p.v. = v 2 (n + t + )aIH 1
120 t=0 lx s̄x 2 x+t+ 2

where
Z ∞
(12)
aIH
x+t+ 21 = v r .r pih ih
x+t+ 1 µx+t+ 1 +r hx+t+ 2 +r äy+t+ 1 +r dr
1
2 2 2
0

(“ih” indicates that a mortality table for men retiring due to ill-health is employed).
For age retirements, there are terms for retirement (i) between ages 60 and 65, (ii) at exact ages
60 and 65.
(i) value =
64−x µ ¶
SAL X t+ 1 rx+t zx+t+ 12 1
v 2 (n + t + )aN H
1
120 t=60−x lx s̄x 2 x+t+ 2

where aN H
x+t+ 1
is as for aIH
x+t+ 1
, but with a mortality table for age retirements.
2 2

µ 0 ¶
SAL 60−x r60 z60
(ii) value = v (n + 60 − x)aN
60
H
120 lx s̄x
µ 0 ¶
SAL 65−x r65 z65
+ v (n + 65 − x)aN
65
H
120 lx s̄x

(rx0 refers to retirements at exact ages 60, 65; note that l65 = r65
0
).

19.14 Preserved Pensions on Leaving Service


Suppose that (for example) “early leavers” get a deferred annual pension of
1
60 × final salary per year of service, vesting at age 65. ‘Final’ salary refers to the average salary in
the 3 years before leaving service.
In practice, this pension may escalate between the date of leaving and age 65; suppose that
escalation is at rate j per annum compound. Also, one must specify a mortality table for the early
leavers: let us use the notation ˆlx for their life table. The m.p.v. of this withdrawal benefit for a life
age x is
64−x
SAL X 65−x wx+t 1 zx+t+ 21 1
ˆl65
v (n + t + ) (1 + j)65−(x+t+ 2 ) · ā65 (19.14.1)
60 t=0 lx 2 s̄x ˆl
x+t+ 1 2

where ā65 is the annuity factor, with the pension possibly payable monthly, etc.
To evaluate (19.14.1) by commutation functions, one needs to define suitable ‘new’ commutation
functions. That is, the expression (19.14.1) is equal to
64−x
SAL X 1
(n + t + )z Cx+t
wa
60s̄x Dx t=0 2

where
" 1 #
wx zx+ 12 (1 + j)65−x− 2
z
Cxwa = ˆl65 a65 v 65
ˆl 1
x+ 2
19.14. PRESERVED PENSIONS ON LEAVING SERVICE 323

We now define
64−x
X
z
Mxwa = z
C wa
x+t ,
t=0
1 z wa
z
M̄xwa = z M wa
x − C
2 x
and
64−x
X
z wa
R̄xwa = z
M̄ x+t
t=0

(Note the similarities with the definitions used in previous sections.) The present value of the
deferred pension is
SAL £ z wa z wa ¤
n. Mx + R̄x (19.14.2)
60s̄x Dx
Note The notation used here is not standard, but is an example of “do-it-yourself” commutation
functions.
324 CHAPTER 19. PENSION FUNDS

Exercises

19.1 Contributions to a pension scheme by employees are made at a rate of 5% of salary when aged
under 35, 6% between ages 35 and 45, and 7 12 % when aged 45 or over. Calculate the present
value of the future contributions payable by a member aged exactly 30 who in the past year
has received a total salary of £12,718.
19.2 A company pension scheme provides the following benefits for all members:
(1) a pension on retirement (on grounds of ill-health or of age) of one-eightieth of final pen-
sionable salary for each year of service (including fractions),
(2) a lump sum on retirement of 3 times the annual pension,
(3) on death in service, a lump sum of £30,000,
(4) on withdrawal from service, a return of the employee’s contributions, accumulated at 3%
per annum compound.
Final pensionable salary is defined as the average annual salary in the three years immediately
before retirement. All members who reach age 65 retire immediately.
Employees contribute to the scheme at the rate of 3% of salary, payable continuously. Salaries
are revised continuously. The employer’s contribution rate is assessed for each member sep-
arately, and is such that the prospective reserve for each new entrant is zero. Expenses are
ignored.
(i) (a) Derive a formula, in terms of suitable commutation functions, for valuing benefit (1)
above in respect of a new entrant aged x with annual salary rate SAL. (You need not define
the service table functions.)
(b) In respect of a new entrant aged x with annual salary rate SAL, give formulae for valuing
benefits (2), (3) and (4) above, using suitable commutation functions. (You need not derive
the formulae.)
(c) Hence find a formula for the employer’s contribution rate for a new member aged x and a
starting salary rate of £10,000 p.a.
(ii)(a) Using the basis given in the pension fund section of the Formulae and Tables (and the
supplement), find the value of each of the benefits (1), (2), (3) and (4) for a new entrant aged
45 with salary rate £10,000 per annum.
(b) Hence or otherwise find the employer’s contribution rate for this new member.
19.3 The pension scheme of a certain company provides an annual pension on retirement (for
‘age’ or ‘ill- health’ reasons) of amount equal to one per cent of the member’s total earnings
throughout his service. The pension is payable weekly. In addition, in the event of a member
dying in service there is payable at the time of death a lump sum of £30,000. There is no
benefit on withdrawal.
The company pays a constant percentage of all the members’ salaries into the pension fund.
The percentage is that which will exactly cover the cost of benefits for a new entrant to the
fund at age 30 with an initial salary rate of £10,000 per annum. Contributions are payable
continuously, and the employees do not contribute to the scheme. Expenses are negligible.
(a) Calculate the contribution rate paid by the company, assuming the last retirement age is
65.
(b) A valuation of the fund is to be conducted. For each active member of the scheme there
is recorded
19.15. EXERCISES 325

(i) the age nearest birthday (which is regarded as the member’s exact age) at the valuation
date,
(ii) the annual salary rate at the valuation date, and
(iii) the total past earnings in service (prior to the valuation date.)
For each age, the totals of (ii) and (iii) are recorded and the following is an extract from the
data.

Age x No. of members Total past earnings Total of annual salary


aged x for members aged x rates for members aged x
£ £
25 11 302,100 70,100

Assuming that the basis of the Tables provided is appropriate, find the liability at the valuation
date for the benefits payable to the members aged 25, and determine whether the future
contributions payable in respect of these members are more or less than sufficient to cover the
benefits.
19.4 A pension scheme provides each member who retires (for any reason) with annual pension
1
equal to 60 × final salary per year of service. Final salary is the average income over the last
3 years of service, and fractions of a year of service are not included when calculating the
pension.
Assuming that equal contributions are payable by the member and his employer, that in the
event of death in service a benefit is payable equal to the return without interest of both the
member’s and the employer’s contributions, and that in the event of withdrawal from service
a return without interest is made of the member’s contributions, calculate the appropriate
contribution rate payable by both the member and his employer in respect of a new entrant
aged 40.
19.5 You are consulting actuary to a small pension scheme, which has just been established. You
have decided to use the pension fund tables in Formulae and Tables for Actuarial Examinations
as the basis for all calculations. The scheme provides the following benefits to employees:
1
(i) on retirement (for ill-health reasons or otherwise), a pension of 80 th of annual pension-
able salary, averaged over the previous three years, for each year of future service including
fractions;
(ii) on withdrawal or death in service, a return of the member’s contributions, accumulated
at 3% per annum compound interest.
Employees contribute 2% of salary to the scheme. Pensionable salary is defined as salary less
£4000. Salaries are revised continuously, and contributions are made continuously.
Details of the current membership are as follows.
member age current salary rate (£)
1 45 30,000
2 45 20,000
3 35 10,000
4 35 10,000

(a) The employer has decided to contribute the proportion of total salaries which, together
with the employees’ contributions, will exactly pay for the benefits. Calculate the employer’s
contribution rate.
(b) A new employee, aged 35 and with current salary rate £8,000, is about to be hired.
Calculate the surplus or deficiency in the pension fund after this new member joins.
326 CHAPTER 19. PENSION FUNDS

19.6 It is desired to set up a pension scheme for the group of employees described below. For each
member there is recorded his exact age, his exact length of service with the company, and his
annual rate of salary.

Past Rate of Past Rate of Past Rate of


Age Service Salary Age Service Salary Age Service Salary
(Years) p.a. (Years) p.a. (Years) p.a.
£ £ £
25 4 8,800 35 12 12,400 45 25 15,000
25 6 8,500 35 18 12,500 45 15 14,000
25 5 8,500 35 15 12,800 45 10 13,000
25 3 8,600 35 5 12,500 45 20 13,200
25 1 8,600 35 10 12,400 45 5 13,000

1
The scheme will provide pensions of 60 th of “pensionable salary” for each year of service
(fractions of a year being included) and on death or withdrawal from service a return will be
made of the member’s contributions with 3% compound interest. All members will contribute
at the same rate and the employer will contribute the same amount as each member. The
contribution rate will be such as to provide exactly the benefits for future service. The basis
of “Formulae and Tables” is to be used.
(a) Assuming that pensionable salary is the average annual earnings over the three-year period
ending on the retirement date, calculate the contribution rate payable by each member.
(b) Calculate also the total liability for past service benefits. The employer wishes to meet
this liability by paying additional contributions proportional to future salary payments. At
what rate should these additional contributions be made?
(c) Immediately after the scheme is set up as described above the 45 year old member with
5 years of service withdraws. Is the position of the fund improved or worsened by this with-
drawal?
19.7 The pension scheme of a large company provides the following benefits, among others:
(1) on death in service of married members: a spouse’s pension of one-third of the member’s
annual salary at the date of death;
(2) on death after normal retirement of the member at age 65 or after ill-health retirement
at an earlier age: a spouse’s pension of 1% of annual salary at the date of retirement for
each year of scheme membership (including fractions of a year); no pension is payable if the
marriage has taken place after the member’s retirement date.

Assume that a service table has been constructed, and that the proportion of members mar-
ried at exact y is hy . Assume further that spouses are the same age as members, and that
a unisex mortality table is used for all calculations, with the age rated up by 5 years on
ill-health retirement. You are given the age nearest birthday, current salary rate and years
of past service (including fractions) of each member. Spouse’s pension is payable monthly in
advance, beginning immediately on the death of the member.
(i) Using the rate of interest i per annum, find formulae for the mean present value of each
of the benefits (1) and (2) above for a member aged x (x < 65). You are NOT required to
construct commutation functions.
(ii) Suppose that the scheme’s rules are to be changed so that benefit (2) is to be payable to
any surviving spouse. Show how to modify the formulae of (i) to accommodate this change.
19.16. SOLUTIONS 327

Solutions

19.1
12, 718
{0.05s N̄30 + 0.01s N̄35 + 0.015s N̄45 }
s29 × D30
= 12, 718 × 1.30162
= £16, 554

19.2 (i)(a) "64−x µ #


X ¶
SAL 1 z ia 1 z ra
m.p.v. = (t + ) Cx+t + (t + ) Cx+t + (65 − x)z C65
ra
80s̄x Dx t=0
2 2
(i+r)a
SAL z R̄x
=
80s̄x Dx

z i+r
3.SAL R̄x
(b)(2): 80s̄x · Dx
Mxd
(3) 30000 · Dx
SAL sj w
(4) 0.03 · s̄x Dx R̄x where j = 0.03
(c) Let employer’s contribution rate be k for this member. Then
SAL s
(k + 0.03) N̄ x = above benefits.
s̄x Dx
So
· ¸
1 1 z (i+r)a 3 z i+r d sj w
k + 0.03 = R̄ x + R̄ + 3s̄ M
x x + 0.03. R̄
s N̄ x 80 80 x x

(ii)(a) Values of benefits are:


(1) 13,088
(2) 3,642 (using Supplement)
(3) 3,016
(4) 0
Hence total value of benefits = £19,746.
(b) value of contributions = 142580(k + 0.03)
19746
Hence k + 0.03 = 142580
and hence k = 10.85%.
19.3 (a) Let k be the contribution rate. Then
(i+r)a
SAL s SAL s R̄30 Md
k· · N̄30 = · + 30000 · 30
s̄30 D30 100s̄30 D30 D30
where SAL = 10, 000.

" #
s (i+r)a
s̄30 R̄30 d
Hence k = s 0.01 + 3.M30
N̄30 s̄30
= 0.0535 = 5.35%
328 CHAPTER 19. PENSION FUNDS

(b) Value of benefits for the group is


(i+r)a (i+r)a
302100 M25 70100 s R̄25 Md
· + · + 11 × 30000 25
100 D25 100s̄25 D25 D25
= 2977.37 + 54730.83 + 11811.67 = £69, 528

Value of future contributions is


70100 s
0.0535 × N̄ 25
s̄25 D25
= £76, 718.

Hence contributions are more than sufficient.


19.4 Let k% of salary be the contribution rate for both the member and the employer. Then if the
member has a salary rate of £SAL, · ¸
2k SAL s SAL z (i+r)a 1 z (i+r)a
· N̄40 = R̄40 − M 40
100 s̄40 D40 60s̄40 D40 2
k SAL s w 2k SAL s d
+ · · R̄40 + · · R̄40
100 s̄40 D40 100 s̄40 D40

Hence 6342.42k = 35513.07 + 13.74k + 562.35k.


Therefore k = 6.16%
19.5 (a) Construct the following table, using j = 0.03:
z
SAL R̄(i+r)a 4000 R̄(i+r)a sj
R̄d+w
Member age salary 80s̄x· xDx 80 · xDx 0.02 SAL
s̄x
x
Dx
1 45 30,000 39,264 3,999 899
2 45 20,000 26,176 3,999 599
3 35 10,000 15,189 3,498 549
4 35 10,000 15,189 3,498 549
95,818 14,994 2,596
Let the employer contribute k% of salary,
· so m.p.v. of contributions ¸ is
50000 s 20000 s
(k + 0.02) N̄ 45 + · N̄35
s̄45 D45 s̄35 D35
= 1093716(k + 0.02)

Therefore k + 0.02 = 95818−14994+2596


1093716
and hence k = 5.627%.
(b) Reserve for new member is
(i+r)a (i+r)a
8000 z R̄35 4000 R̄35
· − ·
80s̄35 D35 80 D35
8000 sj d+w
+ 0.02 × . R̄35
s̄35 D35
8000
− × 0.07627s N̄35
s̄35 D35
= 12, 151 − 3, 498 + 440 − 11, 617
= −2, 524.

Hence there is now a surplus of £2,524.


19.6 (a) For each age x,
P z
(n×SAL) Mx(i+r)a
Value of Past Service Pension = 60s̄x · Dx
19.16. SOLUTIONS 329
P z (i+r)a
SAL R̄x
Value of Future Service Pension = ·
P60s̄x Dx
sj d+w
SAL R̄x
Value of return of contributions = 100s̄x · Dx (where j = 0.03) per 1% of salary

P P return of
x SAL (n × SAL) P.S.P. F.S.P. contributions (per 1% of salary)
25 43,000 163,100 7,776 76,783 2,223.7
35 62,600 752,300 55,421 126,780 1,720.5
45 68,200 1,044,000 103,230 119,012 1,022.7
166,427 322,575 4,966.9
The value of the contributions
P is, per 1% of contributions,
SAL s N̄x
(1)
100s̄x Dx

x Value of (1)
25 8,791.2
35 11,919.1
45 9,724.0
30,434.3
Let members’ contribution be k%. The equation of value is thus
2k × 30, 434.3 = 322, 575 + 4, 966.9k

Hence k = 5.77%.
(b) The past service liability is £166,427. Let the contribution rate per cent needed to pay
for this be p. Then p solves,
p × 30, 434.3 = 166, 427
Hence p = 5.468%.
(c) The reserve for the withdrawn member is
13, 000 z (i+r)a 5 × 13000 z (i+r)a 13000 sj d+w
· R̄45 + M 45 + 0.0577 × · R̄45
60s̄45 D45 60s̄45 D45 s̄45 D45
13000 s
− (2 × 0.0577 + 0.05468) · · N̄45
s̄45 D45
= 30, 238 − 31, 526
= −1, 288

So the position of the pension fund has worsened by £1,288.


19.7 (i) Benefit (1) has value
64−x
SAL X t+ 1 dx+t (12) sx+t
v 2 h 1 ä
lx x+t+ 2 x+t+ 2 s̄x
1
3 t=0

Benefit (2):
On normal retirement the value is
SAL r65 h (12) 1
i
· v 65−x .s̄65 (n + 65 − x) × h65 a65|65 (1 + i) 24
100s̄x lx

On ill-health retirement, the value is


64−x
SAL X t+ 1 sx+t ix+t 1 (12) 1
v 2( ) (n + t + )[hx+t+ 12 ax+t+ 1 +5|x+t+ 1 (1 + i) 24 ]
100 t=0 s̄x lx 2 2 2
330 CHAPTER 19. PENSION FUNDS

(ii) Benefit (1): no change


Benefit (2):
For normal retirement change
(12) 1
h65 a65|65 (1 + i) 24
to Z ∞
(12)
v t .t p65 µ65+t h65+t ä65+t dt
0

For ill-health retirement, change


(12) 1
hx+t+ 12 ax+t+ 1 +5|x+t+ 1 (1 + i) 24
2 2

to
Z ∞
(12)
v r .r px+t+ 12 +5 µx+t+ 12 +5+r hx+t+ 12 +r äx+t+ 1 +r dr
2
0
19.16. SOLUTIONS 331

Appendix: Formulae for valuing a return of contributions


Suppose employee’s contributions are to be returned on withdrawal with interest at rate j per
annum compound. Consider a member aged x whose current salary rate is SAL and whose total
past contributions, accumulated at rate j p.a., are TPC.
Assume that the employee will in future contribute k% of salary. The value of the return of
future contributions is
64−x
k SAL X t+ 1 wx+t 1
v 2 [sx (1 + j)t + sx+1 (1 + j)t−1 + ... + sx+t−1 (1 + j) + sx+t ]
100 s̄x t=0 lx 2
µ ¶
k SAL 1 1 1 1
= {v 2 wx × sx + v 1 2 wx+1 [sx (1 + j) + sx+1 ]
100 s̄x lx 2 2
1 1
+ v 2 2 wx+2 [sx (1 + j)2 + sx+1 (1 + j) + sx+2 ]
2
+ ...
1 1
+ v 64 2 −x w64 [sx (1 + j)64−x + sx+1 (1 + j)64−x−1 + ... + s63 (1 + j) + s64 ]}
2
Collecting the coefficients of sx , sx+1 , ... gives
µ ¶
k SAL 1 1 1 1
{sx [ v 2 wx + v 1 2 wx+1 (1 + j) + ... + v 64 2 −x w64 (1 + j)64−x ]
100 s̄x lx 2
1 1 1 1
+ sx+1 [ v 1 2 wx+1 + v 2 2 wx+2 (1 + j) + ... + v 64 2 −x w64 (1 + j)64−x−1 ]
2
1 1
+ ... + s64 [ v 64 2 −x w64 ]}
2
Define
1 1
j
Cxw = (1 + j)x+ 2 v x+ 2 wx
64−x
X
j w
Mxw = j
C x+t
t=0

j w 1j w
M̄xw = j M x − C
2 x
sj
M̄xw = sx .j M̄xw
64−x
X 1
sj
R̄xw = (1 + j)−(x+t+ 2 ) .sj M̄x+t
w

t=0

The value of the return of future contributions can be written as


µ ¶ · ¸
k SAL −(x+ 21 ) 1 x+ 12 x+1 12 64 12
{s x (1 + j) (v(1 + j)) wx + (v(1 + j)) wx+1 + ... + (v(1 + j)) w64
100 s̄x v x lx 2
· ¸
1 1 1 1
+ sx+1 (1 + j)−(x+1 2 ) (v(1 + j))x+1 2 wx+1 + (v(1 + j))x+2 2 wx+2 + ... + (v(1 + j))64 f rac12 w64
2
· ¸
1 1 1
+ ... + s64 (1 + j)−64 2 (v(1 + j))64 2 w64 }
2
µ ¶
k SAL sj w
= . R̄x
100 s̄x Dx
332 CHAPTER 19. PENSION FUNDS

The value of the return of past contributions with interest at rate j is


64−x
X 1 1 wx+t
(T P C) v t+ 2 (1 + j)t+ 2
t=0
lx
64−x
(T P C) X 1
= (1 + j)−x (v(1 + j))x+t+ 2 wx+t
Dx t=0
j
Mxw
= (T P C)(1 + j)−x
Dx
Hence the total value is
j
Mxw k SAL sj w
(T P C)(1 + j)−x + R̄x
Dx 100 s̄x Dx
Suppose now that the employee’s contributions do not depend on salary, but are instead £A each
year, payable continuously. The value of the return of future contributions is obtained in a similar
way to the previous derivation, with the salary scale function taken as 1. This gives a present value
of
j w
R̄x

Dx
where
64−x
X 1
j
R̄xw = (1 + j)−(x+t+ 2 ) .j M̄x+t
w
.
t=0
Appendix A

Some notes on examination technique

Each candidate should spend some time going over his or her notes, the textbook and (especially)
past examination papers, making a note of subjects and formulae of particular importance. For this
purpose a set of postcards/computer cards is useful: each topic may be summarised on one postcard,
enabling the candidate to learn the most important facts and then have a “revision aid”. (He or she
should ask a friend to choose a card at random, to give the title and to ask for a description of the
topic.)

In actuarial examinations (particularly in life contingencies) it is important to separate the basic


ideas or “rationale” of the solution from

(i) technical aspects, such as the correct use of actuarial tables and commutation functions, and
(ii) arithmetic.
It is sometimes necessary to omit (ii) or even (i) under extreme time-pressure, but one should
leave space to return to the question later if time permits. On the other hand, it is satisfying to give
a complete answer, and this should be attempted if time is not short. One should keep the left-hand
pages of the examination book free for rough working and arithmetic.

On first seeing the examination paper, one should tick the questions about which one is most
confident - especially bookwork. Leave the less familiar questions until later. One should ensure that
an approximate time-scheme (say, 15 minutes per 10-mark question) is adhered to, within reasonable
limits. Routine arithmetic (or even looking up the tables) may sometimes be left till near the end,
when one is too tired to think out new ideas but can still do (or check) arithmetic. As in musical
performances, it is essential to get off to a good start, by being in the right frame of mind and
picking the most suitable questions to attempt first.

333
Appendix B

Some technical points about the tables used in examinations

1. Avoid excessive reliance on commutation functions.


In theoretical/statistical questions, it is nearly always best to avoid the use of commutation func-
tions.

2. In the A1967-70 section of “Formulae and Tables” commutation functions are given only at 4%
interest. For other interest rates one most use the more limited tables provided, e.g.
lx+n
äx:n at 6% = äx − v n äx+n
lx

lx+n Dx+n
(There is no point writing v n as since Dx and Dx+n are not given at 6% in the tables.)
lx Dx

3. Remember to make use of the functions maturing at ages 60, 65, etc. This saves time.

4. If only limited tables are available, one must proceed directly, using suitable approximations. For
1
example, suppose that one is asked to evaluate Ā50.5 :2.5 at 10% interest on A1967-70 ultimate.
This may be done by the trapezoidal rule as follows:
Z 2.5 · ¸
2.5 l53 µ53
v t t p50.5 µ50.5+t dt + µ50.5 + v 2.5
0 2 l50.5

where µ50.5 and l50.5 are estimated by linear interpolation.

5. Remember that for a(55) there are two tables - male and female (each with a select period of 1
year.) Assume that males and females are subject to the table of the appropriate sex (unless the
question states otherwise.)

6. Be careful (especially in A1967-70, but also in a(55)) that you distinguish between “select” and
“ultimate” tables and use the correct rate of interest.

334
Appendix C

Some common mistakes

WRONG CORRECT

1 1
1. Āx:n ; (1 + i) 2 Ax:n Āx:n ; (1 + i) 2 Ax1:n + Ax:n1
1
(1 + i) 2 (Mx − Mx+n ) + Dx+n
=
Dx
i 1 1
2. āx + ax āx ; ax + 2 or äx − 2
δ
µ ¶µ ¶
i m−1 Dx+n
3. ä(m)
x:n
; ax:n ä(m)
x:n
; äx:n − 1−
d(m) 2m Dx

4. ax1:n , ax:n1 do not exist

Z t · ¸t Z t · ¸t
£ ¤t cr+1 cr
5. c dr = cr 0 or
r r
c dr = (Note that cr = er log c )
0 r+1 0 0 log c 0

6. Mean present value = v T M.P.V. = Āx


Present value (as random variable) = Āx P.V. = v T
µ ¶
(1 − v T )
7. var(P āT − v T ) var P − vT
δ
= var(P āT ) − var(v T ) · µ ¶ ¸
P +δ P
= var − vT +
δ δ
or var(P āT ) + var(v T )
µ ¶2
P +δ
= var(v T )
δ

8. äx:n = 1 + ax:n äx:n = 1 + ax:n−1

lx0 lx0
9. µx = µx = −
lx lx
µZ t ¶ µ Z t ¶
10. t px = exp µx+r dr t px = exp − µx+r dr
0 0

Nx − Nx+n Nx+1 − Nx+n+1


11. ax:n = ax:n =
Dx Dx

335
Appendix D

Some formulae for numerical integration

1. Mid-point rule

Z b µ ¶
a+b
f (x)dx + (b − a)f
a 2

2. Trapezoidal rule
Z b µ ¶
b−a
f (x)dx + [f (a) + f (b)]
a 2

3. Simpson’s rule
Z b µ ¶· µ ¶ ¸
b−a a+b
f (x)dx + f (a) + 4f + f (b)
a 6 2

4. Three-eighths rule
Z b µ ¶· µ ¶ µ ¶ ¸
b−a 2a + b a + 2b
f (x)dx + f (a) + 3f + 3f + f (b)
a 8 3 3

336
336
age
i i
x Cxi Mxi Rx z
Cxi z
Mxi z
Rx
20 0.0 188.8 6835.5 0.0 904.4 33753.7 TABLES FOR
21 0.0 188.8 6646.7 0.0 904.4 32849.3 LUMP SUM
22 0.0 188.8 6457.9 0.0 904.4 31944.9 BENEFITS ON
23 0.0 188.8 6269.1 0.0 904.4 31040.5 RETIREMENT
24 0.0 188.8 6080.3 0.0 904.4 30136.1

25 0.0 188.8 5891.5 0.0 904.4 29231.7 Ill-health


26 0.0 188.8 5702.8 0.0 904.4 28327.3 retirement functions
27 0.0 188.8 5514.0 0.0 904.4 27422.9
28 0.0 188.8 5325.2 0.0 904.4 26518.5
29 0.0 188.8 5136.4 0.0 904.4 25614.1

30 0.0 188.8 4947.6 0.0 904.4 24709.7


31 0.9 188.8 4758.8 2.2 904.4 23805.3
32 0.8 187.9 4570.5 2.2 902.2 22902.0
33 0.8 187.1 4383.0 2.2 900.0 22000.9
34 1.6 186.3 4196.3 4.4 897.8 21101.9

35 1.5 184.7 4010.8 4.4 893.5 20206.3


36 1.4 183.2 3826.8 4.3 889.1 19315.0
37 1.8 181.8 3644.3 5.8 884.8 18428.1
38 1.8 180.0 3463.5 5.7 879.0 17546.2
39 1.7 178.2 3284.4 5.7 873.3 16670.1

40 2.0 176.5 3107.1 7.0 867.6 15799.6


SUPPLEMENT 41 2.0 174.4 2931.6 6.9 860.6 14935.5
42 2.3 172.5 2758.1 8.2 853.7 14078.3
43 2.2 170.2 2586.8 8.1 845.5 13228.7
44 2.4 168.0 2417.6 9.4 837.3 12387.3

45 2.4 165.6 2250.8 9.2 828.0 11554.7


46 2.6 163.2 2086.4 10.4 818.7 10731.3
47 2.9 160.7 1924.5 12.2 808.3 9917.8
48 3.1 157.7 1765.3 13.3 796.2 9115.5
49 3.3 154.6 1609.1 14.3 782.9 8326.0

50 3.9 151.3 1456.2 17.1 768.6 7550.3


51 4.2 147.4 1306.9 19.2 751.5 6790.2
52 5.0 143.2 1161.6 22.9 732.4 6048.3
53 5.4 138.2 1020.9 25.3 709.5 5327.4
54 6.3 132.8 885.4 29.8 684.1 4630.6

55 6.9 126.5 755.7 33.5 654.3 3961.3


56 7.7 119.6 632.6 38.0 620.8 3323.7
57 8.7 111.9 516.9 43.3 582.8 2721.9
58 10.0 103.2 409.4 50.3 539.5 2160.8
59 11.4 93.2 311.2 58.3 489.2 1646.5

60 13.6 81.8 223.7 70.2 430.8 1186.5


61 13.7 68.2 148.7 71.5 360.6 790.8
62 15.3 54.4 87.4 80.7 289.1 465.9
63 18.0 39.1 40.7 95.5 208.4 217.1
64 21.1 21.1 10.6 112.9 112.9 56.5

337
SUPPLEMENT
age
r r
x Cxr Mxr Rx z
Cxr z
Mxr z
Rx
20 0.0 1524.0 65671.0 0.0 8048.7 347094.1 TABLES FOR
21 0.0 1524.0 64147.0 0.0 8048.7 339045.4 LUMP SUM
22 0.0 1524.0 62623.1 0.0 8048.7 330996.8 BENEFITS ON
23 0.0 1524.0 61099.1 0.0 8048.7 322948.1 RETIREMENT
24 0.0 1524.0 59575.2 0.0 8048.7 314899.5

25 0.0 1524.0 58051.2 0.0 8048.7 306850.8 Age retirement


26 0.0 1524.0 56527.2 0.0 8048.7 298802.2 functions
27 0.0 1524.0 55003.3 0.0 8048.7 290753.5
28 0.0 1524.0 53479.3 0.0 8048.7 282704.9
29 0.0 1524.0 51955.4 0.0 8048.7 274656.2

30 0.0 1524.0 50431.4 0.0 8048.7 266607.6


31 0.0 1524.0 48907.4 0.0 8048.7 258558.9
32 0.0 1524.0 47383.5 0.0 8048.7 250510.3
33 0.0 1524.0 45859.5 0.0 8048.7 242461.6
34 0.0 1524.0 44335.6 0.0 8048.7 234413.0

35 0.0 1524.0 42811.6 0.0 8048.7 226364.3


36 0.0 1524.0 41287.6 0.0 8048.7 218315.7
37 0.0 1524.0 39763.7 0.0 8048.7 210267.0
38 0.0 1524.0 38239.7 0.0 8048.7 202218.4
39 0.0 1524.0 36715.8 0.0 8048.7 194169.7

40 0.0 1524.0 35191.8 0.0 8048.7 186121.1


41 0.0 1524.0 33667.8 0.0 8048.7 178072.4
42 0.0 1524.0 32143.9 0.0 8048.7 170023.8
43 0.0 1524.0 30619.9 0.0 8048.7 161975.1
44 0.0 1524.0 29096.0 0.0 8048.7 153926.5

45 0.0 1524.0 27572.0 0.0 8048.7 145877.8


46 0.0 1524.0 26048.0 0.0 8048.7 137829.2
47 0.0 1524.0 24524.1 0.0 8048.7 129780.5
48 0.0 1524.0 23000.1 0.0 8048.7 121731.8
49 0.0 1524.0 21476.2 0.0 8048.7 113683.2

50 0.0 1524.0 19952.2 0.0 8048.7 105634.5


51 0.0 1524.0 18428.2 0.0 8048.7 97585.9
52 0.0 1524.0 16904.3 0.0 8048.7 89537.2
53 0.0 1524.0 15380.3 0.0 8048.7 81488.6
54 0.0 1524.0 13856.3 0.0 8048.7 73439.9

55 0.0 1524.0 12332.4 0.0 8048.7 65391.3


56 0.0 1524.0 10808.4 0.0 8048.7 57342.6
57 0.0 1524.0 9284.5 0.0 8048.7 49294.0
58 0.0 1524.0 7760.5 0.0 8048.7 41245.3
59 0.0 1524.0 6236.5 0.0 8048.7 33196.7

60 378.5 1524.0 4712.6 1952.7 8048.7 25148.0


61 212.4 1145.4 3377.9 1107.4 6096.0 18075.7
62 112.3 933.0 2338.7 591.0 4988.6 12533.4
63 93.8 820.7 1461.8 498.0 4397.5 7840.4
64 77.7 726.9 688.1 415.5 3899.6 3691.8

65 649.2 649.2 3484.1 3484.1

338
SUPPLEMENT
Functions for valuing (at interest rate i per annum) refunds on death of contributions with
compound interest at interest rate j per annum
i = .04
j = 0.03

j d d
x Md j
Rx sj
Rx
P64 x j d P64 −(y+.5) j d P 64 d
= y=x Cy = y=x (1 + j) . M y = y=x (1 + j)−(y+.5) .sj M y
20 3108.18 31393.46 87740.98
25 2878.57 23700.42 76568.15
30 2700.59 17508.94 63315.76
35 2524.59 12500.67 49478.97
40 2338.12 8479.14 36324.46
45 2120.85 5291.34 24305.71
50 1804.15 2861.89 13941.80
55 1333.41 1177.45 6005.02
60 655.64 243.17 1286.33
61 490.11 147.36 784.29
62 350.45 79.12 423.29
63 225.80 33.70 181.20
64 108.85 8.09 43.67
Notes.
(1) j Cxd = (1 + j)x+.5 .(1 + i)−(x+.5) .dx
P64−x d
(2) j Mxd = t=0 j C x+t
d d 1 d
(3) j M x = j M x − 2 · jCx
sj d j d
(4) M x = sx · M x

339
SUPPLEMENT
Functions for valuing (at interest rate i per annum) refunds on withdrawal of contributions with
compound interest at interest rate j per annum
i = .04
j = 0.03
j w w
x Mw j
Rx sj
Rx
P64 x j w P64 −(y+.5) j w P64 w
= y=x Cy = y=x (1 + j) . M y = y=x (1 + j)−(y+.5) .sj M y
20 43547.50 119414.16 207941.08
25 19045.16 42216.97 98759.32
30 8041.95 13393.17 38299.39
35 3006.98 3281.50 10746.75
36 2399.53 2334.90 7831.22
37 1875.23 1608.25 5520.49
38 1426.97 1063.28 3732.97
39 1049.20 666.53 2391.95
40 735.82 388.85 1425.63
41 482.26 204.89 767.03
42 284.70 92.43 353.18
43 140.12 31.95 124.58
44 46.19 6.20 24.67
Notes.
(1) j Cxw = (1 + j)x+.5 .(1 + i)−(x+.5) .wx
P64−x w
(2) j Mxw = t=0 j C x+t
w w w
(3) j M x = j M x − 21 · j C x
w w
(4) sj M x = sx · j M x

340
SUPPLEMENT
Functions for valuing (at interest rate i per annum) refunds on death of contributions with
compound interest at interest rate j per annum
i = .04
j=0

j d d
x Md j
Rx sj
Rx
P64 x j d P64 −(y+.5) j d P 64 d
= y=x Cy = y=x (1 + j) . M y = y=x (1 + j)−(y+.5) .sj M y
20 792.85 18575.75 56093.05
25 674.16 14932.38 50789.94
30 594.98 11769.88 44005.80
35 527.62 8965.83 36247.43
40 466.07 6483.42 28117.45
45 404.38 4303.49 19889.59
50 326.91 2467.10 12048.69
55 227.54 1071.57 5469.33
60 104.08 232.42 1229.62
61 76.40 142.17 756.75
62 53.72 77.11 412.57
63 34.07 33.21 178.59
64 16.18 8.09 43.67
Notes.
(1) j Cxd = (1 + j)x+.5 .(1 + i)−(x+.5) .dx
P64−x d
(2) j Mxd = t=0 j C x+t
d d 1 d
(3) j M x = j M x − 2 · jCx
sj d j d
(4) M x = sx · M x

341
SUPPLEMENT
Functions for valuing (at interest rate i per annum) refunds on withdrawal of contributions with
compound interest at interest rate j per annum
i = .04
j=0
j w w
x Mw j
Rx sj
Rx
P64 x j w P64 −(y+.5) j w P64 w
= y=x Cy = y=x (1 + j) . M y = y=x (1 + j)−(y+.5) .sj M y
20 20579.08 103387.17 180950.89
25 7847.29 37164.60 87256.75
30 2916.96 12076.18 34605.99
35 971.53 3051.10 10002.54
36 758.81 2185.93 7337.82
37 580.57 1516.24 5208.20
38 432.61 1009.65 3546.59
39 311.55 637.57 2288.96
40 214.05 374.77 1374.41
41 137.46 199.01 745.20
42 79.53 90.52 345.94
43 38.36 31.58 123.14
44 12.40 6.20 24.67
Notes.
(1) j Cxw = (1 + j)x+.5 .(1 + i)−(x+.5) .wx
P64−x w
(2) j Mxw = t=0 j C x+t
w w w
(3) j M x = j M x − 21 · j C x
w w
(4) sj M x = sx · j M x

342

Das könnte Ihnen auch gefallen